You are on page 1of 506

Allergy&Immunology Allergy&Immunology

Niket Sonpal,MD
ChiefResident LenoxHillHospitalNSLIJ AssistantClinicalProfessor Touro CollegeofMedicine

Anaphylaxis Angioedema Urticaria AllergicRhinitis PrimaryImmunodeficiency Disorders

Anaphylaxis Worst form of acute allergic reaction Synonymous with immediate hypersensitivity Pathogenesis
Initial sensitization to antigen with subsequent reexposure Upon re-exposure, IgE binds to mast cells leading to release of their granules (e.g., histamine, prostaglandins, and leukotrienes) Results in abnormalities that essentially define anaphylaxis

Anaphylaxis Sensitization
Initial sensitization to antigen

Re-exposure Anaphylaxis
Upon re-exposure, IgE binds to mast cells leading to release of their granules
LEUKOTRIENES

DEATH

HISTAMINE

PROSTAGLANDINS

MTBS2CK p.41

MTBS2CK p.41

Hives Itching Constriction of airways Swollen tongue Wheezing Dyspnea Tachycardia Nausea, vomiting, or diarrhea Dizziness or fainting Hypotension

Anaphylaxis Anaphylactoid Reactions


No presensitization to antigen

Anaphylaxis/Etiology

Causes of anaphylaxis = causes of any allergic event


Insect bites and stings Medications: penicillin, phenytoin, lamotrigine, quinidine, rifampin, and sulfa Foods Latex is a very important cause of anaphylaxis in healthcare workers

Identical Tx

Non-IgE related

MTBS2CK p.41

MTBS2CK p.41

Anaphylaxis/Presentation

Anaphylaxis/Treatment

Characterized by Hypotension Tachycardia Respiratory: stridor Rash

The best initial treatment is A Airway Protection


Intubation Cricothyroidodotomy

B Breathing g C - Circulation
Epinephrine Antihistamines H1 and H2 Glucocorticoids
MTBS2CK p.42

Source: Fashad Bagheri. MD

MTBS2CK p.4142

Angioedema Angioedema is sudden swelling of


Face Tongue Eyes Airway
Look for recent start of ACE inhibitors preceding symptoms

Angioedema/Presentation

Hereditary angioedema is a genetic disorder Glucocorticoids dont work

This can be from deficiency of C1 esterase inhibitor Characteristic association of onset with minor physical trauma Angioedema often idiopathic

Source: commons.wikimedia.org

MTBS2CK p.42

MTBS2CK p.43

Angioedema/Tests The best initial test...


C2 and C4 levels in complement pathway
Will be decreased

Angioedema/Treatment Airway Acute Tx Long Term Tx

C1 esterase inhibitor
Also decreased

3 types of hereditary angioedema


Type I - decreased levels of C1INH (85%) Type II - normal levels but decreased function of C1INH (15%) Type III - no detectable abnormality in C1INH

Ensure airway protection first

Fresh frozen plasma Ecallantide E ll tid

Androgens Danazole And Stanazole

MTBS2CK p.43

MTBS2CK p.43

Urticaria Form of allergic reaction that causes sudden swelling of superficial skin layers Can be caused by
Insects Medications Pressure Cold Vibration

Urticaria/Treatment 1. Antihistamines
Hydroxyzine, diphenhydramine, fexofenadine, loratidine, cetirizine, or ranitidine

2 L 2. Leukotriene k ti receptor t antagonists


Source: commons.wikimedia.org

Montelukast or zafirlukast
Source: commons.wikimedia.org

MTBS2CK p.43

MTBS2CK p.44

AllergicRhinitis

AllergicRhinitis/DiagnosticTests

IgE-dependent triggering of mast cells Seasonal allergies such as hay fever are common Presents with recurrent episodes of Watery y eyes, y , sneezing, g, itchy y nose, , and itchy eyes Inflamed, boggy nasal mucosa Pale or violaceous turbinates Nasal polyps
MTBS2CK p.44

Diagnosed clinically Skin testing and blood testing IgE levels may be elevated Nasal smear with eosinophils
Source: commons.wikimedia.org

MTBS2CK p.44

AllergicRhinitis/Treatment

CommonVariableImmunodeficiency(CVID)

1. Prevention & avoidance 2. Intranasal corticosteroid sprays 3. Antihistamines H1 blockers 4. Intranasal anticholinergic medications 5. Desensitization to allergens

B cells are present but decreased Igs Decrease in all Ig subtypes


IgG, IgM, and IgA

LOW B cells output, normal T cells

MTBS2CK p.44

MTBS2CK p.45

CommonVariableImmunodeficiency(CVID) Recurrent sinopulmonary infections Frequent episodes of


Bronchitis Pneumonia Sinusitis And Otitis media

CommonVariableImmunodeficiency(CVID)

Other manifestations are


Giardiasis Sprue-like intestinal malabsorption

Clue to CVID is a decrease in the output of B lymphocytes with a normal number of B cells as well as normal amounts of lymphoid tissue (e.g., nodes, adenoids, and tonsils)

MTBS2CK p.45

MTBS2CK p.45

CommonVariableImmunodeficiency(CVID)

CommonVariableImmunodeficiency(CVID)

Immunoglobulin levels
Decreased

Antigen Stimulation Decreased response


Normal Number of B Cells

Treatment Antibiotics are used for each infection as it develops Chronic maintenance
Regular Reg lar inf infusions sions of IVIG

MTBS2CK p.45

MTBS2CK p.45

Xlinked(Bruton)Agammaglobulinemia Low B, normal T in males Male children with increased sinopulmonary infections B cells and lymphoid tissues are diminished T cells are normal Treatment: Abx for infections as they arise Long-term regular administration of IVIG keeps children healthier

SevereCombinedImmunodeficiency LOW B AND LOW T The word combined in severe combined immunodeficiency (SCID) means that there is deficiency in both B and T cells
Results in infections related to both deficiencies

LOW B cells and LOW T cells. Analogous to HIV LOW B cells, normal T cells in young male children
MTBS2CK p.45 MTBS2CK p.46

SevereCombinedImmunodeficiency

IgADeficiency These patients present with recurrent sinopulmonary infections The difference with this syndrome is: Atopic diseases Anaphylaxis to blood transfusion when blood donor has normal levels of IgA Sprue-like condition with fat malabsorption Increase in risk of vitiligo, thyroiditis, and rheumatoid arthritis

B cells Decreased Ig production T cells Markedly decreased numbers of T cells Long-Term Treatment Bone marrow transplant
MTBS2CK p.46

MTBS2CK p.46

IgADeficiency

HyperIgE Syndrome Presents with recurrent skin infections due to Staphylococcus Treat infections as they arise Consider prophylactic antibiotics (e.g., dicloxacillin or cephalexin)
Source:wikimedia commons

Treat infections as they arise Washed blood transfusions otherwise anaphylaxis IVIG injections will not work The Th t trace amounts t of fI IgA Ai in IVIG may provoke anaphylaxis

MTBS2CK p.46

MTBS2CK p.46

WiskottAldrichSyndrome Normal T cells normal B cells decline with age Immunodeficiency combined with thrombocytopenia and eczema T lymphocytes markedly deficient in blood and lymph nodes Bone marrow transplantation is only definitive treatment

ChronicGranulomatousDisease(CGD)

CGD is genetic disease resulting in extensive inflammatory reactions Leads to lymph nodes with purulent material leaking out

Normal T cells normal B cells decline with age

MTBS2CK p.46

MTBS2CK p.46

ChronicGranulomatousDisease(CGD)

ChronicGranulomatousDisease(CGD)

Aphthous ulcers And Inflammation of nares is common

Granulomas may become obstructive in the GI or urinary tract Look for infections with odd combination of:
Staphylococcus Burkholderia Nocardia Aspergillus
Source:commons.wikimedia.org

MTBS2CK p.46

MTBS2CK p.4647

ChronicGranulomatousDisease(CGD)

PrimaryImmunodeficiencyDisorders
CVID LOW B cell output Normal T cells Hyper IgE Syndrome Skin infections Staphylococcus X-linked (Bruton) Agammaglobulinemia LOW B cells normal T cells in young male children SCID IgA Deficiency Atopic Anaphylaxis

Abnormal nitroblue tetrazolium testing


This is a decrease in NADPH oxidase, which generates superoxide CGD test is negative

LOW B & T cells Analogous to HIV

Wiskott-Aldrich Syndrome
Normal T cells Normal B cells Low Platelets Eczema

CGD Lymph nodes with purulent material Infections, combined with Staphylococcus Burkholderia Nocardia Aspergillus

MTBS2CK p.47

MTBS2CK p.4547

TheCardiovascularSystem
ConradFischer,MD AssociateProfessorofMedicine TouroCollegeofMedicine NewYorkCity

CoronaryArtery Disease Part1


Definition D fi iti RiskFactors ClinicalPresentation

CoronaryArteryDisease(CAD)/Definition

Can be used interchangeably with:


Atherosclerotic heart disease Ischemic heart disease

All imply insufficient perfusion of coronary arteries Abnormal narrowing of vessels Insufficient oxygen delivery to myocardial tissue
MTBS2CK p.49

48-year-old woman in office with chest pain for several weeks. Pain isn't reliably related to exertion. No pain now. The pain is retrosternal and sometimes associated with nausea. No SOB and no radiation beyond chest. She has no past medical history. What is the most likely diagnosis? a. Gastroesophageal reflux disease (GERD) b. Unstable angina Acute, severe pain in ED c. Pericarditis Pain worse with lying down, better when sitting up d. Pneumothorax Sharp, pleuritic pain, tracheal deviation e. Prinzmetal angina Nonexertional chest pain,

MTBS2CK p.49

early in morning, ST segment elevation

RiskFactorsforCAD Menstruating women virtually never have MIs Which of the following is most likely to benefit a patients risk of coronary disease?
Estrogen simply does NOT help CAD

Risk factors are most important with equivocal or uncertain histories

a. Administration of estrogen replacement at time of menopause b Stopping tamoxifen Tamoxifen unrelated to CAD b. c. Stopping aromatase inhibitors Zero relationship to CAD d. Regular exercise e. Relaxation methods (e.g., meditation) Good, but

Women > men eventually die of heart disease


MTBS2CK p.50 MTBS2CK p.50

immeasurable, no evidence of proven benefit

RiskFactorsforCAD

RiskFactorsforCAD

Clear ones:

Diabetes mellitus Tobacco smoking HTN Hyperlipidemia Family history of premature CAD Age > 45 men; > 55 women

Diabetes (worst risk) HTN Defined as > 140/90 More common than diabetes 20% of total population (60 million people) with HTN 50% unaware theyre hypertensive

MTBS2CK p.50

MTBS2CK p.50

RiskFactorsforCAD

Family history not a risk if CAD developed in elderly relatives Relatives were grandparents, cousins, or aunts and uncles Family y history y is a risk ONLY if First-degree relatives (siblings/parents) Premature disease: Defined as:
Male relative < 55 Female relative < 65
MTBS2CK p.5051

Which of the following is the most dangerous to a patient in terms of risk for CAD?
Triglycerides not as dangerous as elevated LDL

a. Elevated triglycerides b. Elevated total cholesterol Its not the TOTAL cholesterol or low l HDL HDL, it its LDL! c. Decreased D d HDL d. Elevated LDL e. Obesity The danger of obesity is from its association

with high LDL, DM, and HTN

MTBS2CK p.51

RiskFactorsforCAD

Less clear Physical inactivity Excess alcohol Insufficient fruits & vegetables Emotional stress CT scan calcium scores Positron emission tomography (PET) scanning
MTBS2CK p.51

Postmenopausal woman develops chest pain immediately on hearing news of her sons death. She develops chest pain, dyspnea, and ST segment elevation in leads V2 to V4. Troponin levels rise with MI. Coronary angiography is normal including an absence of vasospasm. Echocardiography: Apical LV ballooning Whats the presumed mechanism of this disorder? a. Absence of estrogen g Its not absent estrogen g only y causing g MI b. Massive catecholamine discharge c. Plaque rupture There were no plaques on angiography d. Platelet activation Platelets cause MI from CAD, not emotions e. Emboli to coronary arteries It wouldve been seen

MTBS2CK p.52

RiskFactorsforCAD Unreliable (Unproven) Risk Factors for CAD Homocysteine Chlamydia infection C-reactive protein No clear benefit in therapeutic intervention on these factors Theyre the wrong answers

RiskFactorsforCAD

The Most Common Wrong Answer If risk factor question involves: Family history Mistaking CAD in elderly relatives as a risk for patient

Frequently used wrong answers are just as important to learn as right answers
MTBS2CK p.52 MTBS2CK p.52

ChestPain/Presentation Correcting which of the following risk factors for CAD results in the most immediate benefit?

What Is the Most Likely Diagnosis? Ischemic pain


Dull /Sore Squeezing (Pressure-like)

a. b. c c. d. e.

Diabetes mellitus Tobacco smoking Hypertension Hyperlipidemia Weight loss

Fixing all of them is good. But Not as fast as smoking cessation!

Like any muscle thats starved for oxygen Produces sore-muscle type pain when ischemic Qualities that go against ischemia
Sharp (knifelike) or pointlike Lasts for few seconds

MTBS2CK p.53

MTBS2CK p.53

ChestPain/Presentation

ChestPain/Presentation

Three features help tell whether or not the pain is ischemic in nature:
1. Changes with respiration (pleuritic) 2. Changes with position of body 3. Changes with touch of chest wall (tenderness)

< 10% with chest pain in ED end up having an MI 50% have no cardiac disease at all MCC of chest pain that isnt isn t ischemic in nature is GI related

Each (pleuritic, positional, tender) excludes ischemia with 95% negative predictive value (NPV)

MTBS2CK p.53

MTBS2CK p.53

Characteristics of Ischemic Pain

CausesofChestPain
Location Substernal

Ifthecase describes. Chestwalltenderness Radiationtoback, unequalblood pressurebetween arms

Answerasmost likelydiagnosis Costochondritis Aorticdissection

Answerasmost accuratetest Physicalexamination ChestXray withwidened mediastinum,chest CTA,MRA,orTEE confirmsdisease EKGwithSTelevation everywhere,PR depression

Duration Stable angina: > 2 - < 10 min ACS: > 10 - 30 min

Provoking factors Physical activity, activity cold cold, emotional stress

Quality Squeezing, tightness, heaviness, pressure, burning, aching NOT: sharp, pins, stabbing, or knifelike

Alleviating factors Rest

NOT tender NOT positional NOT pleuritic

Associated symptoms SOB, nausea, diaphoresis, dizziness, lightheadedness, fatigue

Radiation Neck, lower jaw and teeth, arms, shoulders

Painworsewithlying flat,betterwhen sittingup

Pericarditis

MTBS2CK p.54

CausesofChestPain
Ifthecase describes. Epigastricdiscomfort, painrelatedtoeating Answerasmost likelydiagnosis Duodenalulcer disease Answerasmost accuratetest Endoscopy Ifthecase describes. Suddenonset,SOB, tachycardia,hypoxia Sharp,pleuriticpain, trachealdeviation

CausesofChestPain
Answerasmost likelydiagnosis Pulmonaryembolus Answerasmost accuratetest SpiralCT,V/Qscan

Pneumothorax

ChestXray

,cough, g , Badtaste, hoarseness

p g Gastroesophageal reflux

Response p toPPIs; ;or liquidantacids

Cough,sputum, hemoptysis

Pneumonia

ChestXray

MTBS2CK p.54

MTBS2CK p.54

ChestPain/Presentation

Features that DONT help answer the diagnosis question Nausea Fever SOB (dyspnea) (d ) Sweating (diaphoresis) Anxiety

CoronaryArtery Disease Part2


Diagnosis

MTBS2CK p.54

DiagnosticTests/Electrocardiogram

DiagnosticTests/Enzymes(CKMB/Troponin)

The best initial test for all forms of chest pain is certainly an electrocardiogram (EKG) In office-based setting:
Expect normal EKG

Cardiac enzymes are not the answer in Office/ambulatory case Chronic or stable chest pain If the patient has acute chest pain then the answer is:
Transfer

But You cant do other testing until you know the EKG
MTBS2CK p.55

to the emergency department

MTBS2CK p.55

DiagnosticTests/Enzymes(CKMB/Troponin)

DiagnosticTests/StressTesting Exercise tolerance testing (ETT) is the answer when: Etiology is unclear and EKG is not diagnostic Answer Stress test ETT without nuclear isotopes: when: Etiology
1. Read EKG 2 Exercise 2.

Enzymes are the answer when: Acute cases of chest pain Emergency department

uncertain & EKG not diagnostic

Exercise means heart rate > 80% of maximum Maximum heart rate = 220 patient age
MTBS2CK p.55 MTBSCK p.55

DiagnosticTests/StressTesting Thallium (Nuclear) Stress Normal myocardium picks up thallium like potassium via Na/K-ATPase Myocardium alive & perfused? Nuclear isotopes will be picked up Abnormalities/Ischemia or Infarction? Decreased thallium or nuclear uptake Stress or Dobutamine Echo Normal myocardium moves on echo Abnormalities decrease wall motion Dyskinesis, Akinesis, or Hypokinesis
MTBSCK p.56

DiagnosticTests/StressTesting

Ischemia versus infarction: Ischemia = Reversible decreased perfusion Reversal of decrease in thallium uptake or wall motion It returns to normal after a period of rest Infarction = Irreversible
Ischemia is reversible wall motion or thallium uptake between rest and exercise. Infarction is irreversible or fixed.
MTBSCK p.56

DiagnosticTests/StressTesting
Cant exercise?

UseofExerciseToleranceTesting Test
Exercisetolerance

Indication

Ischemiadetected

Persantine (dipyridamole) or adenosine with nuclear isotopes (e.g., thallium or sestamibi) Or


Dobutamine in combination with echocardiography Dobutamine increases myocardial oxygen consumption Dobutamine provokes ischemia Ischemia decreases wall motion on echocardiogram

Determinepresence STsegmentdepression ofischemia CantreadEKG Decreaseduptakeof p nuclearisotope Decreasedwallmotion Decreaseduptakeof nuclearisotope Decreasedwallmotion

Exercisethallium

Exerciseecho

CantreadEKG

Dipyridamolethallium Cantexercise

Dipyridamole may provoke bronchospasm. Avoid in asthmatics.


MTBSCK p.56

Dobutamineecho
MTBS2CK p.56

Cantexercise

DiagnosticTests/StressTesting

Chest pain (likelihood of CHD is high)


Resting EKG abnormalities?

Nuclear and Echo:

No Able to exercise? Yes No Chemical stress test (dipyridamole thallium or dobutamine echo)

Yes

Sensitivity = Specificity
Exercise Thallium = Exercise Echo Dipyridamole Thallium = Dobutamine Echo

Exercise stress test

+
ANGIOGRAPHY

Stress echocardiogram or nuclear stress test

1 or 2 vessel disease

3 vessel disease, left main or 2 vessel disease in diabetics CABG

MTBSCK p.56

Stent placement

DiagnosticTests/CoronaryAngiography

DiagnosticTests/CoronaryAngiography

Detects anatomic location of disease Determines surgery, angioplasty, or other methods of revascularization Sometimes used if noninvasive tests are equivocal

Stenosis (narrowing) < 50% of diameter is insignificant Surgery or angioplasty is done for at least 70% stenosis

MTBSCK p.5657

MTBSCK p.57

DiagnosticTests/HolterMonitoring

Continuous ambulatory EKG monitor Records rhythm Usually for 24-hour period May be for 48 to 72 hours Holter monitor detects rhythm disorders: A-fib, atrial flutter Ectopy (e.g., premature beats) V-tach Holter monitor does not detect ischemia Not accurate for evaluating the ST segment
MTBSCK p.57

48-year-old woman comes to office with chest pain occurring over several weeks. Pain isnt reliably related to exertion. Shes comfortable now. The pain is retrosternal. Past medical history and the EKG is normal. What is the most appropriate next step in management?

a. CK-MB Cardiac enzymes evaluate ACS b. Troponin Evaluate valve function, function wall motion motion, EF c. Echocardiogram E h di d. Exercise tolerance testing Detect the anatomic location of stenosis; e. Angiography f. CT angiography determines method of revascularization CT Angiogram and MRI not accurate enough g. Cardiac MRI h. Holter monitor Used for rhythm evaluation

CAD/Treatment USMLE Step 2 CK is most concerned that you know:

CoronaryArtery Disease Part3


Treatment

Medications that lower mortality


Chronic angina (not an acute coronary syndrome)
Aspirin Beta blockers Nitroglycerin

Step 2 will not test dosing Route of administration is tested


MTBS2CK p.57

CAD/Treatment

CAD/Treatment

Nitroglycerin: Chronic Stable Angina Oral Transdermal patch Acute Coronary Syndrome Sublingual Paste Intravenous

Nonspecific beta blockers (e.g.. propranolol) are not used routinely in cardiology Clopidogrel
Acute MI (all forms) Aspirin intolerance (e.g., allergy) Recent angioplasty with stenting Adverse effect
Rare thrombotic thrombocytopenic purpura
MTBS2CK p.58

MTBS2CK p.58

Treatment/Prasugrel

Treatment/ACEInhibitors

Thienopyridine - same class as clopidogrel and ticlopidine Prasugrel: Antiplatelet medication for use in: Angioplasty & stenting All acute MI Intolerant of aspirin 75 increased hemorrhagic stroke
MTBS2CK p.58

Low EF/systolic dysfunction (best mortality benefit) Regurgitant valvular disease Most M t common adverse d effect: ff t Cough
7% of patients Switch to ARBs
MTBS2CK p.58

Treatment/LipidManagement
64-year-old man starts lisinopril for CAD with EF of 24%, and symptoms of breathlessness. He sometimes has rales, but is asymptomatic today. Physical reveals minimal edema of lower extremities. Potassium level is elevated and its present on a repeat measurement. EKG is unchanged. How would you best manage the patient?
Remove K from the body;You y; should eliminate the cause

Statins (HMG-CoA reductase inhibitors)

CAD with LDL > 100 mg/dL

a. b. c. d. e.

Add kayexalate (potassium-binding resin) Insulin and glucose Drive K into the cell, given in acute situation He should get an alternative drug Stop lisinopril Switch lisinopril to candesartan ARBs lead to hyperkalemia Switch lisinopril to hydralazine and nitrates

MTBS2CK p.5859

MTBS2CK p.59

Treatment/LipidManagement

CADEquivalents

LDL > 70: Treat when patient has Coronary disease AND diabetes
Youre tested on national guidelines from nonbiased federal organizations g Not private organizations (e.g., ACC) Everyone will agree: With CAD, goal of LDL at least < 100 mg/dL

CAD equivalents Statins to bring LDL down if > 100:


Peripheral artery disease (PAD) Carotid disease (not stroke) Aortic disease (aortic artery, not valve) Diabetes mellitus

MTBS2CK p.59

MTBS2CK p.59

OtherLipidLoweringTherapies Which is the most common adverse effect of statin medications? a. Rhabdomyolysis b. Liver dysfunction c Renal failure c. d. Encephalopathy e. Hyperkalemia Niacin, gemfibrozil, cholestyramine, and ezetimibe all have beneficial effects on lipid profiles None is Best initial therapy None has clear mortality benefit statins provide Niacin and fibric acid derivatives such as gemfibrozil have some mortality benefit benefit, but not as much as statins Statins: Antioxidant effect on endothelial lining of coronary arteries Gives benefit transcending lowering LDL number
MTBS2CK p.60

The most common wrong answer

They arent adverse effects of statins

MTBS2CK p.60

Niacin

Gemfibrozil

Associated with: Glucose intolerance Elevation of uric acid Uncomfortable itchiness from histamine Niacin is excellent to add to statins if full lipid control not achieved with statins Although statins, exercise, and cessation of tobacco use will all raise HDL level, niacin will raise HDL somewhat more
MTBS2CK p.60

Fibric acid derivatives: Lower triglyceride levels > statins Benefit of lowering triglycerides alone not proven as useful as straightforward mortality benefit of statins Use caution combining fibrates with statins

Fibrates + Statins= Increased myositis

MTBS2CK p.60

Cholestyramine

Ezetimibe

Bile acid sequestrant Significant interactions with medications in gut Potentially blocking their absorption Cholestyramine Ch l t i associated i t d with ith uncomfortable GI complaints Constipation & flatus

Definitely lowers LDL

No clear benefit to patient


LDL levels are an imperfect marker of benefit with cholesterol-lowering therapies Ezetimibe: No better than placebo No Change in MI, stroke, or death

MTBS2CK p.60

MTBS2CK p.60

LipidLoweringTherapy

LipidLoweringMedsandAdverseEffects
Agent Adverseeffect

Lipid-lowering therapy: What is clear? Statins lower mortality the most Questions about adverse effects Besides the benefit of statins in CAD with LDL levels > 100 mg/dL, the only truly clear aspect t of f other th therapies th i i is th their i adverse d effects

Statins

Elevationsoftransaminases(LFTs), myositis Elevationinglucoseanduricacidlevel, Pruritus Increasedriskofmyositiswhencombined withstatins Flatusandabdominalcramping Welltoleratedandnearlyuseless

Niacin

Fibric acid derivatives Cholestyramine Ezetimibe


MTBS2CK p.61

Check AST & ALT with statins


MTBS2CK p.61

CalciumChannelBlockers(CCBs)

CalciumChannelBlockers(CCBs)

Dihydropyridine CCBs:
Nifedipine Nitrendipine Nicardipine Nimodipine Ni di i

May increase mortality with CAD because of raising heart rate

CCBs are: Negative inotropes Should decrease myocardial oxygen consumption Increased heart rate in the aggregate gg g will increase myocardial oxygen consumption Bottom line:
Do not routinely use CCBs in CAD
None of the calcium-channel blockers have been shown to lower morality in CAD

MTBS2CK p.61

MTBS2CK p.61

CalciumChannelBlockers(CCBs)

CalciumChannelBlockers(CCBs)

Verapamil and diltiazem, which do not increase heart rate, are used in those who cannot tolerate beta blockers because of severe asthma 70% with reactive airway diseases (e (e.g., g asthma) still tolerate beta-1 specific BBs

Use CCBs (verapamil/diltiazem) in CAD only with: Severe asthma precluding the use of beta blockers Prinzmetal variant angina Cocaine-induced chest pain Adverse Effects of CCBs Edema You must know adverse effects Constipation Heart block (rare)
MTBS2CK p.6162

MTBS2CK p.61

10

Revascularization

CoronaryArteryBypassGrafting(CABG)

Angiography: Evaluates possibility of revascularization Either coronary bypass surgery or angioplasty Symptoms alone cannot tell the number of vessels involved

Lowers mortality only with: Three vessels with > 70% stenosis in each Left main occlusion Two-vessel T l di disease with ith di diabetes b t Persistent symptoms despite maximal medical therapy

MTBS2CK p.62

MTBS2CK p.62

CoronaryArteryBypassGrafting(CABG)

PercutaneousCoronaryIntervention(PCI)

Benefit greatest with left ventricular dysfunction Internal mammary artery grafts last 10 years Saphenous vein grafts last 5 years

Percutaneous coronary intervention (PCI) (angioplasty) Intervention best therapy in acute coronary syndromes Particularly those with ST segment elevation Mortality benefit of PCI Much harder to demonstrate in chronic stable angina
MTBS2CK p.62

MTBS2CK p.62

PercutaneousCoronaryIntervention(PCI)

Max medical therapy with aspirin, beta blockers, ACEi/ ARBs, and statins has proven benefit thats PCI in stable CAD PCI decreases dependence on medication Decreasing frequency of angina episodes
PCI: Best in acute coronary syndromes, particularly with ST segment elevation. PCI doesnt provide clear mortality benefit for stable patients.
MTBS2CK p.62

AcuteCoronary Syndromes Part1


Definition Presentation Diagnosis

11

AcuteCoronarySyndromes(ACS)

Definition Impossible to determine precise etiology of (ACS) from history & physical alone Risk factors (e (e.g., g hypertension hypertension, diabetes mellitus, tobacco) same for CAD

Acute Coronary Syndrome


EKG ST elevation No ST elevation Cardiac Biomarkers

+
STEMI
MTBS2CK p.62 MTBS2CK p.62

UA

NSTEMI

NSTEMI(Ischemia)

STEMI

S4Gallop 70-year-old woman in ED with crushing substernal chest pain for last hour. Pain radiates to left arm and is associated with anxiety, diaphoresis, and nausea. Pain is sore and dull and clenches fist in front of chest. History of hypertension. Pulsus paradoxus, Which is most likely in this patient? tamponade p a. >10 mmHg decrease in BP on inhalation b. Increase in jugular venous pressure on inhalation Kussmaul sign: constrictive friction rub: pericarditis c. Triphasic scratchy sound Pericardial pericarditis d. Continuous machinery murmur PDA (patent ductus) e. S4 gallop LVH/dilated cardiomyopathy f. Point of maximal impulse displaced to axilla

MTBS2CK p.63

12

AcuteCoronarySyndromes Increased jugulovenous pressure on inhalation Kussmaul sign Constrictive pericarditis Triphasic scratchy sound: Pericardial friction rub Dressler syndrome: Not until several days after MI Much rarer Everyone is on aspirin already!
MTBS2CK p.63

AcuteCoronarySyndromes Continuous machinery murmur Patent ductus arteriosus Displaced point of maximal impulse (PMI) LVH as well as dilated cardiomyopathy Displaced p PMI is an anatomic abnormality y Could not occur with ACS
Dont walk into Step 2 without knowing when you will expect each cardiac physical findings
MTBS2CK p.63

No specific physical finding allows you to answer a most likely diagnosis question in terms of ST elevation or depression without an EKG

70-year-old woman in ED with crushing substernal chest pain for the last hour. Which EKG finding gives the worst prognosis? a. ST elevation in leads II, III, aVF IWMI better prognosis than AWMI b. PR interval >200 milliseconds First degree AV block c ST elevation in leads V2-V4 c. V2 V4 d. Frequent premature ventricular complexes (PVCs) e. ST depression in leads V1 and V2 Posterior MI f. Right bundle branch block (RBBB)

Benign compared to LBBB

MTBS2CK p.6364

PVCs should not be treated, even with acute infarction

70-year-old woman comes to ED with crushing substernal chest pain for the last hour. EKG shows ST segment elevation in V2 to V4. What is the next step in management? a. CK-MB level Neither of them would be elevated b. Oxygen All of them should be given, c. Nitroglycerin sublingual but neither lowers mortality d. Aspirin e. Thrombolytics All will be done, but first aspirin f. Metoprolol Sh ld b Should be given, i ti time d doesnt t matter tt g. Atorvostatin Not in acute settings h. Angioplasty Never the right answer i. Consult cardiology j. Transfer patient to intensive care unit Start treatment first k. Troponin level l. Morphine m. Angiography

MTBS2CK p.6465

AcuteCoronarySyndromes
70-year-old woman comes to ED with crushing substernal chest pain for last hour. EKG shows ST segment elevation in V2 to V4. Aspirin has been given to chew. What is next step in this patient? a. CK-MB level Neither of them would be elevated b. Oxygen All of them should be given, c. Nitroglycerin sublingual but none lowers mortality d. Morphine Angioplasty is superior to thrombolytics e. Thrombolytics BBs and statins should be given, but they are f. Metoprolol not critically dependent upon time g. Atorvostatin h. Angioplasty i. Troponin level j. Lisinopril

Learn the order in which to do things. You must know what is first.

MTBS2CK p.65

MTBS2CK p.65

13

AcuteCoronarySyndromes

ACS/DiagnosticTests
Test Timetobeing abnormal Durationofabnormality

On USMLE Step 2 CK, consultation is almost never the correct choice. Do everything yourself.
In ACS:
Does the treatment lower Which is most important to do

EKG

Immediatelyat onsetofpain

STelevationprogresses toQwavesoverdays toaweek 12days 12days 1014days

Myoglobin

14hours 46hours 46hours

mortality? first?

CKMB Troponin

MTBS2CK p.65

MTBS2CK p.66

ACS/DiagnosticTests

DiagnosticTests/Reinfarction

Troponin Cant distinguish reinfarction several days after the first event Renal insufficiency gives false positive tests Troponin: excreted through the kidney

New episode of pain within a few days of the first? EKG detects new ST segment abnormalities Check CK-MB levels After 2 days, y , the CK-MB level from the initial infarction returns to normal CK-MB elevated several days after initial MI indicates a new ischemic event

MTBS2CK p.66

MTBS2CK p.66

IntensiveCareUnitMonitoring

After initial management patient should be in ICU Continuous rhythm monitoring is essential Monitoring and rapid cardioversion improves survival
Most common cause of death first several days after myocardial infarction is ventricular arrhythmia (ventricular tachycardia, ventricular fibrillation) Rapid electrical cardioversion or defibrillation
MTBS2CK p.66

AcuteCoronary Syndromes Part2


Treatment

14

Treatment/STEMI Angioplasty versus Thrombolytics Angioplasty (PCI) superior to thrombolytics


Better survival and mortality Less bleeding Complications of MI decreased Less arrhythmia, less CHF, fewer ruptures of septum, free wall [tamponade] and papillary muscles [valve rupture]

Treatment/STEMI

Complications of PCI Rupture of coronary artery Restenosis Hematoma at entry site into artery Only 20% of U.S. hospitals can do primary angioplasty Important to have ability to perform emergency cardiac surgery to repair the vessel
MTBS2CK p.67

Standard of care: PCI within 90 minutes of arriving in emergency department with chest pain Door to balloon time: under 90 minutes
MTBS2CK p.67

Treatment/STEMI Which is most important in decreasing the risk of restenosis of the coronary artery after PCI? a. Multistage procedure (i.e., doing 1 vessel at a time, with multiple procedures) Used only at procedure b Use of heparin for 36 b. 3 6 months after the procedure c. Warfarin use after the procedure For DVT and PE not coronary disease d. Placement of bare metal stent e. Placement of drug-eluting stent (paclitaxel, sirolimus)
Has much more risk for restenosis than drug-eluting stent Doesnt change risk with each vessel done

Restenosis within 6 Months of PCI No stent: 30% 40% Bare metal stent:15% 30% Drug-eluting stent:10% Cant use thrombolytics? Transfer to a facility performing PCI

MTBS2CK p.67

MTBS2CK p.67

Treatment/STEMI

Absolute Contraindications to Thrombolytics Major bleeding:


Bowel (melena), brain

Recent surgery (within the last 2 weeks) Severe hypertension (>180/110) Nonhemorrhagic stroke within the last 6 months Heme-positive brown stool is not an absolute contraindication to the use of thrombolytics
MTBS2CK p.6768

Patient comes to small rural hospital without catheterization lab with chest pain and ST segment elevation. What is the next step in management? a. Transfer for angioplasty Should be done within 90 minutes b. Administer thrombolytics now c. Consult cardiology Never the right answer

Time is muscle. Delay = Death Door to needle time: under 30 minutes


MTBS2CK p.68

15

TreatmentIndicationsandBenefits
Therapy Inwhatcasesiseffectgreatest?

TreatmentIndicationsandBenefits
Therapy Inwhatcasesiseffectgreatest?

Aspirin Clopidogrel

Everyone,bestinitialtherapy AllMIs,undergoingangioplastyand stenting Everyone,effectnotdependenton time;startedanytimeduring admission Everyone,benefitbestwithejection fraction<40%

Statins Oxygen,nitrates Heparin

Everyone,bestwithLDL>100mg/dL Noclearmortalitybenefit Afterthrombolytics/PCItoprevent restenosis,initialtherapywithNSTEMI andunstableangina Cantusebetablockers,cocaine inducedpain,Prinzmetal angina

Betablockers

ACEI/ARBs
MTBS2CK p.68

Calciumchannel blockers
MTBS2CK p.68

Treatment/STSegmentDepressionACS
A man comes to the ED with chest pain for the last hour that is crushing in quality and does not change with respiration or position of his body. EKG: ST segment depression, V2 to V4. Aspirin has been given. What is the next step in this patient? a. Low molecular-weight heparin b. Thrombolytics Benefit only with STEMI c. Glycoprotein IIb/IIIa inhibitor (abciximab) PCI with stenting

d. e. f. g.

Nitroglycerin No clear benefit Morphine Angioplasty Heparin first Metoprolol Should be given, but not urgent

Glycoprotein IIb/IIIa Inhibitors (Abciximab, Tirofiban, Eptifibitide) Used in acute coronary syndromes Those who undergo angioplasty and stenting Not beneficial in acute ST elevation infarctions Inhibits I hibi platelet l l aggregation i Reduction in mortality with ST depression, particularly in patients whose troponin or CK-MB levels rise and then develop an MI requiring PCI with stenting
MTBS2CK p.69

MTBS2CK p.69

SummaryofTreatmentDifferencesbetween CardiacEvents Stableangina UA/NSTEMI STEMI

SummaryofTreatmentDifferencesbetween CardiacEvents Stableangina UA/NSTEMI STEMI

Aspirin Betablockers Nitrates Heparin

Yes Yes Yes No

Yes Yes Yes Yes

Yes Yes Yes Yes,onlyafter thrombolytics

Thrombolytics No

No

Yes,butnotas goodasPCI No No

CCBs Warfarin

No No

No No

GPIIb/IIIa meds No
MTBS2CK p.70

Yes

No
MTBS2CK p.70

16

AcuteCoronarySyndromes/Treatment
Bottom line: 1. tPA (thrombolytics) are beneficial only with STEMI 2. Heparin is best for NSTEMI 3. GPIIb/IIIa inhibitors are best for NSTEMI and those undergoing PCI and stenting

Treatment/STSegmentDepressionACS

In non-ST elevation ACS, when all medications have been given and the patient is not better, urgent angiography and possibly angioplasty (PCI) should be done. Not better means: Persistent pain S3 gallop or CHF developing Worse EKG changes Rising troponin levels

Calcium-channel blockers & warfarin: No mortality benefit in ACS LMW heparin superior to unfractionated heparin for mortality benefit
MTBS2CK p.70

MTBS2CK p.70

Acute Coronary Syndromes STEMI Non-STEMI/UA


Aspirin/Clopidogrel Beta Blockers Statin ACE Morphine p Nitrates

ComplicationsofAcuteMI

Excellent source of: What is the most likely diagnosis? questions, (most common question on Step 2) All MI complications can result in low BP Hypotension: Not help determine diagnosis

PCI
If available <90 min after patient arrives
1. 2. Emergency CABG Failed PCI Ischemia refractory ALL therapy

Thrombolytics
If PCI not available. Use within 12 hrs from start of chest pain

Heparin Early PCI


Absolute contraindications to thrombolytics Major bleeding (bowel/brain) Recent surgery (within last 2 weeks) Severe hypertension (>180/110) Nonhemorrhagic stroke last 6 months

MTBS2CK p.70

MTBS2CK p.71

ComplicationsofAcuteMI/Bradycardia

ComplicationsofAcuteMI/Bradycardia

Heart rate: Key to establishing diagnosis Sinus bradycardia: Very common with MI From ischemia of sinoatrial (SA) node

Third-degree (complete) AV block:

Cannon a waves
Distinguishes 3rd degree block from sinus bradycardia before EKG From atrial systole against closed tricuspid Tricuspid valve closed because essence of third-degree block is atria and ventricles contracting separately Atria/ventricles out of coordination with each other
MTBS2CK p.71

MTBS2CK p.71

17

ComplicationsofAcuteMI/Bradycardia

ComplicationsofAcuteMI/Tachycardia

Cannon is jugulovenous wave bouncing up into the neck Look for: right ventricular infarction and third-degree AV block All symptomatic bradycardias: 1. Treated first with atropine 2. Then place pacemaker if the atropine is not effective 3. Pace all permanent 3rd degree blocks
MTBS2CK p.71

Right Ventricular Infarction Look for: New inferior wall MI & clear lungs on auscultation Cant g get blood into lungs g if blood cant enter heart Flip EKG leads from left side to right side of chest Most specific finding: ST elevation in right lead 4 (RV4)
MTBS2CK p.71

ComplicationsofAcuteMI/Tachycardia

ComplicationsofAcuteMI/Tachycardia

Right coronary supplies:

Right ventricle (RV) AV node Inferior wall of the heart


Up to 40% with inferior wall myocardial infarctions (IWMI) have right ventricular infarction Treat RV infarct with high-volume fluid Avoid nitroglycerin (markedly worsens filling)
MTBS2CK p.71

Tamponade/Free Wall Rupture Usually takes several days after infarction for wall to scar & weaken enough for rupture Look for sudden loss of pulse Lungs: Clear Cause of Pulseless Electrical Activity (PEA) Test: Echocardiography Treat: Pericardiocentesis is done on way into operating room for repair
MTBS2CK p.72

ComplicationsofAcuteMI/Tachycardia

ComplicationsofAcuteMI/Tachycardia Valve or Septal Rupture Both present with new onset murmur and pulmonary congestion Mitral regurgitation best heard at apex with radiation to axilla Ventricular septal rupture best heard at lower left sternal border Look for a step-up in oxygen saturation as you go from the right atrium to the right ventricle to hand you the diagnosis of septal rupture
MTBS2CK p.72

Ventricular Tachycardia/Ventricular Fibrillation Both can cause sudden death No way to distinguish without EKG if no pulse Cardiovert/ Defibrillate
These complications are the reason patients with acute MI are monitored in an ICU for the first several days after the infarction
MTBS2CK p.72

18

ComplicationsofAcuteMI/Tachycardia

IntraaorticBalloonPump

Most accurate test: Echocardiogram for both You cant always depend on buzzwords like step-up for oxygenation Often, only numbers are presented: 42% oxygen saturation found on blood from right atrium and 85% saturation found in right ventricular sample

Intraaortic balloon pump (IABP): Answer for pump failure from anatomic problem that can be fixed in operating room IABP contracts & relaxes in sync with natural heartbeat Helps give a push forward to blood IABP is never a permanent device (bridge to surgery)
MTBS2CK p.72

MTBS2CK p.72

ExtensionoftheInfarction/Reinfarction

Aneurysm/MuralThrombus

Second event infarct Look for: Recurrence of pain New rales Bump up in CK-MBs Sudden onset pulmonary edema Actions: Repeat EKG Re-treat with angioplasty or thrombolytics
Continue: aspirin, metoprolol, nitrates, ACE, statins
MTBS2CK p.7273

Detected with echocardiography Most aneurysms dont need specific therapy Mural thrombi are treated with heparin followed by warfarin

MTBS2CK p.73

WhatistheMostLikelyDiagnosis? Diagnosis Keyfeature

WhatistheMostLikelyDiagnosis? Diagnosis Keyfeature

ThirddegreeAV Block Sinusbradycardia Tamponade/wall Rupture RVinfarction

Bradycardia,cannona Waves Nocannonawaves Suddenlossofpulse,JVD

Valverupture

Newmurmur, rales/congestion Newmurmur,increasein oxygensaturationon enteringtherightventricle Lossofpulse,needEKGto answerquestion

Septal rupture

IWMIinhistory,clearlungs, hypotension

Ventricular fibrillation
MTBS2CK p.73

MTBS2CK p.73

19

PreparationforDischargefromHospital

PreparationforDischargefromHospital

Detection of Persistent Ischemia Stress test prior to discharge Stress test determines if angiography needed Angiography A i h d determines t i need df for revascularization (angioplasty or bypass surgery)

Everyone should go home on: Aspirin Clopidogrel Beta blockers (metoprolol) Statins ACE inhibitors
Best for anterior wall infarctions because of high likelihood of developing systolic dysfunction
MTBS2CK p.74

Dipyridamole is never the right choice for coronary artery disease

MTBS2CK p.73

PreparationforDischargefromHospital

SexualIssuesPostinfarction

Clopidogrel: All MIs, or intolerant of aspirin, or post-stenting ARBs: those with cough to ACE inhibitor Ticlopidine: for rare person intolerant of both aspirin and clopidogrel
Prophylactic antiarrhythmic medications: Do not use amiodarone, flecainide, or any rhythmcontrolling medication to prevent the development of ventricular tachycardia or fibrillation. Do not be fooled by the question describing frequent PVCs and ectopy. Prophylactic antiarrhythmics increase mortality.
MTBS2CK p.74

The most commonly tested facts are: Dont combine nitrates with sildenafil; hypotension can result because theyre both vasodilators Erectile dysfunction postinfarction is most commonly from anxiety Most common medication is beta blocker
MTBS2CK p.74

SexualIssuesPostinfarction

Wait after MI for sexual activity


2-6 weeks

If post-MI stress test is normal, any form of exercise program can be started including sex

CongestiveHeartFailure
Definition Presentation Diagnosis Treatment

MTBS2CK p.74

20

CongestiveHeartFailure(CHF)/Definition

CHF/CausesofSystolicDysfunction

Shortness of breath (dyspnea) - essential feature of congestive heart failure (CHF) Dysfunction of heart as a pump of blood Insufficient oxygen delivery and fluid in lungs Either from: S t li dysfunction: Systolic d f ti Low Ejection Fraction (EF) and dilation of heart Diastolic dysfunction: EF is preserved Heart cant relax and receive blood
MTBS2CK p.7475

Most Common: Hypertension resulting in cardiomyopathy Initially theres preservation of EF Over time time, the heart dilates resulting in systolic dysfunction and low EF Valvular heart disease of all types results in CHF
MTBS2CK p.75

CHF/CausesofSystolicDysfunction Myocardial infarction (MI) is a very common cause of dilated cardiomyopathy and decreased EF Dead or infarcted heart wont pump CHF most common cause of hospital admission in USA MI death rate down from:
Thrombolytics Beta blockers Angioplasty Aspirin, clopidogrel

CHF/CausesofSystolicDysfunction

Infarction Dilation Regurgitation CHF


MTBS2CK p.75

Many are normal, are living with CHF


MTBS2CK p.75

CHF/CausesofSystolicDysfunction Less common causes are: Alcohol Postviral (idiopathic) myocarditis Radiation Adriamycin (doxorubicin) use Chagas Ch di disease and d other th i infections f ti Hemochromatosis (also causes restrictive cardiomyopathy) Thyroid disease Peripartum cardiomyopathy Thiamine deficiency
MTBS2CK p.75

CHF/Presentation In addition to dyspnea on exertion look for: Orthopnea (worse when lying flat, relieved when sitting up or standing) Peripheral edema Rales on lung exam Jugulovenous distention (JVD) Paroxysmal nocturnal dyspnea (PND)(sudden worsening at night, during sleep) S3 gallop rhythm (Be prepared to identify the sound on Step 2. It may be played.)
MTBS2CK p.7677

21

HeartSoundsTimingintheCardiacCycle

CHF/Presentation

The most frequently asked USMLE Step 2 CK question is: What is the most likely diagnosis?

MTBS2CK p.76

MTBS2CK p.76

WhatistheMostLikelyDiagnosis? forDyspnea
Keyfeature Mostlikelydiagnosisis

WhatistheMostLikelyDiagnosis? forDyspnea
Keyfeature Mostlikelydiagnosisis

Suddenonset,clearlungs Suddenonset,wheezing, increasedexpiratoryphase Slower,fever,sputum,unilateral rales/rhonchi Decreasedbreathsounds unilaterally,trachealdeviation


MTBS2CK p.77

Pulmonaryembolus Asthma

Circumoral numbness,caffeine use,historyofanxiety Pallor,gradualoverdaysto weeks k

Panicattack

Anemia

Pneumonia Pulsus paradoxus,decreased heartsounds,JVD Pneumothorax Palpitations,syncope


MTBS2CK p.77

Tamponade

Arrhythmia (ofalmostanykind)

WhatistheMostLikelyDiagnosis? forDyspnea
Keyfeature Dullnesstopercussionatbase Longsmokinghistory,barrelchest ,brownblood Recentanestheticuse, notimprovedwithoxygen,clear lungsonauscultation,cyanosis Burningbuildingorcar,wood burningstoveinwinter,suicide attempt
MTBS2CK p.77

WhatistheMostLikelyDiagnosis? forDyspnea

MostLikelydiagnosisis Pleuraleffusion COPD g Methemoglobinemia

All of these will lack: Orthopnea/PND S3 gallop

Carbonmonoxidepoisoning

MTBS2CK p.77

22

CHF/DiagnosticTests

CHF/EjectionFraction What is the best initial test? Transthoracic echo

Echocardiography Most important test of CHF There is no OTHER way to distinguish Systolic vs. Diastolic dysfunction Will NOT distinguish by: History Physical Tests (e.g., EKG, chest X-ray, or BNP)
MTBS2CK p.77

Whats the most accurate test? Multiple Multiple-gated gated acquisition scan (MUGA) or nuclear ventriculography Transesophageal echocardiography (TEE): More accurate for valves
MTBS2CK p.78

CHF/DiagnosticTests

CHF/DiagnosticTests

When should you answer nuclear ventriculography? Rarely needed Person receiving chemotherapy with doxorubicin Trying to give max dose to cure lymphoma But not cause cardiomyopathy
MTBS2CK p.78

When should you answer BNP? Acute SOB Etiology unclear You cant wait for echo Normal BNP excludes CHF

MTBS2CK p.78

TestsUsedtoDetermineEtiologyofCHF

TestsUsedtoDetermineEtiologyofCHF Test
T4/TSH

Test
EKG ChestXray Holter monitor Cardiac catheterization CBC
MTBS2CK p.78

Etiology ofCHF
MI,heartblock Dilatedcardiomyopathy

Etiology ofCHF
Bothhighandlowthyroidlevels causeCHF Excludessarcoid,amyloid Rarelyneeded Canbiopsyothersites

Myocardialbiopsy

Paroxysmalarrhythmias Precisevalvediameters, septal defects Anemia


SwanGanz rightheart catheterization

DistinguishesCHFfromARDS;not routine

MTBS2CK p.78

23

CHF/Treatment

CHF/Treatment

Systolic Dysfunction (Low EF) ACE inhibitors or angiotensin receptor blockers (ARBs) Beta blockers Spironolactone, S i l t E l Eplerenone Diuretics Digoxin

ACE/ARBs All patients with systolic dysfunction All stages of disease Beneficial effects: any drug in class When are ARBs Next best step? ACE induced cough

MTBS2CK p.79

MTBS2CK p.79

CHF/Treatment

CHF/Treatment

Beta Blockers Not clearly any drug in class Evidence only for:
Metoprolol Bisoprolol Carvedilol

Beta Blockers are: Anti-ischemic Decrease heart rate Decrease O2 consumption Antiarrhythmic

MTBS2CK p.79

MTBS2CK p.79

CHF/Treatment Which of the following is the MCC of death from CHF? a. b. c c. d. e. Pulmonary edema We can fix this almost all the time Myocardial infarction CHF doesnt cause MI Arrhythmia/sudden death Emboli Clots rare in CHF unless AFib Myocardial rupture This from MI, not CHF

Spironolactone Benefit from inhibition of aldosterone Only proven for more advanced CHF (class III and IV) with dyspnea on minimal exertion or at rest What is the most common adverse effect? Hyperkalemia Gynecomastia
MTBS2CK p.7980

MTBS2CK p.79

24

CHF/Treatment

CHF/Treatment

When is eplerenone the answer? When spironolactone leads to antiandrogenic effects (e.g., gynecomastia)

Diuretics ED: Acute pulmonary edema Office: Combination with ACEi or ARB Furosemide, torsemide, or bumetanide equal Spironolactone, although a diuretic, is not used at doses where it has a diuretic effect Diuretics control symptoms of CHF. They do not lower mortality.

MTBS2CK p.80

MTBS2CK p.80

CHF/Treatment

CHF/Treatment Devices with mortality benefit

Digoxin Digoxin does NOT lower mortality in CHF This is often the single most important question concerning CHF on USMLE Digoxin will: Control symptoms Decrease frequency of hospitalizations No positive inotropic agent (digoxin, milrinone, amrinone, dobutamine) has been proven to lower mortality
MTBS2CK p.80

1. Implantable defibrillator: Ischemic CM & EF < 35%. Remember: Arrhythmia and sudden death are MCC of death in CHF 2. Biventricular pacemaker: EF < 35% & wide QRS > 120 ms with persistent symptoms

MTBS2CK p.8081

CHF/Treatment

CHF/Treatment

Biventricular pacemaker is NOT a dualchamber pacemaker Dual-chamber pacer has wire in right ATRIUM and right VENTRICLE Biventricular pacemaker resynchronizes the heart when theres a conduction defect Defers/delays need for cardiac transplantation Symptoms markedly improved
MTBS2CK p.81

Transplantation Symptoms despite maximal medical therapy (ACE, BB, spironolactone, diuretics, digoxin) and possibly biventricular pacemaker Warfarin Always wrong in absence of clot in heart
MTBS2CK p.81

25

CHF/Treatment

CHF/Treatment

Mortality Benefit in Systolic Dysfunction ACEi/ARBs Beta blockers Spironolactone Hydralazine/nitrates Implantable defibrillator Calcium-channel blockers (CCBs) can actually raise mortality
MTBS2CK p.81

Diastolic Dysfunction (CHF with preserved EF) Less clear Beta blockers have clear benefit No mortality benefit in diastolic dysfunction Digoxin clearly has no benefit

MTBS2CK p.81

CHF/Treatment

Diuretics control symptoms of fluid overload Do not confuse diastolic dysfunction from hypertrophic CM with hypertrophic obstructive cardiomyopathy (HOCM) Diuretics are contraindicated in HOCM because they increase obstruction

AcutePulmonaryEdema
Definition Presentation Diagnosis Treatment

MTBS2CK p.81

AcutePulmonaryEdema

AcutePulmonaryEdema/Presentation How do I answer What is the most likely diagnosis? Acute shortness of breath with: Rales JVD S3 gallop Edema Ed Orthopnea MAY be: Ascites & enlargement of liver/spleen from chronic passive congestion of right side of heart
MTBS2CK p.82

Definition Worst (most severe) form of CHF Rapid fluid accumulation in lungs

MTBS2CK p.82

26

S3GallopRhythm

JVD

PittingEdema

WetCrackles

Whatisthebestinitialtest? Brain Natriuretic Peptide (BNP) is the answer if: Diagnosis not clear Normal BNP excludes CHF Chest X-ray shows: Vascular congestion Filling Filli of f bl blood d vessels l t toward dh head d( (cephalization h li ti of flow) Flow mostly at base because of gravity Enlargement of heart Effusion
MTBS2CK p.82 82

PulmonaryEdema

Pulmonary edema with cephalization of flow and engorged pulmonary veins


Source: Saba Ansari, MD.

MTBS2CK p.83

27

AcutePulmonaryEdema/DiagnosticTests

Oximetry/Arterial Blood Gas (ABG) Hypoxia expected Respiratory alkalosis from hyperventilation CO2 leaves l more easily il than th oxygen enters

AcutePulmonaryEdema/ WhichTestismostlikelytoalteracutemanagement?

Answer!!! EKG A-fib, Atrial flutter, or V-tach What to do first? Synchronized cardioversion Restore atrial systole = Return atrial contribution to cardiac output
MTBS2CK p.83

MTBS2CK p.83

AcutePulmonaryEdema/DiagnosticTests

AcutePulmonaryEdema/DiagnosticTests

EKG Diseased hearts need atrial systole more than normal hearts Up from 10% to 30 - 40% of cardiac output Diseased hearts means:
Dil Dilated t d cardiomyopathy di th Valvular heart disease

If acute pulmonary edema is from arrhythmia fix it fast with..


Cardioversion!!!!!
MTBS2CK p.83

Echocardiography MUST be done on all patients ONLY WAY to determine systolic or diastolic dysfunction No difference in initial therapy Huge difference in chronic therapy

MTBS2CK p.83

AcutePulmonaryEdema/Treatment 74-woman comes to ED with the acute onset of SOB, RR of 38/minute, rales to apices, S3 gallop, and JVD. What is the best initial step in management?

Will be done, but first you have to treat a. Oximeter Should be done, but not urgently b. Echocardiography c. Intravenous furosemide d. Ramipril Although they are used in CHF, they are not e. Metoprolol Part of the treatment of acute setting f. Nesiritide No mortality benefit. Not better than nitrates

Preload Reduction Initial therapy: Oxygen Loop diuretics (e.g., furosemide or b bumetinide) ti id ) Morphine Nitrates

MTBS2CK p.84

MTBS2CK p.84

28

AcutePulmonaryEdema/Treatment

AcutePulmonaryEdema/Treatment

The majority in acute pulmonary edema can be managed with preload reduction Removing 1 to 2 liters of fluid is best Nesiritide does NOT work better than other agents

What do you do if the questions say: Preload reduction hasnt been effective? Positive Inotropic Agents Dobutamine in ICU Amrinone and milrinone Phosphodiesterase inhibitors that perform the same role
Increase contractility

Decrease afterload

MTBS2CK p.84

MTBS2CK p.84

AcutePulmonaryEdema/Treatment

AcutePulmonaryEdema/Treatment

Positive Inotropic Agents Digoxin too slow! Increases contractility, but Needs several weeks to work No benefit of digoxin in acute setting

Afterload Reduction ACEi and ARBs: Used on discharge Long-term use with systolic dysfunction (low EF) Nitroprusside in ICU Hydralazine alternate for ACE/ARB
Heparin is always wrong for acute pulmonary edema, use for clots

MTBS2CK p.85

MTBS2CK p.85

ValvularHeartDisease/Definition

ValvularHeartDisease
Definition Presentation Diagnosis Treatment

All can be congenital in nature Rheumatic fever can lead to any form and mitral stenosis is most common Aging = Aortic stenosis Regurgitant R it t via i HTN & i ischemia h i Infarction regurgitation Regurgitation dilation

MTBS2CK p.85

29

ValvularHeartDisease/Presentation

ValvularHeartDisease/DiagnosticTests

All forms associated with SOB and CHF Only murmurs are specific for presentation Right-sided heart lesions (tricuspid and pulmonic valve) increase with inhalation Inhalation venous return to right side Left-sided lesions (mitral and aortic valve) with exhalation Exhalation squeezes blood out of lungs into left side

Best initial test: Echocardiogram Transesophageal: more sensitive & specific than transthoracic echo Most accurate test: Catheterization C th t i ti : Catheterization Most precise valvular diameter Most exact pressure gradient across valve
MTBS2CK p.85

MTBS2CK p.85

ValvularHeartDisease/DiagnosticTests

ValvularHeartDisease/Treatment

EKG: Not specific with valvular heart disease Shows hypertrophy of chambers Cannot confirm diagnosis from EKG Chest X X-ray: ray: Shows enlargement chambers Precise anatomic correlation poor X-ray is neither the most accurate test nor the best initial test
MTBS2CK p.85

All forms associated with fluid overload All benefit from diuretics Meds alone cant improve stenotic lesions Mitral Mit l stenosis: t i Dilated Dil t d with ith balloon b ll Aortic stenosis: Surgical replacement

MTBS2CK p.86

ValvularHeartDisease/Treatment

ValvularHeartDisease/Treatment

Regurgitant lesions Respond best to vasodilators ACEi/ARBs, nifedipine, or hydralazine Surgical replacement must be done before heart dilates too much If heart dilates excessively valve replacement will not be able to correct decrease in systolic function If myocardium stretches too much it wont return to normal size and shape
MTBS2CK p.86

Assess ventricular size based on: End-systolic diameter Ejection fraction Expansion of end-systolic diameter (must replace p the valve) )

Endocarditis prophylaxis is not indicated for any of these valve lesions.

MTBS2CK p.86

30

MitralStenosis (MS)

MitralStenosis

Definition/Etiology Most often from rheumatic fever Extremely uncommon in US Low incidence of acute rheumatic fever Treatment if symptomatic Do not treat asymptomatic MS

Look for: Pregnancy and Immigrant Pregnancy: 50% increase in plasma volume Contraction of uterus squeezes squeezes 500 mL extra into central circulation Most with MS are immigrants to the US Come from places where acute rheumatic fever is still common
MTBS2CK p.86

MTBS2CK p.86

MitralStenosis

MitralStenosis/Presentation

Presentation SOB and CHF AND! Unique features of presentation:


Dysphagia: Left atrium (LA) presses on esophagus Hoarseness: LA presses on laryngeal nerve Atrial fibrillation & stroke from enormous LA MS: Look for Hemoptysis

Enlarged left atrium in mitral stenosis compresses the esophagus causing dysphagia

Source: Andrew Peredo

young adult patients

MTBS2CK p.8687

MTBS2CK p.87

MitralStenosis

MitralStenosis headphonesrequired

Physical findings Diastolic murmur after opening snap Squatting & leg raising increase it! Increased venous return increases murmur

MTBS2CK p.87

MTBS2CK p.87

31

MitralStenosis

Diagnostic tests Echo TTE: Best initial test TEE more accurate than TTE Catheterization: Most accurate test EKG Atrial rhythm disturbance, particularly atrial fibrillation, very common LA hypertrophy: Biphasic P wave: V1 and V2
MTBS2CK p.87

MitralStenosis/DiagnosticTests Diagnostic tests Chest X-ray: Left Atrial Hypertrophy Straightening of left heart border Elevation of left main mainstem bronchus Second bubble behind heart

MitralStenosis/Treatment

Diuretics & sodium restriction: When fluid overload present Balloon valvuloplasty With a catheter Valve replacement Only O l when h a catheter h procedure d cannot b be d done or fails Warfarin A-fib to INR 2 to 3 Rate control: Digoxin, beta blockers, or diltiazem/ verapamil
MTBS2CK p.88

Source : Andrew Peredo

MTBS2CK p.87 88

AorticStenosis

AorticStenosis

Definition/Etiology Congenital bicuspid valve Increasing calcification with age Presentation Angina (most common) Syncope CHF:
Poorest prognosis 2-year average survival

Murmur Systolic, crescendo-decrescendo Peaks in diamond-shape mid-systole Heard best at 2nd right intercostal space Radiates to carotids

Source: Shwan Christian

MTBS2CK p.88

MTBS2CK p.88

32

AorticStenosis

AorticStenosis headphonesrequired

Murmur Valsalva & standing Decrease intensity of murmur Less venous return = Less Murmur Handgrip Softens murmur Less blood ejected = Less murmur

MTBS2CK p.88

MTBS2CK p.88

AorticStenosis/DiagnosticTests TTE, then TEE, then catheterization Chest X-ray


Left ventricular hypertrophy

AorticStenosis/Treatment

EKG LV hypertrophy (LVH)


Source: Nihar Shah, MD.

S V1 + R V5 > 35 mm

MTBS2CK p.89

Cardiac enlargement is defined as a heart greater in diameter than 50% of the total transthoracic diameter

Replacement: Only truly effective therapy for AS Diuretics CHF but dont tolerate volume depletion well Balloon valvuloplasty p y Not routine for AS AS calcification doesnt improve well with balloon valvuloplasty Only if surgery isnt an option Unstable/fragile patients
MTBS2CK p.89

MitralRegurgitation

MitralRegurgitation

Definition/Etiology MR is abnormal backward flow of blood through mitral valve that doesnt fit together Hypertension Endocarditis Myocardial infarction Papillary muscle rupture Any heart dilation leads to MR
MTBS2CK p.89

Presentation Signs/Symptoms of CHF Unique finding: Pansystolic (holosystolic) murmur Obscures S1 and S2 Radiates to axilla Handgrip worsens murmur of MR Handgrip increases afterload Pushes blood backwards Handgrip worsens AR and MR
MTBS2CK p.90

33

MitralRegurgitation/Presentation

MitralRegurgitation headphonesrequired

Squatting & leg raising worsen MR Increase venous return to heart All left-sided murmurs except mitral valve prolapse (MVP) and hypertrophic obstructive cardiomyopathy will increase with expiration MR diagnosed with echo

MTBS2CK p.90

MTBS2CK p.90

MitralRegurgitation

MitralRegurgitation/Treatment

Treatment Vasodilators: ACE or ARBs are best Decrease rate of progression Digoxin & diuretics for symptomatic CHF

Valve replacement: Indicated when heart dilates Dont wait for left ventricular end systolic diameter (LVESD) to become large Replace R l when: h LVESD > 40 mm or EF < 60% Valve repair:
Placing a clip or sutures across valve to tighten

MTBS2CK p.90

MTBS2CK p.90

AorticRegurgitation

AorticRegurgitation

Definition/Etiology AR caused by: Anything that makes the heart or aorta dilate: MI HTN Endocarditis Marfan syndrome or cystic medial necrosis Inflammatory disorders (e.g., Ankylosing spondylitis, Reiter syndrome) Syphilis
MTBS2CK p.9091

Presentation Besides CHF unique physical findings are: Wide pulse pressure Water-hammer (wide, bounding) pulse Quincke pulse (pulsations in nail bed) Hill sign (BP in legs as much as 40 mmHg above arm BP) Head bobbing (de Musset sign)
MTBS2CK p.91

34

AorticRegurgitation

AorticRegurgitation headphonesrequired

Murmur Diastolic, decrescendo murmur Heard best: Lower left sternal border Valsalva & Standing: Softer Handgrip (increases afterload): Worse

MTBS2CK p.91

MTBS2CK p.91

AorticRegurgitation

AorticRegurgitation

Diagnostic tests EKG & Chest X-ray: LVH

Treatment ACEi/ARBs or nifedipine: Vasodilators Increase forward flow of blood Delay progression Digoxin & Diuretics: Little benefit Surgical valve replacement: Acute valve rupture (MI) Replace valve before LV dilates excessively EF < 55% LVESD > 55 mm
MTBS2CK p.91

MTBS2CK p.91

MitralValveProlapse(MVP)

MitralValveProlapse

Common Considered normal anatomic variant 2% to 5% of population Particularly in women Marfan or Ehlers-Danlos syndrome

Presentation Most often asymptomatic When symptoms do occur: Symptoms of CHF usually absent Most common is:
Atypical chest pain Palpitations Panic attack

MTBS2CK p.91

MTBS2CK p.91

35

MitralValveProlapse

MitralValveProlapse headphonesrequired

Murmur Presents with: Midsystolic click When severe associated with murmur Mitral regurgitation Valsalva & Standing worsen MVP Squatting & Handgrip improve (diminish) MVP
MTBS2CK p.92 MTBS2CK p.92

MitralValveProlapse Redundant mitral valve leaflet doesnt seal allowing regurgitation

MitralValveProlapse

Diagnostic tests Echocardiography: Best choice Catheterization: Rarely, if ever, done Valve replacement: Rarely needed
Source: Andrew Peredo

MTBS2CK p.92

MTBS2CK p.92

MitralValveProlapse

Treatment Beta blockers: When symptomatic Valve repair With catheter Place clip to tighten valve Stitches valve to tighten leaflets Surgical repair rarely necessary Endocarditis prophylaxis not recommended
MTBS2CK p.92

Cardiomyopathy & PericardialDisease


DilatedCardiomyopathy Hypertrophic h Cardiomyopathy d h RestrictiveCardiomyopathy AcutePericarditis PericardialTamponade ConstrictivePericarditis

36

Cardiomyopathy/Definition

Cardiomyopathy/Etiology

Abnormal function of heart muscle Frequent valve or auscultory abnormalities But! It originates i i t from f an abnormally b ll contracting or relaxing myocardium

Can be dilated, hypertrophic, or restrictive The terms dilated cardiomyopathy, systolic dysfunction, and low EF are often used interchangeably Hypertrophic cardiomyopathy interchangeable with diastolic dysfunction Or!! Cardiac failure with preserved EF

MTBS2CK p.92

MTBS2CK p.93

Cardiomyopathy/Presentation

Cardiomyopathy/DiagnosticTests

All forms give: SOB, particularly worsened by exertion Edema Rales JVD

Echocardiography: Best initial test Often Most accurate test used EKG & Chest X-ray: Should be performed Nothing specific on them confirm the diagnosis Murmurs not increasing with expiration: HOCM MVP
MTBS2CK p.93

MTBS2CK p.93

Cardiomyopathy/Treatment

DilatedCardiomyopathy

All treated with diuretics Other treatments based on type of cardiomyopathy The only real functional difference in management and answers to questions is: TREATMENT

In addition to previous MI and ischemia, dilated cardiomyopathy can be from:


Alcohol Postviral myocarditis Radiation Toxins (e.g., doxorubicin) Chagas disease

MTBS2CK p.93

MTBS2CK p.93

37

DilatedCardiomyopathy

DilatedCardiomyopathy/Treatment

Dyspnea, gallop, edema, testing same as section on CHF

Multiple meds mortality ACEi (or ARBs) Beta blockers (metoprolol, carvedilol) Spironolactone (or eplerenone) Symptom Control ONLY: Diuretics & Digoxin

MTBS2CK p.93

MTBS2CK p.93

DilatedCardiomyopathy/Treatment

HypertrophicCardiomyopathy

QRS wide (> 120 ms) Biventricular pacemaker Improve both symptoms and survival Automated implantable cardioverter/defibrillator di t /d fib ill t h has mortality t lit benefit

HTN - MCC MUST distinguish between hypertrophic cardiomyopathy (HCM) and HOCM HCM: Reaction to BP Heart hypertrophies yp p to carry y load Develops difficulty relaxing in diastole Cant relax = Cant receive blood Patient becomes short of breath

MTBS2CK p.9394

MTBS2CK p.94

HypertrophicObstructiveCardiomyopathy

HypertrophicCardiomyopathy

MTBS2CK p.9495

Source: Andrew Peredo

Genetic disorder Abnormal shape of septum Asymmetrically hypertrophied septum and valve leaflet blocks blood leaving the heart

S4 gallop Fewer signs of right heart failure Less ascites Less enlargement of liver and spleen

MTBS2CK p.94

38

HOCM/Presentation
Dyspnea Chest pain Syncope & lightheadedness Sudden death, particularly in healthy athletes Worsened by heart rate
(e.g., exercise, dehydration, and diuretics)

HOCM/DiagnosticTests

Echo (best initial test) Septum 1.5x thickness of posterior wall


Systolic anterior motion (SAM) of mitral valve is classic for HOCM. It contributes t ib t t to obstruction. b t ti Catheterization is most accurate test to determine precise gradients of pressure across the chamber. Septal Q waves in the inferior and lateral leads are common in HOCM.

Worsened by in left ventricular chamber size


(e.g., ACEi, ARBs, digoxin, hydralazine, valsalva, and standing suddenly)
MTBS2CK p.94

EKG: Nonspecific ST and T wave changes are common. LVH is common. EKG can be normal in a quarter
MTBS2CK p.95

HypertrophicCardiomyopathy/Treatment

HOCM/SpecificTherapy Implantable defibrillator: HOCM with syncope Ablation of septum: Catheter placing absolute alcohol in muscle Causes small infarctions If symptoms t persist: i t Surgical S i l myomectomy t Surgical myomectomy is the therapy only if all medical and catheter procedures fail.
MTBS2CK p.95

Beta blockers: Best initial therapy both HOCM & HCM Diuretics help HCM Diuretics harm HOCM
Digoxin and spironolactone are definitely always wrong in hypertrophic cardiomyopathy.
MTBS2CK p.95

In HOCM, ACEi and diuretics definitely dont help. This is the major difference between HOCM and HCM

RestrictiveCardiomyopathy Hypertrophic versusDilatedCardiomyopathy

Hypertrophic
BetaBlockers Diuretics ACEi/ARB Spironolactone Digoxin
MTBS2CK p.95

Dilated
Yes Yes Yes Yes Yes

Yes Yes Unclearbenefit No No

Combines worst aspects of dilated and hypertrophic cardiomyopathy Heart neither contracts nor relaxes Infiltrated with substances creating immobility

MTBS2CK p.96

39

RestrictiveCardiomyopathy

RestrictiveCardiomyopathy

Causes are: Sarcoidosis Amyloid Hemochromatosis Endomyocardial fibrosis Scleroderma

Presentation Dyspnea: Most common Right heart failure Ascites, edema, JVD Enlargement g of liver & spleen p Pulmonary hypertension: Common Kussmaul sign: jugulovenous pressure on inhalation

MTBS2CK p.96

MTBS2CK p.96

RestrictiveCardiomyopathy

RestrictiveCardiomyopathy

Diagnosis Echocardiography: Initial test EF normal or elevated EKG: Low voltage Amyloid: Speckling of septum on echo or cardiac MRI Most accurate test: Endomyocardial biopsy Rarely done Diagnosis made from biopsies elsewhere
MTBS2CK p.96

Treat underlying cause Diuretics relieve some pulmonary HTN and signs of right heart failure No other clear therapy

MTBS2CK p.96

MurmursandtheEffectsofManeuvers

MurmursandtheEffectsofManeuvers Standing/Valsalva Decreasesboth

Lesion

Squatting/Legraising

Mitralandaortic Increasesboth stenosis Mitralandaortic Increasesboth regurgitation Mitralvalve prolapse HOCM


MTBS2CK p.96

Decreasesboth

More blood increases all murmurs except MVP and HOCM.

Decrease

Increase

Decrease

Increase
MTBS2CK p.96

Less blood decreases all murmurs except MVP and HOCM.

40

EffectsofManeuvers

EffectsofManeuvers

Standing from a squatting position: Opens vessels of legs Decreases venous return to heart Valsalva: Exhalation against a closed glottis Increases intrathoracic pressure Decreases venous return to heart
MTBS2CK p.97

Standing or Valsalva = Diuretic use Stenotic & regurgitant murmurs all treated with diuretics and salt restriction Standing St di & Valsalva V l l will ill i improve th them

MTBS2CK p.97

EffectsofManeuvers

EffectsofManeuvers

MVP & HOCM: Worsen with diuretics Diuretics decrease LV size Diuretics worsen MVP and obstruction of HOCM Standing St di and d valsalva l l worsens th them

Handgrip Handgrip increases afterload Contraction of arm muscles compresses arteries Decreases emptying of heart Opposite of ACE inhibitor Worsens AR and MR

MTBS2CK p.97

MTBS2CK p.97

EffectsofManeuvers Amyl nitrate: Direct arteriolar vasodilator Decreases afterload Simulates ACE inhibitors or ARBs on heart Valvular disease treated with ACEi/ARB will improve with amyl nitrate Improves AR and MR Handgrip = Fuller left ventricle Amyl nitrate = ACEi = Emptier left ventricle
MTBS2CK p.97

EffectofManeuverson Intensity(loudness)ofMurmurs Lesion Aorticstenosis Mitralstenosis A i regurgitation Aortic i i Mitralregurgitation Mitralvalveprolapse HOCM
MTBS2CK p.97

Handgrip Decreases Noeffect I Increases Increases Decreases Decreases

Amylnitrate Increases Noeffect D Decreases Decreases Increases Increases

41

EffectofManeuverson Intensity(loudness)ofMurmurs

PericardialDisease

Handgrip and amyl nitrate have no meaningful effect on mitral stenosis in the same way ACEi stenosis, has no meaningful effect on MS

Causes of pericarditis, pericardial tamponade & constrictive pericarditis overlap If the etiology of pericarditis extravasates fluid, then tamponade can occur If the cause of pericarditis is chronic, fibrosis and calcification of pericardium lead to constrictive pericarditis

MTBS2CK p.97

MTBS2CK p.98

Pericarditis/Etiology

Pericarditis/Etiology

Any Infection Inflammatory disorder Connective tissue disorder Chest trauma or cancer near the heart can cause pericarditis Most common infection is viral
MTBS2CK p.98

Systemic lupus erythematosus: Most common connective tissue disorder But Wegener granulomatosis Goodpasture syndrome Rheumatoid arthritis Polyarteritis nodosa and others can cause pericarditis
MTBS2CK p.98

Pericarditis/Presentation

Pericarditis/DiagnosticTests EKG shows ST segment elevation in all leads, but the most specific finding is PR segment depression

What Is the Most Likely Diagnosis? Sharp chest pain Changes with respiration Changes with position of body Worsened by lying flat Improved by sitting up

MTBS2CK p.98

MTBS2CK p.9899

42

Pericarditis/Treatment

PericardialTamponade/Etiology

Treat cause Majority: Idiopathic


Idiopathic presumed viral Coxsackie B virus

T Treated t d with ith NSAIDs NSAID (e.g., ( ibuprofen, ib f naproxen) Colchisine - recurrences

Any cause of pericarditis can extravasate enough fluid to cause tamponade Compression heart starts on right side Walls are thinner 50 mL acutely y cause tamponade p Over weeks to months, pericardium stretches for 2 L of fluid Can also be from trauma Bleed requires emergent thoracotomy
MTBS2CK p.99

MTBS2CK p.99

PericardialTamponade/Presentation

PericardialTamponade

What Is the Most Likely Diagnosis? Hypotension Tachycardia Distended neck veins Clear lungs
Which of the following physical findings is most likely to be associated with this patient? Pulsus paradoxus Decrease of more than 10 mmHg in systolic blood pressure on inhalation
MTBS2CK p.99 MTBS2CK p.100

PericardialTamponade

EKG: Electrical alternans (different heights of QRS complexes between beats)

PericardialTamponade

Chest X-ray: Enlarged cardiac shadow expanding in both directions (globular heart) Echocardiogram: Right atrial and ventricular diastolic collapse Right heart catheterization: Equalization of pressures in diastole

MTBS2CK p.100

MTBS2CK p.100101

43

PericardialTamponade
78-year-old man with lung cancer experiences several days of increasing SOB. Hes lightheaded today. BP 106/70; pulse 112; JVD present; lungs: clear, BP drops to 92/58 on inhalation. Which is most appropriate to confirm the diagnosis?

a. b b. c. d. e.

EKG Low voltage non-specific. Rare alternans y; globular g heart later Chest X-ray X ray Normal acutely; Echocardiogram Most accurate; Right heart catheterization should do echo first Cardiac MRI Not clearly right for

Treatment Pericardiocentesis Needle rapidly reexpands the heart IV fluids A hole or window recurrent cases
Diuretics will decrease intracardiac filling pressure and may markedly worsen collapse of right side of heart

ANYTHING at this time

MTBS2CK p.100

MTBS2CK p.101

ConstrictiveDisease

ConstrictiveDisease

Any cause of pericarditis with calcification and fibrosis Prevents filling of right side of heart Tuberculosis

What Is the Most Likely Diagnosis? Signs of right heart failure such as:
Edema Ascites Enlargement of liver and spleen JVD

Constrictive pericarditis is a combination of physical findings described above with calcification on chest X-ray
MTBS2CK p.101 MTBS2CK p.101

ConstrictiveDisease

PericardialKnock headphonesrequired

Which physical findings is most likely associated with this patient? Kussmaul sign: Increase in JVD on inhalation Normally neck veins go down on inhalation Knock: Extra heart sound in diastole From ventricular filling Heart fills to its maximum, it hits the stiff, rigid pericardium with a knock
MTBS2CK p.101

44

ConstrictiveDisease

ConstrictiveDisease

Best initial test Chest X-ray: Calcification & fibrosis

CT scan or MRI more accurate, not 1st Echocardiogram: Excludes RV hypertrophy or cardiomyopathy as cause Myocardium moves normally with constrictive pericarditis

MTBS2CK p.101

Commons.wikimedia.org. Used with permission

MTBS2CK p.101

ConstrictiveDisease

Treatment Diuretics: Decompress filling of heart Relieves edema and organomegaly Surgical removal of pericardium

PeripheralArteryDisease,Aortic Disease,&HeartDisease inPregnancy


Definition Etiology Presentation Diagnosis Treatment

MTBS2CK p.101

PeripheralArteryDisease(PAD)/Etiology

PeripheralArteryDisease/Presentation What Is the Most Likely Diagnosis? Pain in calves on exertion Relieved by rest Walking up or down hills Severe disease causes loss of:
Hair H i f follicles lli l Sweat glands Sebaceous glands

Stenosis of peripheral arteries from: Diabetes mellitus Hyperlipidemia Hypertension Tobacco smoking

Same causes as Coronary Disease!!

The skin becomes smooth and shiny


Spinal stenosis pain is worse when walking down hills because of leaning back
MTBS2CK p.102 MTBS2CK p.102

45

PeripheralArteryDisease Diagnostic tests Best initial test = Ankle-brachial index (ABI) ABI: Ratio of BP in ankles to brachial arteries Normally equal or slightly greater in ankles because of gravity If difference between them is > 10% (ABI < 0 0.9), 9) disease is present Most accurate test = angiogram Angiogram not necessary unless revascularization will be done
There is no routine screening for PAD since there is no mortality benefit to be obtained
MTBS2CK p.102

PeripheralArteryDisease

Treatment The best initial therapy is:


Aspirin Smoking cessation Cilostazol

Single most effective medication: Cilostazol Surgery or angioplasty if medical therapy not effective
In all major vascular disease control the following: BP LDL < 100 Diabetes

MTBS2CK p.102103

AorticDissection 67-year-old man in ED with sudden onset chest pain is also felt between his scapulae. He has a history of HTN and tobacco smoking. BP 169/108. What is the best initial test?
a. b. c. d. e. f. g. Chest X-ray Chest CT Dont show specific changes MRA, TEE, and CTA MRA have same accuracy; Transesophageal echocardiogram not most accurate, Transthoracic echocardiogram neither best initial CT angiogram Angiography Most accurate, but not best initial test
Author: J. Heuser; commons.wikimedia.org Used with permission

MTBS2CK p.103

AorticDisease

AorticDisease/Treatment For dissection, Most important step is: Control BP This can be done with:
Beta blockers Nitroprusside

Most frequently tested points in aortic disease Diagnosis & treatment of acute dissection Screening recommendations
Key points for presence of aortic dissection Pain in between the scapulae Difference in blood pressure between the arms

Beta blockade decreases shearing forces that worsens dissection Beta blockers must start before nitroprusside to protect against reflex tachycardia of nitroprusside, which worsens shearing forces

MTBS2CK p.103

MTBS2CK p.104

46

AbdominalAorticAneurysm Which of the following is the most appropriate screening for aortic aneurysm?

a. b. c. d d. e.

Everyone > 50 with CT angiography Men who ever smoked > 65 with ultrasound Everyone > 50 with ultrasound Everyone > 65 with ultrasound Men > 65 with ultrasound
Incidence of AAA is less in both nonsmokers and in women, so there is no recommendation for screening in those groups

MTBS2CK p.104

PeripartumCardiomyopathy Which of the following is most dangerous to a pregnant woman? a. b. c. d. e. Mitral stenosis Peripartum cardiomyopathy Choose this if p peripartum p Ei Eisenmenger phenomenon h cardiomyopathy is not one of the choices Mitral valve prolapse Atrial septal defect

Not as dangerous as choices b and c

Unknown why antibodies are made against the myocardium in some pregnant women LV dysfunction often reversible and short term If LV dysfunction does not improve, then must undergo cardiac transplantation

MTBS2CK p.104

MTBS2CK p.105

PeripartumCardiomyopathy Therapy the same as for dilated cardiomyopathy of any cause:


ACEi/ARB Beta blockers Spironolactone Diuretics Digoxin

EisenmengerSyndrome

Repeat pregnancy with peripartum cardiomyopathy will kill 50-70% of women!! Peripartum cardiomyopathy develops after delivery in most cases; ACEi/ARBs must be used
MTBS2CK p.105

From right-to-left shunt from pulmonary HTN Develops in person with ventricular septal defect with left-to-right shunt that eventually develops pulmonary HTN When pulmonary HTN becomes very severe shunt reverses Right-to-left shunting develops

MTBS2CK p.105

47

EisenmengerSyndrome

EisenmengerSyndrome

If peripartum cardiomyopathy is not one of the choices in asking, What is the worst cardiac disease in pregnant women? then Look for Eisenmenger in the choices
Pregnancy increases plasma volume by 50%. Mitral stenosis will worsen in pregnancy, But not as much as peripartum cardiomyopathy or Eisenmenger

Large left-to-right shunt (congenital heart defect)

P l Pulmonary HTN
Reverse of the shunt: right-to-left shunt
MTBS2CK p.105

MTBS2CK p.105

48

Dermatology Part1 Dermatology Part1


ConradFischer,MD AssociateProfessorofMedicine TouroCollegeofMedicine NewYorkCity CutaneousMalignancies AtopicDermatitis(Eczema) Psoriasis PityriasisRosea SeborrheicDermatitis(Dandruff) BlisteringDiseases

Cutaneousmalignancies

MalignantMelanoma

Greater with pale skin More sun-exposed areas Diagnosis: Biopsy Treatment: Surgical removal No truly effective chemotherapy
Skin Cancer More sun, more cancer Biopsy Remove
MTBS2CK p. 363

More frequent in sun-exposed areas But! Not exclusive to those areas Benign vs. Malignant Melanoma clinically ABCDE: A: Asymmetry B: Border irregularity C: Color irregularities D: Diameter > 6 mm E: Evolution (changing over time)
MTBS2CK p.363

MalignantMelanoma

DistinctionBetweenBenignandMalignant

Diagnosis? Suspicious?

Benign

Malignant

Biopsy!
Include entire thickness of lesion if possible ibl Worst prognosis!!

Round Evenborders Colorevenlyspread Diameterconstant


MTBS2CK p.364

Asymmetric Bordersuneven Coloruneven Diameterincreases

Growing lesions
MTBS2CK p. 363

BenignLesion

MalignantMelanoma

Diagnostic Test Full thickness biopsy: indispensible Dont perform shave biopsy Treatment/Prognosis Surgical removal Must include normal skin surrounding lesion Interferon injection helpful in widespread disease Melanoma has strong tendency to metastasize to brain
MTBS2CK p.364
Source: Andrew Peredo

MTBS2CK p.364

SquamousCellCancer

SquamousCellCancer

Sunlight

Organ transplant!!!!
Immunosuppressive drugs! Squamous starts looking like ulcer Doesnt heal, grows Biopsy and remove

Source: James Heilman MD

MTBS2CK p.364

MTBS2CK p.364

BasalCellCarcinoma

BasalCellCarcinoma

Most common skin cancer Question describes: Waxy lesion

Shiny like a pearl


U Unlike lik melanoma, l wide id margins i not t necessary Shave biopsy fine! (diagnostic) Recurrence < 5% Basal cell: use Mohs microsurgery
MTBS2CK p.364 MTBS2CK p.364

BasalCellCarcinoma/ MohsMicrographicSurgery

BasalCellCarcinoma/ MohsMicrographicSurgery

Remove skin cancer under dissecting microscope Very thin slices of skin Immediate frozen section VERY precise way to treat skin cancer Mohs:
Remove cancer Keep normal
MTBS2CK p.365

Stop resecting when margin is cancerfree No removal of wide margins Mohs Best: Delicate areas Eyelid & Ear

MTBS2CK p.365

KaposiSarcoma(KS)

KaposiSarcoma(KS)

In past, KS seen only in older men of Mediterranean origin Now: AIDS From HHV-8, which is oncogenic Reddish/purplish R ddi h/ li h More vascular than others

MTBS2CK p.365

MTBS2CK p.365

KaposiSarcoma(KS)

KaposiSarcoma/Treatment

Also in GI tract and lung Only from AIDS via sexual contact associated with KS IDU AIDS rarely associated with KS

Unlike other skin cancers, KS not routinely removed surgically 1. Treat AIDS with antiretrovirals 2. Majority disappear as CD4 improves 3 Intralesional 3. I t l i li injections j ti of f vincristine i i ti or interferon 4. If these fail, use chemotherapy with liposomal doxorubicin
MTBS2CK p.366

MTBS2CK p.365

ActinicKeratoses

ActinicKeratoses

Premalignant From high-intensity sun exposure in fairskinned people Very small risk of SCC for each individual lesion

Richard Usatine, M.D. Used with permission.

MTBS2CK p.366

MTBS2CK p.366

ActinicKeratoses

ActinicKeratoses

Many occur in a single person Risk is cumulative Like cervical dysplasia and cervical cancer Slow to progress Remove with:
Curettage Cryotherapy Laser Topical 5-fluorouracil Imiquimod
MTBS2CK p.366

Richard Usatine, M.D. Used with permission.

MTBS2CK p.366

ActinicKeratoses

SeborrheicKeratoses

Imiquimod Local immunostimulant Also for molluscum contagiosum Condyloma acuminatum

Extremely common in elderly Hyperpigmented lesions Commonly referred to as liver spots Stuck on appearance May look like melanoma to some people Seborrheic keratoses = Zero malignant potential

MTBS2CK p.366

MTBS2CK p.366

SeborrheicKeratoses

SeborrheicKeratoses

Removed with: Cryotherapy Surgery Laser Removal (cosmetic reasons)

Source: James Heilman MD

MTBS2CK p.366

MTBS2CK p.366

AtopicDermatitis(Eczema)

AtopicDermatitis(Eczema)/Presentation

Common skin disorder Associated with overactivity of mast cells & immune system Look for history of: Asthma Allergic rhinitis Family history of atopic disorders Onset < age 5, very rare to start > 30

From premature and idiosyncratic release of transmitters (e.g., histamine) Pruritus & scratching is most common presentation Scratching leads to scaly rough areas of thickened skin On face, neck & skin folds Popliteal area behind knee

MTBS2CK p.366

MTBS2CK p.366

AtopicDermatitis(Eczema)/Presentation

AtopicDermatitis(Eczema)/Presentation

MTBS2CK p.366

Richard Usatine, M.D. Used with permission.

MTBS2CK p.366

Richard Usatine, M.D. Used with permission.

AtopicDermatitis(Eczema)/Presentation Itching scratching Scratching more itching Superficial skin infections from Staphylococcus are common Microorganisms driven under epidermis by scratching This, in turn, more itching Skin thickened because of scratching and drying = lichenified
MTBS2CK p.367

AtopicDermatitis(Eczema)/SkinCare

1. Stay moisturized: dry skin is more itchy 2. Use a humidifier, especially in winter 3. Avoid bathing, soap, and washcloths (skin hyperirritable) 4. Avoid brushes, , washcloths, , hot water, , and anything that rubs on skin 5. Cotton less irritating than wool
IgE levels: Elevated in atopic dermatitis
MTBS2CK p.367

Food allergies dont exacerbate atopic dermatitis

AtopicDermatitis(Eczema)/MedicalTherapy

AtopicDermatitis(Eczema)/MedicalTherapy

1. Topical corticosteroids: used in flares of disease (oral steroids only in most severe acute flares) 2. Tacrolimus and pimecrolimus
T cellinhibiting agents L Longer-term t control t l Help get patient off steroids Used systemically in organ transplant recipients Prevent organ rejection Used topically for atopic dermatitis
MTBS2CK p.367

3. Antihistamines:
Mild disease: nonsedating drugs (cetirizine, fexofenadine, loratadine) Severe disease: hydroxyzine, diphenhydramine, doxepine

4. Antibiotics (e.g., cephalexin, mupirocin, retapamulin) when impetigo occurs 5. UV light (phototherapy) for severe recalcitrant disease
MTBS2CK p.367

Psoriasis/Definition/Presentation

Psoriasis/Definition/Presentation

Incredibly common 2 million in US Characterized by silvery, scaly plaques Not itchy most of the time Arthritis < 10% Extensive disease associated with depression

MTBS2CK p.367

MTBS2CK p.368

Psoriasis/Treatment Local Disease 1. Topical high-potency steroids: fluocinonide, amcinolone, betamethasone, clobetasol 2. Vitamin A & Vitamin D ointment Helps get patient off steroids Vitamin D agent is calcipotriene Steroids cause skin atrophy 3. Coal tar preparation 4. Pimecrolimus and tacrolimus Used in delicate areas (e.g., face & penis) Alternative to steroids Less deforming
MTBS2CK p.368

Psoriasis/Treatment

Steroids cause atrophy Inhibit collagen formation and growth Convert AAs glucose Gluconeogenesis

MTBS2CK p.368

Psoriasis/Treatment

Extensive Disease 1.UV light 2.Antitumor necrosis factor (TNF) inhibitors Etanercept Adalimumab Infliximab Miraculous for severe disease 3.Methotrexate: Last because of effects on liver & lung First for psoriatic arthritis
MTBS2CK p.368

TNF inhibitors reactivate TB Screen with PPD prior to use

MTBS2CK p.368

PityriasisRosea

PityriasisRosea

Idiopathic, transient dermatitis Starts with a single lesion (herald patch) Then disseminates Can look like secondary syphilis But spares palms & soles Transient If symptomatic: treat with steroids or UV light
MTBS2CK p.369
Source: Andrew Peredo

MTBS2CK p.368

SeborrheicDermatitis(Dandruff)

SeborrheicDermatitis(Dandruff)

Hypersensitivity reaction Dermal infection Noninvasive dermatophyte organisms Both topical steroids and antifungal agents t (e.g., ( ketoconazole) k t l ) are useful f l

Increased in:

AIDS Parkinsons disease

Seborrheic = Benign

MTBS2CK p.369

MTBS2CK p.369

PemphigusVulgaris

PemphigusVulgaris

Both idiopathic/autoimmune form and Drug induced form Associated with: ACE inhibitors Penicillamine Phenobarbital Penicillin

Autoantibodies split epidermis, resulting in: Bullae that easily rupture Thin-walled Involvement I l t of f mouth th Fluid loss & infection Widespread (acts like burn)

MTBS2CK p.369

MTBS2CK p.369

PemphigusVulgaris

PemphigusVulgaris

Source: commons.wikimedia.org
Source: phil.cdc.gov

PemphigusVulgaris

PemphigusVulgaris

Characteristic finding: Nikolsky sign Loss or denuding of skin from mild pressure Nikolsky y sign: g Removal of superficial layer of skin Single sheet while pulling on it Fingers worth of pressure
MTB S2CK - p. 370

Most accurate test = biopsy Biopsy shows: autoantibodies on immunofluorescent (IF) studies

Without treatment, pemphigus is fatal

MTBS2CK p.370

PemphigusVulgaris/Treatment

BullousPemphigoid

1. Systemic steroids (prednisone) 2. Azathioprine or mycophenolate to wean patient off steroids 3. Rituximab (anti-CD20 antibodies) or IVIG in refractory cases

Much milder than pemphigus: Bullae stay intact Less loss of fluid Less infection Mouth involvement uncommon

MTBS2CK p.370

MTBS2CK p.370

BullousPemphigoid

BullousPemphigoid

Biopsy with IF stains is Most accurate test Best initial therapy: Prednisone To get patients off steroids, use azathioprine, cyclophosphamide, or mycophenolate Mild bullous p pemphigoid p g responds p to erythromycin, dapsone, and nicotinamide (not niacin)
Nikolsky sign absent in bullous pemphigoid
Richard Usatine, M.D. Used with permission.

MTBS2CK p.370

PorphyriaCutaneaTarda

PorphyriaCutaneaTarda

Blistering skin disease Sun-exposed areas With history of: Liver disease Hepatitis C Alcoholism Estrogen use Iron overload (hemochromatosis)
MTBS2CK p.370

Hepatitis C: Most frequently tested association with PCT Look for involvement of: backs of hands & face

MTBS2CK p.370

PorphyriaCutaneaTarda/ DiagnosticTests/Treatment

Most accurate test Increased uroporphyrins on 24-hr urine Deficiency of uroporphyrin decarboxylase activity Treatment Correct underlying cause Stop alcohol Stop estrogens Remove iron with phlebotomy
MTBS2CK p.370

Dermatology Part2
SkinInfections DrugReactions StaphylococcalScaldedSkinSyndromeand ToxicShockSyndrome Acne

Impetigo

Impetigo/Treatment

Most superficial bacterial skin infection Staphylococcus and Streptococcus invade epidermis Results in weeping, crusting, oozing, & draining of skin

Mild disease with topical agents: Mupirocin Bacitracin Retapamulin Severe disease with oral agents: Dicloxacillin or cephalexin Community-acquired MRSA with: Doxycycline Clindamycin Trimethoprim/sulfamethoxazole (TMP/SMZ)
MTBS2CK p.371

MTBS2CK p.371

10

Erysipelas

Erysipelas/Presentation

Much more severe than impetigo Occurs at deeper level in skin Streptococcus > Staphylococcus Invades dermal lymphatics Causes bacteremia, bacteremia leukocytosis leukocytosis, fever fever, and chills Fatal, if untreated
Skin infections can cause glomerulonephritis, but not rheumatic fever.
MTBS2CK p.371

Bright, red, hot swollen Lesion on face Leukocytosis Often systemic disease

MTBS2CK p.371

Erysipelas/Presentation

Erysipelas/Treatment Although erysipelas is more often from Streptococci, you must treat for Staphylococcus as well unless you have a definitive diagnostic test such as blood cultures. Treatment of all skin infections is similar Same answers as for:
Cellulitis Folliculitis Furuncles Carbuncles

Source: Thomas F Sellers

MTBS2CK p.371

Erysipelas/Treatment

Erysipelas/Treatment

Mild disease - oral medications: Dicloxacillin, cephalexin, cefadroxyl Penicillin allergic: erythromycin, clarithromycin, or clindamycin MRSA: Doxycycline, clindamycin, or TMP/SMX

Severe disease (e.g., fever): IV medications: Oxacillin, nafcillin, cefazolin Penicillin allergic: Clindamycin, vancomycin MRSA: Vancomycin, linezolid, daptomycin, tigecycline, ceftaroline
Cross reaction Between penicillin and cephalosporins unusual (< 5%)

MTBS2CK p.372

MTBS2CK p.372

11

Erysipelas/Treatment

Cellulitis

Step 2 CK tests: Route of administration (O l vs Intravenous) (Oral I t )

Soft tissue infection of skin Extends from dermis into subcutaneous tissue Skin: Warm, red, swollen, and tender

MTBS2CK p.372

MTBS2CK p.372

Cellulitis

Cellulitis Involves legs > arms Doesnt have collections of walled-off infection, which is an abscess Cellulitis isnt only at hair follicle; that's folliculitis, furuncles, and carbuncles

Skin infection: Caused by S. aureus

NOT S. epidermidis
S. epidermidis lives on skin as normal flora
MTBS2CK p.372 MTBS2CK p.372

Cellulitis/DiagnosticTests

Cellulitis/Treatment

No diagnostic testing needed Most accurate test


Inject sterile saline into skin & aspirate for culture
Yield only 20%

Same as for erysipelas Topical antibiotics will not cover cellulitis Below dermal/epidermal junction Topical T i l antibiotics tibi ti wont t reach h

Staphylococcus > Streptococcus

MTBS2CK p.372

MTBS2CK p.372

12

Folliculitis,Furuncles,Carbuncles

Originate around hair follicles Different terms dont have precise definitions Indistinguishable, no cutoff in size
Severe disease = fever, chills, bacteremia

Folliculitis,Furuncles,Carbuncles/ SizeoftheInfection

Folliculitis: Earliest & mildest Furuncle: Small abscess or collection of infected material Carbuncle: Collection of furuncles

Folliculitis < Furuncle < Carbuncle

Treat: Ox/Clox/Diclox/Naf
MTBS2CK p.372 MTBS2CK p.372

Folliculitis,Furuncles,Carbuncles

Folliculitis,Furuncles,Carbuncles

Source: cdc.gov

Source: commons.wikimedia.org

Folliculitis,Furuncles,Carbuncles

Folliculitis,Furuncles,Carbuncles/ PenicillinAllergy

Reaction - Rash Use cephalosporins Reaction - Anaphylaxis Mild infection: Macrolides, clindamycin, doxycycline, or TMP/SMZ Severe infection: Vancomycin, linezolid, daptomycin, or tigecycline
MTBS2CK p.373 MTBS2CK p.373

13

Folliculitis,Furuncles,Carbuncles/ OtherAntistaphylococcal Medications

FungalInfections

Beta-lactam/beta-lactamase combinations
Amoxicillin/clavulanate Ticarcillin/clavulanate Ampicillin/sulbactam Piperacillin/tazobactam

Dermatophyte = superficial fungal infection = tinea For example: Tinea corporis p = body y Tinea manum = hand Tinea pedis = foot Tinea cruris = groin (jock itch)
MTBS2CK p.374

Carbapenems (imipenem, meropenem)

MTBS2CK p.373374

FungalInfections

FungalInfections/DiagnosticTests/Treatment

Best initial test KOH (potassium hydroxide) Dissolves epidermal skin cells Leaves fungi intact Most accurate test Fungal culture

Richard Usatine, M.D. Used with permission.

MTBS2CK p.374

FungalInfections/DiagnosticTests/Treatment

FungalInfections/Treatment

Best initial therapy Topical antifungal agent if no hair or nails involved For hair (tinea capitis) and nail (tinea unguium)
Terbinafine Itraconazole

Topical antifungal agents: Clotrimazole Ketoconazole Oral ketoconazole Econazole causes gynecomastia Miconazole Mi l Nystatin Ciclopirox

MTBS2CK p.374

MTBS2CK p.375

14

FungalInfections/OralandVaginalCandidiasis

DrugReactions Hypersensitivity reactions to medications vary in severity When severity of reaction changes, the name of reaction changes Drugs causing hypersensitivity reactions: Penicillins Sulfa drugs (including thiazides, furosemide, and sulfonylureas) Allopurinol Phenytoin Lamotrigine NSAIDs
MTBS2CK p.375

Same answers for both KOH: Best initial test Fungal culture: Most accurate test With clear presentation: Treat Topical antifungal from previous list

MTBS2CK p.375

DrugReactions

MorbilliformRash

Drugs that cause hypersensitivity reactions of the skin are the same that cause: Hemolysis Interstitial I t titi l nephritis h iti Thrombocytopenia

Mildest reaction Skin stays intact No mucous membrane involvement No specific therapy

MTBS2CK p.375

MTBS2CK p.375

MorbilliformRash

ErythemaMultiforme

Widespread Target lesions Mostly on trunk Mucous membrane uninvolved From herpes or mycoplasma Prednisone may benefit

Source: commons.wikimedia.org

MTBS2CK p.376

MTBS2CK p.375

15

ErythemaMultiforme

StevensJohnsonSyndrome

Erythema multiforme is characterized by multiple small target-shaped l i lesions th that t can b be confluent

Very severe Involves mucous membranes Sloughs off respiratory epithelium May lead to respiratory failure Steroids not beneficial Use intravenous immunoglobulins (IVIG)

MTBS2CK p.375

Source: Andrew Peredo

MTBS2CK p.375

ToxicEpidermalNecrolysis

ToxicEpidermalNecrolysis

Mucous membrane involvement Nikolsky sign Steroids definitely dont help Treat with IVIG

The skin comes off in a sheet, simulating a burn.

MTBS2CK p.375

MTBS2CK p.376

Source: Conrad Fischer, MD

StaphylococcalScaldedSkinSyndrome(SSSS) andToxicShockSyndrome(TSS)

StaphylococcalScaldedSkinSyndromeand ToxicShockSyndrome

Different severities of same event Reaction to toxin in surface of Staphylococcus SSSS looks similar to TEN, including Nikolsky sign life TSS has skin involvement as well as lifethreatening multi-organ involvement:
Hypotension Renal dysfunction ( BUN and creatinine) Liver dysfunction CNS involvement (delirium)

Both treated with: Antistaphylococcal medications Oxacillin or nafcillin are most effective Cefazolin is essentially equal Antibiotics dont reverse disease Kills Staphylococcus that produces toxin

MTBS2CK p.377

MTBS2CK p.377

16

Acne/Treatment

Acne/Treatment

Mild acne Topical antibacterials: Benzoyl peroxide If ineffective add topical antibiotics (e.g., clindamycin or erythromycin)

Moderate acne Add topical vitamin A derivatives: tretinoin, adapalene, or tazarotene to topical antibiotics If no response to topical vitamin A derivatives and antibiotics, use oral antibiotics (e.g., minocycline or doxycycline)

MTBS2CK p.377

MTBS2CK p.377

Acne/Treatment

Severe acne Add oral vitamin A, isotretinoin to oral antibiotics Isotretinoin causes hyperlipidemia

Vitamin A derivatives extremely teratogenic

MTBS2CK p.377

17

EMERGENCYMEDICINE
NiketSonpal,MD
ChiefResident LenoxHillHospitalNSLIJ AssistantClinicalProfessor TouroCollegeofMedicine

Toxicology,Poisoning,& Overdose

TreatmentofOverdose
32-year-old woman with a history of depression comes to ED 30 minutes after taking a bottle of pills in a suicide attempt. BP 118/70, pulse 90, and respirations normal. She refuses to tell you what she took. What is the next step?

Gastric Lavage
OVERDOSE TREATMENTS

a. Induce emesis with ipecac p th way up the b. Gastric lavage c. Psychiatric consultation Consultations are 99% wrong on USMLE Takes too long d. Serum chemistry e. Urine toxicology screen Urine wont show up yet Pills will not make that transit time f. Cathartics/laxatives g. Whole bowel irrigation Doesnt change outcomes We dont know if its opiates h. Naloxone We dont know if its benzos i. Flumazenil
MTBS2CK p.533

It could be caustic and injure on

Cathartics

Bowel Irrigation Fluids and Diuresis

Ipecac

MTBS2CK p.534

InitialManagementofPoisoning

InitialManagementofPoisoning

Gastrointestinal Emptying Gastric lavage


Gastric emptying of any kind is always wrong with Caustics (acids and alkali) Altered mental status

Gastric lavage is rarely done. Removes 50% of pills at 1 hour Removes 15% of pills at 2 hours

Ipecac is always a wrong answer in ED

MTBS2CK p.534

MTBS2CK p.534

InitialManagementofPoisoning

InitialManagementofPoisoning

Ipecac No inpatient benefit 15-20 minutes onset Hinders antidotes

Cathartics Cathartic agents WRONG answer Prokinetics WRONG answer

MTBS2CK p.534

MTBS2CK p.534

InitialManagementofPoisoning

InitialManagementofPoisoning

Forced Diuresis Fluids and diuretics is always a wrong answer Risk of PE > benefit

Whole Bowel Irrigation

NGT with polyethylene glycol-electrolyte solution is almost always wrong Only for:
Iron ingestion Lithium Drug-filled packets

MTBS2CK p.534

MTBS2CK p.534

InitialManagementofPoisoning

When answer is unclear and cause of overdose is asked say:


Acetaminophen 1st Most Common Aspirin 2nd Most Common

Woman comes to ED one hour after taking a bottle of pills. BP 118/70, pulse 90/min, and respirations 14/min. She is confused, disoriented, and lethargic. What is the next step in the management?
Seizure risk too high 2 line a. Flumazenil Dangerous in AMS b. Gastric lavage c. Psychiatric consultation Never get a consult youre an MD d. Naloxone and dextrose Not the first step e. Intubation
nd

What to do is often unclear. What is useless or dangerous (ipecac, forced diuresis, cathartics) is very clear.
MTBS2CK p.535

MTBS2CK p.535

InitialManagementofPoisoning

InitialManagementofPoisoning

Psychiatric consultation is indicated for suicide attempt, but is a wrong answer on USMLE S2 CK when specific antidotes and diagnostic tests are needed.

Opiate overdose is fatal: Give naloxone immediately Benzodiazepine overdose by itself is not fatal and acute withdrawal causes seizures Dont give flumazenil

MTBS2CK p.535

MTBS2CK p.535

InitialManagementofPoisoning

Acetaminophen

Charcoal Charcoal is benign Charcoal is not dangerous Charcoal is superior to lavage and ipecac
When you dont know what to do in toxicology, give charcoal
MTBS2CK p.535536

Legal drugs kill more people in the United States than illegal drugs because theyre less expensive and more available Toxicity of acetaminophen with ingestion > 8 to 10 g Fatality with ingestions > 12 to 15 g

MTBS2CK p.536

Acetaminophen Four Most Common Acetaminophen Overdose Questions: 1. If a clearly toxic amount of acetaminophen has been ingested (> 810 g) Ans er N-acetylcysteine Answer: N acet lc steine 2. If the overdose was > 24 hours ago No therapy

Acetaminophen 3. Amount of ingestion unclear? Drug level 4. Charcoal does not make N-acetylcysteine ineffective Charcoal isnt contraindicated with Nacetylcysteine

MTBS2CK p.536

MTBS2CK p.536

AspirinOverdose

AspirinOverdose

What is the most likely diagnosis?


Renal Toxicity Altered Mental Status
Respiratory Alkalosis Metabolic Acidosis

Aspirin causes diffuse, multisystem toxicity


Increased PT
ARDS
Aspirin Overdose

Tinnitus

Aspirin Overdose

Increased Anion Gap

Respiratory Failure Lactic Acidosis

Increased PTT

Hyperventilation
MTBS2CK p.536

MTBS2CK p.536

AspirinOverdose

Treatment is alkalinizing urine, which increases rate of aspirin excretion Know blood gas in aspirin overdose
Tinnitus, respiratory alkalosis, and metabolic acidosis are the key to diagnosing aspirin overdose.

Which of the following is most likely to be found in aspirin overdose?


(Normal values: pH 7.40, pCO2 40, HCO3 24)

a. pH 7.55 pCO2 50 HCO3 24 pH 7.25 p pCO2 62 HCO3 38 b. p c. pH 7.46 pCO2 22 HCO3 16 d. pH 7.35 pCO2 32 HCO3 20

This is a distracter p overdose never g gives Aspirin respiratory acidosis Metabolic acidosis with respiratory compensation

MTBS2CK p.536

MTBS2CK p.536537

Depressed patient presents with altered mental status from ingesting multiple toxic substances. You know for certain that he took some lorazepam only today, for the first time. There is no response to naloxone or dextrose. The patient is given flumazenil and immediately seizes. What is the most likely cause of the seizure?

What is the best initial test for the patient previously described?
a. Urine toxicology b. Electroencephalogram c. EKG Wont show the cause of the seizure d. Head CT in this patient e. Potassium level

a. Cocaine withdrawal Cocaine toxicity causes seizures, not withdrawal Doesnt cause seizures b. Opiate withdrawal c. Tricyclic antidepressants d. SSRIs SSRIs toxicity causes serotonin syndrome (SS) Causes tinnitus and hyperventilation, not seizures e. Aspirin
MTBS2CK p.537

MTBS2CK p.537

TricyclicAntidepressantToxicity

TricyclicAntidepressants Seizures Arrhythmia

Widened QRS Complex Dry mouth

TCA Toxicity

Urinary retention Constipation

MTBS2CK p.537

MTBS2CK p.538

TricyclicAntidepressants

Caustics

The best initial treatment of TCA overdose is with sodium bicarbonate


Bicarbonate will protect the heart against arrhythmia Bicarbonate does not increase urinary y excretion of TCAs as it does for aspirin

Caustic ingestion of acids and alkalis (e.g., drain cleaner) causes


Mechanical damage to oropharynx Esophageal perforation Stomach perforation

Dont give alkali to reverse acids

MTBS2CK p.538

MTBS2CK p.538

Caustics

CarbonMonoxidePoisoning

Flush out caustics Use water in high volumes Endoscopy is performed to assess degree of damage
Steroids dont prevent injury from caustics

Carbon monoxide (CO) poisoning is the MCC of death in fires

Thermal injury is always the wrong answer

MTBS2CK p.538

MTBS2CK p.538

CarbonMonoxidePoisoning

CarbonMonoxidePoisoning

CO binds oxygen to Hb so tightly that carboxyhemoglobin will not release oxygen to tissues Carboxyhemoglobin acts functionally like anemia

Myocardial infarction Carbon Monoxide

Functional Anemia

Dyspnea

Confusion Seizures

Lightheadedness

Source:cdc.gov

MTBS2CK p.538

MTBS2CK p.538

CarbonMonoxidePoisoning

No functional difference between absence of blood and carboxyhemoglobin; 60% carboxyhemoglobin = loss of 60% of blood The left ventricle cant can t distinguish between anemia, carboxyhemoglobin, and stenosis of coronary arteries.

Which of the following blood gas results would you find in carbon monoxide poisoning?
a. b. c. d. pH 7.55 pCO 50 HCO 24 pH 7.25 pCO 62 HCO 38 pH 7.46 pCO 22 HCO 16 pH 7.35 pCO 26 HCO 18
2 3 2 3 2 3 2 3

Distracter Respiratory acidosis with metabolic alkalosis Respiratory alkalosis with metabolic acidosis

MTBS2CK p.538

MTBS2CK p.538

CarbonMonoxidePoisoning

CarbonMonoxidePoisoning

Diagnostic tests
Carbon monoxide poisoning gives normal pO2 because oxygen doesnt detach from hemoglobin Routine oximetry will be falsely normal Most accurate test is a level of carboxyhemoglobin

Treatment
Remove patient from exposure Give 100% oxygen

MTBS2CK p.538

MTBS2CK p.539

CarbonMonoxidePoisoning

Methemoglobinemia

Treatment Severe disease is treated with hyperbaric oxygen Severe symptoms are defined as:
CNS symptoms Cardiac symptoms Metabolic acidosis

Methemoglobin is oxidized Hb Ferric Ferric = Fe3+ Ferrous = Fe2+ Oxidized hemoglobin is brown and will ill not t carry oxygen Chocolate-brown blood

MTBS2CK p.539

Wikimedia

MTBS2CK p.539

Methemoglobinemia

Methemoglobinemia/Presentation

Methemoglobinemia from idiosyncratic reaction of hemoglobin to drugs such as: Benzocaine and anesthetics Nitrites and nitroglycerin Dapsone p

The effects of methemoglobinemia are similar to carboxyhemoglobin Oxygen isnt delivered to tissues
Metabolic Acidosis Methemoglobinemia Functional Anemia

Ferrous Hemoglobin

Benzocaine Nitrites Dapsone

Methemoglobinemia Ferric Hemoglobin Confusion Seizures Headache


MTBS2CK p.539

Dyspnea Cyanosis

Lightheadness

MTBS2CK p.539

Methemoglobinemia/Presentation

Methemoglobinemia/DiagnosticTests/ Treatment

Carbon monoxide: blood abnormally red Methemoglobinemia: blood abnormally brown

Both methemoglobinemia and carboxyhemoglobin give normal pO2 on blood gas Most accurate test is methemoglobin level Best initial therapy is 100% oxygen Most effective therapy is methylene blue, which half-life of methemoglobin

Cyanosis + normal pO2 = methemoglobinemia


MTBS2CK p.539 MTBS2CK p.539540

Organophosphate(Insecticide)Poisoningand NerveGas

Organophosphate(Insecticide)Poisoningand NerveGas Bronchospasm Bronchorrhea Organophosphate g p p Poisoning Salivation Respiratory arrest

Organophosphates and nerve gas identical in their effects Nerve gas faster and more severe Massive increase in acetylcholine by i hibiti metabolism inhibiting t b li

Polyuria y

Lacrimation

MTBS2CK p.540

MTBS2CK p.540

Organophosphate(Insecticide)Poisoningand NerveGas

Acetylcholine constricts bronchi and increases bronchial secretions

56-year-old military commander attacked with nerve gas. Presents with salivation, lacrimation, urination, defecation, and SOB. Pupils are constricted. What is the first step in management?

a. Atropine p b. Decontaminate (wash) the patient Stabilize first c. Remove his clothing Stabilize first d. Pralidoxime Takes too long e. No therapy is effective Never the right answer

MTBS2CK p.540

MTBS2CK p.540

Organophosphate(Insecticide)Poisoningand NerveGas

DigoxinToxicity/Etiology

Nerve gas and organophosphates g p p are absorbed through the skin

Hypokalemia predisposes to digoxin toxicity because potassium and digoxin compete for same site on the Na+/K+ATPase Less potassium is bound, more digoxin is bound
K+ Increased Digoxin Binding

MTBS2CK p.540

MTBS2CK p.540

DigoxinToxicity/Presentation

DigoxinToxicity/Presentation

Confusion

Yellow Halo Vision

Hypokalemia
Digoxin g Toxicity Arrhythmias y

Digoxin toxicity Hyperkalemia

Digoxin toxicity

Hyperkalemia

Nausea Vomiting Abdominal Pain


MTBS2CK p.541

MTBS2CK p.540541

DigoxinToxicity/DiagnosticTests

DigoxinToxicity/DiagnosticTests

Most accurate test: digoxin level Best initial tests: potassium level and EKG EKG: downsloping of ST segment in all leads
Atrial tachycardia with variable AV block is the most common digoxin toxic arrhythmia

Digoxin can produce any arrhythmia


MTBS2CK p.541

DigoxinToxicity/Treatment

LeadPoisoning Renal Tubule Toxicity (ATN) Abdominal pain lead colic

Control potassium and give digoxinspecific antibodies Digoxin-binding antibodies rapidly remove digoxin from circulation
Strongest indication for digoxin-binding antibodies: CNS and cardiac involvement.

Lead Toxicity Sideroblastic Anemia

Memory y loss confusion Peripheral Neuropathy wrist drop

MTBS2CK p.541

MTBS2CK p.541

LeadPoisoning

LeadPoisoning/Treatment

Best initial test: level of free erythrocyte protoporphyrin Most accurate test: lead level

Chelating agents remove lead from body


Succimer: only oral Ethylenediaminetetraacetic acid (EDTA) and dimercaprol (BAL) are parenteral

Most accurate test for sideroblastic anemia: Prussian blue stain

MTBS2CK p.541

MTBS2CK p.541

MercuryPoisoning Oral ingested Neurological problems Nervous, jittery, twitchy, and sometimes hallucinatory Inhaled Lung toxicity & interstitial fibrosis

MercuryPoisoning

Theres no therapy to reverse pulmonary toxicity Chelating agents such as dimercaprol and succimer are effective in removing mercury from body and decreasing neurological t i it toxicity

MTBS2CK p.541

ToxicAlcohols/MethanolandEthyleneGlycol Methanol Ethylene glycol

DifferencesbetweenMethanoland EthyleneGlycol
Methanol Source Woodalcohol,cleaning solutions,paintthinner Ethyleneglycol Antifreeze

1. Intoxication 1 2. Metabolic Acidosis 3. Increased Anion Gap 4. Osmolar Gap

Treated w/ 1. Fomepizole 2. Dialysis

Toxic metabolite Presentation

Formicacid/formaldehyde Oxalicacid/oxalate

Oculartoxicity

Renaltoxicity Hypocalcemia, envelopeshaped oxalatecrystalsin urine

Initialdiagnostic Retinalinflammation abnormality


MTBS2CK p.542

MTBS2CK p.542

10

ToxicAlcohols/MethanolandEthyleneGlycol Osmolar Gap Osmolar gap = measured serum osmolality - calculated osmolality Serum osmolality = 2(Na+) + BUN/2.8 + Glucose/18
Ex: Measurement 350 - Calculated 300 = gap of 50 osmoles

ToxicAlcohols/MethanolandEthyleneGlycol

Treatment Best initial therapy: fomepizole, which inhibits alcohol dehydrogenase and prevents production of toxic metabolite Only dialysis removes methanol and ethylene glycol

Ordinary alcohol (ethanol) also osmolar gap


MTBS2CK p.542 MTBS2CK p.542

SnakeBites

SnakeBites/Treatment
Ineffectiveordangerous treatment Beneficialtherapy

Most common injury from snake bites is local wound Death from snake bites:
Hemolytic toxin: hemolysis, DIC, and damage to endothelial lining of tissues Neurotoxin: can result in respiratory paralysis, ptosis, dysphagia, and diplopia

Tourniquetsblocking arterialflow Ice

Pressure

Immobilizationdecreases movementofvenom Antivenin

Incisionandsuction, especiallybymouth

MTBS2CK p.542543

MTBS2CK p.543

SpiderBites/Presentation

DifferencesbetweenTypesofSpiderBites
Blackwidow Brownrecluse

All spider bites present with a sudden, sharp pain that patient may describe as: 1.I stepped on a nail 2. A piece of glass was in my shoe.

Presentation

Abdominal pain, musclepain Hypocalcemia

Localskin necrosis

Labtest abnormalities Treatment

None

Calcium, antivenin

Debridement, steroids, dapsone

MTBS2CK p.543

MTBS2CK p.543

11

Dog,Cat,andHumanBites

Dog,Cat,andHumanBites

Management of dog, cat, and human bites is essentially identical Theyre managed with:
Amoxicillin/clavulanate Tetanus vaccination booster if > 5 years since last injection

Dog and Cats: Pasteurella multocida Humans: Eikenella corrodens


Human bites are more damaging than dog and cat bites
MTBS2CK p.543

Rabies vaccine only if: Animal has altered mental status/bizarre behavior Attack was unprovoked, by a stray dog that cannot be observed or diagnosed

MTBS2CK p.544

RabiesandBats

HeadTrauma
If a bat was noted to be in the room and the patient was asleep VACCINATE!

MTBS2CK p.544

HeadTrauma

HeadTrauma

Any head trauma resulting in altered mental status or loss of consciousness (LOC) is managed first with a head CT Head CT without contrast is best initial test to detect blood
Contrast detects mass lesions such as cancer and abscess, not blood

LOC = CT Concussion:
No focal neurological abnormalities Normal CT scan

Contusion:
Occasionally (rarely) has focal findings Ecchymoses found on CT (blood mixed in with brain parenchyma)

MTBS2CK p.544

MTBS2CK p.544

12

Contusion

Headtrauma

Subdural and epidural hematomas: usually associated with more severe trauma than a concussion Impossible to distinguish without head CT, epidural hematoma more frequently with skull k ll f fracture t

MTBS2CK p.544

MTBS2CK p.544

EpiduralHematoma

SubduralHematoma

MTBS2CK p.545

Source: Brendan T. Doherty

LucidInterval

Treatment

Lucid interval is a second loss of consciousness occurring several minutes to several hours after initial loss of consciousness
Patient wakes up after initial LOC, but loses consciousness a second time due to accumulation of blood

Concussion: no specific therapy


Wait at least 24 hours before returning to sports

Contusion: vast majority need no specific treatment


Rarely need surgical debridement
Those with concussion safe to go home. Hospitalization isnt necessary. Observe at home for altered mental status.
MTBS2CK p.545

Time between first and second episodes of LOC is lucid interval


Both epidural and subdural hematomas are associated with a lucid interval.
MTBS2CK p.545

13

Treatment

DefinitionofaLargeIntracranialhemorrhage

Subdural and epidural hematoma: Treatment is based on size and signs of compression of brain Small ones are left alone Large L h hematomas t are managed d with: ith
1. Intubation and hyperventilation 2. Mannitol 3. Drainage
MTBS2CK p.545

Compression of ventricles or sulci Herniation with abnormal breathing and unilateral dilation of pupil Worsening mental status or focal findings

MTBS2CK p.546

SummaryofSevereHeadTrauma
Concussion Contusion Subdural Epidural

SummaryofSevereHeadTrauma
Concussion Contusion Subdural Epidural

Nofocal findings Nolucid Interval

Rarelyfocal +/ focal findings Nolucid Interval +/ lucid Interval

+/ focal findings +/ lucid interval

NormalCT

Ecchymoses

Venous, Arterial, crescent biconvexor lensshaped hematoma Drainlarge Drainlarge ones ones

Nospecific treatment; observeat home


MTBS2CK p.546

Nospecific treatment; observein hospital

MTBS2CK p.546

HeadTrauma 25-year-old man sustains head trauma in MVA. A large epidural hematoma is found. Immediately after intubation and mannitol, surgical evacuation is successfully performed. Which of the following will benefit the patient?
a. Repeated doses of mannitol Doesnt reduce mortality Doesnt always work b. Continued hyperventilation c. Proton pump inhibitor (PPI) For SAH only d. Nimodipine e. Dexamethasone Doesnt change outcomes
MTBS2CK p.546 MTBS2CK p.546

Steroids dont benefit intracranial bleeding. They decrease edema around mass lesions.

14

Burns

Burns&Hypothermia

Best initial therapy for those caught in a fire is 100% oxygen to treat smoke inhalation and CO poisoning Airway burn is 2nd MCC of death from burns only if theres been airway injury

MTBS2CK p.547

Burns

Burns

Stridor

Burns inside mouth

Burn Victim Indications for Intubation

Burns inside nasopharynx

If airway burn is not present, the 2nd MCC of death is volume loss Fluid replacement is based on percentage of body surface area (BSA) burned

Hoarseness

Wheezing
MTBS2CK p.547

MTBS2CK p.547

VolumeofFluidReplacement

VolumeofFluidReplacement

Replace with Ringer lactate Give one-half in first 8 hours, a quarter in second 8 hours, and a quarter in the third 8 hours Give 4 mL for each percentage of BSA burned (including 2nd and 3rd degree burns) for each kilogram of body weight
Head: 9% BSA Arms: 9% BSA each Legs: 18% BSA each Chest or back: 18% BSA each
MTBS2CK p.547

Patchy burns that arent continuous make the percentage of BSA burned hard to assess. Use the width of patients hand to make an estimate Each hand width is 1% of BSA The short answer is: give the largest amount of Ringer lactate or normal saline listed as a choice; its probably the right answer
Fluid replacement: (4 mL) (% BSA burned) weight (kg)
MTBS2CK p.547

15

HeatDisorders What is the MCC of death several days to weeks after a burn? a. Infection Rhabdomyolysis causes renal failure b. Renal failure c. Cardiomyopathy Not affected so quickly Most common immediate cause of death d Lung injury d. Fluid loss doesnt mean malnutrition e. Malnutrition
Heatcramps/ exhaustion Heatstroke

Risk

Exertion;high outsidetemp Normal Normal

Exertion;high outsidetemp Elevated Elevated

Bodytemp CPKandK+ level Treatment

Oralfluidsand electrolytes

IVfluids; evaporation

MTBS2CK p.547

MTBS2CK p.548

HeatDisorders
Neuroleptic malignantsyndrome Risk Antipsychotic medications Malignant hyperthermia Anesthetics administered systemically Elevated Elevated Dantrolene

Hypothermia

Look for intoxicated person with hypothermia MCC of death: cardiac arrhythmia Best initial step: EKG

Bodytemp CPKandK+ level Treatment

Elevated Elevated Dantroleneor dopamineagonists: bromocriptine,cabergoline

MTBS2CK p.548

MTBS2CK p.548

Hypothermia

Drowning

Manage with airway and administer positive pressure ventilation


Steroids and antibiotics are not beneficial Salt water drowning: acts like CHF with wet, heavy lungs

Wrong answers for drowning include:


Steroids Antibiotics
MTBS2CK p.548 MTBS2CK p.549

16

Drowning

Fresh water drowning: causes hemolysis from absorption of hypotonic fluid into vasculature

CardiacRhythmDisorders

RBC RBC

Fresh H20

RBC Hemolysis

MTBS2CK p.549

InitialManagementofCardiacArrest

InitialManagementofCardiacArrest

First step :
Make sure patient is truly unresponsive Call for help: call 911/activate Emergency Medical Services (EMS)

After patient has been shown to be unresponsive, and EMS activated, the next step is:
1. Open airway: head tilt, chin lift, jaw thrust p and start chest compressions p if 2. Check pulse pulseless 3. Give rescue breaths if not breathing
CPR doesnt restart the heart; CPR keeps patient alive until cardioversion can be performed.
MTBS2CK p.549

Truly unresponsive Chest compressions Rescue breaths

MTBS2CK p.549

InitialManagementofCardiacArrest

Pulselessness

When is a precordial thump the answer?


Very recent onset of arrest (<10 minutes) with no defibrillator available You know its recent because you saw it happen (witnessed)

Sudden loss of a pulse can be caused by:


Asystole Ventricular fibrillation (VF) Ventricular tachycardia (VT) Pulseless electrical activity (PEA)

Best initial management of all forms of pulselessness is CPR


MTBS2CK p.549550

MTBS2CK p.549

17

Pulselessness

Pulselessness

Asystole Besides CPR, therapy for asystole is with epinephrine Vasopressin is alternative to epinephrine They both constrict blood vessels in tissues (e.g., skin) Shunts blood into critical central areas (e.g., heart and brain)
MTBS2CK p.550

Ventricular Fibrillation Best initial therapy for VF is an immediate, unsynchronized cardioversion followed by CPR Unsynchronized = defibrillation All electrical cardioversions synchronized except VF and pulseless VT

MTBS2CK p.550

Pulselessness

Ventricularfibrillation

Only VF and VT without a pulse l get t unsynchronized h i d cardioversion.

MTBS2CK p.550

MTBS2CK p.550

Pulselessness

Pulselessness

After defibrillation, then is epinephrine or vasopressin followed by another electrical shock Amiodarone or lidocaine Magnesium

Ventricular Tachycardia (VT) Wide complex tachycardia with regular rate Management entirely based on hemodynamic status
Pulseless VT: manage exactly same way as VF Hemodynamically stable VT: Amiodarone, then lidocaine, then procainamide. If all medical therapy fails, then cardiovert patient

Bretylium is always a wrong answer


MTBS2CK p.550551

MTBS2CK p.551

18

Pulselessness

VentricularTachycardia

Ventricular Tachycardia Hemodynamically unstable VT: Perform electrical cardioversion several times followed by medications (e.g., amiodarone or lidocaine)
VT is managed with shock, drugs, and CPR at all times in between the shocks.
MTBS2CK p.551 MTBS2CK p.551

Pulselessness

Pulselessness

Hemodynamic instability is defined as: Chest pain Dyspnea/CHF Hypotension Confusion These qualities of instability are the same for all rhythm disturbances Direct intracardiac medication administration is always a wrong answer
MTBS2CK p.551

Pulseless Electrical Activity (PEA) PEA = electrically normal, but no motor contraction PEA = no cardiac output

MTBS2CK p.551

Pulselessness

Pulselessness/Treatment

PEA treated by correcting underlying cause We synchronize delivery of electricity in cardioversion of VT to prevent worsening of arrhythmia into ventricular fibrillation or asystole asystole. Tamponade
Tension Pneumothorax

Causes of PEA

HYPOvolemia HYPOglycemia HYPO l i HYPOxia HYPOthermia

PEA: look for patient with a normal EKG and no pulse


MTBS2CK p.551552

Massive PE

MTBS2CK p.552

Metabolic acidosis HYPERkalemia HYPOkalemia

19

AtrialArrhythmias

AtrialArrhythmias

Atrial rhythm disturbances rarely associated with hemodynamic compromise Look for :
Palpitations, p , dizziness, , or lightheadedness g Exercise intolerance or dyspnea Embolic stroke

Irregularly irregular rhythm suggests Afib as the most likely diagnosis even before EKG is done A-fib: A fib: most common arrhythmia in the United States

MTBS2CK p.552

MTBS2CK p.552

AtrialArrhythmias

AtrialFibrillation

Atrial Fibrillation and Atrial Flutter Two disorders with nearly identical management Major points of difference are:
Flutter Fl tt is i a regular l rhythm h th vs. fibrillation is irregular Flutter changes to sinus rhythm or deteriorates into fibrillation
Source: Abhay Vakil, MD.

MTBS2CK p.552

MTBS2CK p.553

AtrialFlutter

AtrialArrhythmias/Treatment

Hemodynamically unstable atrial arrhythmias = synchronized cardioversion Synchronization prevents electricity from being delivered during refractory period (ST-T wave) Synchronization prevents change into VT or VF

MTBS2CK p.553

MTBS2CK p.553

20

AtrialArrhythmias/Treatment

AtrialArrhythmias/Treatment

Chronic Atrial Fibrillation A-fib > 2 days > 7 days risk of clot formation Routine cardioversion is not i di t d indicated Chronic is usually secondary atrial or valvular anatomic change

Chronic Atrial Fibrillation Shocking doesnt correct a dilated left atrium causing A-fib Over 90% will revert to fibrillation Rate control and warfarin is standard of care f A-fib for A fib
Chronic A-fib should be anticoagulated before cardioversion. Unstable, acute disease doesnt need anticoagulation.

MTBS2CK p.553554

MTBS2CK p.553554

AtrialArrhythmias/Treatment

AtrialArrhythmias/Treatment

Best initial therapy for fibrillation and flutter is to control the rate 1. Goal HR < 100/minute 2. INR between 2-3
1. 1 Slow rate 2. Anticoagulate Rate control drugs do not convert patient into sinus rhythm.
MTBS2CK p.554

Calcium-channel blockers used to control HR with atrial arrhythmias are diltiazem and verapamil
Reliably block AV node Other calcium-channel blockers control BP
No matter how much you might think it better to shock every patient into sinus, it just doesnt work in long run.
MTBS2CK p.554

AtrialArrhythmias/Treatment

AtrialArrhythmias/Treatment

Warfarin Without anticoagulation 6% a year stroke risk INR 2-3, rate: 2-3% stroke risk Dabigatran and Rivaroxaban Alternatives to warfarin For non-valvular A-Fib No INR monitoring
MTBS2CK p.554

A-fib is caused by anatomic cardiac defects dilating atrium Thats why vast majority revert Acute disease normalizes spontaneously; dont force it Chronic disease reverts into arrhythmia; dont force it either

MTBS2CK p.554

21

AtrialArrhythmias/Treatment

AtrialArrhythmias/Treatment

Atrial rhythm problems can cause acute PE from loss of atrial kick kick in those with cardiomyopathy.

Lone Atrial Fibrillation Patients with low risk of stroke can use ASA 2-3% per year vs. 1% per year bleeding Scoring S i is i called ll d CHADS score CHADS 2 = warfarin, dabigatran, rivaroxiban

MTBS2CK p.554

MTBS2CK p.554

AtrialArrhythmias/Treatment

AtrialArrhythmias/Treatment
Definition/Criteria for Low Risk of Stroke from A-fib No cardiomyopathy/CHF/atherosclerosis No HTN Age 75 No diabetes No stroke in past The answer for management of lone atrial fibrillation is: Rate control Aspirin No warfarin or dabigatran

Major bleeding from warfarin is defined as: Intracranial I t i lh hemorrhage h Requires transfusion

MTBS2CK p.554

MTBS2CK p.555

AtrialArrhythmias/Treatment Supraventricular Tachycardia

AtrialArrhythmias/Treatment

SVT
Vagal Maneuvers
Carotid Massage Valsalva Dive Reflex Ice immersion

Adenosine

Beta Blockers CCBs Digoxin


MTBS2CK p.555 MTBS2CK p.555

Source: Abhay Vakil, MD.

22

AtrialArrhythmias/Treatment

AtrialArrhythmias/Treatment

Wolff-Parkinson-White Syndrome (WPW) WPW is an anatomic abnormality in cardiac conduction pathway Answer most likely diagnosis question by looking for:
SVT alternating with ventricular tachycardia SVT gets worse after diltiazem or digoxin Observing delta wave on EKG Vagal maneuvers slow and convert SVT. They dont convert atrial fibrillation.
MTBS2CK p.556

Most accurate test for WPW is cardiac electrophysiology (EP) studies


MTBS2CK p.556

AtrialArrhythmias/Treatment

AtrialArrhythmias

Acute therapy: Procainamide or amiodarone Chronic therapy: Radiofrequency catheter ablation is curative for WPW EP studies tell you where the anatomic defect is Digoxin and calcium-channel blockers are dangerous in WPW

Multifocal Atrial Tachycardia Multifocal atrial tachycardia (MAT) is associated with chronic lung disease such as COPD Treat underlying lung disease Treat MAT as you would A-fib, but avoid beta blockers because of lung disease
MTBS2CK p.556

MTBS2CK p.556

AtrialArrhythmias Woman comes to office for routine evaluation. Shes found to have a pulse of 40 and an otherwise completely normal history and physical examination. What is the most appropriate next step in the management of this patient?

MTBS2CK p.557

a. Atropine You dont know the rhythm y yet y b. Pacemaker Too invasive c. EKG Too invasive d. Electrophysiology studies Can result in ischemia e. Epinephrine f. Isoproterenol Old and no longer used; always wrong g. Nothing; reassurance Without EKG cannot say
MTBS2CK p.557

23

BradycardiaandAVblock

BradycardiaandAVblock

Sinus Bradycardia Isoproterenol is never the right answer to anything Sinus Bradycardia
Asymptomatic Symptomatic

No Treatment

Atropine 1st Pacemaker Long Term

MTBS2CK p.557

MTBS2CK p.558

BradycardiaandAVblock

BradycardiaandAVblock Second-Degree AV block Mobitz I or Wenckebach block: progressively lengthening PR interval results in a dropped beat Mobitz I is most often a sign of normal aging of conduction system. y If there are no symptoms, y p , its managed the same way as sinus bradycardia Dont treat if asymptomatic

First-Degree AV block Use same management as sinus bradycardia


Atropine p and p pacemaker are used for sinus bradycardia only if symptomatic.

MTBS2CK p.558

MTBS2CK p.558

BradycardiaandAVblock
Second-Degree AV block Mobitz II block: far more pathologic than Mobitz I Mobitz II just drops a beat without progressive lengthening of PR interval. Mobitz II progresses or deteriorates into third-degree AV block. Treat it like third-degree AV block. Everyone with Mobitz II block gets a pacemaker even if they are asymptomatic

BradycardiaandAVblock

MTBS2CK p.558

MTBS2CK p.558

24

58-year-old woman is admitted to hospital with an acute MI. On the second hospital day she develops sustained VT even though she is on aspirin, heparin, lisinopril, and metoprolol. What is the most appropriate next step in management?

Which of the following tests would you do for this patient to determine a risk of recurrence?

a. Increase the dose of metoprolol Wont treat ischemia b. Add diltiazem Will not affect rhythm c. Angiography for angioplasty or bypass d. Implantable defibrillator Underlying cause can be fixed e. EP studies Not ectopy, but rather from ischemia
MTBS2CK p.559

For unexplained syncope a. EP studies b. Echocardiography c. MUGA scan (nuclear ventriculography) For perfusion d. Ventilation/perfusion p scan For PE study e. Tilt-table testing For syncope

MTBS2CK p.559

73-year-old man has his third syncopal episode in last 6 months. An EKG done in the field shows VT. His stress test is normal. What is the most appropriate next step in the management of this patient?
a. Metoprolol Not enough g to p prevent death b. Diltiazem Have no affect in VT c. Angiography Normal stress therefore no need for angio d. Implantable defibrillator e. EP studies EKG shows cause so need for EP

46-year-old man has intermittent episodes of palpitations, lightheadedness, and near-syncope. His EKG is normal. The echo shows an EF of 42%. Holter monitor shows several runs of wide complex tachycardia lasting 5-10 seconds. Which of the following is most likely to benefit this patient?
a. Pacemaker placement Dont know underlying cause yet b. Digoxin Does nothing for vent arrhythmias c. Warfarin Low risk for clot d. EP studies e. Swan-Ganz catheter Swan if for diagnosing SHOCK
MTBS2CK p.560

MTBS2CK p.559560

25

PituitaryDisorders Endocrinology
Dr.ConradFischer,MD AssociateProfessorofMedicine TouroCollegeofMedicine NewYorkCity

Panhypopituitarism DiabetesInsipidus p Acromegaly Hyperprolactinemia

Panhypopituitarism/Etiology

ThePituitaryGland

Compression or damage of the pituitary gland Tumors, cancer, adenomas, cysts, meningiomas, craniopharyngiomas, or lymphoma Trauma and radiation are damaging

MTBS2CK p.107

MTBS2CK p.107

Panhypopituitarism/Etiology

Panhypopituitarism/Presentation

Conditions such as: Hemochromatosis Sarcoidosis Histiocytosis X Infection with fungi fungi, TB TB, or parasites Autoimmune and lymphocytic infiltration damages gland

Prolactin deficiency Men


No symptoms

Women:
Prolactin = In favor of or pro p lactation If deficient, the patient cannot lactate normally after childbirth

MTBS2CK p.107

MTBS2CK p.107

Panhypopituitarism/Presentation Luteinizing hormone (LH) and follicle-stimulating hormone (FSH) deficiency Both genders will have decreased libido and decreased axillary, pubic, and body hair Men
Unable to produce testosterone or sperm Erectile dysfunction and decreased muscle mass

Panhypopituitarism/Presentation

Growth hormone (GH) deficiency Adults


Few symptoms

Children
Dwarfism

Women
Unable to ovulate or menstruate normally and become amenorrheic

Catecholamines, glucagon, and cortisol act as stress hormones

MTBS2CK p.107

MTBS2CK p.108

Panhypopituitarism/Presentation

Panhypopituitarism/DiagnosticTests

Kallman Syndrome Decreased FSH and LH Decreased GnRH Anosmia

Hyponatremia from: Hypothyroidism Glucocorticoid underproduction Potassium levels remain normal


Aldosterone is unaffected and aldosterone excretes potassium

MTBS2CK p.108

MTBS2CK p.108

Panhypopituitarism/DiagnosticTests MRI detects compressing mass lesions on pituitary

Panhypopituitarism/DiagnosticTests Growth Hormone (GH) IGF level Arginine Stimulation: increases GH GHRH stimulation: increases GH ACTH and Cortisol levels High Hi h or normal ll levels l excludes l d panhypopituitarism h it it i Sex Hormones LH, FSH level Testosterone level Thyroid TSH
MTBS2CK p.108

MTBS2CK p.108

Source:JamesG.Smirniotopoulos,MD

Panhypopituitarism/DiagnosticTests Older, Less Useful Tests Metyrapone Inhibits 11-beta hydroxylase and decreases cortisol Normal: ACTH and 11deoxycortisol levels rise I Insulin li stimulation ti l ti Normal: decreased glucose levels raise GH Failure of GH to rise in response to insulin indicates pituitary insufficiency

Panhypopituitarism/Treatment Replace deficient hormones with...


Thyroxine Cortisone Testosterone and estrogen Recombinant human growth hormone Antidiuretic hormone (ADH) and oxytocin (Posterior Pituitary)

Note: No deficiency disease described for oxytocin


Oxytocin helps uterine contraction during delivery, but delivery still occurs even if its absent ADH deficiency also known as central diabetes insipidus

MTBS2CK p.109

MTBS2CK p.109

DiabetesInsipidus/Etiology Decrease in amount of ADH from pituitary (central DI) or its effect on kidney (nephrogenic DI) Central Diabetes Insipidus (CDI) Damage to brain:
Stroke Tumor Trauma Hypoxia Infiltration (sarcoidosis, hemochromatosis) Infection

DiabetesInsipidus/Etiology

Nephrogenic DI (NDI): Chronic pyelonephritis Amyloidosis Myeloma Sickle cell disease Lithium Hypercalcemia or hypokalemia inhibits ADH effect

MTBS2CK p.109

MTBS2CK p.109

DiabetesInsipidus/Presentation Excessive thirst Extremely high-volume urine Volume depletion Severe Hypernatremia Neurological symptoms Confusion, C f i di disorientation, i t ti l lethargy, th and d eventually t ll seizures and coma Only when volume losses are unmatched by fluid intake

DiabetesInsipidus/DiagnosticTests

Urine osmolality: Low Urine sodium: Low Serum osmolality: High Urine volume: Enormous

MTBS2CK p.109

MTBS2CK p.110

DiabetesInsipidus/DiagnosticTests Difference between central and nephrogenic DI is:

DiabetesInsipidus/Treatment

Response to vasopressin
Central DI: Urine volume decrease & urine osmolality increase Nephrogenic DI: No effect of vasopressin use on urine volume or osmolality
MTBS2CK p.110

Central DI: Long-term vasopressin (desmopressin) Nephrogenic DI: 1. Correct the cause (hypokalemia or hypercalcemia) 2. Hydrochlorothiazide, NSAIDs, amiloride
MTBS2CK p.110

DiabetesInsipidus/Evaluation
High-volume urine, plus excessive thirst Volume depletion, plus hypernatremia

DiabetesInsipidus/Evaluation
Vasopressin (Desmopressin) stimulation test
Urine: Volume decrease + osmolality increase Urine: No effect

Serum: Osmolality: Elevated Sodium: Elevated

Urine: Volume: HIGH Osmolality: Decreased Sodium: Decreased

Effect

Diagnosis

Central diabetes insipidus Vasopressin

Nephrogenic diabetes insipidus

Proceed with vasopressin (desmopressin) stimulation test


MTBS2CK p.109

Treatment

Treat underlying cause, hydrochlorothiazide, amiloride, NSAIDs

MTBS2CK p.110

Acromegaly

Acromegaly/Etiology

Soft tissue overgrowth throughout the body

Pituitary adenoma Part of Multiple Endocrine Neoplasias (MEN) Combined with parathyroid and pancreatic disorders (e (e.g., g gastrinoma or insulinoma) Rarely caused by ectopic GH or GHRH production
MTBS2CK p.110

MTBS2CK p.110

Copyright Richard Usatine, M.D. Used with permission.

Acromegaly/Presentation
Increased hat, ring, and shoe size Carpal tunnel syndrome Obstructive sleep apnea from soft tissues enlarging Body odor from sweat gland hypertrophy Teeth widening from jaw growth Deep voice and macroglossia (big tongue)

Acromegaly/DiagnosticTests Colonic polyps Arthralgias from joints growing out of alignment Hypertension for unclear reasons in 50% Cardiomegaly, CHF, and erectile dysfunction from increased prolactin cosecreted with pituitary adenoma Hyperglycemia Glucose intolerance Hyperlipidemia Best initial test... Insulin-like growth factor (IGF) M t accurate Most t t test t... Glucose suppression test Normal: Glucose should suppress growth hormone MRI? Only after the laboratory identification of acromegaly
MTBS2CK p.111

MTBS2CK p.110111

Acromegaly/Treatment 1. Surgery
Transphenoidal resection of pituitary Cures 70%

Hyperprolactinemia/Etiology

Many causes not associated with pituitary adenoma Prolactin increases via: Cosecretion with GH in acromegaly g y Hypothyroidism with pathologically high TRH levels

2. Medications
Cabergoline: Dopamine agonist inhibits GH release Octreotide or lanreotide: Somatostatin inhibits GH release Pegvisomant: GH receptor antagonist

3. Radiotherapy
Only when not responsive to surgery or medications
MTBS2CK p.111 MTBS2CK p.111

Hyperprolactinemia/Etiology

Hyperprolactinemia/Presentation

Pregnancy Chest wall stimulation Cutting pituitary stalk Antipsychotic medications, tricyclic antidepressants, p , and SSRIs Methyldopa Metoclopromide Opioids

Women Galactorrhea Amenorrhea Infertility Men Erectile dysfunction Decreased libido Galactorrhea (very rare in men)
MTBS2CK p.111

MTBS2CK p.111

Hyperprolactinemia/DiagnosticTests After the prolactin level is found to be high, perform: Thyroid function tests Pregnancy test BUN/creatinine (kidney disease elevates prolactin) Liver function tests (cirrhosis elevates prolactin) MRI is done after... High prolactin level is confirmed AND Secondary causes like medications are excluded AND Patient is not pregnant
MTBS2CK p.112

Hyperprolactinemia/Treatment

1. Dopamine agonists
Cabergoline is better tolerated than bromocriptine

2 Transphenoidal surgery when NOT 2. responding to medications 3. Radiation is rarely needed


MTBS2CK p.112

Whattolookfor... Hypothyroidism Hyperthyroidism Tachycardia,palpitations, arrhythmia(Afib) Diarrhea(hyperdefecation) Weightloss Anxiety,nervousness, restlessness Hyperreflexia Heatintolerance Fever

ThyroidDisorders
Hypothyroidism Hyperthyroidism ThyroidNodules

Bradycardia Constipation Weightgain Fatigue,lethargy,coma Decreasedreflexes Coldintolerance Hypothermia(hairloss, edema)


MTBS2CK p.113

Hypothyroidism Etiology Most from failure of thyroid gland from burnt-out Hashimoto thyroiditis
Acute phase is rarely perceived

Hypothyroidism Diagnostic tests Best initial test for all thyroid disorders is...
TSH
TSH levels are markedly elevated if gland has failed

Dietary deficiency of iodine Amiodarone


What to look for... Hypothyroidism: Everything is SLOW! Except menstrual flow, which is increased
MTBS2CK p.112

If the TSH level is suppressed, measure...


T4

Treatment Replace thyroxine (synthroid)

MTBS2CK p.113

Hyperthyroidism/Findings

Myxedema

Diagnosis Gravesdisease

Uniquefeature

Eyeproptosis (2040%) Skinfindings(5%) Subacutethyroiditis Tenderthyroid Painlesssilent silent thyroiditis Nontender, Nontender normalP/E Involutedglandisnt Exogenousthyroid palpable hormoneuse HighTSHlevel Pituitaryadenoma

MTBS2CK p.113

MultinodularGoiter

ThyroidBruit

Hyperthyroidism/DiagnosticTests T4 (thyroxine) level Elevated in all forms of hyperthyroidism TSH level Pituitary release of TSH is inhibited in all forms EXCEPT... Pituitary adenomas, will have high TSH level Graves disease (unique features): Eye and skin abnormalities Elevated radioactive iodine uptake TSH receptor antibodies
MTBS2CK p.113114

Hyperthyroidism/Treatment Graves Ophthalmopathy Best initial therapy...


Steroids

For those unresponsive to steroids...


Radiation

What if unresponsive to other therapy...


Decompressive surgery
MTBS2CK p.114

Source: Jonathan Trobe, MD

Hyperthyroidism/Treatment

AcuteHyperthyroidism/ThyroidStorm

Diagnosis Gravesdisease Subacutethyroiditis y Painlesssilent thyroiditis Exogenousthyroid hormoneuse Pituitaryadenoma


MTBS2CK p.114

Treatment Radioactiveiodine Aspirin None Stopuse

Treatment: Propranolol Blocks target organ effect Inhibits peripheral conversion of T4T3 Thiourea drugs Methimazole and propylthiouracil Block hormone production

Surgery

MTBS2CK p.114

AcuteHyperthyroidism/ThyroidStorm

ThyroidNodules

Treatment: Iodinated contrast material Blocks peripheral conversion of T4 to T3 Blocks release of existing hormone Steroids (hydrocortisone) Role in treating hyperthyroidism? Radioactive iodine Ablates gland for permanent cure
MTBS2CK p.114

5% women 1% men 95% benign


Adenoma, colloid nodule, cyst

Thyroid nodules rarely hyperfunctioning

MTBS2CK p.114

ThyroidNodules/DiagnosticTests 46-year-old woman with a small mass on palpation of thyroid. No tenderness and otherwise asymptomatic. What is the most appropriate next step? a. Fine-needle aspiration Done if TFTs are normal b Radionuclide iodine uptake scan Determines etiology b. of hyperfunctionality c. T4 and TSH levels Guides biopsy d. Thyroid ultrasound e. Surgical removal (excisional biopsy)

Biopsy with a fine-needle aspirate if theres normal thyroid function (T4/TSH) Ultrasound or radionuclide scanning not required (tests cannot exclude cancer)

Follicular adenoma when you cant be sure


MTBS2CK p.115 MTBS2CK p.115

46-year-old woman with thyroid nodule and normal thyroid function testing has a biopsy showing indeterminate for follicular adenoma. What is the most appropriate next step?
g , determines extent a. Neck CT Doesnt make a diagnosis, a b. Surgical removal (excisional biopsy) c. Ultrasound Cant excluded cancer, still need biopsy d. Calcitonin levels Suggests extent of medullary

CalciumDisorders
Hypercalcemia Hyperparathyroidism Hypocalcemia

carcinoma only

MTBS2CK p.115

Hypercalcemia/Etiology

Hypercalcemia/Etiology

Vitamin D intoxication Sarcoidosis and other granulomatous diseases Thiazide diuretics Hyperthyroidism yp y Metastases to bone and multiple myeloma

Most common cause is...


Primary hyperparathyroidism (PTH) Most cases are asymptomatic Severe, acute symptomatic hypercalcemia a high prevalence of cancer

MTBS2CK p.116

MTBS2CK p.116

Hypercalcemia/Presentation

Hypercalcemia/Treatment

Acute, symptomatic hypercalcemia


Confusion, stupor, lethargy Constipation Bone lesions: Osteoporosis Renal: Nephrolithiasis, Nephrolithiasis DI, DI renal insufficiency Cardiovascular: Short QT syndrome

Acute hypercalcemia, treat with... Saline hydration at high volume Bisphosphonates: pamidronate, zoledronic acid Calcitonin Prednisone: ONLY for sarcoidosis and granulomatous disease
MTBS2CK p.116

MTBS2CK p.116

Hyperparathyroidism
75-year-old man with history of malignancy admitted with lethargy, confusion, and abdominal pain. Found to have a markedly elevated calcium level. After 3L normal saline and pamidronate, his calcium level is still markedly elevated the following day. What is the most appropriate next step in management? a Calcitonin a. Doesnt add to pamidronate b. Zolendronic acid c. Plicamycin Less efficacy than pamidronate. Always wrong d. Gallium Less efficacy than pamidronate. Always wrong e. Dialysis Not needed. Renal failure has low Ca++

Primary hyperparathyroidism: Solitary adenoma (80%85%) Hyperplasia of all 4 glands (15%20%) Parathyroid malignancy (1%)

f. Cinacalcet Inhibits PTH


MTBS2CK p.116117 MTBS2CK p.117

Hyperparathyroidism

Hyperparathyroidism

Elevation in calcium levels often asymptomatic When symptomatic: Osteoporosis Nephrolithiasis and renal insufficiency Muscle weakness Anorexia, nausea, vomiting, and abdominal pain Peptic ulcer disease (calcium stimulates gastrin)
MTBS2CK p.117

Besides high calcium and PTH levels, you also find: Low phosphate level Short QT on EKG Sometimes an elevated BUN and creatinine Alkaline phosphatase elevated from effect of PTH on bone

MTBS2CK p.117

Hyperparathyroidism/Management

Hyperparathyroidism/Treatment

Bone X-ray is NOT a good test for bone effects of PTH DEXA densitometry is better Preoperative imaging of neck with sonography or nuclear scanning may be helpful in determining the surgical approach

Surgical removal of involved parathyroid glands

MTBS2CK p.117

MTBS2CK p.117 118

10

Hypocalcemia

Hypocalcemia/Presentation Neural hyperexcitability: Chvostek sign (facial nerve hyperexcitability) Carpopedal spasm Perioral numbness Mental irritability Seizures Trousseau sign (tetany)

Low calcium = twitchy, hyperexcitable High calcium = lethargic, slow

Souce: nih.gov

MTBS2CK p.118

MTBS2CK p.118

Hypocalcemia/Diagnosis&Treatment EKG: Prolonged QT May cause arrhythmia


Ventricular tachycardia

AdrenalDisorders
Hypercortisolism Hyperparathyroidism Hypocalcemia

Treatment Replace calcium and vitamin D

MTBS2CK p.118

PituitaryAdrenalAxis

Hypercortisolism Cushing disease Pituitary overproduction of ACTH Cushings syndrome Due to ectopic production of ACTH
C Carcinoid i id ( (most t common i is small ll cell ll carcinoma i of f the lung) Overproduction autonomously in adrenal gland

Prednisone (glucocorticoid use) causes same manifestations of hypercortisolism


Source:nih.gov

MTBS2CK p.119

11

Hypercortisolism/Etiology CauseofHypercortisolism PituitaryACTH (Cushingdisease) Adrenals Ectopic E i ACTH (carcinoid) UnknownsourceofACTH Frequency 70% 15% 10% 5%

Hypercortisolism/Presentation Fat redistribution


Moon face, truncal obesity, buffalo hump, thin extremities, increased abdominal fat

HTN
From increased sodium reabsorption in kidney and increased vascular reactivity

Skin
Striae, easy bruising, decreased wound healing, thinning of skin

Menstrual disorders Erectile dysfunction Polyuria


From hyperglycemia and increased free water clearance

Osteoporosis

MTBS2CK p.119

MTBS2CK p.119

Hypercortisolism/DiagnosticTests Always confirm the source of hypercortisolism with biochemical tests before you perform imaging studies

Hypercortisolism/DiagnosticEvaluation
CC: I feel weak and tired, and I notice hair growth on my face and strange marks on my stomach Low-dose (1mg) dexamethasone suppression test
Decreased = Disease Excluded

10% of population has an abnormality of pituitary on MRI If you start with a scan, you may remove the pituitary when the source is in the adrenals

24-hour urine cortisol Increased

Late-night salivary cortisol Increased

High?
ACTH-dependent Cushings syndrome

Serum ACTH

Low?
ACTH-independent Cushings syndrome

MTBS2CK p.120

MTBS2CK p.119120

Hypercortisolism/DiagnosticEvaluation
ACTH-dependent Cushings syndrome
Pituitary vs. Ectopic ACTH production? High-dose dexamethasone suppression i t test t

ACTH-independent Cushings syndrome


Adrenal Mass? CT Adrenals Increased ACTH & cortisol? Pituitary Mass? Pituitary MRI No mass seen? Petrosal sinus sampling for ACTH

ConfirmatoryLaboratoryFindingsin AdrenalDisorders
Adrenal ACTHlevel Low Pituitary High Ectopic High

Ectopic ACTHsecreting tumor

Chest CT

Petrosalsinus ACTH

Notdone

High g ACTH

LowACTH

Supression of cortisol? Pituitary adenoma (Cushing disease)


MTBS2CK p.120

Highdose Nosuppression dexamethasone

Suppresses Cortisol

Nosuppression

MTBS2CK p.121

12

Hypercortisolism/OtherFindings Cortisol - anti-insulin stress hormone Hyperglycemia Hyperlipidemia Theres some aldosterone-like effect of cortisol Hypokalemia H k l i Metabolic alkalosis Leukocytosis from demargination of WBCs

Hypercortisolism/Treatment

Surgically remove source of hypercortisolism


Transsphenoidal surgery for pituitary sources Laparoscopic removal for adrenal sources

MTBS2CK p.120

MTBS2CK p.121

EvaluationofAdrenalIncidentaloma How far should you go in the evaluation of an unexpected, asymptomatic adrenal lesion found on CT? 1. Metanephrines of blood or urine to exclude pheochromocytoma 2 Renin and aldosterone levels to exclude 2. hyperaldosteronism 3. 1 mg overnight dexamethasone suppression test

Hypoadrenalism(Addisons)/Etiology Addisons disease Chronic hypoadrenalism Etiology


Autoimmune destruction ( (80%) ) Infection (TB) Adrenoleukodystrophy Metastatic cancer to adrenal gland

Adrenal crisis Acute adrenal insufficiency Etiology


Hemorrhage, surgery, hypotension, yp , trauma Suddenly stopping chronic high-dose prednisone

MTBS2CK p.121

MTBS2CK p.121

Hypoadrenalism/Presentation Weakness, fatigue Acute adrenal crisis can also present with Altered mental status profound hypotension, Nausea, vomiting, fever, confusion, and anorexia, hypotension coma Hyperpigmentation from chronic adrenal insufficiency

Hypoadrenalism/DiagnosticTests

Pituitary failure: Hypoglycemia ACTH is low Hyperkalemia Metabolic acidosis Adrenal failure: Hyponatremia ACTH is high High BUN

Eosinophilia is common in hypoadrenalism


MTBS2CK p.121122
CopyrightRichardUsatine,MD Usedwithpermission.

MTBS2CK p.122

13

Hypoadrenalism
Signs & Symptoms Weakness Hypotension Weight Loss Hyperpigmentation

Hypoadrenalism/Treatment

Treatment is more important than testing in acute adrenal crisis Replace p steroids with hydrocortisone y Fludrocortisone
For postural instability A steroid hormone particularly high in mineralocorticoid or aldosterone-like effect
MTBS2CK p.122

Cosyntropin Stimulation Test Plasma cortisol before & after 250 ug cosyntropin IM or IV

Cortisol fails to rise


High ACTH Aldosterone Low too Low ACTH Aldosterone an increase

Primary adrenal insufficiency


MTBS2CK p.121122

Secondary adrenal insufficiency Adrenal atrophy from pituitary insufficiency

PrimaryHyperaldosteronism
Patient brought to ED after sustaining severe abdominal trauma in MVA. On second hospital day, he becomes markedly hypotensive without evidence of bleeding. Theres fever, high eosinophil count, hyperkalemia, hyponatremia, and hypoglycemia. What is the next step?

The autonomous overproduction of aldosterone despite a high pressure with a low renin activity

a. b. c. d. e.

Norepinephrine will constrict CT scan of the adrenals vessels more effectively Draw cortisol level and administer hydrocortisone Treating severe hypotension Cosyntropin stimulation testing more important than finding ACTH level etiology Dexamethasone suppression testing

Solitary adenoma: 80% Bilateral hyperplasia:15-20% Malignant: 1%

MTBS2CK p.122123

MTBS2CK p.123

PrimaryHyperaldosteronism All forms of secondary HTN are more likely in those whose onset is...
< 30 or > 60 years Uncontrolled by 2 antihypertensive medications

PrimaryHyperaldosteronism/DiagnosticTests Best initial test... Plasma aldosterone to plasma renin ratio A low plasma renin with high aldosterone = Primary hyperaldosteronism Most accurate test... Adrenal venous blood sampling high aldosterone! CT Scan? Only after laboratory testing reveals...
Low potassium, low plasma renin, and high aldosterone despite a high-salt diet
MTBS2CK p.123

Has a characteristic finding on the history, physical, or labs Primary hyperaldosteronism = High BP + Low K+

MTBS2CK p.123

14

PrimaryHyperaldosteronism/Treatment Unilateral adenoma Resected by laparoscopy Bilateral hyperplasia Eplerenone or spironolactone Spironolactone causes... Gynecomastia Decreased libido Anti-androgenic
MTBS2CK p.124

Pheochromocytoma Nonmalignant lesion of adrenal medulla Autonomous overproduction of catecholamines despite high BP Pheochromocytoma is the answer when theres: Episodic HTN Headache Sweating Palpitations and tremor

MTBS2CK p.124

Pheochromocytoma/DiagnosticTests Best initial test... Plasma catecholamines Confirmed with... 24-hour urine metanephrines and catecholamines
More accurate than VMA level

Pheochromocytoma/DiagnosticTests Imaging of adrenal glands (CT or MRI) Done only after biochemical testing MIBG scanning Nuclear isotope scan Detects D t t location l ti of f pheochromocytoma h h t that th t originates outside adrenal gland

MTBS2CK p.124

MTBS2CK p.124

Pheochromocytoma/DiagnosticTests
MIBG scan showing unilateral pheochromocytoma

Pheochromocytoma/Treatment

1. 2. 3. 4.

Phenoxybenzamine: (IV alpha blocker) Propranolol Calcium-channel blocker (possible) Laparoscopic removal

Source:LauraNModzelewski

MTBS2CK p.124

15

DiabetesMellitus(DM)

DiabetesMellitus
Presentation Diagnosis Treatment DiabeticKetoacidosis HealthMaintenance Complications

Persistently fastingbloodglucoselevels>125 onatleast2separateoccasions


Type 1 DM
Onset in childhood Insulin dependent from early age Not related to obesity Insulin deficiency

Type 2 DM
Onset in adulthood Directly related to obesity Insulin resistance

MTBS2CK p.124125

DiabetesMellitus/Presentation

DiabetesMellitus/DiagnosticTests

Polyuria, polyphagia, and polydipsia Type 1 DM: thinner than Type 2 diabetics Type 2 DM: more resistant to diabetic ketoacidosis (DKA) Both types: wound healing

2 FBG measurements > 125 mg/dL One glucose level about 200 mg/dL with symptoms Abnormal oral glucose tolerance testing Hemoglobin A1c > 6.5%

MTBS2CK p.125

MTBS2CK p.125

DiabetesMellitus/Treatment

DiabetesMellitus/Treatment Oral Hypoglycemic Medications Best initial drug: metformin Metformin blocks gluconeogenesis Sulfonylureas insulin release from pancreas weight gain Dipeptidyl Peptidase Inhibitors Increase insulin Decrease glucagon Sitagliptin, Saxagliptin, Linagliptin
MTBS2CK p.125

Best initial treatment... Diet, exercise, and weight loss Weight loss controls as much as 25% of cases of type yp 2 DM without medications, , Exercising muscle doesnt need insulin

MTBS2CK p.125

16

DiabetesMellitus/Treatment

DiabetesMellitus/Treatment

Thiazoladinediones (glitazones) Rosiglitazone, Pioglitazone Relatively contraindicated in CHF Increase fluid overload Nateglinide and repaglinide Stimulates insulin release Similar to sulfonylureas No additional therapeutic benefit to sulfonylureas
MTBS2CK p.125126

Incretins (exenatide, liraglutide) Raise insulin Decrease glucagon levels Decrease gastric motility Helps in weight loss

MTBS2CK p.126

DiabetesMellitus/Treatment Alpha-glucosidase inhibitors (acarbose, miglitol) Block glucose absorption in bowel Cause flatus, diarrhea, and abdominal pain Pramlintide Analog A l of f protein t i called ll d amylin li thats th t secreted t d normally with insulin Amylin
decreases gastric emptying decreases glucagon levels decreases appetite
MTBS2CK p.125126

DiabetesMellitus/Treatment Insulin Added if patient isnt controlled with oral hypoglycemic agents Insulin glargine gives steady state of insulin for entire day Dosing isnt isn t tested Glargine provides much more steady blood levels than NPH insulin Combined with short-acting insulin (e.g., lispro, aspart, or glulisine)

MTBS2CK p.126

DiabetesMellitus

DiabetesMellitus/Treatment Pharmacokinetics of insulin formulations


Drug Aspart Glulisine Lispro Regular NPH Lente Ultralente Glargine Type Rapid-acting Onset (hr) 0.20.5 Peak (hr) 0.52 Duration (hr) 34

Short-acting Intermediate Intermediate Long-acting Long-acting

0.51 1.5 1.53 34

23 410 715 915 No peak

68 1624 1624 2228 2436

MTBS2CK p.126

MTBS2CK p.126

17

DiabeticKetoacidosis(DKA) More common with type 1 diabetes Can definitely present in type 2 diabetes Presents with... Hyperventilation Altered mental status Metabolic M t b li acidosis id i with ith i increased d anion i gap Hyperkalemia in blood, but decreased total body potassium because of urinary spillage Increased anion gap on blood testing Serum is positive for ketones
MTBS2CK p.126

DiabeticKetoacidosis/Treatment

Treat with... 1. Large-volume saline and insulin replacement 2. Replace potassium when potassium level approaches normal 3. Correct the underlying y g cause
Noncompliance with medications Infection Any serious illness

MTBS2CK p.127

DiabetesMellitus/HealthMaintenance 57-year-old man admitted to ICU with altered mental status, hyperventilation, and markedly elevated glucose level. Which of the following is the most accurate measure of the severity of his condition? a. Glucose level Can be elevated without DKA If very low, theres risk of death b. Serum bicarbonate Mean very little c. Urine ketones Not all blood ketones are detected d. Blood ketones e. pH level on blood gas Need to know whats

All patients with DM should receive... Pneumococcal vaccine Yearly eye exam to check for proliferative retinopathy, which needs laser therapy Statins to g get LDL < 100 mg/dL g ACEi or ARBs to get BP < 130/80 mmHg ACEi or ARB if urine tests positive for microalbuminuria Foot exam for neuropathy and ulcers
MTBS2CK p.127

responsible for pH level


MTBS2CK p.127

ComplicationsofDiabetes

ComplicationsofDiabetes

Gastroparesis Immobility of bowels Bloating, constipation Early satiety, vomiting Abdominal discomfort Treat with metoclopramide or erythromycin

Non-proliferative retinopathy Tighter control of glucose Aspirin doesnt help retinopathy Proliferative retinopathy Neovascularization and vitreous hemorrhages Treated with laser photocoagulation

MTBS2CK p.128

MTBS2CK p.128

18

ComplicationsofDiabetes

Neuropathy Decreased sensation in feet Main cause of skin ulcers Leads to osteomyelitis Treatment pain with Pregabalin Gabapentin Tricyclic antidepressants

MTBS2CK p.128

19

Autonomy Ethics
Dr.ConradFischer,MD AssociateProfessorofMedicine TouroCollegeofMedicine NewYorkCity AdvanceDirectives Minors Brain i Death h

Ethics Every human being of adult years and sound mind has the right to determine what shall be done with his own body; and a surgeon who performs an operation without his patients consent commits an assault, for which he is liable in damagesexcept in cases of emergency where the patient is unconscious and where h it i is necessary t to operate t b before f consent t can be obtained.
Justice Benjamin Cardozo, Schloendorff v. Society of New York Hospital, 211 NY 125, 105 NE 92 (1914)

Ethics

This states the premise underlying half the ethics questions on S2 CK of USMLE: 1. Autonomy 2. Adult 3 Capacity 3. C it t to understand d t d

MTBS2CK p.561

MTBS2CK p.561

Autonomy

Patients have sole right to determine what treatment they shall and shall not accept Autonomy beats Beneficence Beneficence: trying to do good for others Trying y g to help p not as important p as following g wishes Patients have the right to refuse treatments that are good for them if they dont want them.
MTBS2CK p.561

A man has an ugly house you offer to paint free in his favorite color. Everyone in neighborhood agrees the house is ugly & what you offer is clearly superior. He understands everything you are offering, including the clear benefit to him. The man Cost and benefit and the still refuses. common good arent as What do you do? important as the autonomy have to just do a. Honor the mans man s wishes: no paint job individuals what h t they th want t with ith their th i own property. b. Paint his house against his will c. Ask the neighborhood council to consent to the paint job d. Get a psychiatric evaluation on the man e. Get a court order to allow the paint job f. Ask his family for consent to the paint job g. Wait until he is out of town, then paint his house
MTBS2CK p.561562

AdvanceDirectives
Man comes to ED after MVA that causes a ruptured spleen. Hes fully conscious. He understands that hell die without splenectomy, and that hell live if he has the splenectomy. He refuses the repair and blood transfusion. Entire family including brother who is healthcare proxy and document completed only a few weeks ago clearly state, Everything possible should be done, including surgery. What do y you do? a. Honor his current wishes, no surgery b. Wait until he loses consciousness, then perform the surgery You must follow the last known wishes of the c. Psychiatric consult patient, even if they are verbal, and even if they contradict the written proxy. You cannot wait until d. Ethics committee his consciousness is lost, then go against his e. Emergency court order wishes. f. Follow what is written in the documented health-care proxy g. See if there is consensus from the family
MTBS2CK p.562

Tell caregivers parameters of care the patient wanted Agent = person designated by patient to carry out patients wishes Term Agent sometimes used interchangeably with healthcare proxy Healthcare proxy is written document outlining parameters of care Major problem with proxy is details of care often unclear
MTBS2CK p.562

AdvanceDirectives

AdvanceDirectives

Not helpful to just say, No heroic measures To be useful, document must specifically state, No intubation, no CPR, no chemotherapy, no dialysis Can also specifically state wishes about fluid and nutrition If proxy says, No NG tube, no artificial feeding, then its useful Proxy takes effect only when patient has lost capacity to make decisions
MTBS2CK p.561562

Order of Decision Making 1. Patient with capacity supersedes all else 2 Healthcare 2. H lth proxy: an agent t( (person) ) to carry out wishes

MTBS2CK p.563

AdvanceDirectives

AdvanceDirectives

Order of Decision Making 3. Living will


Document outlining patients wishes Stating, I never want dialysis is more valid than a family member or friend saying, From what I know about b t him, hi h he wouldnt ld t want t dialysis, di l i or He H t told ld me he never wants dialysis. Documentation!
Written living will with concrete statements I never want blood transfusion or chemotherapy is valid

Order of Decision Making 4. Persons clearly familiar with the patients wishes
Problem with documentation Difficult for friends to document they knew the patients wishes If case clearly states friend knows and can prove that she knew the patients wishes, then this is the plan of care thats followed

MTBS2CK p.563

MTBS2CK p.563

AdvanceDirectives

AdvanceDirectives

Order of Decision Making 5. Family


General order of decision making
1. 2 2. 3. 4. Spouse Adult children Parents Siblings

Ethics Committee
The answer when: 1. Patient has lost capacity to make decisions 2 Advance directive is missing or unclear 2. Critically important for medical futility E.g. when the patient or proxy asking for tests and treatments that may have no benefit
MTBS2CK p.563

Unlike life, USMLE S2 CK must be clear If family is split, then answer is:
Ethics committee or court order
MTBS2CK p.563

AdvanceDirectives

AdvanceDirectives

Court order is right answer when: Theres no advance directive and 1. Patient has no capacity 2. Family in disagreement
Like a house being g left equally q y to four children who cannot agree what to do with it

Psychiatric Evaluation of Patient The answer when... It is not clear if patient has capacity Question clearly states patient has capacity? Not necessary! Clearly delirious or psychotic? Not necessary!
MTBS2CK p.563

Caregivers want to withdraw care and ethics committee cannot reach a conclusion
MTBS2CK p.563

Minor

Minor

Minors do not have decision-making capacity Cannot consent to or refuse medical treatments Only parents or legal guardian can consent and refuse Exceptions are: Contraception Prenatal care Substance abuse treatment Sexually transmitted diseases (STDs) including HIV/AIDS
MTBS2CK p.564

Abortion States are split on parental notification laws Some require it, some dont Your Y answer is: i Tell the minor patient to notify her parents.

MTBS2CK p.564

BrainDeath

Brain death = death in our legal system If brain dead, you dont need consent to stop therapy such as mechanical ventilation or antibiotics Court order and ethics committee arent correct answers USMLE S2 CK will want you to discuss, educate, explain, and confer before everything else.
MTBS2CK p.564

Consent
DoNotResuscitateOrders PhysicianAssistedSuicide Euthanasia TerminalSedationand LawofDoubleEffect FutileCare OrganandTissueDonation

Consent Only adults can consent to procedures Each procedure needs individual consent Consent implied in emergency Person doing procedure must obtain consent Adverse effects of procedure must be explained to make consent valid Consequences of refusing procedure must be explained to make consent valid Pregnant women can refuse procedures and treatments for their unborn children Telephone consent is valid
MTBS2CK p.564

Consent

Patient signs consent for ovarian biopsy on left side. At surgery you find cancer of right side. What do you do? Wake patient up & obtain consent to remove ovary on right side.

MTBS2CK p.564

Consent Patient needs colonoscopy. Gastroenterologist asks you to obtain consent for procedure. What do you do? The gastroenterologist who will perform the procedure needs to obtain consent. Do you know all complications of the procedure and alternatives? If you dont explain the possibility of perforation because you are unfamiliar with it, the consent isnt valid If patients colon perforates and you didnt explain alternate procedures, the consent isnt valid
MTBS2CK p.564

DoNotResuscitateOrders

DNR orders refer only to withholding CPR They dont refer to withholding any other form of therapy

MTBS2CK p.565

DoNotResuscitateOrders

PhysicianAssistedSuicide

Patient with capacity consents to DNR before losing consciousness. She needs a surgical procedure, but the surgeon refuses because the patient is DNR. What do you do?
Perform the surgery DNR doesnt mean withholding antibiotics, chemotherapy, or surgery DNR means only that, if the patient dies, you wont attempt resuscitation
MTBS2CK p.565

Always a wrong answer!


This includes states in which its legal Ethical requirements for physicians supersede legality Physician Physician-assisted assisted suicide is administered by the patient, but this is still unethical for physicians
Physician ethics come before legal requirements. You cannot do something unethical even if its legal at the moment.
MTBS2CK p.565

Euthanasia

Physician-administered treatment intended to end or shorten patients life

TerminalSedationand LawofDoubleEffect

Always y wrong g

Is it acceptable to administer pain medication even if theres the possibility treatment shortens life? For example, its acceptable to give pain medications to a person with COPD who has metastatic cancer even if the only way to relieve pain is to give enough opiates that breathing may be impaired, causing the patient to die earlier
MTBS2CK p.565

MTBS2CK p.565

TerminalSedationand LawofDoubleEffect

FutileCare

The question is one of intent: If the meds are given with intent to relieve pain, and as an adverse effect they shorten life, its ethical If the primary intent is to shorten life, its unethical

Physician not obligated to render care thats futile even if the family or patient wants it If brain dead & family insists on continued mechanical ventilation ventilation, you are under no obligation to do so You are under no obligation to perform tests and treatments you consider worthless
MTBS2CK p.565

MTBS2CK p.565

OrganandTissueDonation

ConsentforOrganDonation

Payment for ORGANS is unacceptable Payment for RENEWABLE tissues (sperm & eggs) is acceptable

Only organ donor network should ask for consent for organs Ethical conflict of interest for physician to ask for consent for organ donation Organ donor network has fewer refusals than physician Organ donor cards give an indication of patients wishes, but family can refuse organ donation even if patient has organ donor card
MTBS2CK p.566

MTBS2CK p.565

Confidentiality

Physician Responsibilities
Confidentiality Doctor/PatientRelationship GiftsfromIndustry Abuse ImpairedDrivers ExecutionofPrisoners Torture

Patients right to confidentiality can be broken when theres danger to others STDs HIV/AIDS Airborne Ai b communicable i bl diseases di ( (e.g., tuberculosis) Court order demanding information

MTBS2CK p.566

Confidentiality

Right to confidentiality cannot be broken for: Employers Coworkers Government agencies Family Confidentiality is Friends important, but not as
important as protecting others from harm.
MTBS2CK p.566

Patient with HIV/AIDS has repeatedly refused to disclose his HIV status to his sexual partner. The partner accompanies the patient to the office visits and is in the waiting room. The patient insists you not tell the partner. What do you do?
a. b. c. d. Honor the patients wishes is not as important as protecting the health of the partner. Obtain a court order Consult the ethics committee Either the physician or the department of health can notify the partner
The confidentiality of the patient

MTBS2CK p.566

Confidentiality Woman comes to your office with valid identification from a law enforcement/government agency. She requests a copy of your patients medical records. What do you do? Provide health-related protected records to government agencies, including those from law enforcement, f t only l if: if
Valid warrant or subpoena from courts Otherwise violates the constitutional protection against illegal search and seizure of property This violates HIPAA, which protects health information

Confidentiality

HIV-positive healthcare workers do not have to disclose their status to their patients or their employers.

MTBS2CK p.566

MTBS2CK p.566

Doctor/PatientRelationship

GiftsfromIndustry

Physicians arent obligated to accept everyone coming to him or her as a patient You have the right to end the doctor/patient relationship but must give the patient sufficient time to obtain another caregiver Small S ll gifts ift f from patients ti t are acceptable t bl as long as they arent tied to a specific treatment request Romantic or sexual contact between patients and their current physicians is never acceptable
MTBS2CK p.566567

Never acceptable!
Even pens, penlights, pads, and cups are unacceptable Meals in direct association with educational activities arent considered gifts

MTBS2CK p.567

ElderAbuse

DomesticViolenceandSpousalAbuse

You can report elder abuse against the consent of patient Abused older adults may be too weak, fragile, or vulnerable to protect themselves Elder abuse is treated ethically like child abuse

Unlike child abuse, domestic abuse cannot be reported against the patients wishes You can report and intervene only with patients patient s consent

MTBS2CK p.567

MTBS2CK p.567

ImpairedDrivers (SeizureDisordersandDriving)

ExecutionofPrisoners

Least clear area nationally No uniformity of laws between states Answer suggest that the patient find another means of transportation Wrong answers would be:
a. Confiscating car keys and reporting to law enforcement b. Hospitalizing patient c. Refusing to let the patient get in car
MTBS2CK p.567

Never ethical for a physician to participate in executions at any level You cannot ethically formulate a lethal injection or even do so much as pronounce a prisoner dead Even if state law makes execution legal, physicians should never participate at any level

MTBS2CK p.567

Torture

Torture

Physicians are never to participate in torture of prisoners or detainees Even if the question states that youre in the military, your ethical obligation as a physician supersedes your obligation to the military ilit This would include:
a. Refusing orders from military superiors to participate in torture b. Keeping the torture safe so that its not fatal or damaging
MTBS2CK p.567568

Torture is the ethical equivalent of child abuse. Your p participation p is never acceptable; youre obligated only to report it.

MTBS2CK p.568

Esophagus Gastroenterology
NiketSonpal,MD
ChiefResident LenoxHillHospitalNSLIJ AssistantClinicalProfessor TouroCollegeofMedicine
Achalasia EsophagealCancer EsophagealSpasm Esophagitis RingsandWebs ZenkersDiverticulum Scleroderma MalloryWeissTear BoerhaavesSyndrome

EsophagealDisorders/Definitions

EsophagealDisorders/Presentation Both can lead to weight loss Hence, weight loss cannot be used to answer What is the most likely diagnosis? With the symptoms Weight loss Anemia Heme-positive stool

Dysphagia Difficulty swallowing Odynophagia Pain while swallowing

Endoscopy!

MTBS2CK p.237

MTBS2CK p.237

Achalasia/Diagnosis

Achalasia/Diagnosis

Inability of lower esophageal sphincter (LES) to relax due to a loss of nerve plexus within lower esophagus Etiology unclear Aperistalsis of esophageal body

Look for... Young patient (< 50) Progressive worsening dysphagia to both solids and liquids at the same time No association with alcohol and tobacco use Complain of...
Regurgitation and halitosis

Chagas Disease = recent travel to South America and new onset dysphagia
MTBS2CK p.237 MTBS2CK p.237

Achalasia/DiagnosticTests

MarkedDilationoftheEsophagus

Barium esophagram Will show a birds beak

Source:commons.wikimedia.com

Source:NiketSonpal,MD

MTBS2CK p.238

Achalasia/DiagnosticTests

Achalasia/DiagnosticTests

Most accurate test is Manometry

Chest X-ray May show abnormal widening of esophagus, but is neither very sensitive nor very specific

Chest X-ray is never the right answer to diagnose achalasia

Source:NiketSonpal,MD

MTBS2CK p.238

MTBS2CK p.238

Achalasia/DiagnosticTests

Achalasia/Treatment

Upper endoscopy Shows normal mucosa in achalasia

Cannot exactly be cured All treatment is based on simple mechanical dilation of esophagus

Source:NiketSonpal,MD

MTBS2CK p.238

MTBS2CK p.238

Achalasia/Treatment

Achalasia/Treatment

Pneumatic dilation

Effective > 80% to 85% of patients Repeat sessions y are necessary

Botulinum toxin injection Will relax LES, but effects wear off in about 3-6 months, requiring reinjection
Site of Botulinum Injection

Source:commons.wikimedia.com

Freedictionary.com

MTBS2CK p.238

MTBS2CK p.238

Achalasia/Treatment

EsophagealCancer/Diagnosis

Surgical sectioning or myotomy can help to alleviate symptoms Known as Heller M t Myotomy

Look for: Age 50 or older Dysphagia with solids first then progresses to liquids Association with p prolonged g alcohol and tobacco use > 5 years of GERD symptoms

NiketSonpalMD

MTBS2CK p.238

MTBS2CK p.239

EsophagealCancer/DiagnosticTests

EsophagealCancer/DiagnosticTests

Endoscopy is indispensible, since only a biopsy can diagnose cancer NO TISSUE, NO ISSUE
Barium cannot diagnose cancer

CT and MRI scans Only shows extent of tissue PET scan Gives information about other anatomic lesions

Source:commons.wikimedia.com

MTBS2CK p.239

MTBS2CK p.239

EsophagealCancer/Treatment

EsophagealCancer/Treatment

Surgical resection is always the thing to try No resection (removal) = no cure Chemotherapy and radiation

Stent placement Purely palliative

Source:NiketSonpal,MD

MTBS2CK p.239

MTBS2CK p.239

EsophagealSpasm/Background

EsophagealSpasm/Diagnosis

Diffuse esophageal spasm (DES) and nutcracker esophagus are clinically indistinguishable Both present
Sudden onset of chest pain Not related to exertion

EKG and stress


Normal

Esophagram best initial test


Normal

Manometry y most accurate test


Abnormal contraction in various section of the esophagus

MTBS2CK p.239

MTBS2CK p.239

EsophagealSpasm/Treatment

EsophagealSpasm/BariumStudy

Treated with: Nitrates


Relax smooth muscle

Calcium-channel blockers No Ca+2= no smooth muscle contraction

MTBS2CK p.240

MTBS2CK p.240

InfectiousEsophagitis/Management
43-year-old man recently diagnosed with AIDS comes to ED with pain on swallowing thats become progressively worse over the last several weeks. Theres no pain when not swallowing. His CD4 count is 43 mm3. The patient isnt currently taking any medications. What is the most appropriate next step in management? Doesnt t diagnose candida a Esophagram Doesn a. b. Upper endoscopy Too invasive c. Oral nystatin swish and swallow Only for oral candida
Dysphagia with HIV CD4 < 100

Yes
Empirically start fluconazole

Yes

Improvement

No
Perform upper endoscopy with biopsy

d. Intravenous amphotericin Too big gun e. Oral fluconazole

Continue therapy and HAART

CMV large ulcerations. Tx: ganciclovir or foscarnet

HSV small ulcerations. Tx: acyclovir

MTBS2CK p.240

InfectiousEsophagitis

RingsandWebs Schatzki ring Schatzki rings and Plummer-Vinson syndrome both cause dysphagia

Source:AaronCho Source:commons.wikimedia.com

MTBS2CK p.241

RingsandWebs Schatzki ring Associated with intermittent dysphagia

RingsandWebs Plummer-Vinson syndrome Triad of: 1. Dysphagia due to Esophageal Webs 2. Iron Deficiency Anemia 3. Glossitis More M proximal i l than th Schatzki S h t ki rings i Diagnosis
Barium esophagram

Treatment
Iron replacement
MTBS2CK p.241
Source:AzmeenaLaila,MD

MTBS2CK p.241

ZenkersDiverticulum Outpocketing of posterior pharyngeal constrictor muscles

ZenkersDiverticulum
Best test for diagnosis is
Esophogram

No medical treatment and surgical intervention is best

StanfordHospitalMedia

Source:commons.wikimedia.com

MTBS2CK p.241

MTBS2CK p.241

Scleroderma

MalloryWeissSyndrome/Presentation Upper GI bleed secondary to repetitive retching Self-limited Dx: endoscopy

Presentation
Symptoms of reflux Scleroderma or progressive systemic sclerosis

Diagnosis
Manometry

Treatment
PPIs Screening for Barretts esophagus
Source:commons.wikimedia.com

MTBS2CK p.242

MTBS2CK p.242

MalloryWeissSyndrome/Treatment

BoerhaavesSyndrome

No specific therapy and will resolve spontaneously Severe cases with persistent bleeding are managed with injection of epinephrine or electrocautery to stop bleeding

Esophageal rupture due to prolonged retching


A full thickness tear

Physical exam
Hammens sign crepitus Subcutaneous air = snap crackle and pop

Source:commons.wikimedia.com

EMERGENCY!
MTBS2CK p.242

Stomach
EpigastricPain GastroesophagealRefluxDisease BarrettsEsophagus Gastritis PepticUlcerDisease NonulcerDyspepsia Gastrinoma DiabeticGastroparesis

44-year-old woman comes to see you because of epigastric pain for several months. She denies nausea, vomiting, weight loss, or blood in her stool. On physical examination you find no abnormalities. What is the most likely diagnosis?
a. b. c. d. e. f. Duodenal ulcer disease No anemia or heme (+) stools Gastric ulcer disease Heme (+) stools Gastritis Need EGD to diagnose Pancreatitis Acute not chronic Non-ulcer dyspepsia Pancreatic cancer No painless jaundice

MTBS2CK p.242243

AbdominalPain/CausesofPainbyLocation
Right Upper Quadrant Cholecystitis Biliary colic Cholangitis Perforated duodenal ulcer Right Lower Quadrant Appendicitis Ovarian torsion Ectopic pregnancy Cecal diverticulitis Left Upper Quadrant Splenic rupture IBS Splenic flexure syndrome

EpigastricPain/MostLikelyDiagnosis Ifthisisinthehistory: Themostlikelydiagnosisis:

Mid-Epigastrium Pancreatitis Aortic dissection Peptic ulcer disease

Left Lower Quadrant Sigmoid volvulus Sigmoid diverticulitis Ovarian torsion Ectopic pregnancy

Painworsewithfood Painbetterwithfood Weightloss Tenderness Badtaste,cough,hoarse Diabetes,bloating Nothing

Gastriculcer Duodenalulcer Cancer,gastriculcer Pancreatitis Gatroesophageal reflux Gastroparesis Nonulcerdyspepsia

MTBS2CK p.243

EpigastricPain/DiagnosticTests Endoscopy Only way to truly understand the etiology of epigastric pain from ulcer disease Only y way y to g give a precise diagnosis

EpigastricPain/Treatment Proton pump inhibitors (PPIs) First line therapy Empiric Minimum 4 weeks H2 blockers Ranitidine, nizatidine, cimetidine, famotidine Not as effective, but will work in about 70% of patients
MTBS2CK p.243

Liquid antacids Roughly the same efficacy as H2 blockers Misoprostol Artificial prostaglandin analogue Used to treat NSAIDinduced gastric damage When PPIs arrived, misoprostol became wrong answer

Wikimedia

MTBS2CK p.243

GastroesophagealRefluxDisease(GERD) Inappropriate relaxation of LES Results in acid contents of stomach coming up into esophagus Patient complains of... Heartburn Metallic taste Cough 42-year-old man comes to office with epigastric pain radiating up under his chest, which becomes worse after lying flat for an hour. He also has a brackish taste in his mouth. What is the most appropriate next step in the management of this patient?

a. Ranitidine Not as effective b. Liquid antacid Not as effective c. Lansoprazole Not the first step must fail 1st d. Endoscopy e. Barium swallow Not an anatomic disease f. 24-hour pH monitoring Too invasive
MTBS2CK p.244

MTBS2CK p.244

GERD/DiagnosticTests Most often diagnosed using patient history When not clear, the most accurate test is...
24-hour pH monitoring

GERD/Treatment

All patients should try lifestyle changes


Lose weight, avoid alcohol, nicotine and certain foods, eat within 3 hours of bedtime Elevate head 6-8 inches

Endoscopy when: Dysphagia or odynophagia Weight loss Anemia or heme-positive stools > 5 years of symptoms to exclude Barretts esophagus
MTBS2CK p.244245

Mild or Intermittent Symptoms May be treated with liquid antacids or H2 blockers Persistent Symptoms or Erosive Esophagitis PPIs for 4-6 weeks
MTBS2CK p.245

GERD/Treatment

BarrettsEsophagus

5% will not respond to PPIs Patients require surgery to tighten LES:

Nissen fundoplication
Stomach wrapped around LES

Due to long-standing GERD Change to columnar metaplasia Typically takes 5 years

Endocinch
Scope used to place a suture around LES

Local heat or radiation of LES


Causes scarring to tighten LES Last resort
MTBS2CK p.245

Diagnosis Biopsy via endoscopy


Only way to assess presence of Barretts esophagus
MTBS2CK p.245

BarrettsEsophagus/Treatment

Gastritis Inflammation or erosion of gastric lining Sometimes called gastropathy


3 levels: mild, moderate and severe With or without hemorrhage

Finding Barrettsalone (metaplasia) Lowgradedysplasia Highgradedysplasia

Management PPIsandrescope every2 3years PPIsandrescope every 612months Ablationwithendoscopy, photodynamictherapy, radiofrequencyablation, orsurgicalremoval

Many causes
Alcohol NSAIDs Helicobacter pylori Portal HTN Stress
Burns, trauma, sepsis, multiorgan failure

MTBS2CK p.246

MTBS2CK p.246

Gastritis/Presentation

Gastritis/Diagnosis

Often with GI bleeding without pain Severe, erosive gastritis can present with epigastric pain Look for NSAIDs or alcoholism in history No unique q p physical y findings g
You cannot answer the most likely diagnosis question from history and physical alone

The most accurate test is... EGD H. py pylori testing g


Treated if associated with gastritis

Capsule endoscopy is not appropriate for upper GI bleeding if endoscopy is one of the choices

MTBS2CK p.246

MTBS2CK p.246

Source: commons.wikimedia.org

TestingforHelicobacterpylori Test
Whatisgood aboutthistest? Whatisbad aboutthistest?

Gastritis/Treatment
Invasivetest (endoscopy) Lacksspecificity, cantdistinguish current/previous infection Requiresexpensive equipment Requiresstool sample
MTBS2CK p.247

Endoscopic biopsy Serology

Mostaccurateof alltests Inexpensive, excludesinfection ifnegative

Treat with PPIs H2 blockers, sucralfate, and liquid antacids arent effective as PPIs

UreaC13 orC14 Positiveonlyin breathtesting activeinfection,


noninvasive

Sucralfate is an inert substance (aluminum hydroxide complex) that coats the stomach. If sucralfate is presented as a choice, its nearly always the wrong answer.

H.Pylori stoolantigen
MTBS2CK p.246247

Positiveonlyin activeinfection, noninvasive

Gastritis

PepticUlcerDisease(PUD)

Stress ulcer prophylaxis is indicated in: Mechanical ventilation Burns Curlings ulcers Head trauma Cushings ulcers Coagulopathy

PUD refers to both duodenal ulcer and gastric ulcer disease Endoscopy is key to diagnosis and treatment MCCs H. pylori and NSAIDs
Note: Alcohol and tobacco dont cause ulcers. They delay healing of ulcers

MTBS2CK p.247

MTBS2CK p.247248

PepticUlcerDisease

PepticUlcerDisease/Presentation

PUD presents with recurrent episodes of epigastric pain


Abdominal Pain (dull, sore, gnawing) Pain that is improved with eating Pain that is worsened by eating

Duodenal Ulcer

Gastric Ulcer

Source: commons.wikimedia.org

Source: commons.wikimedia.org

MTBS2CK p.248

PepticUlcerDisease/Diagnosis

PepticUlcerDisease/Treatment

Upper endoscopy: most accurate test Allows for intervention and biopsy
Ulcer on endoscopy Biopsy positive for H. pylori Bleeding ulcer

PUD responds to PPIs...


But must treat for H. pylori
Duodenal Ulcer Gastric Ulcer

H. pylori in more than 80% to 90% of cases


Antibiotic treatment Clip or epinephrine injection

H. pylori in 50% to 70% of cases

Radiologic Testing
Poor sensitivity and no histology testing
MTBS2CK p.248

Cancer is present in 4% of those with GU but in none of those with DU.


MTBS2CK p.248249

10

PepticUlcerDisease/Treatment

PepticUlcerDisease/Treatment
56-year-old woman comes to clinic with epigastric pain from endoscopically confirmed duodenal ulcer, which is unresponsive to several weeks of PPI, clarithromycin, and amoxicillin. What is the most appropriate next step?
PCN allergy?

H. pylori Treatment
Biopsy positive for H. Pylori

PPI combined with clarithromycin and amoxicillin

Replace amoxicillin with metronidazole

30-60 day post therapy testing for eradication

a. b. c. d. e. f. g.

Refer for surgery g y Too invasive Switch the PPI to ranitidine PPI is superior to ranitidine Abdominal CT scan CT cant detect H. pylori Capsule endoscopy Cant detect H. pylori Urea breath testing Vagotomy Not necessary and too invasive Add sucralfate Always the wrong answer

MTBS2CK p.243

MTBS2CK p.249

PepticUlcerDisease/Treatment Treatment of Refractory Ulcers If initial therapy doesnt resolve the DU then detecting persistent H. pylori and switching the antibiotics to metronidazole and tetracycline is appropriate or adding bismuth For those with refractory y GU, , a repeat p endoscopy is done to exclude cancer

GastricUlcersvs.DuodenalUlcers

GU is routinely biopsied
Routinely repeating the endoscopy to confirm healing is standard with GU

GU vs. DU

GU is associated with cancer in 4%

GU pain is more often worsened by food

MTBS2CK p.249

MTBS2CK p.250

NonulcerDyspepsia

NonulcerDyspepsia The best initial therapy is with PPIs


NUD < 45-55 years NUD > 45-55 years

Non-ulcer (functional) dyspepsia is epigastric pain that has no identified etiology MCC of epigastric pain in U.S.

Empiric PPI therapy

PPIs and EGD to r/o cancer

Symptoms persist and H. Pylori is present = treat for H. pylori


MTBS2CK p.250 MTBS2CK p.250

11

Gastrinoma(ZollingerEllisonSyndrome) Look for patient with ulcers that are: 1. Large > 1 cm 2. Multiple 3. Past 3rd portion of duodenum 4. Presents with: Diarrhea Abdominal pain Anemia Heme-positive stools
MTBS2CK p.251

Gastrinoma/DiagnosticTests
After endoscopy confirms an ulcer, diagnosis made using one of the following three: 1. High gastrin levels after antisecretory therapy (PPIs/H2) 2. High gastrin levels despite a high gastric acid output 3. Persistent high gastrin levels despite injecting secretin (most accurate)

Source:commons.wikimedia.org

MTBS2CK p.251

Source:NiketSonpalMD

Gastrinoma/DiagnosticTests What is the next best step after diagnosing ZE? Exclude metastatic disease
Somatostatin receptor scintigraphy + Endoscopic ultrasound

Gastrinoma/Treatment

Local disease is removed surgically Metastatic disease requires lifelong PPIs to block acid production

Ultrasound, CT, and MRI of abdomen have poor sensitivity and are never the most accurate test for diagnosing ZE
MTBS2CK p.251 MTBS2CK p.251

DiabeticGastroparesis
64-year-old patient with diabetes for 20 years comes to the office with several months of abdominal fullness, intermittent nausea, constipation, and a sense of bloating. On physical examination, a splash is heard over the stomach. What is the next step? a. Abdominal CT scan Can only diagnose static conditions b. Colonoscopy Wrong end! c. Erythromycin d. Upper endoscopy Wouldnt reveal anything e. Nuclear gastric emptying study Most accurate test for
MTBS2CK p.252

Long standing diabetes

Autonomic neuropathy from high glycemic index

+
Resultant dysmotility from an inability to sense stretch in the GI tract

=
Diabetic gastroparesis
MTBS2CK p.252

Nausea, vomiting, and early satiety

diabetic gastroparesis, but rarely used

12

DiabeticGastroparesis
The patients most common complaint is abdominal discomfort with eating large or small meals Nausea & vomiting Bloating & constipation Early satiety
Upper and Lower Symptoms

Colon
GastrointestinalBleeding Diarrhea IrritableBowelSyndrome InflammatoryBowelDisease DiverticularDisorders ColonCancerScreening

Anorexia

Neurologic Symptoms

Endoscopy - large amount of retained food

Succussion splash

Signs elicited by exam

GastrointestinalBleeding
69-year-old woman comes to ED with multiple red/black stools. Past medical history significant for aortic stenosis. Pulse 115/ minute. BP 94/62 mmHg. Examination otherwise normal. What is the next step in management? a. b. c. d. e. f. g. h. Colonoscopy Nasogastric tube placement Upper endoscopy Patient is unstable, Bolus of normal saline Stablized first, THEN CBC diagnose Bolus of 5% dextrose in water Consult gastroenterology D5w doesnt stay intravascular Check for orthostasis

Most common: Ulcer disease

Esophagitis

Upper GI bl di bleeding

Varices

Gastritis Cancer
MTBS2CK p.253

Duodenitis

MTBS2CK p.252253

GastrointestinalBleeding
Most common: Diverticulosis

GastrointestinalBleeding/PhysicalFindings SeverityofBloodLossBasedonHemodynamics PhysicalFinding Orthostasis Pulse>100/minute SystolicBP<100mmHg Percentageofbloodloss 1520% 30% 30%

Polyps

Lower GI bl di bleeding

Hemorrhoids

Remember: orthostasis is defined as...


IBD Cancer
MTBS2CK p.253

UGIB

>10-point rise in pulse when going from supine to sitting or standing up or BP drop of 20 points or more when sitting up
MTBS2CK p.253

13

VaricealBleeding
Vomiting blood +/ black stool

GastrointestinalBleeding/DiagnosticTests

For acute severe bleeding


Replace fluids and check Hct, platelet count, and coagulation tests (PT or INR)

Cirrhosis

Variceal bleeding

Asterixis

For variceal bleeding


Octreotide and urgent endoscopy to control the bleed by banding

Spider angiomata and caput medusa


MTBS2CK p.253 254

Palmar erythema Splenomegaly

MTBS2CK p.254

AdditionalDiagnosticTestsforGIBleeding Test
Nuclearbleedingscan Angiography

GastrointestinalBleeding/Treatment

Indication
Endoscopyunrevealinginamassive acutehemorrhage Specificsiteofbleedingneeds tobeidentifiedpriortosurgeryor yin embolizationofvessel;usedonly massive,nonresponsivebleeding Smallbowelbleedingwhenupperand lowerendoscopydontshowetiology NotusefulinGIbleeding Showsischemiainseverebleeding

Stabilization Treatment
Fluid replacement
1-2 liters an hour

Packed RBCs
Hct < 30 in those who are older or suffer from CAD

Capsuleendoscopy CTorMRIofabdomen EKG

MTBS2CK p.254

MTBS2CK p.255

GastrointestinalBleeding/Treatment

GastrointestinalBleeding/Treatment

Stabilization Treatment
Fresh frozen plasma
INR > 1.3

Platelets
< 50,000 when bleeding

Treat Underlying Cause Endoscopy to determine diagnosis and administer treatment (band varices, cauterize ulcers, inject epinephrine into bleeding gastric vessels) IV PPI for upper GI bleeding Surgery to remove site of bleeding if fluids, blood, platelets, and plasma will not control bleed

MTBS2CK p.255

MTBS2CK p.255

14

GastrointestinalBleeding/Treatment

Diarrhea/Types
Lactose intolerance Carcinoid syndrome

Esophageal and Gastric Varices Octreotide (somatostatin) Banding Transjugular intrahepatic portosystemic shunting h ti (TIPS) Propranolol

Diarrhea

Antibioticassociated diarrhea

Chronic pancreatitis Malabsorption

MTBS2CK p.255

Diarrhea/AntibioticAssociated

Diarrhea/AntibioticAssociated Best initial test is...


Stool C. diff. toxin

Clindamycin associated with highest incidence of antibiotic-associated diarrhea and Clostridium difficile Blood and white cells may be present in stool Presents several days or weeks after start of antibiotics ANY antibiotic can potentially cause diarrhea

Most accurate test is...


C. diff. PCR

Best initial therapy is...


Metronidazole

If theres no response to metronidazole, next step in management is to...


Switch to oral vancomycin or fidaxomicin
IV vancomycin is always wrong for antibiotic-associated diarrhea since it will not pass bowel wall.

MTBS2CK p.255

MTBS2CK p.255 256

Diarrhea/AntibioticAssociated
75-year-old man is admitted to hospital with pneumonia. Several days after start of antibiotics, he has diarrhea. Stool C. diff toxin is positive, and hes started on metronidazole, which leads to resolution of diarrhea over a few days. Two weeks later diarrhea recurs and C. diff toxin is positive again. What is the next step?
a. b. c. d. e. Retreat with metronidazole orally Use vancomycin orally Repeat metronidazole before switching Sigmoidoscopy and treat only if pseudomembranes are found Not necessary Intravenous metronidazole Only if patient cannot take orally Wait for stool culture C. diff. doesnt grow in culture f. Intravenous vancomycin Repeat metronidazole before switching / IV vancomycin does not work
MTBS2CK p.256 Diarrhea after antibiotic Use Yes C. difficile positive? No Consider alternative causes Yes T t with Treat ith metronidazole No Switch to oral vancomycin or fidaxomicin

Improvement?

Yes Continue metronidazole until end of course

15

Diarrhea/Malabsorption All present with steatorrhea: Oily Greasy Stools Floating Foul smelling Causes: Celiac disease Whipples disease Chronic pancreatitis
MTBS2CK p.256

Diarrhea/Malabsorption All forms of fat malabsorption present with deficiency of fat-soluble vitamins (A, D, E, and K) Hence, they can all present with... Deficiency VitaminD VitaminK VitaminB12 Manifestation Hypocalcemia Bleeding,easybruising Anemia,hypersegmented neutrophils,neuropathy
Vitamin B12 needs an intact bowel wall and pancreatic enzymes to be absorbed
MTBS2CK p.256257

Diarrhea/DiagnosticTests Celiac disease Best initial test is...


Anti-tissue transglutaminase

Diarrhea/Malabsorption Whipples Disease Arthralgias Ocular findings Neurologic abnormalities (dementia, seizures) Fever Lymphadenopathy L h d th
Source:commons.wikimedia.org

Most accurate diagnostic test is...


Small bowel biopsy
Flattening of villi Can exclude lymphoma

Other tests IgA antigliadin antibody Antiendomysial antibody

MTBS2CK p.257

MTBS2CK p.257

Diarrhea/Malabsorption Whipples disease and tropical sprue Most accurate diagnostic test is...
Bowel wall biopsy showing specific organism Treat with ceftriaxone or TMP/SMZ

ChronicPancreatitis/DiagnosticTests Abdominal X-ray 50% to 60% sensitive for calcification of pancreas and very specific when test is abnormal

MTBS2CK p.257

Source:commons.wikimedia.org

MTBS2CK p.257258

16

ChronicPancreatitis/DiagnosticTests Abdominal CT scan 80% to 90% sensitive for pancreatic calcification

ChronicPancreatitis/DiagnosticTests Secretin stimulation testing Most accurate diagnostic test


Place NG tube; normal pancreas releases large volume of bicarbonate-rich fluids after IV secretin

D-xylose testing Old test to distinguish pancreatitis from bowel wall abnormalities D-xylose normal in pancreatic disorders

MTBS2CK p.257258

MTBS2CK p.257258

Malabsorption/Treatment Disease Chronicpancreatitis Celiacdisease SpecificTreatment Enzymereplacement Avoidglutencontaining foods(wheat,oats,rye, barley) Ceftriaxone,TMP/SMX TMP/SMX,tetracycline

CarcinoidSyndrome Presentation Flushing Wheezing CV murmurs tricuspid regurgitation Diarrhea Best initial diagnostic test is... Urinary 5-hydroxyindoleacetic acid (5-HIAA) test Treatment Octreotide

Whipplesdisease Tropicalsprue

MTBS2CK p.258

MTBS2CK p.258259

LactoseIntolerance Presents like malabsorptive diarrhea, except...


Intermittently No weight loss Normal vitamin levels

IrritableBowelSyndrome(IBS) Pain syndrome with either diarrhea, constipation, or both Pain of IBS is...
Relieved by BM Less at night Relieved by a change in bowel habit (e.g., diarrhea)

Best initial diagnostic test is...


Stool osmolality test

No specific diagnostic test


Diagnosis of exclusion in association with complex of symptoms

Most accurate test is...


Cessation of symptoms after diet change

Best treatment is...


Stop eating milk products and consider exogenous lactase therapy
MTBS2CK p.259

Not associated with...


Weight loss Blood or white cells in stool
MTBS2CK p.259

17

IrritableBowelSyndrome/Treatment

InflammatoryBowelDisease

Fiber in diet Antispasmodic agents


Hyoscyamine Dicyclomine

Tricyclic y antidepressants p
Amitriptyline

Crohns disease (CD) and ulcerative colitis (UC) present with... Diarrhea and abdominal pain Blood in stool anemia Weight g loss Fever

Antimotility agents
Loperamide for diarrhea Lubiprostone (chloride-channel activator)
Increases BM frequency
MTBS2CK p.259 MTBS2CK p.259

InflammatoryBowelDisease Extraintestinal manifestations in both CD and UC Arthralgias Uveitis, iritis Skin manifestation (erythema nodosum, pyoderma gangrenosum) Sclerosing cholangitis (more frequent in UC) Risk of cancer associated with CD and UC Both forms of IBD can lead to colon cancer Cancer risk with duration CD involving colon has same cancer risk as UC
MTBS2CK p.259260

InflammatoryBowelDisease ErythemaNodosum PyodermaGangrenosum

Source:commons.wikimedia.org Source:commons.wikimedia.org

InflammatoryBowelDisease/Differences

InflammatoryBowelDisease/DiagnosticTests Most accurate test is...


Endoscopy

Crohnsdisease
Skiplesions Transmuralgranulomas Fistulasandgranulomas Massesandobstruction Perianaldisease

Ulcerativecolitis
Curablebysurgery Entirelymucosal Nofistulas,noabscesses Noobstruction Noperianaldisease

Frequentquestion:Whenshouldscreeningoccur?
Source:commons.wikimedia.org Source:commons.wikimedia.org

Answer:After8to10yearsofcolonicinvolvement, withcolonoscopyevery1to2years.
MTBS2CK p.260

CD thats mainly in small bowel, radiologic tests such as barium studies will detect lesions
MTBS2CK p.260

18

InflammatoryBowelDisease/DiagnosticTests

InflammatoryBowelDisease/Treatment

ANCAandASCAResultsinIBD
Test
Antineutrophilcytoplasmic antibody(ANCA) Antisaccharomyces cerevesiaeantibody(ASCA)

Crohnsdisease Ulcerativecolitis
Negative Positive

Positive

Negative

Acute exacerbations of either CD or UC are treated with steroids Chronic maintenance of remission is with 5ASA derivatives (mesalamine) Asacol for UC Pentasa for CD Perianal CD
Ciprofloxacin and metronidazole

MTBS2CK p.260

MTBS2CK p.260261

InflammatoryBowelDisease/Treatment Azathioprine and 6-mercaptopurine are used to wean patients off steroids Fistulae and severe unresponsive disease Anti-tumor necrosis factor (TNF) agents (infliximab) Surgery only if theres no response to anti-TNF agents NOTE Neither form of IBD is routinely treated with surgery UC can be cured, however, with colectomy In CD, surgery is used exclusively for bowel obstruction as it recurs at surgical site
MTBS2CK p.261

Diverticulosis

Outpocketings of colon Where arteries meet mucosa Vegetarians At risk? Asymptomatic Infection?

MTBS2CK p.261

Diverticulosis

Diverticulitis

Most accurate test is... Colonoscopy Barium studies Fiber Fiber Fiber

Diagnosis LLQ pain and tenderness Fever Leukocytosis Palpable mass sometimes occurs Symptoms such as nausea, constipation, and bleeding can be present, but are nonspecific

MTBS2CK p.261

Source:commons.wikimedia.org

MTBS2CK p.261

19

Diverticulitis

Diverticulitis

Best initial test is... CT scan of the abdomen 98% accuracy Colonoscopy and barium enema are dangerous g in acute diverticulitis
Increased risk of perforation because infection weakens colonic wall

Source:commons.wikimedia.org

MTBS2CK p.261

Diverticulitis/Treatment Treatment Antibiotics that cover E. coli and anaerobes that are present in bowel such as...
Ciprofloxacin combined with metronidazole Amoxicillin/clavulanate Ticarcillin/clavulanate Piperacillin/tazobactam

Surgery is for those with...


No response to medical therapy Frequent recurrences of infection Perforation, fistula formation, abscess, strictures, or obstruction
MTBS2CK 261262

Which of the following is the most effective method of screening for colon cancer? a. Colonoscopy b. Sigmoidoscopy All less sensitive c. Fecal occult blood testing (FOBT) than colonoscopy d Barium enema d. e. Virtual colonoscopy with CT scanning Low sensitivity; misses small f. Capsule endoscopy
For small bowel bleeding polyps

MTBS2CK p.262

ColonCancer/FrequencyofScreening

ColonCancer/FrequencyofScreening

Routine testing Patients should have a colonoscopy every 10 years beginning at age 50 Single g family y member with colon cancer? Begin 10 years earlier than the age at which the family member developed their cancer or age 40, whichever is younger Screen every 10 years if relative > 60 or every 5 years if relative < 60
MTBS2CK p.262

Hereditary nonpolyposis colon cancer syndrome (HNPCC) comprises... 3 family members 2 generations 1p premature ( (< 50) ) Start screening at age 25 with colonoscopy every 1 to 2 years

MTBS2CK p.262

20

ColonCancer/FrequencyofScreening

ColonCancer/FrequencyofScreening

Familial adenomatous polyposis (FAP) Presence of thousands of polyps with abnormal genetic test known as adenomatous polyposis coli (APC) test Start screening with sigmoidoscopy at age 12 every year

Previous adenomatous polyp Patient should have colonoscopy...


Every 3 to 5 years

Previous history of colon cancer Patient should have colonoscopy...


1 year after resection Then at 3 years Then every 5 years

MTBS2CK p.263

MTBS2CK p.263

OtherPolyposisSyndromes Peutz-Jeghers syndrome


Multiple hamartomatous polyps, melanotic spots on lips and skin, frequency of breast cancer, gonadal and pancreatic cancer

Pancreas&Liver

Gardner syndrome
Colon cancer associated with: osteomas, desmoid tumors, and other soft tissue tumors

Turcot syndrome
Colon cancer in association with CNS malignancy

Juvenile polyposis
Colon cancer in association with multiple hamartomatous polyps
MTBS2CK p.263

AcutePancreatitis
Drug allergy Most common 1. GB stones 2. ETOH Drug toxicity

AcutePancreatitis/Presentation Acute epigastric pain + tenderness + nausea/vomiting = Pancreatitis Pain intensity is subjective and doesnt correlate with degree of organ damage In severe cases theres hypotension and fever
The pain of pancreatitis goes straight through to the back like a spear stabbed into the abdomen. Cholecystitis pain goes around the side to the back.
MTBS2CK p.264

Ductal obstruction

Pancreatitis

Scorpion sting

Trauma

Hypertriglyceridemia Hypercalcemia

Infection

MTBS2CK p.263264

21

AcutePancreatitis/DiagnosticTests Which of the following is associated with the worst prognosis in pancreatitis? a. Elevated amylase Levels do not correlate with severity b. Elevated lipase c. Intensity of the pain Pain doesnt predict d Low calcium d. e. C-reactive protein (CRP) rising
Elevated with all inflammation Best initial tests are... Amylase and lipase Most specific diagnostic test is CT scan Labs CBC: leukocytosis, drop in Hct over time with rehydration Elevated LDH and AST Hypoxia Hypocalcemia Elevated urinary trypsinogen activation peptide
MTBS2CK p.264265

MTBS2CK p.264

AcutePancreatitis/DiagnosticTests Imaging CT or MRI scan are best


Also detect pseudocysts

AcutePancreatitis/DiagnosticTests

ERCP
Can help determine etiology (stones, stricture, tumor)

Plain X X-ray ray


Sentinel loop of bowel (air-filled piece of small bowel in LUQ) Limited utility

Ultrasound has very poor accuracy


Overlying bowel blocks precise imaging
MTBS2CK p.265

Source:NiketSonpal,MD

NOTE: Abdominal CT scan is always performed with IV and oral contrast to better define and outline abdominal structures.

AcutePancreatitis/Treatment

AcutePancreatitis/Treatment

NPO (no food) IV hydration Analgesia PPIs pancreatic stimulation from acid entering g duodenum

> 30% necrosis on CT or MRI, add antibiotics (e.g., imipenem) Infected, necrotic pancreatitis should be resected with surgical debridement to prevent ARDS and death Pseudocysts are drained with a needle if they are enlarging or painful

MTBS2CK p.265

MTBS2CK p.265

22

AcutePancreatitis/Complication

LiverDisease
Spider angiomata and palmar erythema Portal hypertension leading to varices

Coagulopathy

Hepatorenal syndrome

Chronic Li Liver Disease

Asterixis and encephalopathy h l th

Hepatopulmonary syndrome

Thrombocytopenia

Hypoalbuminemia & edema and ascites

Source:commons.wikimedia.org

MTBS2CK p.265

Ascites Paracentesis is performed with... New-onset ascites Abdominal pain and tenderness Fever Low albumin L lb i i in th the ascitic iti fluid? fl id? Portal HTN from cirrhosis is the etiology Serum ascites albumin gradient (SAAG) The difference or gradient of albumin between serum and ascitic fluid
MTBS2CK p.266

Ascites

SAAG:CorrelatingLevelwithSpecificDiseases <1.1g/dL
Infections Cancer Nephroticsyndrome

>1.1g/dL
PortalHTN CHF Hepaticveinthrombosis Constrictivepericarditis

MTBS2CK p.266

SpontaneousBacterialPeritonitis Infection without perforation of bowel Causative organisms


E. coli (most common) Other gram-negative bacilli Pneumococcus Anaerobes (rare)

SpontaneousBacterialPeritonitis Best initial test is... Cell count with > 250 neutrophils Most accurate test is... Fluid culture, but takes too long for results Gram G stain t i is i negative/LDH ti /LDH nonspecific ifi Treatment: cefotaxime or ceftriaxone NOTE: SBP frequently recurs. When the ascites fluid albumin level is quite low, use prophylactic norfloxacin or TMP/SMX.

MTBS2CK p.266

MTBS2CK p.266

23

TreatmentofSpecificFeaturesofCirrhosis

AlcoholicLiverDisease Diagnosis of exclusion No specific therapy Most accurate test is...


Liver biopsy

Feature
Ascitesandedema Coagulopathyand thrombocytopenia Encephalopathy Hypoalbuminemia lb Spiderangiomata andpalmar erythema Varices Hepatorenal syndrome Hepatopulmonary syndrome

Treatment
Spironolactoneandotherdiuretics.Serial pericentesis forlargevolumeascites. FFPand/orplateletsonlyifbleedingoccurs Lactulose,neomycin,orrifaximin f therapy h Nospecific Nospecifictherapy Propranololandbandingviaendoscopy Somatostatin (octreotide),midodrine Nospecifictherapy

Source:commons.wikimedia.org

Alcohol and drugs causing liver disease give a greater elevation in AST compared to ALT. Viral hepatitis: Higher ALT than AST. Binge drinking: Sudden rise in GGTP.
MTBS2CK p.267

MTBS2CK p.267

PrimaryBiliaryCirrhosis(PBC)

PrimaryBiliaryCirrhosis(PBC) Most unique features of PBC are... Xanthelasma/xanthoma Osteoporosis Most accurate test is... Liver biopsy Most accurate blood test: Antimitochondrial antibody Treatment with... Ursodeoxycholic acid

PBC is most likely diagnosis with... Woman in 40s or 50s Fatigue and itching Normal bilirubin Elevated alkaline phosphatase

MTBS2CK p.267

MTBS2CK p.267268

PrimaryBiliaryCirrhosis(PBC)

PrimarySclerosingCholangitis Pruritus Elevated alkaline phosphatase GGTP Elevated bilirubin level Treatment Cholestyramine Ursodeoxycholic acid

Most accurate test is... ERCP, not liver biopsy Shows beading, narrowing, or strictures in biliary system
Source:commons.wikimedia.org

MTBS2CK p.268

Source:commons.wikimedia.org

24

PrimarySclerosingCholangitis

Alpha1AntitrypsinDeficiency Look for... Combination of...

PSC doesnt improve or resolve with resolution of IBD. Even after a colectomy in UC, patient may still progress to needing a liver transplantation.

Liver disease Emphysema (COPD)

Young patient (< 40) Nonsmoker Treatment Replace enzyme exogenously

MTBS2CK p.268

MTBS2CK p.268

Hemochromatosis

Hemochromatosis

Genetic disorder leading to overabsorption of iron in duodenum Mutation: C282y gene Presentation
Patient in their 50s with mild increases in AST and alkaline phosphatase
Erectile dysfunction

Amenorrhea Skin Darkening

Hemochromatosis
Cardiomegaly

Fatigue and joint pain (pseudogout)

Diabetes

MTBS2CK p.268

MTBS2CK p.268269

Hemochromatosis Best initial test is... Iron studies


Increased serum iron and ferritin Decreased iron binding capacity

ChronicHepatitisBandC EKG Conduction defects Echocardiogram Dilated or restrictive cardiomyopathy Best therapy Phlebotomy

Both are associated with... Cirrhosis Liver cancer PAN

Most accurate test is... Liver biopsy


Increased iron

MTBS2CK p.269

MTBS2CK p.269

25

ChronicHepatitisBandC Chronic hepatitis B Surface antigen positive > 6 months Hepatitis B DNA PCR is the best way to determine viral replication activity Liver biopsy for fibrosis Biopsy tracks progress Chronic hepatitis C 80% have chronic infection Never symptomatic when illness contracted Hepatitis C DNA PCR
Determines disease activity

ChronicHepatitisBandC/Treatment Chronic hepatitis B Adefovir Lamivudine Telbivudine Entecavir Tenofovir T f i Interferon Chronic hepatitis C Combination of... Interferon + Ribavirin + Telapavir ( Boceprevir) (or B i)

MTBS2CK p.270

MTBS2CK p.270

HepatitisTreatmentSideEffects Drug AdverseEffects

WilsonDisease/Presentation

Interferon Ribavarin Adefovir Lamivudine Bocepevir Telaprevir


MTBS2CK p.270

Arthralgias,thrombocytopenia, depression,leukopenia Anemia Renaldysfunction None Anemia Rash

Decrease in ceruloplasmin causes buildup of copper Neurological symptoms Psychosis, tremor, dysarthria, ataxia, or seizures Coombs negative hemolytic anemia Renal tubular acidosis or nephrolithiasis

MTBS2CK p.270271

WilsonDisease/Diagnosis

WilsonDisease

Best initial test is... Slit-lamp examination for Kayser-Fleischer rings Most accurate diagnostic g test is... Abnormally increased amount of copper excretion into urine after giving penicillamine

MTBS2CK p.271

MTBS2CK p.271

26

WilsonDisease/Treatment Penicillamine chelates copper and removes it Additional therapies are... Zinc: interferes with intestinal copper absorption Trientine: alternate copper-chelating compound

AutoimmuneHepatitis Look for... Young women Signs of liver inflammation Positive ANA More specific tests: Liver-kidney microsomal antibodies Anti-smooth muscle antibodies
MTBS2CK p.271

Most accurate test: Liver biopsy Treatment Prednisone and/or azathioprine

Decreased ceruloplasmin level is not the most accurate test. This is the most common wrong answer. All plasma proteins can be decreased in those with liver dysfunction and cirrhosis.
MTBS2CK p.271

AutoimmuneHepatitis

NonalcoholicSteatohepatitis Nonalcoholic Fatty Liver Disease Extremely common cause of mildly abnormal liver function tests Disorder is associated with: Obesity Diabetes Hyperlipidemia Corticosteroid use Most accurate test: biopsy Management: weight loss
MTBS2CK p.272

Source:commons.wikimedia.org

NonalcoholicSteatohepatitis

Source:commons.wikimedia.org

27

Anemia Hematology
Dr.ConradFischer,MD AssociateProfessorofMedicine TouroCollegeofMedicine NewYorkCity
Presentation DiagnosticTests y p Symptoms MeanCorpuscularVolume MicrocyticAnemia MacrocyticAnemia NormocyticAnemia Treatment

Anemia Presentation All forms of anemia present with identical symptoms if they have the same hematocrit (Hct) Symptoms based on severity, not etiology What is the most likely diagnosis?
Cannot be answered using symptoms alone

HematocritandSymptoms

Hematocrit
>30 35% 25 30% 20 25% <20 25%

ExpectedSymptoms
None Dyspnea(worseon exertion), ),fatigue g Lightheadedness, angina Syncope,chestpain

Diagnostic Tests Best initial test to evaluate anemia?


Always a complete blood count (CBC)

MTBS2CK p.203

MTBS2CK p.203

Anemia/DiagnosticTests

MeanCorpuscularVolume

Ultimately, cardiac ischemia from anemia proves fatal. Myocytes cant distinguish between: Anemia Hypoxia CAD Carbon monoxide poisoning All of these conditions result in decreased oxygen delivery to tissues
MTBS2CK p.203204

Mean corpuscular volume (MCV)


First clue to knowing etiology
Mean Corpuscular Volume (Normal 80100 fL)

Smaller?

Larger?

Microcytosis

Macrocytosis

MTBS2CK p.204

Microcytosis

Microcytosis Similarities among microcytic anemias Low reticulocyte count Only alpha thalassemia has 3 genes deleted elevated reticulocyte count Microcytic anemias are due to production problems
Nearly synonymous with reticulocyte counts

Causes of low MCV: Iron deficiency Thalassemia Sideroblastic anemia Anemia of chronic disease

MTBS2CK p.204

MTBS2CK p.204

Microcytosis

MacrocyticAnemia/Etiology Causes of high MCV B12 & folate deficiency Alcoholism Sideroblastic anemia Liver disease or hypothyroidism Medications M di ti ( (e.g., zidovudine id di or phenytoin) h t i ) Antimetabolite medications: azathioprine, 6mercaptopurine, hydroxyurea Myelodysplastic syndrome (MDS)

Routine blood smear


Not effective in telling the difference between types of microcytosis
All hypochromic All potentially give target cells

MTBS2CK p.204

MTBS2CK p.204

NormocyticAnemia Acute blood loss & hemolysis rapid drop in Hct (no time for MCV change) Blood loss leads to iron deficiency & microcytosis Hemolysis increases reticulocyte count
Reticulocytes raise the MCV Reticulocytes slightly larger than normal cells

Anemia/Treatment Treatment If severe: give packed RBCs Answering the question At what Hct do I transfuse a patient? depends on the following factors:
1. Symptomatic? Transfuse. 2. Hct very low in elderly person? Heart disease? Transfuse.

MTBS2CK p.204

MTBS2CK p.205

Anemia/Treatment Symptomatic from anemia means: SOB Lightheaded, confused, and sometimes syncope Hypotension and tachycardia Chest pain

BloodProducts Packed Red Blood Cells (PRBCs) Concentrated form of blood

Whole blood - 150 mL plasma = PRBCs


Hct of PRBCs: 70% - 80% Removal of plasma doubles Hct Each unit PRBCs raises Hct by 3 points/unit

Remember: No transfusion if young & asymptomatic

MTBS2CK p.205

MTBS2CK p.205

BloodProducts Fresh Frozen Plasma (FFP) Replaces clotting factors for elevated prothrombin time, aPTT, or INR Most important if actively bleeding FFP used as replacement with plasmapheresis

BloodProducts Cryoprecipitate Used to replace fibrinogen Some utility in disseminated intravascular coagulation (DIC) Provides high amounts of clotting factors in small plasma volume

Whole blood is never correct Whole blood is divided into either PRBCs or FFP
MTBS2CK p.205 MTBS2CK p.205

MicrocyticAnemia

Microcytosis = MCV < 80 fL

MicrocyticAnemia
Etiology Presentation DiagnosticTests Treatment

Microcytic,hypochromicanemiaseenonbloodsmear. MTBS2CK p.206


CopyrightJamesVanRhee. Usedwithpermission.

MicrocyticAnemia/Etiology Iron deficiency Blood loss One teaspoon (5 mL/day) blood loss leads to iron deficiency over time Body only needs very tiny amount of iron
1 to 2 mg/day

MicrocyticAnemia/Etiology Chronic disease Initially MCV is normal, then decreases Unclear etiology Any cancer or chronic infection Clear mechanism only in renal failure deficiency of erythropoietin Hemoglobin synthesis will not occur because iron does not move forward Iron is locked in storage or trapped in macrophages or in ferritin

Menstruating women need a little more


2-3 mg/day

Pregnant women need 5-6 mg/day Duodenum absorbs only about 4 mg/day
MTBS2CK p.206

MTBS2CK p.206

MicrocyticAnemia/Etiology Sideroblastic anemia Can be macrocytic when associated with myelodysplasia (MDS) Most common cause: Alcohol effect on marrow Less common causes...
Lead poisoning Isoniazid Vitamin B6 deficiency

MicrocyticAnemia/Etiology Thalassemia Extremely common cause of microcytosis Most with thalassemia trait are asymptomatic

MTBS2CK p.206

MTBS2CK p.206

MicrocyticAnemia/Presentation

MicrocyticAnemia/Presentation

You cant distinguish these anemias based on symptoms Might have a suggestion from history

How to answer What is the most likely diagnosis? for anemia FeatureintheHistory Mostlikelydiagnosis? Bloodloss(GIbleeding) Menstruation Cancerorchronicinfection Rheumatoidarthritis Alcoholic Asymptomatic Irondeficiency Irondeficiency Chronicdisease Chronicdisease Sideroblastic Thalassemia

MTBS2CK p.206

MTBS2CK p.206

MicrocyticAnemia/DiagnosticTests

MicrocyticAnemia/DiagnosticTests

Peripheral smear not useful! All hypochromic All can be associated with target cells

Target cells: most common with thalassemia

MTBS2CK p.207

MTBS2CK p.207

CopyrightJamesVanRhee. Usedwithpermission.

MicrocyticAnemia/IronStudies Unique features and diagnoses of iron studies

MicrocyticAnemia/IronStudies Iron deficiency

UniqueFeature Lowferritin Highiron Normalironstudies

Diagnosis Irondeficiency Sideroblasticanemia Thalassemia

Low ferritin = Iron deficiency


However, one third of patients with iron deficiency will have normal or increased ferritin Ferritin is an acute phase reactant This means any infection or inflammation can raise the ferritin level

MTBS2CK p.207

MTBS2CK p.207

MicrocyticAnemia/IronStudies Both iron deficiency and anemia of chronic disease are associated with low serum iron Iron deficiency: increase in total iron binding capacity (TIBC) TIBC measures unbound sites on transferrin Open sites on transferrin = increased capacity or number of unbound sites

MicrocyticAnemia/IronStudies Chronic disease Serum iron: low in circulation Iron trapped in storage Ferritin (stored iron): elevated or normal Circulating iron: decreased Major M j diff difference i is TIBC i is low l

MTBS2CK p.207

MTBS2CK p.207

MicrocyticAnemia/IronStudies Sideroblastic anemia Only microcytic anemia elevated circulating iron level Thalassemia Genetic disease with normal iron studies

UniqueLaboratoryFeatures Iron deficiency Red cell distribution of width (RDW) increased


Newer cells are more iron deficient Newer cells are smaller in iron deficiency As body runs out of iron, newer cells have less Hb & get p g progressively g y smaller

Elevated platelet count common Single most accurate test is...


Bone marrow biopsy for stainable iron (decreased) Rarely done, but most accurate

MTBS2CK p.207

MTBS2CK p.207

UniqueLaboratoryFeatures Sideroblastic anemia The most accurate test is... Prussian blue staining for ringed sideroblasts Basophilic stippling can occur in any cause of sideroblastic anemia

UniqueLaboratoryFeatures Thalassemia Most accurate test is...

Hb electrophoresis!

MTBS2CK p.207208

MTBS2CK p.208

Source:commons.wikimedia.org

UniqueLaboratoryFeatures Thalassemia Most accurate test for alpha thalassemia


Genetic studies 3-gene deletion alpha thalassemia
Hb H (beta-4 tetrads) Increased reticulocyte count

ElectrophoresisFindings

Alphathalassemia
Onegenedeleted:normal Twogenesdeleted:mildanemia, normalelectrophoresis Threegenesdeleted:moderate anemiawithHbH(beta4 tetrads),increasedreticulocytes Fourgenesdeleted:gamma4 tetradsorhemoglobinBart;CHF causesdeathinutero
MTBS2CK p.208

Betathalassemia
Inc.HbFandA2 N/A Betathalassemiaintermedia NormalHbF Notransfusiondependence N/A

All thalassemia types have normal RDW

MTBS2CK p.208

MicrocyticAnemia/Treatment Iron deficiency Replace iron with oral ferrous sulfate If insufficient, patients get IM iron Chronic disease Correct underlying disease Only O l end-stage d t renal lf failure il responds d t to erythropoietin Sideroblastic anemia Correct the cause Some respond to vitamin B6 (pyridoxine)
MTBS2CK p.208

MicrocyticAnemia/Treatment Thalassemia Trait not treated Beta thalassemia major (Cooley anemia) Chronic transfusion lifelong Iron I overload l d managed d with ith d deferasirox f i ( (oral li iron chelator) Deferoxamine is parenteral

MTBS2CK p.209

MacrocyticAnemia
Etiology Presentation DiagnosticTests Treatment

73-year-old man in office with fatigue progressively worse over several months. He is short of breath when he walks up one flight of stairs. He drinks 4 vodka martinis a day. He complains of numbness and tingling in his feet. Physical: decreased sensation in feet. Hct: 28% MCV: 114 fL (elevated) What is the next step? 1st is peripheral smear Once hypersegmented neutrophils are seen, a. Vitamin B12 level THEN get B12 & folate levels b. Folate level c. Peripheral blood smear d. Schilling test To see if B12 deficiency is due to pernicious anemia e. Methylmalonic acid level
Confirms diagnosis of B12 deficiency when B12 levels are equivocal

MTBS2CK p.209

MacrocyticAnemia Megaloblastic Hypersegmented neutrophils

MacrocyticAnemia/Etiology Vitamin B12 deficiency is caused by... Pernicious anemia Pancreatic insufficiency Dietary deficiency (vegan/strict vegetarian) Crohns disease damaging terminal ileum Blind loop syndrome (gastrectomy or gastric bypass for weight loss) Diphyllobothrium latum

Many factors raise MCV Only B12 & folate deficiency & antimetabolite medications cause hypersegmentation

Source: Alireza Eghtedar, MD

MTBS2CK p.209210

MTBS2CK p.209

MacrocyticAnemia/Etiology

MacrocyticAnemia/Presentation

Folate deficiency Dietary deficiency (goats milk has no folate and limited iron/B12) Psoriasis & skin loss or turnover Drugs: g p phenytoin, y , sulfa

Alcohol Gives macrocytosis & neurological problems Will not give hypersegmented neutrophils

MTBS2CK p.209

MTBS2CK p.209

MacrocyticAnemia/Presentation B12 deficiency Can give any neurological abnormality Peripheral neuropathy most common Dementia least common Posterior column damage to position & vibratory sensation or subacute subacute combined degeneration degeneration of cord is classic Look for ataxia

MacrocyticAnemia/DiagnosticTests Labs common to both B12 & folate deficiency Megaloblastic anemia Increased LDH & indirect bilirubin levels Decreased reticulocyte count Hypercellular bone marrow Macroovalocytes M l t Increased homocysteine levels Only B12 deficiency is associated with increased methylmalonic acid level

MTBS2CK p.210

MTBS2CK p.210

MacrocyticAnemia/DiagnosticTests B12 & folate deficiency Identical hematologically on blood smear


73-year-old woman with decreased position & vibratory sensation of lower extremities. Hct: 28%, MCV: 114 fL, and hypersegmented neutrophils. B12 level decreased, but near the borderline of normal. What is the next step in management?
a. b. c. d. e. f. Methylmalonic acid level Confirms cause as pernicious Anti-intrinsic factor antibodies anemia after B12 deficiency confirmed Anti-parietal cell antibodies Schillings test Rare test of etiology; premature Folate level Folate doesnt give neurological deficits Homocysteine level Doesnt add anything

Source: Alireza Eghtedar, MD

MTBS2CK p.210

MTBS2CK p.210211

MacrocyticAnemia/DiagnosticTests

MacrocyticAnemia/DiagnosticTests Tested facts about macrocytic anemia B12 & folate deficiency can cause pancytopenia as well as macrocytic anemia Pancreatic enzymes are needed to absorb B12
They free it from carrier (R) protein

Tested facts about macrocytic anemia Schilling test is never the right answer Pernicious anemia is confirmed with antiintrinsic factor and anti-parietal cell antibodies Red cells are destroyed as they leave the marrow, so reticulocyte count is low

Neurological abnormalities will improve as long as they are minor (e.g., peripheral) and short duration

MTBS2CK p.211

MTBS2CK p.211

MacrocyticAnemia/Treatment

Replace what is deficient Folate replacement corrects hematologic problems of B12 deficiency, but not the neurological problems

Which of the following is a complication of B12 or folate replacement? a. Seizures Not associated with treatment b. Hemolysis Cells are produced rapidly, not hemolyzed c Hypokalemia c. High K+ from massive tissue or d. Hyperkalemia cellular breakdown e. Diarrhea
Cause hypokalemia, but not associated with B12 or folate replacement therapy

MTBS2CK p.211

MTBS2CK p.211

HemolyticAnemia All forms lead to: Sudden in Hct Increased LDH, indirect bilirubin & reticulocytes Decreased haptoglobin Slight increase in MCV
Reticulocytes R ti l t > normal l cells ll

HemolyticAnemia Part1
SickleCellDisease HereditarySpherocytosis AutoimmuneHemolysis y

Hyperkalemia

MTBS2CK p.211212

SickleCellDisease Chronic, well-compensated hemolytic anemia Reticulocyte count always high Acute painful vaso-occlusive crisis is caused by... Hypoxia Dehydration Infection I f ti Cold temperatures

SickleCellDisease/Presentation Look for... African American Sudden, severe pain in chest, back & thighs May have fever

Rare for adult to present with acute crisis without clear history of sickle cell disease

MTBS2CK p.212

MTBS2CK p.212

SickleCellDisease/Manifestations Bilirubin gallstones Increased infection from autosplenectomy


Encapsulated organisms

SickleCellDisease/DiagnosticTests Best initial test... - Peripheral smear Most accurate test.. - Hemoglobin electrophoresis Trait (AS disease) does not give sickled cells

Osteomyelitis y
Most commonly from Salmonella

Stroke Enlarged heart with hyperdynamic features and systolic murmur Skin ulcers Avascular necrosis of femoral head

Retinopathy

MTBS2CK p.212

MTBS2CK p.212213

SickleCellDisease/Treatment Which is found on smear in sickle cell disease? a. b. c. d d. e. Basophilic stippling Associated with lead poisoning Howell-Jolly bodies Bite cells Seen with G6PD deficiency Fragmented red cells seen with S hi t Schistocytes t intravascular hemolysis Morulae 1. Oxygen/hydration/analgesia 2. Fever or white cell count higher than usual? Antibiotics given! Ceftriaxone, levofloxacin, or moxifloxacin 3. Folic acid in everyone 4. Pneumococcal vaccine: autosplenectomy 5. Hydroxyurea: prevent recurrences, increases Hb F Dont wait for results of testing! Start antibiotics with fever. No spleen = Overwhelming infection = Death!!
MTBS2CK p.213 MTBS2CK p.213

Seen inside neutrophils in Ehrlichia infections

10

SickleCellDisease/Treatment 43-year-old man with sickle cell disease admitted with acute pain crisis. Only med is folic acid. Hct on admission: 34%. On 3rd hospital day, hematocrit drops to 22%. What is the initial test? y count Not best way to gauge severity of a. Reticulocyte b. Peripheral smear hemolysis Causes aplastic crisis which freezes growth of c. Folate level marrow d. Parvovirus B-19 IgM level Most accurate test for parvovirus B-19 is e. Bone marrow PCR for DNA. IVIG is best initial therapy.
Marrow shows giant pronormoblasts, but this isnt better than reticulocyte count at measuring severity of hemolysis
MTBS2CK p.214 MTBS2CK p.214

Exchange transfusion for severe vasoocclusive crisis with: Acute chest syndrome Priapism Stroke Visual disturbance from retinal infarction

SickleCellTrait

HereditarySpherocytosis Genetic defect in cytoskeleton of RBCs Leads to abnormal round shape Loss of normal flexibility characteristic of biconcave disc that allows red cells to bend in spleen

Patient is heterozygous for sickle gene (AS) Only manifestation is inability to concentrate urine (isosthenuria) Clinically asymptomatic Normal CBC Normal smear Hematuria sometimes occurs No treatment for sickle cell trait

MTBS2CK p.214

MTBS2CK p.214215

HereditarySpherocytosis/Presentation

HereditarySpherocytosis/Diagnosis

Look for... Recurrent episodes of hemolysis in a young child or newborn Intermittent jaundice Splenomegaly S l l Family history of anemia or hemolysis Bilirubin gallstones

Low MCV Increased mean corpuscular hemoglobin concentration (MCHC) Negative Coombs test
Most acc accurate rate test: test Osmotic fragility fragilit
Cells are placed in slightly hypotonic solution Increased swelling leads to hemolysis

MTBS2CK p.214215

MTBS2CK p.215

11

HereditarySpherocytosis/Treatment

AutoimmuneHemolysis

1. Chronic folic acid replacement supports red cell production 2. Splenectomy stops hemolysis but doesnt eliminate spherocytes

Also known as warm or IgG hemolysis Idiopathic in 50% Clear causes are...
Chronic lymphocytic leukemia (CLL) Lymphoma Systemic lupus erythematosus (SLE) Drugs: penicillin, alpha-methyldopa, rifampin, phenytoin

MTBS2CK p.215

MTBS2CK p.215

AutoimmuneHemolysis/DiagnosticTests

AutoimmuneHemolysis/DiagnosticTests Autoantibodies remove small amounts of red cell membrane and lead to smaller membrane Forces cell to become round Biconcave discs have greater surface than sphere

Most accurate diagnostic test is... Coombs test


Detects IgG on surface of red cells Direct & indirect tests tell basically the same thing Indirect associated with greater amount of antibody

Smear doesnt show fragmented cells because destruction occurs inside spleen or liver, not in blood vessel
MTBS2CK p.215216 MTBS2CK p.215216

AutoimmuneHemolysis/Treatment 1. Glucocorticoids (prednisone) best initial therapy 2. Recurrent episodes: splenectomy 3. Severe, acute hemolysis not responding to prednisone can be controlled with intravenous immunoglobulin g (IVIG) 4. Rituximab when splenectomy doesnt control hemolysis

AutoimmuneHemolysis/Treatment

Alternate treatments to diminish need for steroids: Cyclophosphamide Cyclosporine Azathioprine Mycophenolate mofetil

MTBS2CK p.216

MTBS2CK p.216

12

ColdAgglutininDisease
IgM antibodies against red cells in association with Epstein-Barr virus, Waldenstrm macroglobulinemia, or Mycoplasma pneumoniae Presentation Symptoms in colder parts of body Numbness or mottling of nose, ears, fingers, and toes Symptoms resolve on warming body part

HemolyticAnemia Part2
ColdAgglutininDisease G6PDDeficiency HUSandTTP PNH

Diagnosis
Direct Coombs test positive only for complement Smear normal in most May show spherocytes
MTBS2CK p.216

ColdAgglutininDisease Treatment 1. Stay warm 2. Administer rituximab 3. Cyclophosphamide, cyclosporine, or other immunosuppressive agents stop production of antibody 4. Plasmapheresis in some

ColdAgglutininDisease/Treatment

Cryoglobulins often mixed up with cold agglutinins. Both are IgM and dont respond to steroids. Cryoglobulins are associated with: Hepatitis C Joint pain Glomerulonephritis

Steroids and splenectomy usually dont work


MTBS2CK p.216217 MTBS2CK p.217

G6PDDeficiency X-linked recessive Inability to generate glutathione reductase and protect red cells from oxidant stress Most common oxidant stress is infection Other causes: dapsone, quinidine, sulfa drugs, primaquine nitrofurantoin primaquine, nitrofurantoin, and fava beans

G6PDDeficiency/Presentation

Look for...
African American or Mediterranean males Sudden anemia & jaundice Normal-sized spleen With infection i f ti or drug d

G6PD deficiency is X-linked recessive, it manifests almost exclusively in males

MTBS2CK p.217

MTBS2CK p.217

13

G6PDDeficiency/Diagnosis Best initial test is... Peripheral smear


Look for Heinz bodies and bite cells

G6PDDeficiency

Treatment Avoid oxidant stress Nothing reverses hemolysis

Most accurate test... G6PD level


1 to 2 months after acute episode of hemolysis

MTBS2CK p.217

MTBS2CK p.218

HUSandTTP Hemolytic uremic syndrome (HUS) and thrombotic thrombocytopenic purpura (TTP) are different versions of the same basic disease
HUS Associated with E. coli 0157:H7 More frequent in children TTP Associated with ticlopidine clopidogrel, ticlopidine, clopidogrel cyclosporine, and AIDS Neurological disorders (confusion and seizures) Fever More common in adults

HUSandTTP Both characterized by:

Intravascular hemolysis with fragmented red cells (schistocytes) Thrombocytopenia Renal insufficiency

Source:commons.wikimedia.org

MTBS2CK p.218

MTBS2CK p.218

HUSandTTP
Diagnosis No one specific test diagnoses either disorder Normal PT/aPTT Negative Coombs test Treatment Severe cases treated with plasmapheresis or plasma exchange

ParoxysmalNocturnalHemoglobinuria
Clonal stem cell defect with increased sensitivity to complement in acidosis Etiology Deficiency of complement regulatory proteins CD 55 and 59, also known as decay accelerating factor (DAF) Gene for phosphatidylinositol glycan class A (PIG (PIG-A) A) is defective overactivation of complement system During sleep, relative hypoventilation leads to pCO2 and acidosis This does nothing to an unaffected person In PNH it leads to hemolysis and thrombosis
MTBS2CK p.218

MTBS2CK p.218

14

ParoxysmalNocturnalHemoglobinuria
Presentation Episodic dark urine with first urination of day from hemoglobin Pancytopenia and iron deficiency anemia

ParoxysmalNocturnalHemoglobinuria

Pancytopenia Most accurate test is...


Decreased CD55 & CD59

Ham test and sucrose hemolysis are never the correct answer
Flow cytometry is another way of saying CD55/CD59 testing

Thrombosis MCC of death


MTBS2CK p.218 MTBS2CK p.218219

ParoxysmalNocturnalHemoglobinuria

1. Prednisone 2. Allogeneic bone marrow transplant is the only method of cure 3. Eculizumab inactivates C5 in complement pathway and decreases red cell destruction; its a complement inhibitor 4. Folic acid and iron replacement with transfusions as needed

HematologicMalignancies
AplasticAnemia PolycythemiaVera EssentialThrombocytosis Myelofibrosis

MTBS2CK p.219

AplasticAnemia
Pancytopenia of unclear etiology Any infection or cancer can invade marrow causing decreased production or hypoplasia Other causes:
Radiation and toxins (e.g., toluene, insecticides (DDT), and benzene) Drug effect: sulfa, phenytoin, carbamazepine, chloramphenicol, alcohol, chemotherapy SLE PNH Infection: HIV, hepatitis, CMV, EBV B12 and folate deficiency Thyroid-inhibiting medications (e.g., propylthiouracil (PTU) and methimazole)
MTBS2CK p.219

AplasticAnemia

Fatigue of anemia Infections from low white cell counts Bleeding from thrombocytopenia Most accurate test is... is Bone marrow biopsy Aplastic anemia is confirmed by excluding all causes of pancytopenia

MTBS2CK p.219

15

AplasticAnemia/Treatment Treat any underlying cause thats identified THEN... Blood transfusion, antibiotics for infection, and platelets for bleeding Aplastic anemia acts as autoimmune disorder T cells attack patients own marrow Cyclosporine inhibits T cells This brings the marrow back to life!

AplasticAnemia/Treatment True aplastic anemia treated with allogeneic bone marrow transplantation (BMT) if young enough and matched When too old for BMT (> 50) or no matched donor y y g globulin ( (ATG) ) and treatment is anti-thymocyte cyclosporine Tacrolimus is alternative to cyclosporine

MTBS2CK p.219

MTBS2CK p.219220

PolycythemiaVera Unregulated overproduction of all 3 cell lines Red cell overproduction is most prominent Mutation in JAK2 protein which regulates marrow production Red cells grow wildly despite low erythropoietin

PolycythemiaVera Symptoms of hyperviscosity from increased red cell mass such as: Headache, blurred vision, and tinnitus Hypertension Fatigue Splenomegaly Bleeding from engorged blood vessels Thrombosis from hyperviscosity

MTBS2CK p.220

MTBS2CK p.220

PolycythemiaVera
Hematocrit markedly elevated > 60% Platelets and WBC count Erythropoietin Total red cell mass You must exclude hypoxia as cause of erythrocytosis Oxygen levels: normal Vitamin B12 levels are elevated for unclear reasons it s Iron levels because its been used to make red cells

PolycythemiaVera

Most accurate test...


JAK2 mutation (95%)

Increased basophils
All myeloproliferative disorders

Small number converted to AML

MTBS2CK p.220

MTBS2CK p.220

16

EssentialThrombocytosis(ET) Markedly elevated platelet count > 1 million Leads to both thrombosis & bleeding Very difficult to distinguish from an elevated platelet count as a reaction to another stress (e.g., infection, cancer, or iron deficiency)

EssentialThrombocytosis(ET) < 60 & asymptomatic with platelets < 1.5 million Best initial therapy...
Hydroxyurea

Anagrelide with red cell No treatment necessary suppression from hydroxyurea > 60 & thromoboses or platelet count > 1.5 Aspirin for million erythromelalgia (painful, Begin treatment red hands in ET)

MTBS2CK p.221

MTBS2CK p.221

Myelofibrosis
Older persons with pancytopenia Bone marrow shows marked fibrosis Blood production shifts to spleen & liver, which become markedly enlarged Look for teardrop-shaped cells and nucleated red blood cells on smear Thalidomide and lenalidomide: tumor necrosis factor inhibitors that increase bone marrow production Occasional patient < 50-55, allogeneic bone marrow transplantation is attempted

Leukemia
AcuteLeukemia ChronicMyelogenousLeukemia LeukostasisReaction MyelodysplasticSyndrome ChronicLymphocyticLeukemia HairyCellLeukemia

MTBS2CK p.221

AcuteLeukemia/Presentation History of myelodysplasia suggests acute leukemia Signs of pancytopenia (fatigue, infection, bleeding) Even if WBC is normal or increased Infection is common presentation Leukemic cells (blasts) dont function normally

AcuteLeukemia/M3 Most frequently tested acute leukemia is M3, known as acute promyelocytic leukemia (APL) M3 is associated with disseminated intravascular coagulation (DIC)

M3 DIC
No distinct clinical presentation bet between een the 3 subtypes of acute lymphocytic leukemia (ALL)

MTBS2CK p.221

MTBS2CK p.221

17

AcuteLeukemia/Diagnosis Best initial test is... Blood smear


Shows blasts

AcuteLeukemia/Diagnosis Auer rods Eosinophilic inclusions Associated with acute promyelocytic leukemia (M3)

Most accurate test is... Flow cytometry


Distinguishes different subtypes of acute leukemia Detects specific CD subtypes associated with each type of leukemia

Myeloperoxidase Characteristic of acute myelocytic leukemia (AML)

MTBS2CK p.221

MTBS2CK p.221222

AcuteLeukemia/Treatment Both AML & ALL treated initially with chemotherapy to remove blasts from peripheral blood smear Known as inducing remission The question is: proceed to BMT after remission or give more chemotherapy Prognosis poor? go straight to BMT Prognosis good? more chemotherapy

AcuteLeukemia/Treatment
The best indicator of prognosis in acute leukemia is...

Cytogenetics
Assesses specific chromosomal characteristics found in each patient Good cytogenetics = less chance of relapse = more chemotherapy Bad cytogenetics = more chance of relapse = immediate BMT

MTBS2CK p.222

MTBS2CK p.222

AcuteLeukemia/Treatment

AcuteLeukemia/Treatment

1. Add all-trans-retinoic acid (ATRA) to those with M3 (acute promyelocytic leukemia) 1. Add intrathecal chemotherapy such as methotrexate to ALL treatment; prevents relapse of ALL in CNS

Most tested facts for acute leukemia are... M3 (promyelocytic leukemia) gives DIC Add ATRA to M3 Auer rods = AML Add intrathecal methotrexate to ALL

MTBS2CK p.222

MTBS2CK p.222

18

ChronicMyelogenousLeukemia/Presentation Look for... High WBC count: all neutrophils Vague symptoms of fatigue, night sweats, and fever from hypermetabolic syndrome Splenomegaly: early satiety, abdominal fullness, and LUQ pain Pruritus is common after hot baths/showers
Histamine release from basophils

ChronicMyelogenousLeukemia/Diagnosis

Determine if its a reaction to another infection, stress (leukemoid reaction), or leukemia CML may give small numbers of blasts Should be < 5% Basophils increased

MTBS2CK p.223

MTBS2CK p.223

ChronicMyelogenousLeukemia/Diagnosis If leukocyte alkaline phosphatase score (LAP) is a choice, then this is first
Leukemic cells dont have normal amounts of alkaline phosphatase; LAP score low in CML

ChronicMyelogenousLeukemia/Treatment
Imatinib (Gleevec), dasatinib, or nilotinib are the best initial therapy Only BMT cures CML
Never the first therapy

If LAP score is not a choice, or the question is What is the most accurate test? then answer BCR-ABL
Can be done by PCR on peripheral blood

MTBS2CK p.223

MTBS2CK p.223

MyelodysplasticSyndrome(MDS)
54-year-old man with SOB, blurry vision, confusion, and priapism. WBC count 225,000/L. Predominantly neutrophils with about 4% blasts.

a. b. c. d. e. f. g.

What is next step in management of this case?


Leukapheresis In acute leukostasis reaction, BCR ABL testing BCR-ABL i important t t to t remove excessive i Bone marrow biopsy white cells from the blood than Bone marrow transplant to establish a specific Consult hematology/oncology diagnosis, no matter what the etiology Flow cytometry Hydroxyurea Will lower the cell count, but not as rapidly as leukapheresis

Preleukemic disorder in those > 60 Pancytopenia despite hypercellular bone marrow Most never develop AML Infection & bleeding lead to death before leukemia occurs

MTBS2CK p.223224

MTBS2CK p.224

19

MyelodysplasticSyndrome Many present with asymptomatic pancytopenia on routine CBC Symptoms can be... Fatigue & weight loss Infection Bleeding Sometimes splenomegaly No single pathognomonic finding in history or physical examination

MyelodysplasticSyndrome/Diagnosis
CBC: anemia with increased MCV, nucleated red cells, and small number of blasts Marrow: hypercellular Prussian blue stain shows ringed sideroblasts Severity based on percentage of blasts

MTBS2CK p.224

MTBS2CK p.224225

MyelodysplasticSyndrome/Treatment

ChronicLymphocyticLeukemia
CLL is a clonal proliferation of normal, mature-appearing B lymphocytes that function abnormally Occurs over age 50 in 90% affected Most asymptomatic at presentation Most common symptom is fatigue Other symptoms:
Lymphadenopathy (80%) Spleen or liver enlargement (50%) Infection

Transfuse blood products as needed Erythropoietin: about 20% response Lenalidomide for those with 5q deletion Decreases transfusion dependence

No unique physical findings Richter phenomenon: Conversion of CLL into high-grade lymphoma Occurs in 5%
MTBS2CK p.225 MTBS2CK p.225

ChronicLymphocyticLeukemia/Diagnosis WBC usually > 20,000/L 80% to 98% lymphocytes Half are hypogammaglobulinemic Anemia & thrombocytopenia occur from marrow infiltration or autoimmune warm IgG antibodies

ChronicLymphocyticLeukemia/Treatment Stage 0 (elevated WBC) & stage I (lymphadenopathy); no treatment Stage II (hepatosplenomegaly), stage III (anemia), and stage IV (thrombocytopenia) treat with fludarabine If choices list fludarabine and rituximab, , this is the best initial therapy for advanced-stage disease (II, III, IV) or any patient who is symptomatic (severe fatigue, painful nodes) PCP prophylaxis is indicated in CLL

Source:commons.wikimedia.org

Smudge cell: Lab artifact Nucleus crushed by cover slip

MTBS2CK p.225

MTBS2CK p.226

20

ChronicLymphocyticLeukemia/Treatment Refractory cases: cyclophosphamide (more efficacy, but more toxic) Mild cases: chlorambucil Severe infection: IVIG Autoimmune thrombocytopenia or hemolysis: prednisone

HairyCellLeukemia Middle-aged men with:


Pancytopenia Massive splenomegaly Monocytopenia Inaspirable dry tap, despite hypercellularity of marrow

The most accurate test is...


Tartrate resistant acid phosphatase (TRAP) or CD11c

Source:commons.wikimedia.org

Treat with cladribine

MTBS2CK p.226

MTBS2CK p.226

NonHodgkinLymphoma Proliferation of lymphocytes in nodes and spleen NHL most often widespread at presentation NHL and CLL are extremely similar NHL is solid mass and CLL is liquid or circulating

Lymphoma
NonHodgkinLymphoma HodgkinDisease MultipleMyeloma MGUS WaldenstrmMacroglobulinemia

MTBS2CK p.226

NonHodgkinLymphoma Painless lymphadenopathy May involve pelvic, retroperitoneal, or mesenteric structures Nodes not warm, red, or tender B symptoms:
Fever, Fever weight loss loss, drenching night sweats

NonHodgkinLymphoma Best initial test is...


Excisional biopsy

CBC normal in most cases High LDH correlates with worse severity Staging determines intensity of therapy Typical staging procedures are:
CT scan of chest, abdomen, and pelvis Bone marrow biopsy

Advanced stages in 80% to 90% of cases

MTBS2CK p.227

MTBS2CK p.227

21

NonHodgkinLymphoma Staging Stage I: 1 lymph node group Stage II: 2+ lymph node groups on same side of diaphragm Stage III: both sides of diaphragm Stage IV: widespread disease Most common wrong answer is needle aspiration of node. Aspiration not enough because individual lymphocytes appear normal.

NonHodgkinLymphoma/Treatment Local disease (Stage Ia and IIa): local radiation Advanced disease (Stage III and IV, any B symptoms): combination chemotherapy with CHOP and rituximab (antibody against CD20) C = cyclophosphamide H = adriamycin (doxorubicin or hydroxydaunorubicin) O = vincristine (oncovin) P = prednisone Mucosal Associated Lymphoid Tissue
Lymphoma of stomach is associated with Helicobacter pylori Treat with clarithromycin and amoxicillin
MTBS2CK p.227

MTBS2CK p.227

HodgkinDisease Presentation, diagnostic tests, B symptoms, and staging of Hodgkin disease (HD) are same as NHL Hodgkin disease has Reed-Sternberg cells on pathology
Normal lymphocyte ReedSternberg cell

DifferencesbetweenHDandNHL HodgkinDisease
Local,StageI,andStageIIin 8090% Centersaroundcervicalarea gcellson ReedSternberg pathology

NonHodgkinLymphoma
StageIIIandStageIVin8090%

Disseminated gcells NoReedSternberg

Pathologicclassification: Pathologicclassification: Lymphocytepredominanthas Burkitt andimmunoblastic bestprognosis haveworstprognosis Lymphocytedepletedhas worstprognosis


Source:commons.wikimedia.org

MTBS2CK p.228

MTBS2CK p.228

HodgkinDisease/Treatment Stage Ia and IIa: local radiation and Chemo Stage III and IV or anyone with B symptoms: ABVD A = adriamycin (doxorubicin) B = bleomycin Relapses after radiation V = vinblastine therapy are treated with D = dacarbazine chemotherapy. Relapses after chemotherapy are treated with extra high dose chemotherapy and bone marrow transplantation.
MTBS2CK p.228

HodgkinDisease/Treatment Complications of Radiation and Chemotherapy Radiation increases the risk of solid tumors such as breast cancer or lung cancer Screening for breast cancer earlier after treatment Radiation increases premature coronary disease The risk of acute leukemia, MDS, and NHL from chemotherapy is about 1% per year

MTBS2CK p.228

22

Which of the following is most useful to determine dosing of chemotherapy in HD? The other choices arent as a. Echocardiogram accurate in determining left b. Bone marrow biopsy ventricular function c. Gender d MUGA or nuclear ventriculogram d. e. Hematocrit f. Symptoms

HodgkinDisease/ AdverseEffectsofChemotherapy

ChemotherapeuticAgent Doxorubicin Vincristine Bleomycin Cyclophosphamide Cisplatin

Toxicity Cardiomyopathy Neuropathy Lungfibrosis Hemorrhagiccystitis Renalandototoxicity

MTBS2CK p.228229

MTBS2CK p.229

MultipleMyeloma Abnormal proliferation of plasma cells Unregulated production of useless immunoglobulin Usually IgG or IgA Immunoglobulins dont fight infection, but clog up the kidney IgM is a separate disease called Waldenstrm macroglobulinemia

MultipleMyeloma Most common presentation is...


Bone pain from pathologic fractures

Pathologic fracture: bone breaks under normal use Due to osteoclast activating factor (OAF), which attacks the bone causing lytic lesions OAF hypercalcemia Infection common because abnormal plasma cells dont make immunoglobulins effective against infection

MTBS2CK p.229

MTBS2CK p.229

MultipleMyeloma Hyperuricemia: increased turnover of nuclear material of plasma cells Anemia: from infiltration of marrow with plasma cells Renal failure: from immunoglobulins and BenceJones p protein in kidney; y; hypercalcemia yp and hyperuricemia also damage the kidney Renal failure and infection are MCCs of death

MultipleMyeloma First test done is X-ray of affected bone shows lytic lesions Serum protein electrophoresis (SPEP) shows an IgG (60%) or IgA (25%) spike of a single type or clone. One clone = Monoclonal or M spike p Fifteen percent have light chains or Bence-Jones protein only

MTBS2CK p.229

MTBS2CK p.229230

23

MultipleMyeloma Additional abnormalities Hypercalcemia Bence-Jones protein on urine immunoelectrophoresis Beta-2 microglobulin levels correspond to severity of disease Smear with rouleaux Elevated BUN and creatinine Bone marrow biopsy: >10% plasma cells defines myeloma Elevated total protein with normal albumin

MultipleMyeloma

Rouleaux IgG paraprotein sticks to red cells causing them to adhere to each other in a stack or roll

MTBS2CK p.230

MTBS2CK p.230

Source:commons.wikimedia.org

MultipleMyeloma
69-year-old woman admitted with severe back pain that suddenly worsened--pop felt when she coughs and tenderness over ribs. X-ray: lytic lesions. Calcium is 2 points above normal, Hct 27%, creatinine elevated. UA: trace protein, but 24-hour urine show 5 grams of protein. What do you expect on technetium bone scan? a. Normal b. Lytic lesions at site of fractures c. Increased uptake diffusely d. Decreased uptake Nuclear bone scan shows increased

M-spike on SPEP doesnt mean IgM


Myeloma has a decreased anion gap. IgG is cationic. Increased cationic substances will increase chloride and bicarbonate levels. This decreases the anion gap.

uptake with osteoblastic activity, which is absent in myeloma.

MTBS2CK p.230

MTBS2CK p.230

Why the difference between the protein on urinalysis and 24-hour urine?
a. b. c. d. e. False positive 24-hour urine is common in myeloma Calcium in urine creates false negative urinalysis Uric acid creates a false positive 24-hour 24 hour urine Bence-Jones protein isnt detected by dipstick. IgG in urine inactivates urine dipstick Bence-Jones protein is detected by urine immunoelectrophoresis. Urine dipstick detects only albumin.
MTBS2CK p.230

What is the single most accurate test for myeloma? a. Skull X-rays Will show lytic lesions, but not as specific b. Bone marrow biopsy Nothing besides myeloma is associated with > 10% plasma c. 24-hour urine Of those with an M-spike cells on bone marrow biopsy of immunoglobulins, 99% d. SPEP dont have myeloma. e. Urine U i immunoelectrophoresis i l t h i Most IgG spikes are (Bence-Jones protein) from monoclonal
gammopathy of unknown significance that doesnt progress or need treatment.
MTBS2CK p.230231

24

MultipleMyeloma/Treatment Best initial therapy is...


Combination of steroids with lenalidomide, bortezomib, or melphalan

MonoclonalGammopathyof UnknownSignificance IgG or IgA spike on SPEP is common in older patients Evaluate with bone marrow biopsy to exclude myeloma Monoclonal gammopathy of unknown significance (MGUS) has small numbers of plasma cells No therapy for MGUS 1% a year transform into myeloma The quantity of immunoglobulin in the spike is main correlate of risk for myeloma: More MGUS = More myeloma
MTBS2CK p.231

Most effective therapy under age 70 is autologous bone marrow transplant with stem cell support

Myeloma therapy is in a state of rapid flux due to numerous advances

MTBS2CK p.231

WaldenstrmMacroglobulinemia Overproduction of IgM Malignant B cells lead to hyperviscosity Presents with... Lethargy Blurry Bl vision i i and d vertigo ti Engorged blood vessels in eye Mucosal bleeding Raynaud phenomenon

WaldenstrmMacroglobulinemia Anemia common IgM spike on SPEP results in hyperviscosity No bone lesions Plasmapheresis is best initial therapy Removes IgM and viscosity L Long-term t treatment t t t with ith chlorambucil hl b il or fludarabine and prednisone Control cells that make abnormal Igs Decrease means of production

MTBS2CK p.231

MTBS2CK p.231

BleedingDisorders First step in evaluation is determining if bleeding is from platelets or clotting factors Platelet Bleeding Superficial Epistaxis, gingival, petechiae, purpura, gums, vaginal bleeding Factor Bleeding Deep Joints and muscles

BleedingDisorders
ITP VonWillebrandDisease Hemophilia FactorXIDeficiency DIC Thrombophilia HeparinInducedThrombocytopenia

Bleeding in brain or GI system can be from either platelet or clotting factor deficiency
MTBS2CK p.232

25

Immune(Idiopathic)ThrombocytopenicPurpura(ITP)

Look for... Isolated thrombocytopenia


Normal Hct Normal N l WBC count t

23-year-old woman comes to ED with increased menstrual bleeding, gum bleeding when she brushes her teeth, and petechiae. Platelet count: 17,000/L (low) What is the next step in therapy? a. Bone marrow biopsy b. Intravenous immunoglobulins c. Prednisone d. Antiplatelet antibodies e. Platelet transfusion
MTBS2CK p.232

Normal-sized spleen

MTBS2CK p.232

Prednisone is more important than checking for increased megakaryocytes or antiplatelet antibodies, which is characteristic of ITP.

ITP

ITP/Treatment

Diagnosis of exclusion Occasional tests are:


Antiplatelet antibodies lack specificity, limited benefit Ultrasound or CT scan to exclude hypersplenism Megakaryocytes are elevated in number

Presentation
Nobleeding,count>30,000 Mildbleeding,count<30,000 Severebleeding(GI/CNS), count<10,000 Recurrentepisodes,steroid dependent Splenectomyorsteroidsnot effective

Management
Notreatment Glucocorticoids IVIG,AntiRho(antiD)

Splenectomy Romiplostimoreltrombopag for recurrencesaftersplenectomy Rituximab,azathioprine, cyclosporine,mycophenolate

MTBS2CK p.232

MTBS2CK p.233

ITP

VonWillebrandDisease(VWD) Most common inherited bleeding disorder Von Willebrand factor (VWF) level or function Autosomal dominant Look for bleeding related to platelets (epistaxis, gingival, gums) with normal platelet count Markedly worsened after aspirin aPTT elevated in 50% of patients

Before splenectomy, give vaccination to: Neisseria meningitidis Hemophilus influenzae Pneumococcus

MTBS2CK p.233

MTBS2CK p.233

26

VonWillebrandDisease(VWD) Diagnosis Bleeding time: increased duration of bleeding VWF (antigen) level may be decreased Ristocetin cofactor assay: detects VWF dysfunction Treatment Initial therapy: DDAVP (desmopressin), which releases subendothelial stores of VWF If no response, use factor VIII replacement or VWF concentrate
MTBS2CK p.233

Hemophilia Look for... Delayed joint or muscle bleeding in male child


Bleeding delayed because primary hemostatic plug is with platelets

Most accurate test is...


Specific assay for factor VIII or IX

Prothrombin time (PT) normal aPTT prolonged

Mixing studies with normal plasma correct aPTT to normal Treat mild cases with DDAVP Severe bleeding treated with replacement of specific factor

MTBS2CK p.234

FactorXIDeficiency Most of time, no increase in bleeding with factor XI deficiency With trauma or surgery, theres increased bleeding Look for... Normal PT with p prolonged g aPTT Mixing study: corrects aPTT to normal Treatment Use FFP to stop bleeding

DisseminatedIntravascularCoagulation(DIC) DIC doesnt occur in otherwise healthy people Look for a definite risk such as...
Sepsis Burns Abruptio placenta or amniotic fluid embolus Snake bites Trauma resulting in tissue factor release Cancer

Theres bleeding related to both clotting factor deficiency as well as thrombocytopenia

MTBS2CK p.234

MTBS2CK p.234

DisseminatedIntravascularCoagulation(DIC)

DisseminatedIntravascularCoagulation(DIC)

Diagnostic Tests Elevated PT and aPTT Low platelet count Elevated D-dimer & fibrin split products Decreased fibrinogen level (has been consumed)

Treatment Replace platelets < 50,000/L as well as clotting factors with FFP Heparin has no benefit Cryoprecipitate C i it t may be b effective ff ti to t replace fibrinogen levels if FFP doesnt control bleeding

MTBS2CK p.234

MTBS2CK p.235

27

HypercoaguableStates/Thrombophilia

HeparinInducedThrombocytopenia HIT more common with unfractionated heparin


Can still occur with LMW heparin

MCC:
Factor V Leiden mutation

No difference in intensity of anticoagulation Warfarin to INR of 2 2-3 3 for 6 months

Presents 5 to 10 days after start of heparin with a marked drop in platelet count (> 30%) Both venous and arterial thromboses occur
Venous clots more common

HIT rarely leads to bleeding Platelets just precipitate out Diagnostic Tests HIT confirmed with ELISA for platelet factor 4 (PF4) antibodies or serotonin release assay
MTBS2CK p.235

MTBS2CK p.235

HeparinInducedThrombocytopenia Treatment Immediately stop all heparin-containing products Cant just switch unfractionated heparin to LMW heparin Direct thrombin inhibitors
Argatroban, Argatroban lepirudin, lepirudin and bivalirudin

AntiphospholipidSyndromes Two main syndromes - lupus anticoagulant and anticardiolipin antibody Both cause thrombosis Anticardiolipin antibodies associated with multiple spontaneous abortions Antiphospholipid (APL) syndromes specifically cause thrombophilia with abnormal aPTT

After direct thrombin inhibitor is started, use warfarin

MTBS2CK p.235

MTBS2CK p.235

AntiphospholipidSyndromes Best initial test is...


Mixing study

Because its a circulating inhibitor, aPTT remains elevated after mix Most accurate test for lupus anticoagulant is...
Russell viper venom test

Treatment Treat with heparin and warfarin as you would for any cause of DVT or PE APL syndrome may require lifelong anticoagulation

MTBS2CK p.235

28

IntroductiontoAntibiotics InfectiousDiseases
ConradFischer,MD AssociateProfessorofMedicine TouroCollegeofMedicine NewYorkCity
PrinciplesofAnsweringQuestions BetalactamAntibiotics Fluoroquinolones Aminoglycosides Doxycycline Trimethoprim/Sulfamethoxazole Betalactam/BetalactamaseCombinations SpecificOrganismGroupsandTheirTreatments

IntroductiontoAntibiotics

IntroductiontoAntibiotics

Organisms associated with diseases dont change But antibiotics that treat them change M Most t important i t t thing thi Antibiotics associated with each group of organisms

Principles of Answering Infectious Diseases Questions The radiologic test is never the most accurate test Risk factors for an infection aren arent t as important as individual presentation Beta-lactam antibiotics have greater efficacy than other classes
MTBS2CK p.3

MTBS2CK p.3

BetalactamAntibiotics

Penicillins

Penicillins, Cephalosporins, Carbapenems, Aztreonam Penicillins Penicillin ( (G, , VK, , benzathine) )


Viridans group streptococci Streptococcus pyogenes Oral anaerobes Syphilis Leptospira
MTBS2CK p.3

Ampicillin and amoxicillin


Cover same organisms as penicillin And E. coli Lyme disease Gram-negative bacilli (few)

MTBS2CK p.3

Penicillins

Penicillins

Answer as Best Initial Therapy for Gram-negative bacteria covered by amoxicillin H. influenzae E. coli Listeria Proteus Salmonella
Otitis media Dental infection and endocarditis prophylaxis Lyme disease limited to rash, joint, or 7th CN involvement UTI in pregnant women Listeria monocytogenes Enterococcal infections
MTBS2CK p.4

MTBS2CK p.3

Penicillins

Penicillinaseresistantpenicillins(PRPs)

Oxacillin, Cloxacillin, Dicloxacillin, and Nafcillin Skin infections: Cellulitis Endocarditis, meningitis, and bacteremia from staphylococci Osteo & Septic arthritis only when proven sensitive Not against Methicillin-Resistant Staphylococcus aureus (MRSA) or Enterococcus
MTBS2CK p.4

Methicillin is never right Causes allergic interstitial nephritis Methicillin sensitive or resistant means oxacillin sensitive or resistant.

MTBS2CK p.4

Penicillins

Piperacillin,Ticarcillin,Azlocillin,Mezlocillin

Piperacillin, Ticarcillin Gram-negative bacilli (e.g., E. coli, Proteus) enterobacteriaciae & pseudomonads Best initial therapy
Cholecystitis & ascending cholangitis Pyelonephritis Bacteremia Hospital-acquired and ventilator-associated pneumonia Neutropenia and fever
MTBS2CK p.4

Also for: streptococci and anaerobes BUT NOT the answer, infection exclusively from these single organisms Use narrower agent Combined with a beta-lactamase inhibitor such as tazobactam or clavulanic acid
MTBS2CK p.4

Cephalosporins Cross-reaction penicillin & cephalosporins is

Cephalosporins

Very small (3% to 5%)


All cephalosporins, in every class, will cover group A, B, and C streptococci, viridans group streptococci, E. coli, Klebsiella, and Proteus mirabilis

If the case describes a rash to penicillin Answer cephalosporins If the case describes anaphylaxis You must use a non-beta-lactam antibiotic

Listeria, MRSA, and Enterococcus are resistant to all forms of cephalosporins.


MTBS2CK p.4 MTBS2CK p.5

FirstGeneration

SecondGeneration

Cefazolin, Cephalexin, Cephradrine, Cefadroxyl 1st-gen cephalosporins treat Staphylococci


Methicillin sensitive = oxacillin sensitive = cephalosporin sensitive

Cefotetan, Cefoxitin, Cefaclor, Cefprozil, Cefuroxime, Loracarbef


Cover same organisms as 1st-gen cephalosporins AND Add coverage for anaerobes & more gram-negative bacilli

Streptococci (except Enterococcus) Some gram-negative bacilli such as E. coli, but not Pseudomonas Osteomyelitis, septic arthritis, endocarditis, cellulitis
MTBS2CK p.5

MTBS2CK p.5

SecondGeneration Cefotetan, Cefoxitin, Cefaclor, Cefprozil, Cefuroxime, Loracarbef Cefotetan or cefoxitin Best initial therapy for pelvic inflammatory disease (PID) combined with doxycycline Warning Cefotetan and cefoxitin risk of bleeding and give a disulfiram-like reaction with alcohol Cefuroxime, loracarbef, cefprozil, cefaclor Respiratory infections (e.g., bronchitis, otitis media, and sinusitis)
MTBS2CK p.5

ThirdGeneration

Ceftriaxone, Cefotaxime, Ceftazidime Ceftriaxone First-line for pneumococcus, including partially insensitive organisms
Meningitis Community-acquired pneumonia (in combination with macrolides) Gonorrhea Lyme involving heart or brain Avoid ceftriaxone in neonates because of impaired biliary metabolism
MTBS2CK p.5

ThirdGeneration

FourthGeneration

Cefotaxime

Superior to ceftriaxone in neonates Spontaneous bacterial peritonitis


C ft idi Ceftazidime has h pseudomonal d l coverage

Cefepime Better staphylococcal coverage compared with the 3rd-generation cephalosporins Neutropenia and fever Ventilator-associated pneumonia

Ceftaroline: only cephalosporin to cover MRSA

MTBS2CK p.5

MTBS2CK p.5

Carbapenems

Monobactams

Imipenem, Meropenem, Ertapenem, Doripenem Cover gram-negative bacilli, including many thatre resistant Anaerobes Streptococci and staphylococci Neutropenia and fever Ertapenem differs from other carbapenems. Ertapenem does not cover Pseudomonas
MTBS2CK p.6

Aztreonam Only monobactam used

Exclusively
Gram-negative bacilli Including I l di Pseudomonas P d

No cross-reaction with penicillin

MTBS2CK p.6

Fluoroquinolones

Fluoroquinolones

Ciprofloxacin, Gemifloxacin, Levofloxacin, Moxifloxacin

Community-acquired pneumonia, including penicillin-resistant pneumococcus Gram-negative bacilli including most pseudomonads Cipro: NOT for pneumococcus

Ciprofloxacin: Cystitis and pyelonephritis Moxifloxacin: Not for urine/cystitis Diverticulitis and GI infections Combined with metronidazole Don Dont t cover anaerobes except moxifloxacin Moxifloxacin as single agent for diverticulitis without metronidazole

MTBS2CK p.6

MTBS2CK p.6

Fluoroquinolones

Fluoroquinolones

Quinolones cause Bone growth abnormalities in children and pregnant women Tendonitis and achilles tendon rupture Gatifloxacin removed because of glucose abnormalities

Source: Grook Da Oger, commons.wikimedia.org

MTBS2CK p.6

MTBS2CK p.6

Aminoglycosides

Doxycycline

Gentamicin, Tobramycin, Amikacin Gram-negative bacilli (bowel, urine, bacteremia) Synergistic with beta-lactam beta lactam antibiotics for enterococci and staphylococci No effect against anaerobes (need oxygen to work) Nephrotoxic and ototoxic
MTBS2CK p.6

Chlamydia Lyme Rash Joint 7th CN palsy MRSA skin

MTBS2CK p.7

Target-shaped rash of Lyme disease or erythema migrans. Source: Nishith Patel.

Doxycycline

Trimethoprim/Sulfamethoxazole

Rickettsia Syphilis: Primary & secondary ONLY if allergic to penicillin Borrelia, Ehrlichia, and Mycoplasma Ad Adverse effects ff t Tooth discoloration (children) Fanconi syndrome (Type II RTA proximal), photosensitivity Esophagitis
MTBS2CK p.7

Cystitis Pneumocystis pneumonia treatment and prophylaxis MRSA of skin and soft tissue (cellulitis) Rash Hemolysis (G6PD deficiency) Marrow suppression (folate antagonist)
Nitrofurantoin has one indication: Cystitis, especially in pregnant women.
MTBS2CK p.7

Betalactam/BetalactamaseCombinations

Amoxicillin/clavulanate Ticarcillin/clavulanate Ampicillin/sulbactam Piperacillin/tazobactam Beta lactamase adds staphylococci effect Beta-lactamase Sensitive Staph only

GramPositiveCocci:Staphylococciand Streptococci

Best initial therapy: Oxacillin, cloxacillin, dicloxacillin, nafcillin First-generation cephalosporins: Cefazolin, cephalexin Fluoroquinolones Macrolides (azithromycin, clarithromycin, erythromycin) are third-line
Less efficacy than oxacillin or cephalosporins Erythromycin more toxic

MTBS2CK p.7

MTBS2CK p.7

Oxacillin(Methicillin)ResistantStaph

Anaerobes

Vancomycin Linezolid: Reversible bone marrow toxicity Daptomycin Tigecycline Ceftaroline


Minor MRSA infections of skin are treated with: TMP/SMX Clindamycin Doxycycline

Oral (above diaphragm) Penicillin (G, VK, ampicillin, amoxicillin) Clindamycin Metronidazole (GI) Piperacillin, carbapenems, and 2nd-generation cephalosporins also cover anaerobes

MTBS2CK p.7

MTBS2CK p.8

GramNegativeBacilli(E.coli,Klebsiella, Proteus,Pseudomonas,Enterobacter,Citrobacter)

Bowel (peritonitis, diverticulitis) Urinary tract (pyelonephritis) Cholecystitis or Cholangitis


Quinolones Aminoglycosides Carbapenems Piperacillin, ticarcillin Aztreonam Cephalosporins
MTBS2CK p.8

Man admitted with E. coli bacteremia. Which of the following is the most appropriate therapy?

a. Vancomycin b. Linezolid c. Quinolones, aminoglycosides, carbapenems, piperacillin, ticarcillin, or aztreonam d. Doxycycline They dont cover grame. Clindamycin negative bacilli. f. Oxacillin

MTBS2CK p.8

CentralNervousSystem(CNS)Infections

CentralNervousSystemInfections
Meningitis g Encephalitis

All CNS infections give Fever Headache Vomiting Seizures

MTBS2CK p.9

CluestoAnsweringthe MostLikelyDiagnosisQuestion

Meningitis/Definition

Symptom Stiffneck Photophobia Meningismus Confusion Focalneurological findings


MTBS2CK p.

Diagnosis Meningitis

Infection/inflammation of CNS covering

Encephalitis Abscess
Source: SVG by Mysid, original by SEER Development Team commons.wikimedia.org

Meningitis/Etiology:4bugs95%ofcases Streptococcus pneumonia (60%) Group B streptococci (14%) Haemophilus influenzae (7%) Neisseria meningitidis (15%) Listeria (2%) Staphylococcus with recent neurosurgery

Meningitis/Presentation

Fever Headache Neck stiffness (nuchal rigidity) Photophobia


Patient with Neck stiffness. Source: Sophian, Abraham: Epidemic cerebrospinal meningitis (1913), St. Louis, C.V Mosby , commons.wikimedia.org

S. Pneumoniae Source: CDC/Dr. M.S. Mitchell

MTBS2CK p.9

MTBS2CK p.9

Meningitis/Presentation

WhatistheMostLikelyDiagnosis?
AIDS with <100 CD4 cells/L Stiff Neck Photophobia Adolescent, petechial rash

Acute bacterial (presents in several hours) Focal abnormalities: 30% If confusion occurs, you wont be able to answer What is the most likely diagnosis? without a CT and lumbar puncture (LP) Cryptococcal meningitis: Slow several weeks

None Cryptococcus Camper/hiker Rash R h target-like lik shape Joint pain Facial palsy Tick remembered in 20% Camper/hiker Rash moves from arms/legs t trunk to t k Tick remembered in 60%

Neisseria

Viral

Pulmonary TB in 85%

Lyme disease
MTBS2CK p.9 MTBS2CK p.9

Rocky Mountain spotted fever (Rickettsia)

Tuberculosis

Meningitis/DiagnosticTests

CerebrospinalFluidEvaluation
Bacterial Cell count 1000s, Neutrophils Cryptococcus, Tuberculosis Viral Lyme,Rickettsia 10s100s 10s100s 10s100s Lymphocytes Lymphocytes Lymphocytes Possibly elevated Possibly decreased Negative Markedly elevated Maybelow Usually normal Usually normal Negative

Best initial test and Most accurate test Lumbar Puncture

Protein Elevated level Glucose Decreased level Stain Stain: and 5070%; Culture Culture: 90%
MTBS2CK p.10
Source: commons.wikimedia.org

Negative

MTBS2CK p.10

Meningitis/DiagnosticTests

Meningitis/DiagnosticTests

When Is Head CT the Best Initial Test? If before the LP, there is

Papilledema Seizures Focal neurological abnormalities Confusion

Blurred disc margin

Neuro exam needs patient who understands and follows instructions and answers questions

You cannot do an accurate neurologic exam if the patient is severely confused.


MTBS2CK p.10

Papilledema is a blurred, fuzzy disc margin from increased intracranial pressure. Source: Conrad Fischer, MD

MTBS2CK p.10

Meningitis/DiagnosticTests

BacterialAntigenDetection (LatexAgglutinationTests)

If theres a contraindication to immediate LP, giving antibiotics is best initial step in management

Better B tt to t treat t t and d decrease the accuracy of a test than to risk permanent brain damage.
MTBS2CK p.10

Similar accuracy to Gram stain If positive, theyre extremely specific If negative, could still have infection Not sufficiently sensitive to exclude bacterial meningitis i iti When is a bacterial antigen test indicated? Those receiving antibiotics prior to LP Culture may be falsely negative
MTBS2CK p.11

WhatistheMostAccurateDiagnosticTest?
Tuberculosis Lyme and Rickettsia Cryptococcus Viral

Meningitis/Treatment Bacterial: Ceftriaxone, Vancomycin, and Steroids Base treatment answer on cell count Culture Needs 2 to 3 days Never available when treatment decision is made G Gram stain t i Good if positive False negative: 30% to 50% Protein & Glucose Nonspecific Doesnt allow treatment decision
MTBS2CK p.11

Specific serologic testing, ELISA, western blot, PCR Acid fast stain and culture on 3 high-volume lumbar punctures Centrifuge to concentrate the organisms TB has high CSF protein Uncentrifuged sample of CSF: 10% sensitivity

Diagnosis of exclusion

India ink is 60% to 70% sensitive

Cryptococcal antigen: > 95% sensitive and specific

MTBS2CK p.11

Meningitis/Treatment Steroids (dexamethasone) Lowers mortality only in S. pneumoniae Give when thousands of neutrophils present No culture results for several days

ListeriaMonocytogenes Resistant to cephalosporins Sensitive to penicillins Add Ampicillin to Ceftriaxone and Vancomycin if case describes risk factors for Listeria. Risks
Elderly Neonates Steroid use AIDS or HIV Immunocompromised (includes alcoholism) Pregnant

Thousands of neutrophils on CSF = ceftriaxone, vancomycin, and steroids. Add ampicillin if immunocompromised for Listeria.
MTBS2CK p.11

MTBS2CK p.11

Neisseriameningitidis

Neisseriameningitidis

Respiratory isolation Rifampin or Ciprofloxacin to close contacts Close contacts Major respiratory fluid contact Household contacts Kissing Sharing cigarettes or eating utensils

Routine school and work contacts are not close contacts Healthcare workers qualify only if they
Intubate patient Perform suctioning Or Have contact with respiratory secretions

MTBS2CK p.12

MTBS2CK p.12

Meningitis

Man comes to emergency department with fever, severe headache, neck stiffness, and photophobia. He has weakness in his left arm and leg. Whats the next step in management? a. Ceftriaxone, Vancomycin, and Steroids b. Head CT Focal neurological deficits, initiate therapy c. Ceftriaxone Almost always d. Neurology consultation wrong e. Steroids Not sufficient

Consultation is almost always a wrong answer on USMLE Step 2 CK

What is the most common neurological l i ld deficit fi it of f untreated bacterial meningitis? Eighth cranial nerve deficit or deafness.
MTBS2CK p.12

MTBS2CK p.12

Encephalitis Acute onset


Fever, and Confusion

What is the most accurate test of herpes encephalitis? a.Brain biopsy Less accurate than PCR b.PCR of CSF c MRI c.MRI
The radiologic test is never the most acc accurate rate test genital, skin lesions

Many causes
Herpes simplex (most common) ( )

Must do head CT first because of confusion

d.Viral culture of CSF Most accurate test for e.Tzanck prep Best initial test for genital lesion f. Serology for herpes (IgG, IgM) population
Useless, 95% positive

MTBS2CK p.12

MTBS2CK p.13

10

Encephalitis/Treatment

Acyclovir Best initial therapy (Herpes encephalitis) Famciclovir & Valacyclovir IV unavailable Foscarnet Acyclovir-resistant herpes
MTBS2CK p.13

A woman with herpes encephalitis confirmed by PCR gets 4 days of acyclovir. Her creatinine level rises. Whats the most appropriate next step in management? a. Stop acyclovir Important to treat

a Reduce the dose of acyclovir and hydrate a. b. Switch to oral famciclovir or valacyclovir c. Switch to foscarnet

Insufficient for herpes encephalitis > Nephrotoxicity than acyclovir

MTBS2CK p.13

OtitisMedia

HeadandNeckInfections
OtitisMedia Sinusitis Pharyngitis Influenza(TheFlu)

Redness Immobility Bulging Decreased light reflex Pain Decreased hearing Fever

Otitis media. Source: www.sharinginhealth.ca MTBS2CK p.13

OtitisMedia/DiagnosticTests/Treatment

Which of the following is the most sensitive physical finding for otitis media? a. Redness b. Immobility c. Bulging d. Decreased light reflex e. Decreased hearing

Tympanocentesis: Sample of fluid for culture Most accurate diagnostic test Choose tympanocentesis if: Multiple recurrences No response to multiple antibiotics

Theyre less sensitive


MTBS2CK p.1314

Radiologic tests for otitis are always the wrong answer


MTBS2CK p.14

11

OtitisMedia/Treatment

Amoxicillin: Best initial therapy If no response to amoxicillin or it recurs, answer: Amoxicillin/clavulanate Azithromycin A ith i or clarithromycin l ith i Cefuroxime or loracarbef Levofloxacin, gemifloxacin, moxifloxacin Quinolones are relatively contraindicated in children
MTBS2CK p.14

34-year-old woman with facial pain, discolored nasal discharge, bad taste in mouth, fever and facial tenderness. Which of the following is the most accurate diagnostic test? a Sinus biopsy or aspirate a.Sinus Radiologic test never the most b.CT scan accurate test c.X-ray d.Culture of the discharge Always the wrong answer for sinusitis e.Transillumination Doesnt provide precise microbiological diagnosis MTBS2CK p.14

Sinusitis

Culture of nasal discharge is always the wrong answer for sinusitis

34-year-old woman with facial pain, discolored nasal discharge, bad taste in her mouth, and fever. She has facial tenderness. What is the most appropriate next step, action, or management? a. a b. c. d. e. Linezolid doesnt cover Haemophilus CT scan Diagnosis is clear, radiologic testing X-ray unnecessary Amoxicillin and decongestant Erythromycin and decongestant
Poor coverage for S. pneumoniae Excellent for resistant g gram-positive, p

Use Sinus Biopsy or Aspirate Infection frequently recurs No response to different empiric therapies

MTBS2CK p.14

MTBS2CK p.15

Pharyngitis Pain on swallowing Enlarged lymph nodes in neck Exudate in pharynx Fever No cough and no hoarseness
Streptococcal pharyngitis. Source: James Heilman, MD

Pharyngitis/DiagnosticTests Best initial test: Rapid strep test An office/clinic-based test Finds group A beta-hemolytic streptococci in minutes Negative test excludes disease and no antibiotics are needed Positive rapid strep test = positive pharyngeal culture Small vesicles or ulcers: HSV or herpangina Membranous exudates: Diphtheria, Vincent angina, or EBV
MTBS2CK p.15

These features make the likelihood of streptococcal pharyngitis exceed 90%

MTBS2CK p.15

12

Pharyngitis/Treatment

Influenza(TheFlu)

1. Penicillin or amoxicillin is best initial therapy 2. Penicillin allergic:


Cephalexin (reaction only rash)

Clindamycin or macrolide (anaphylaxis)

Streptococcal pharyngitis treated to prevent rheumatic fever

Arthralgias/myalgias Cough Fever Headache Sore throat Nausea, vomiting, or diarrhea, especially in children

Source: Mikael Hggstrm

MTBS2CK p.15

MTBS2CK p.16

Influenza(TheFlu)

Influenza(TheFlu)/Treatment < 48 hours of symptoms: Oseltamivir, zanamivir Neuraminidase inhibitors shorten duration of symptoms Treats both influenza A and B > 48 hours of symptoms: Symptomatic treatment only Analgesics, rest, antipyretics, hydration

The most appropriate next step in management depends on time course from presentation Within 48 hours of onset Perform nasopharyngeal swab or wash Rapidly detects antigen associated with influenza

Oseltamivir and zanamivir dont successfully treat complications of influenza such as pneumonia.
MTBS2CK p.16 MTBS2CK p.16

BloodandWBCsinStool

InfectiousDiarrhea
BloodandWBCsinStool NoBloodorWBCsinStool

When you see...in the history the answer is. Poultry Salmonella Campylobacter: MCC associated with Guillain-Barre Syndrome E. coli 0157:H7Hemolytic Uremic Syndrome (HUS) Shigella: 2nd most common association with HUS
MTBS2CK p.16

13

BloodandWBCsinStool

BloodandWBCsinStool The Best initial test is: Blood and/or fecal leukocytes Wont determine specific organism Stool lactoferrin: Greater sensitivity iti it & specificity ifi it than stool leukocytes

Shellfish and cruise ships Vibrio parahaemolyticus: Shellfish, liver & skin disease Vibrio vulnificus: Hemochromatosis blood transfusions Hemochromatosis, Yersinia: High affinity for iron Antibiotics, white and red cells in stool Clostridium difficile
MTBS2CK p.16

WBCs in a stool sample (Methylene Blue Stain). Source: Bobjgalindo

Lactoferrin: Better answer than fecal leukocytes if one of the choices Most accurate test is stool culture
MTBS2CK p.16

No BloodNo WBCsinStool
Viral Giardia:
Camping/hiking and unfiltered fresh water

Treatment

Cryptosporidiosis:
AIDS <100 CD4 cells Test acid fast stain

Mild disease: Oral fluid replacement Severe disease: Fluid replacement and ciprofloxacin
Severe infectious diarrhea means: Hypotension Tachycardia Fever Abdominal pain Bloody diarrhea Metabolic acidosis
MTBS2CK p.17

Bacillus cereus: Vomiting Staphylococcus: Vomiting


Scombroid Most rapid onset Wheezing, flushing, rash Found in fish Treat with antihistamines

MTBS2CK p.1617

DiseaseSpecificTreatment
Organism Treatment

Disease SpecificTreatment Metronidazole, Giardia

Tinidazole Cryptosporidiosis TreatunderlyingAIDS, Ni Nitazoxanide id Fluidsupportasneeded Fluidsupportasneeded

Hepatitis
AcuteHepatitis ChronicHepatitis p

Viral B.cereus, Staphylococcus


MTBS2CK p.17

14

AcuteHepatitis/Definition/Etiology

AcuteHepatitis

Infection or inflammation of liver Majority of acute cases are viral hepatitis A/B Hepatitis C Rarely presents with acute infection Found on blood tests for liver function Evaluation of cirrhosis Hepatitis D Exclusively as coinfection of hepatitis B IDUs
MTBS2CK p.17

Hepatitis E Worst in pregnancy East Asia S Sex, Blood, Bl d M Mom: H Hepatitis titi B B, C C, D Food and water (enteric): Hepatitis A and E You Ate hepatitis A & you Eat hepatitis E

MTBS2CK p.18

AcuteHepatitis/Presentation

AcuteHepatitis/DiagnosticTests

No way to detect specific type from symptoms All forms present with: Jaundice Fever, F weight i ht loss, l and df fatigue ti Dark urine Hepatosplenomegaly Nausea, vomiting, abdominal pain
MTBS2CK p.18

Increased direct bilirubin Increased ratio of alanine aminotransferase (ALT) to aspartate aminotransferase (AST) Increased alkaline phosphatase

Aplastic anemia is a rare complication of acute hepatitis

MTBS2CK p.18

DiseaseSpecificDiagnosticTests

Which correlates with increased mortality?

a. b. c. d d. e.

Bilirubin Prothrombin time ALT AST Alkaline phosphatase

All can be markedly elevated and better fast, except PT

Hepatitis A, C, D, and E: Best initial diagnostic test IgM antibody: Acute infection IgG antibody: Resolution of infection Hepatitis C: Disease activity PCR RNA level = Amount of active viral replication Hepatitis C PCR level = First thing to change PCR drop = Improvement with treatment PCR rises = Treatment failure
MTBS2CK p.18

MTBS2CK p.18

15

SerologicPatterns
Acuteorchronic infection Resolved,old,past infection

SerologicPatterns
Vaccination Windowperiod

Surface antigen eantigen

Positive

Negative

Surface antigen eantigen

Negative

Negative

Positive

Negative PositiveIgG Positive

Negative

Negative PositiveIgM,then IgG Negative

Coreantibody PositiveIgMorIgG Surface antibody Negative

Coreantibody Negative Surface antibody Positive

MTBS2CK p.19

MTBS2CK p.19

Which becomes abnormal first after acquiring hepatitis B infection? a. b. c. d. e. f. Bilirubin Viral replication rises after S Ag e-antigen Surface antigen Measure of bodys Core IgM antibody response to infection ALT Anti-hepatitis B e-antibody Resolution
starting

Which of the following is the most direct correlate with the amount/quantity of active viral replication?

a. b. c. d. e. f.

Bilirubin e-antigen Doesnt tell quantity Surface antigen Core IgM antibody Measure of bodys response to infection ALT Anti-hepatitis B e-antibody

Will appear prior to resolution of all DNA polymerase activity

MTBS2CK p.19

MTBS2CK p.19

Which indicates you cant transmit infection (i.e., active infection has resolved)? a. Bilirubin normalizes b No e-antigen b. e antigen found Normalize long before viral replication stops. No viral replication No serological evidence of disease Will appear prior to resolution of all DNA polymerase activity

Which is the best indication of the need for treatment in chronic disease? b. e-antigen c. Surface antigen a. Bilirubin

c. No surface antigen found d. No core IgM antibody found e. ALT normalizes f. Anti-hepatitis B e-antibody
MTBS2CK p.1920

At least some active disease, it might be on the way to spontaneous resolution and d. Core IgM antibody wouldnt benefit e. ALT Measure of bodys f. Anti-hepatitis B e-antibody response to infection

MTBS2CK p.20

16

Acutehepatitis/Treatment Which of the following is the best indicator that a pregnant woman will transmit infection to her child? At least some active disease, it might a. Bilirubin be on the way to spontaneous b. e-antigen resolution and wouldnt benefit. c. Surface antigen Perinatal transmission: 10% if d. Core IgM antibody positive surface antigen, but eantigen is negative; 90% when e. ALT both positive f. Anti-hepatitis B, e-antibody Hepatitis A and E: Resolve spontaneously over weeks Almost always benign conditions Only acute hepatitis C Hepatitis B: gets medical therapy Ch i i Chronic in 10% No treatment for acute disease Hepatitis C: Use interferon, ribavirin and either telaprevir or boceprevir!!! Treatment decreases likelihood of chronic infection
MTBS2CK p.21

Measure of the bodys response to infection


MTBS2CK p.20

ChronicHepatitis/Treatment Chronic hepatitis B = Surface antigen > 6 months e-antigen = Elevated level of DNA polymerase Treat when BOTH surface and e-antigen are positive Entecavir, or Adefovir, or T Tenofovir, f i or Lamivudine, or Telbivudine, or Interferon (joint & muscle pain, depression, the flu) Interferon is an injection Interferon has most adverse effects
MTBS2CK p.21

ChronicHepatitis/Treatment Adverse effects of interferon: Arthralgia/myalgia Leukopenia & thrombocytopenia Depression & flu-like symptoms

Goal of hepatitis therapy: Reduce DNA polymerase Convert e-antigen to anti-hepatitis e-antibody

MTBS2CK p.21

RoleofLiverBiopsy

ChronicHepatitis

Fibrosis is a strong indication to begin therapy for either hepatitis B or C right away Fibrosis + Active Viral replication will progress to cirrhosis Cirrhosis: Irreversible Who will progress? Old terms chronic active or chronic persistent hepatitis are unhelpful DNA polymerase level is helpful
MTBS2CK p.21

ALT levels arent a good indication of chronic hepatitis activity. You can have significant infection with normal transaminase levels.

MTBS2CK p.21

17

TreatmentofChronicHepatitisC

TreatmentofChronicHepatitisC

Theres no way to determine the duration of infection with hepatitis C, since theres no equivalent of surface antigen test Most patients dont have acute symptoms PCR RNA viral load is elevated, patients If PCR-RNA should be treated with interferon and ribavirin and either boceprevir or telaprevir

Three Drugs! 80% resolution with 3 drugs Interferon AND Ribavirin AND either Telaprevir OR Boceprevir

Ribavirin causes anemia


MTBS2CK p.21 MTBS2CK p.2122

Urethritis

SexuallyTransmittedDiseases
Urethritis Cervicitis PelvicInflammatoryDisease(PID) Syphilis GenitalWarts(CondylomataAcuminata) Pediculosis(Crabs) Scabies

Look for urethral discharge to answer What is the most likely diagnosis? Both urethritis & cystitis give: Dysuria Frequency Burning Cystitis: No discharge

MTBS2CK p.22

Urethritis/DiagnosticTests Best initial test Men: Urethral swab for Gram stain & WBCs Intracellular gram-negative diplococci = Neisseria gonorrhoeae Urine for nucleic acid amplification test (NAAT) detects gonorrhea and Chlamydia NAAT or DNA probe: Most accurate test Other causes of urethritis: Mycoplasma genitalium Ureaplasma Women: Self-administered vaginal swab
MTBS2CK p.22

Urethritis/Treatment

Combine: One drug for gonorrhea & one for Chlamydia Quinolones arent the best initial therapy because of resistance
Gonorrhea Chlamydia

Cefixime Ceftriaxone

Azithromycin Doxycycline

MTBS2CK p.22

18

Cervicitis

PID/Presentation

Cervical discharge Inflamed strawberry cervix Testing & treatment are identical to previous description for urethritis Except E t self-administered lf d i i t d vaginal i l swab b for NAAT

Lower abdominal tenderness Lower abdominal pain Fever Cervical motion tenderness Leukocytosis

Always to exclude pregnancy first!!

MTBS2CK p.22

MTBS2CK p.2223

PID/DiagnosticTests

LaparoscopyinPID

Cervical swab for: Culture DNA probe or nucleic acid amplification (NAAT) Tests clarify y need for treating gp partner Culture preferred to determine resistance Cervical testing isnt the most accurate test for PID.
MTBS2CK p.23

The most accurate test for PID Only rarely needed Laparoscopy is answer if: Diagnosis is unclear Symptoms persist despite therapy Recurrent episodes for unclear reasons

MTBS2CK p.23

PID/Treatment

Combination of medications for gonorrhea and Chlamydia Inpatient: Cefoxitin or cefotetan combined with doxycycline Outpatient: Ceftriaxone and doxycycline (possibly with metronidazole) Anaphylaxis to penicillin?: Levofloxacin and metronidazole (outpatient) Clindamycin and gentamicin (inpatient)
MTBS2CK p.23

UlcerativeGenitalDisease/ WhatIstheMostLikelyDiagnosis? Often impossible to determine specific diagnostic, visual characteristics Question must have sufficient evidence to g give answer

All genital ulcers can have inguinal adenopathy


MTBS2CK p.23

19

PresentationofSTDs
Historyandphysicalfindings Mostlikelydiagnosis

DiagnosticTests
Diagnosis Syphilis DiagnosticTest Darkfieldmicroscopy(Bestinprimary) VDRLorRPR(75%sensitiveinprimary) FTAorMHATP(confirmatory) Stain&culture(specialmedia)

Painlessulcer Painfululcer

Syphilis Chancroid (Haemophilus ducreyi) Lymphogranuloma venereum Herpessimplex

Chancroid(Haemophilus ducreyi) Lymphogranuloma venereum

Lymphnodestenderand suppurating Vesiclespriortoulcer andpainful


MTBS2CK p.23

Complementfixationtitersinblood Nucleicacidamplificationtestingon swab Tzanckprep:Bestinitialtest Viralculture:Mostaccuratetest

Herpessimplex

MTBS2CK p.24

DiagnosticTests

Treatment
Diagnosis Treatment

If dark-field is positive for spirochetes no further testing for syphilis is necessary necessary.

Syphilis

OnedoseIMbenzathine penicillin Doxycycline,ifpenicillinallergic

Chancroid Azithromycin (singledose) (Haemophilus ducreyi) Lymphogranuloma venereum Herpessimplex Doxycycline Acyclovir,valacyclovir,famciclovir Foscarnet foracyclovirresistant herpes

MTBS2CK p.24

MTBS2CK p.24

Syphilis/Presentation Woman comes to clinic with multiple painful genital vesicles. What is the next step in management? Primary Syphilis Painless genital ulcer with heaped-up indurated edges (it becomes painful if it becomes secondarily infected with bacteria) Painless adenopathy

a. Acyclovir orally y topically p y Worthless b. Acyclovir c. Tzanck prep not necessary


If the presentation is diagnostic testing is Most accurate test, but not necessary, if

RichardUsatine,M.D.Usedwithpermission

d. Viral culture vesicles are clear


infection in the past

e. Serology Cant distinguish genital infection from oral f. PCR


MTBS2CK p.24

PCR encephalitis, not genital

Chancres heal spontaneously even without treatment. Penicillin prevents later stages.
MTBS2CK p.25

20

Syphilis/Presentation

Syphilis/Presentation Tertiary Syphilis Neurosyphilis


Meningovascular (stroke from vasculitis) Tabes dorsalis (loss of position and vibratory sense, incontinence, cranial nerve)

Secondary Syphilis Rash (palms and soles) Alopecia areata Mucous patches Condylomata lata

Tabes Dorsalis . Source: CDC/Susan Lindsley

MTBS2CK p.25

phil.cdc.gov

MTBS2CK p.25

Syphilis/Presentation

SensitivityofDiagnosticTestsbyStage
Test VDRLorRPR FTAABS

Tertiary Syphilis Neurosyphilis


General paresis (memory and personality changes) Argyll Robertson pupil

Primary Secondary Tertiary

75%85% 99% 95%

95% 100% 98%

Aortitis (aortic regurgitation, aortic aneurysm) Gummas (skin and bone lesions)
MTBS2CK p.25

MTBS2CK p.25

Syphilis False positive VDRL/RPR Infection, older age, injection drug use, AIDS, malaria, antiphospholipid syndrome, and endocarditis Titers of VDRL or RPR are reliable at > 1:8 Lower titer is more often falsely positive High titers (> 1:32) are rarely false positive
MTBS2CK p.25

Syphilis/Treatment

Primary & secondary syphilis: Single IM injection of penicillin Oral doxycycline: Penicillin allergy Tertiary syphilis: IV penicillin Desensitize to penicillin if allergic

MTBS2CK p.25

21

Syphilis Jarisch-Herxheimer reaction Fever, headache, myalgias after treatment Give aspirin & antipyretics (itll pass)

GenitalWarts(CondylomataAcuminata)/ Diagnosis

Desensitization the answer for neurosyphilis and pregnant women

Papillomavirus Diagnose based on the visual appearance Wrong answers: Biopsy Serology Stain, smear, or culture
MTBS2CK p.26

Condylomata acuminata (genital warts). Source: Farshad Bagheri, MD.

MTBS2CK p.26

GenitalWarts(CondylomataAcuminata)/ Treatment

Pediculosis(Crabs)

Remove by physical means: Cryotherapy with liquid nitrogen Surgery for large ones Laser Melt with podophyllin or trichloroacetic acid Imiquimod: Apply locally (immunostimulant leads to sloughing off of lesion)
MTBS2CK p.26

Hair-bearing areas (axilla, pubis) Itchy Visible on surface Treat with permethrin; li d lindane i is equal li in efficacy, but more toxic

Pediculosis pubis. Source: commons.wikimedia. Used with permission

MTBS2CK p.26

Scabies Web spaces between fingers and toes or at elbows Around nipples near the genitals Burrows visible (they dig) but smaller than pediculosis Scrape & magnify

Scabies/Treatment

Permethrin Widespread disease: Oral Ivermectin Severe disease needs repeat dosing

Scabies burrow under the skin and must be scraped out to establish a diagnosis. Source: Conrad Fischer, MD.

MTBS2CK p.27

MTBS2CK p.27

22

UrinaryTractInfections(UTI)

UrinaryTractInfections
Cystitis Pyelonephritis y p AcuteProstatitis PerinephricAbscess

Present with dysuria (frequency, urgency, burning), maybe fever Urinalysis shows WBCs E. coli MCC Quinolones Best initial therapy Anatomic defects lead to UTIs:
Stones Strictures Tumor or prostate hypertrophy Diabetes
MTBS2CK p.27

UrinaryTractInfections

Cystitis

Foley catheter is a foreign body Neurogenic bladder is an obstruction


Frequency = Multiple episodes of micturation Polyuria = Increase in volume of urine

Presents with dysuria and: Suprapubic pain/discomfort Mild/absent fever


Men have anatomic abnormalities Women most often dont Best initial test: Urinalysis with > 10 WBCs Most accurate test: Urine culture

MTBS2CK p.27

MTBS2CK p.2728

Cystitis/Treatment

Trimethoprim/sulfamethoxazole (TMP/SMZ) if local resistance is low Ciprofloxacin Cephalexin Nitrofurantoin ( (Pregnant g women) ) All beta-lactam antibiotics are considered safe in pregnancy

36 year old generally healthy woman comes with urinary frequency and burning. UA: > 50 WBC What is the next step in management?

a. b. c. d. e.

TMP/SMZ for 3 days TMP/SMZ for 7 days With anatomic abnormality Urine culture No need of culture Ultrasound of urinary system or imaging when symptoms of cystitis CT scan of urinary system

MTBS2CK p.28

MTBS2CK p.28

are clear & WBCs in urine

23

Pyelonephritis

Pyelonephritis

Dysuria with: Flank or CVA tenderness High fever Occasional abdominal pain UA with high WBCs Imaging studies (CT or sonogram) look for anatomic abnormalities causing infection
MTBS2CK p.28

Treat with: Ampicillin & gentamicin Ciprofloxacin Change based on culture/sensitivity


Any drugs for gram-negative bacilli would be effective for pyelonephritis

MTBS2CK p.28

AcuteProstatitis

PerinephricAbscess

Dysuria with: Perineal pain Tender prostate on examination Diagnostic yield of urine culture greatly increased with p prostate massage g Treat same as you would for pyelonephritis Longer duration TMP/SMZ or ciprofloxacin for 6 to 8 weeks for chronic prostatitis
MTBS2CK p.29

Look for: Pyelonephritis not resolving with appropriate therapy When drug choice & dose are correct failure of infection to resolve is an anatomic problem Do sonogram or CT scan Drainage of fluid collection is mandatory Culture of infected fluid is essential to guide therapy
MTBS2CK p.29

Endocarditis/Definition

Endocarditis

Heart valve infection Leads to fever & murmur Diagnosis: Positive blood cultures Vegetations on echocardiogram

MTBS2CK p.29

24

Endocarditis/Etiology

Endocarditis/Etiology

Rare on normal heart valves (except IDUs) Risk proportional to degree of damage of valves Regurgitant & stenotic lesions have increased risk Prosthetic valves: Highest risk

Can develop on normal valves if: Severe bacteremia Highly pathogenic organisms (Staphylococcus aureus) Dental procedures with blood confer a small risk of endocarditis Surgery of mouth & respiratory tract confers no risk unless severe valve disease is present Artificial valve or cyanotic heart disease Endoscopy confers no risk even with biopsy
MTBS2CK p.29

MTBS2CK p.29

Endocarditis/ WhatIstheMostLikelyDiagnosis?

Endocarditis/Presentation/ WhatIstheMostLikelyDiagnosis?
Osler nodes (raised and painful) Roth spots in eyes Brain (mycotic aneurysm) Kidney (hematuria, glomerulonephritis) Conjunctival petechiae Splenomegaly Septic emboli to lungs

Fever New murmur or change in a murmur Complications of endocarditis


Splinter hemorrhages Janeway lesions (flat and painless)

MTBS2CK p.2930

Source: Splarka http://en.wikipedia.org/wiki/Splinter_hemorrhages

MTBS2CK p.30

Endocarditis/DiagnosticTests

Best initial test Blood culture (95%99% sensitive) If positive do an Echo Transthoracic echo (60% sensitive, but 95% specific) Transesophageal T h l echo h (95% sensitive iti and d specific) EKG rarely shows atrioventricular (AV) block if dissection of conduction system occurs (5% 10%)

A man comes into the ED with fever and a murmur. Blood cultures grow Streptococcus bovis. Transthoracic echocardiography shows vegetation. What is the next step in the management?

a. b b. c. d. e.

Colonoscopy vegetation. Transesophageal echocardiogram Will not show diverticuli CT of the abdomen Repeat the blood cultures No point, already positive Surgical valve replacement Premature

Transthoracic showed

Fever + murmur = endocarditis

MTBS2CK p.30

MTBS2CK p.3031

25

HowtoDiagnose CultureNegativeEndocarditis

Endocarditis/Treatment

1. Oscillating vegetation on echocardiography 2. Three minor criteria: Fever Risk: IDU or prosthetic valve Embolic phenomena

Empiric: Vancomycin and gentamicin When culture results are available Treat based on sensitivities

MTBS2CK p.31

MTBS2CK p.31

Endocarditis/Treatment
Organism Treatment

Endocarditis/Treatment
Organism Treatment

Viridans streptococci Staphylococcus aureus (sensitive) Fungal

Ceftriaxonefor4weeks Oxacillin,nafcillin,or cefazolin Amphotericinandvalve replacement

Staphylococcus epidermidis orresistant Staphylococcus Enterococci

Vancomycin

Ampicillin&gentamicin

MTBS2CK p.31

MTBS2CK p.31

Endocarditis

Endocarditis

Treatment of Resistant Organisms Add aminoglycoside & extend duration of treatment When Is Surgery the Answer? CHF or ruptured valve or chordae tendineae Prosthetic valves Fungal endocarditis Abscess AV block Recurrent emboli while on antibiotics
MTBS2CK p.31

Add rifampin for prosthetic valve endocarditis with Staphylococcus. The single strongest indication for surgery is acute valve rupture and CHF.
MTBS2CK p.3132

26

TreatmentofCultureNegativeEndocarditis

ProphylaxisforEndocarditis

Most common culture negative = Coxiella HACEK: Acronym for organisms difficult to culture causing endocarditis
Haemophilus aphrophilus Haemophilus parainfluenza Actinobacillus Cardiobacterium Eikenella Kingella

Use prophylaxis if theres BOTH: 1. Significant cardiac defect


Prosthetic valve Previous endocarditis Cardiac transplant recipient with valvulopathy Unrepaired cyanotic heart disease

AND! 2. Risk of bacteremia


Dental work with blood Respiratory tract surgery that produces bacteremia
MTBS2CK p.32

Use ceftriaxone for HACEK group


MTBS2CK p.32

ProphylaxisforEndocarditis

ProphylaxisforEndocarditis

Amoxicillin prior to procedure If penicillin allergic use clindamycin, azithromycin, or clarithromycin Procedures and anatomic abnormalities that do not need prophylaxis are: Flexible endoscopy, even with biopsy, ERCP Obstetrical and gynecologic procedures
MTBS2CK p.32

Urology procedures (including prostate biopsy) Mitral valve prolapse, even with murmur Mitral regurgitation, mitral stenosis, aortic regurgitation regurgitation, aortic stenosis stenosis, hypertrophic obstructive cardiomyopathy (HOCM), atrial septal defect (ASD)

MTBS2CK p.32

LymeDisease/Definition

LymeDisease

Arthropod-borne Spirochete: Borrelia burgdorferi What is the most common manifestation? Fever & Rash What is the most common manifestation if untreated? Joint pain Also: Cardiac or neurologic disease
MTBS2CK p.33

27

LymeDisease/Etiology

LymeDisease/Etiology

Transmitted by deer tick (Ixodes scapularis) Tick very small Only 20% patients recall bite

Patients recall being outdoors Hiking or camping Experimental models: Tick must be attached for at least 24 hours in order to transmit organism

Richard Usatine, M.D. Used with permission

MTBS2CK p.33

MTBS2CK p.33

LymeDisease/Etiology

LymeDisease/Presentation

Ixodes tick Northeast states: Connecticut (where the town of Lyme gave the disease its name) Massachusetts New York New Jersey

Rash: Most common manifestation (85% - 90%) Occurs 5-14 days after bite Fever often present

MTBS2CK p.33

cdc.gov

MTBS2CK p.33

LymeDisease/Presentation

LymeDisease/Presentation

Erythema migrans Round, red Pale center Target or bulls-eye

Joint pain 60% without treatment Oligoarthritis = few joints Joint fluid: ~ 25,000 WBCs/L Similar number to gout, pseudogout and many infections i f ti
The knee is the most commonly affected joint in Lyme disease.

Target-shaped rash of Lyme disease or erythema migrans. Source: Nishith Patel.

MTBS2CK p.3334

MTBS2CK p.33

28

LymeDisease/Presentation

LymeDisease/Presentation

Neurological manifestations 10% 15% CNS or peripheral Meningitis, encephalitis, or CN palsy


Bells palsy or 7th CN is most common neurological manifestation of Lyme disease.

Cardiac: 4% - 10% Any part of myocardium or pericardium Myocarditis or ventricular arrhythmia

Transient AV block is the most common cardiac manifestation in Lyme disease.

MTBS2CK p.33

MTBS2CK p.33

LymeDisease/DiagnosticTests

LymeDisease/Treatment
Manifestation Treatment

Typical rash doesnt need serologic testing to start treatment Serologic testing for Lyme is essential for: Joint Neurologic g Cardiac manifestations Most 7th CN palsy, arthralgia & AV block are not caused by Lyme Testing is with IgM, IgG, ELISA, Western blot, and PCR testing
MTBS2CK p.34

Asymptomatictickbite

Notreatment routinely Doxycycline Amoxicillin Doxycycline Amoxicillin IVceftriaxone

Rash

Joint,7th CNpalsy

Cardiacandneurologic manifestationsotherthan7th CN palsy


MTBS2CK p.34

AsymptomaticTickBite

Bites without symptoms rarely need treatment Who DOES need treatment? Single dose of doxycycline Within 72 hours of tick bite when:
Ixodes scapularis clearly identified as tick Tick attached for > 24-48 hours Engorged nymph-stage tick Endemic area
MTBS2CK p.34

HIV/AIDS

29

HIV/AIDS/Definition

A retrovirus infecting CD4 (T-helper) cells CD4 cells drop from normal (600-1000) at a rate of 50 to 100/year untreated 5 and 10 years before clinical manifestations Not HIV itself that leads to death Low CD4 leads to illness
MTBS2CK p.35

HIVlifecycle

Source: nih.gov

HIV/AIDS/Etiology

Transmitted by: IDU Sex, particularly men who have sex with men Transfusion (extremely rare since 1985) Perinatal Needle stick or blood-contaminated sharp instrument injury Kissing is not proven to transmit HIV.
MTBS2CK p.35

RiskofTransmissionofHIV WithoutProphylacticTreatment
Modeof transmission Vaginaltransmission Percentageofriskwitheachevent 1:30001:10,000forinsertiveintercourse 1:1000forreceptiveintercourse 1:1000forreceptivefellatiowithejaculation Unclearforinsertivefellatioorcunnilingus 1:300 1:100forreceptiveanalintercourse 25%30%perinataltransmissionwithout medication

Oralsex

Needlestickinjury Analsex Mothertochild


MTBS2CK p.35

HIV/AIDS/Presentation

HIV/AIDS/DiagnosticTests

Infections when CD4 drops < 50/L PCP < 200/L CD4 > 200/L (few infections occur) Infections > 200/L are:
Varicella zoster (shingles) Herpes simplex TB Oral and vaginal candidiasis Bacterial pneumonia

Best initial test: ELISA test Confirmed with Western blot Infected infants: Diagnose with PCR or viral culture ELISA testing is unreliable in infants Maternal HIV antibodies are present for up to 6 months after delivery

MTBS2CK p.35

MTBS2CK p.36

30

ViralLoadTesting(PCRRNAlevel)

ViralLoadTesting(PCRRNAlevel)

Measures response to therapy Decreasing levels good Detect treatment failure Rising levels bad Diagnoses HIV in babies

The goal of therapy is to drive down viral load Undetectable levels (< 50/L): CD4 will rise CD4 rises = Opportunistic infections stop Life expectancy when viral load is undetectable by PCR-RNA is equal in duration to HIV-negative person
MTBS2CK p.36

MTBS2CK p.36

ViralResistanceTesting(Genotyping)

HIV/AIDS/Treatment

Perform prior to initiating antiretroviral medications Decreases likelihood of starting medication to which patients virus is resistant Resistance testing: Evaluates treatment failure Guides choice of medications Select 3 drugs from 2 different classes to which patients virus is susceptible
MTBS2CK p.36

For HIV: Treatment is initiated when: CD4 < 500/L or Viral load is very high (>100,000/L) or Opportunistic infection occurs

MTBS2CK p.36

HIV/AIDS/Treatment

ChoiceofInitialAntiretroviralMedication Initial drug regimen:

Treatment failure first manifests with rising PCRRNA viral load

Emtricitabine, Tenofovir, and Efavirenz


USMLE Step 2 CK doesnt doesn t test dosing

MTBS2CK p.36

MTBS2CK p.36

31

ChoiceofInitialAntiretroviralMedication

AntiretroviralFirstlineMedicationsbyClass
Nucleosideand Non nucleotidereverse nucleoside transcriptase RTIs inhibitors(RTIs) Zidovudine Didanosine Stavudine Lamivudine Emtricitabine Abacavir Tenofovir Efavirenz Etravirine Nevirapine Proteaseinhibitors

Treatment failure: Rising viral load CD4 count decreases or fails to rise CD4 changes are slower than changes in viral load Alternate Drug Regimens If emtricitabine/tenofovir/efavirenz cannot be used because of resistance alternate regimens are based on combination of 3 drugs from at least 2 different classes
MTBS2CK p.37

Ritonavir Saquinavir Nelfinavir Amprenavir Fosamprenavir

Lopinavir Atazanavir Indinavir Tipranavir Darunavir

MTBS2CK p.37

AdditionalClassesofSecondlineAgents Used with drug resistance to multiple classes of first-line agents Entry inhibitors:
Enfuvirtide Maraviroc

PostexposureProphylaxis

Integrase inhibitor:
Raltegravir

All significant needle stick injuries Sexual exposures Bites Give 4 weeks of combination therapy Urine & stool arent an indication for postexposure prophylaxis (PEP)

USMLE Step 2 CK wont require you to know details of efficacy differences between classes.
MTBS2CK p.37 MTBS2CK p.37

PostexposureProphylaxis

AdverseEffectsofHIVMedications
Drug Zidovudine Adverseeffect Anemia Peripheralneuropathyandpancreatitis Hypersensitivity,StevensJohnson Reaction Hyperlipidemia,hyperglycemia Nephrolithiasis Renalinsufficiency

Postexposure prophylaxis isnt routinely indicated for needle stick injury if HIV status of needle is unknown

Stavudine anddidanosine Abacavir

Proteaseinhibitors Indinavir Tenofovir


MTBS2CK p.37 MTBS2CK p.38

32

PreventionofPerinatalTransmission

PreventionofPerinatalTransmission

Pregnant Patients If already on antiretroviral medications Continue same treatment If pregnant and not already on medications di ti Start combination antiretrovirals Only Efavirenz is NOT used in pregnancy
MTBS2CK p.38

After Delivery for Mother CD4 > 500 and she doesnt need them Stop after delivery CD4 < 500 or viral load high Continue after delivery Baby Zidovudine during delivery (intrapartum) AND For 6 weeks after to help prevent transmission
MTBS2CK p.38

IndicationsforAntiretrovirals DuringPregnancy
Condition Action

CesareanDeliveryforHIVPositiveMothers

Patientonantiretroviralsat Continuesamemedications, thetimeofpregnancy exceptswitchefavirenzto proteaseinhibitor Notonantiretrovirals, CD4loworviralloadhigh Initiateantiretrovirals immediately.Continueafter delivery Antiretroviralsimmediately, stoptheminmotherafter birth

Viral load >1000 L: Perform Cesarean delivery Everyone: Intrapartum zidovudine Fully controlled HIV (viral load undetectable) gives <1% transmission.

Notonantiretrovirals, CD4high andviralloadlow


MTBS2CK p.38

MTBS2CK p.39

33

Nephrology
EmmaHolliday, Holliday MD ResidentPhysician RadiationOncology UniversityofTexasMDAndersonCancerCenter

DiagnosticTests inNephrology
Urinalysis WhiteBloodCells Hematuria Cytoscopy Casts

Urinalysis Measures:
Protein WBCs or leukocyte esterase RBCs Specific gravity and pH Nitrites

Dipstickvs.Urinalysis Both give some quantitative values Dipstick described as direct (e.g., 300 mg protein) or scale number: 0, 1+, 2+, 3+, or 4+ Microscopic urinalysis reports number of cells/highpowered field

Pyuria with positive nitrites = UTI

Dont worry about the precise scale Every USMLE Step 2 CK test comes with a range of normal values attached to assess severity

MTBS2CK p.299

MTBS2CK p.299

Urinalysis/Protein Renal tubules secrete slight amounts of TammHorsfall protein


Should be < 30 to 50 mg/24 hours

Urinalysis/Protein 2 - 10% of population has transient proteinuria Benign reasons for proteinuria:
Orthostatic proteinura or physical exertion
If symptoms of proteinuria - best initial test? Urinalysis/Dipstick

Greater amounts of protein associated with either tubular disease or glomerular disease Very large amounts of protein excreted with glomerular disease 1+ proteinuria ~ 1g/day 2+ proteinura ~ 2g/day 3+ proteinuria ~ 3g/day 4+ proteinuria ~ 4g/day
MTBS2CK p.300

If positive, order urine protein to creatinine ratio or 24 hour urine collection Most accurate for determining etiology?
MTBS2CK p.300

Renal biopsy

Urinalysis/Proteinuria Limitations of dipstick for proteinuria: Only detects albumin Cant detect Bence Jones protein
Present in multiple myeloma Best test for Bence Jones proteinuria is UPEP (urine protein electrophoresis)

Urinalysis/Microalbuminuria

Microalbuminuria
Tiny amounts of protein too small to detect on UA

An important screening test in diabetic patients


Should be performed yearly

Cant detect very low amounts of protein

Long-term microalbuminuria
Leads to worsening renal function in diabetic patient and should be treated
MTBS2CK p.301 MTBS2CK p.301

Pyuria Diabetic patient is evaluated, UA shows no protein. Microalbuminuria is detected (level between 50 - 300 mg/24 hours). Whats the next best step in management? a. Enalapril b Kidney biopsy Extreme! We know the etiology already b. c. Hydralazine Less effective & with more adverse effects d. Renal consultation NEVER consult on Step 2 e. Low-protein diet Bad for glycemic control f. Repeat UA annually and treat when trace protein is Starting early will delay disease progression detected
MTBS2CK p.301

White blood cells in urine:


Inflammation Infection Allergic interstitial nephritis

Neutrophils cant be distinguished from eosinophils on UA Neutrophils indicate infection (UTI or urethritis) Eosinophils indicate allergic or acute interstitial nephritis (NOT NSAID-induced renal disease) Wright and Hansel stains detect eosinophils in the urine.
MTBS2CK p.301

Hematuria Etiology: Stones Coagulopathy Infection (cystitis, pyelonephritis) Cancer Cancer C T Treatments t t (cyclophosphamide) ( l h h id ) Trauma Glomerulonephritis False positive = hemoglobinuria or myoglobinuria Dysmorphic RBCs = glomerulonephritis
MTBS2CK p.301302 MTBS2CK p.302

Woman is admitted with trauma and dark urine. The dipstick is markedly positive for blood. What is the best initial test? a. Microscopic examination of urine Overkill - best test to look for bladder mass b. Cystoscopy Best test for hydronephrosis c Renal ultrasound c. d. Renal/bladder CT scan Most accurate test for stones e. Abdominal X-ray Best test for ileus or to detect free air f. Intravenous pyelogram Never the right answer - slow
test uses nephrotoxic contrast

WhenIsCystoscopytheAnswer?

Casts Microscopic collections of material clogging up the tubules and being excreted in urine Type of cast Red cell White cell Eosinophil Hyaline Broad, waxy Granular, muddybrown
MTBS2CK p.302303

When theres hematuria without infection or prior trauma and:


The renal ultrasound or CT doesnt show etiology Bladder sonography shows mass for possible biopsy
Cystoscopy is the most accurate test of the bladder.
MTBS2CK p.302

Association Glomerulonephritis Pyelonephritis Acute interstitial nephritis Dehydration Chronic renal disease Acute tubular necrosis (are dead tubular casts)

AcuteKidneyInjury

Acute kidney injury (AKI)

AcuteKidneyInjury
AcuteTubularNecrosis Hepatorenal p Syndrome y Atheroemboli Acute(Allergic)InterstitialNephritis AnalgesicNephropathy

Decrease in creatinine clearance Results in sudden rise in BUN and creatinine

The definition is not based on specific values of BUN and creatinine Categories:
Prerenal azotemia (decreased perfusion) Postrenal azotemia (obstruction) Intrinsic renal disease (ischemia and toxins)
MTBS2CK p.303

Acute Kidney Injury


Prerenal Intrinsic Renal

AcuteKidneyInjury/Presentation
Post Renal
BPH/Prostate cancer Ureteral stone Cervical cancer Urethral stone Neurogenic bladder Retroperitoneal fibrosis (chemo or XRT)

AKI usually = asymptomatic rise in BUN and creatinine; when symptomatic:


Nausea and vomiting Fatigue/malaise Weakness SOB, edema (fluid overload)

ATN Hypotension - Toxins - Sepsis NSAIDs - Anaphylaxis AG, ampho Cis, Cis cyclosporine - Bleeding - Prolonged ischemia - Dehydration AIN Hypovolemia - PCN, sulfa Rhabdo/hemoglobinuria - Diuretics Contrast - Burns Crystals - Pancreatitis Bence-Jones proteins Post-strep infection - pump fxn - Low albumin - Cirrhosis MTBS2CK p.303304 artery stenosis Renal

Very severe disease presents with:


Confusion Arrhythmia from hyperkalemia and acidosis Sharp, pleuritic chest pain from pericarditis
MTBS2CK p.304305

AcuteKidneyInjury/DiagnosticTests

AcuteKidneyInjury/DiagnosticTests

The best initial test is...


BUN and creatinine

What is the best initial imaging test?


Renal sonogram because it isnt invasive and doesnt need contrast

Nonfunctional kidneys creatinine rises ~ 1 point/day (1 mg/dL) If the BUN:creatinine BUN creatinine ratio is > 20 20:1 1 the etiology is either prerenal or postrenal damage Intrinsic renal disease ratio closer to 10:1
MTBS2CK p.305

Kidney biopsy is rarely the right answer for AKI

MTBS2CK p.305

AcuteKidneyInjury/DiagnosticTests When cause is unclear next best diagnostic step is:


Urinalysis Urine sodium (UNa) Fractional excretion of sodium (FENa) or urea (FEUrea) Urine osmolality

AcuteKidneyInjury/DiagnosticTests Urine Sodium and Fractional Excretion of Sodium Normal kidney:


blood pressure aldosterone aldosterone sodium reabsorption sodium in the urine (< 20)

Damaged kidney (e.g., ATN):


Kidneys cant reabsorb Na appropriately (> 20) Prerenal azotemia: low UNa (below 20) = low FENa (less than 1%) You can answer all the questions on USMLE Step 2 CK without knowing the mathematical formula for FENa.

If all of these are choices always go with urinalysis first

MTBS2CK p.306

MTBS2CK p.306

AcuteKidneyInjury/DiagnosticTests

Urine Osmolality Normal kidney:


intravascular volume ADH ADH water reabsorption at collecting duct urine osmolarity

20-year-old African-American man has screening test for sickle cell. Hes found to be heterozygous (trait or AS) for sickle cell. Sickle cell trait rarely What is the best advice for him?experience pain crises

Damaged kidney (e.g., ATN):


Kidneys cant concentrate urine appropriately - will see Osm similar to blood (~300mOsm)
MTBS2CK p.306

a. b. c. d. e.

Nothing needed until he has a painful crisis y Avoid dehydration Hydroxyurea Only if > 4 pain crises per year. Folic acid supplementation Indicated if hemolysis Pneumococcal vaccination present (high RBC
Indicated if functionally asplenic turnover)

MTBS2CK p.307

ClassificationofAcuteRenalFailure
Test Prerenal azotemia Acutetubular necrosis

AcuteTubularNecrosis/Definition

BUN:Creatinine UrineSodium(UNa) F ti Fractional lexcretion ti ofsodium(FENa) Urineosmolality (UOsm)

>20:1 <20mEq/L <1%

<20:1 >20mEq/L >1%

Injury to kidneys from ischemia and/or toxins result in sloughing of tubular cells into urine Proteinuria isnt significant since protein, not tubules, spill into urine when glomeruli are damaged

>500 mOsm/kg

<300 mOsm/kg

Acute renal failure and a toxin in history are your clues to What is the most likely diagnosis? question for ATN
MTBS2CK p.307

MTBS2CK p.307

Patient presents with fever and acute LLQ abdominal pain. Blood cultures grow E. coli and Candida albicans. Patient started on vancomycin, metronidazole, gentamicin, and amphotericin. CT scan reveals diverticulitis. After 36 hours, her creatinine rises dramatically. Which of the following is most likely the cause of her renal insufficiency? a. Vancomycin Need 5-10 days exposure b. Gentamicin Need 5-10 days exposure c. Contrast media d. Metronidazole Hepatically excreted e. Amphotericin Need 5-10 days exposure

74-year-old blind man admitted with obstructive uropathy and chest pain. History of hypertension and diabetes. Creatinine drops from 10 mg/dL to 1.2 mg/dL 3 days after catheter placement. The stress test shows reversible ischemia. This is still What is the most appropriate management? considered

MTBS2CK p.308

MTBS2CK p.308

a. Coronary artery calcium score on CT scan experimental a b. One to two liters of normal saline hydration prior and during angiography Evidence not as strong c. N-acetylcysteine prior to angiography d. Mannitol during angiography Doesnt help e. Furosemide during angiography Loop diuretics are dangero worsen it f. Intravenous sodium bicarbonate before and during angiography Evidence not as strong

ExtraDifficultQuestion A patient with mild renal insufficiency undergoes angiography and develops 2 mg/dL rise in creatinine from ATN despite the use of saline hydration before and after procedure. What do you expect to find on laboratory testing?
a. b. c. d. Urine sodium FENa Urine specific gravity 8 (low) >1% 1.035 (high) 58 (high) >1% 1.005 (low) 5 (very low) <1% 1.040 (very high) 45 (high) >1% 1.005 (low) Spasm of afferent arteriole leads to reabsorption of Na (and thus water) very concentrated urine ( specific gravity)

ExtraDifficultQuestion

A patient with extremely severe myeloma with a plasmacytoma is admitted for combination chemotherapy. Two days later creatinine rises. What is the most likely cause?

a. b b. c. d. e.

Cisplatin Takes 5-10 days Hyperuricemia Can cause renal insufficiency Bence-Jones proteinuria but would not get worse with treatment Hypercalcemia Hyperoxaluria Oxalate crystals can be present in
urine in cases of bowel resection

MTBS2CK p.309

MTBS2CK p.309

ExtraDifficultQuestion

ExtraDifficultQuestion Suicidal patient ingests an unknown substance and develops renal failure 3 days later. Her calcium level is low and urinalysis shows an abnormality. What did she take?

What wouldve prevented this event? Allopurinol, hydration, and rasburicase Given prior to chemotherapy to prevent renal failure from tumor lysis syndrome

a. b. c. d. e. f.

Aspirin Doesnt affect Ca levels or cause crystals p Hepatotoxic p Acetaminophen Ethylene glycol Constrict afferent arteriole ATN and Ibuprofen papillary necrosis Opiates Can cause FSGS, but not AKI Methanol Affects retinal inflammation not AKI

MTBS2CK p.309

MTBS2CK p.310

AcuteTubularNecrosis/Toxins

AcuteTubularNecrosis

3 things increase risk of toxic/insult ATN: Hypoperfusion of kidney Underlying renal insufficiency Hypertension Diabetes Older age We lose 1% of renal function every year past age of 40.
MTBS2CK p.310

Summary of Causes: Slow onset (5 - 10 days) Drug-related injury


Aminoglycosides, amphotericin, cisplatin, vancomycin, acyclovir, cyclosporine Dose dependent
Mag risk for aminoglycoside and cisplatin

Immediate toxicity (24 48 hrs) Contrast media


Best prevented with saline hydration N-acetylcysteine and sodium bicarbonate arent consistently proven as beneficial
MTBS2CK p.310

AcuteTubularNecrosis

Rhabdomyolysis

Summary of Causes: Hemoglobin and myoglobin Hyperuricemia Ethylene Glycol Multiple Myeloma NSAIDs

Etiology?
Trauma, prolonged immobility, snake bites, seizures, and crush injuries

The best initial test?


UA (dipstick AND microscopic analysis)

Characteristic findings?
Blood is positive on dipstick but NO RBCs are seen on microscopic exam
MTBS2CK p.311

MTBS2CK p.310

Rhabdomyolysis

Rhabdomyolysis

Most specific lab test?


Urine myoglobin

Abnormal lab findings?


Creatine phosphokinase (CPK) Hyperkalemia H k l i Hyperuricemia Hyperphosphatemia Hypocalcemia
MTBS2CK p.311

Treat with: Saline hydration Mannitol Bicarbonate


Dont treat hypocalcemia in rhabdomyolysis if asymptomatic. In recovery, the calcium will come back out of muscles.
MTBS2CK p.311

AcuteTubularNecrosis/Treatment A man comes to the ED after a triathlon followed by status epilepticus. He takes simvastatin at triple the recommended dose. His muscles are tender and urine is dark. IV fluids are started. What is the next best step in management?

No therapy proven to benefit ATN Patients should be managed with


Hydration, if theyre volume depleted and correction of electrolyte abnormalities

a. b. c. d. e.

CPK level Would suggest gg rhabdo as cause, , but not life threatening complications. EKG Potassium replacement Would be fatal Urine dipstick Same reason as A Urine myoglobin Same reason as A

Diuretics increase urine output, but dont change overall outcome More urine output with diuretics doesnt mean renal failure is reversing.
MTBS2CK p.312

MTBS2CK p.311312

AcuteTubularNecrosis/Treatment

AcuteTubularNecrosis/Treatment

Answering treatment questions for ATN is based on recognizing the most common wrong answers:
Low-dose dopamine Diuretics Mannitol Steroids

All these are ineffective in reversing ATN Correct underlying cause


MTBS2CK p.312

When Is Dialysis the Answer? Dialysis is initiated if theres: Fluid overload Encephalopathy Pericarditis Metabolic acidosis A- acidosis E- electrolytes Hyperkalemia

I- intoxications O- overload of volume U-uremia

MTBS2CK p.312

AcuteTubularNecrosis/Rhabdomyolysis Patient develops ATN from gentamicin. Shes vigorously hydrated and treated with high doses of diuretic, low-dose dopamine, and calcium acetate as a phosphate binder. Urine output increases, but she still progresses to end-stage renal failure. She also becomes deaf. What caused her hearing loss?
a. b. c. d. e. Hydrochlorothiazide No otoxicity Dopamine No ototoxicity Furosemide Chlorthalidone No ototoxicity Calcium acetate No ototoxicity

HepatorenalSyndrome

Renal failure developing secondary to liver disease Kidneys normal Look for:
Severe liver disease (cirrhosis) New-onset renal failure with no other explanation Very low urine sodium (> 10 15 mEq/dL) FeNa < 1% Elevated BUN:creatinine ratio (> 20:1)

Treat with:
Albumin, midodrine, octreotide
MTBS2CK p.313

MTBS2CK p.313

Atheroemboli/Etiology

Atheroemboli/Etiology Livedo reticularis

Can occur during catheter procedures:


Cardiac catheterization, angiogram

Emboli can go to:


Kidney, leading to AKI Eye Cutaneous vessles livedo reticularis Peripheral pulses are normal in atheroemboli atheroemboli. Theyre too small to occlude vessels such as the radial or brachial artery.

Lab findings:
Eosinophilia/Eosinophiluria, compliment, ESR

Most accurate test:


Biopsy cholesterol crystals

Source: Farshad Bagheri, MD MTBS2CK p.313314 MTBS2CK p.314

Acute(Allergic)InterstitialNephritis Antibodies and eosinophils attack cells lining tubules Reaction to drugs (70%), infection, and autoimmune disorders
Penicillins and cephalosporins Sulfa drugs (including diuretics like furosemide and thiazides which are sulfa derivatives) thiazides, Phenytoin The medications that cause AIN Rifampin are the same as those that cause: Quinolones Drug allergy and rash Allopurinol Stevens-Johnson syndrome PPI Toxic epidermal necrolysis
Hemolysis
MTBS2CK p.314315

Acute(Allergic)InterstitialNephritis Presentation: What Is the Most Likely Diagnosis? Look for acute renal failure (rising BUN and creatinine) with: Fever (80%) Rash (50%) Arthralgias Eosinophilia and eosinophiluria (80%) BUN/Cr ratio < 20:1 WBC and RBC in urine What is the most accurate test? Wright/Hansel stain to identify eosinophiluria
MTBS2CK p.315316

Acute(Allergic)InterstitialNephritis

AnalgesicNephropathy

Treatment AIN usually resolves spontaneously with stopping drugs or controlling infection Severe disease is managed with dialysis, which may be temporary When creatinine continues to rise after stopping the drug, giving glucocorticoids (prednisone, hydrocortisone, methylprednisolone) is the answer

Analgesic nephropathy presents with: ATN from direct toxicity to tubules AIN Membranous glomerulonephritis Papillary necrosis

MTBS2CK p.316

MTBS2CK p.316

AnalgesicNephropathy

PapillaryNecrosis

Presentation Vascular insufficiency of kidney from inhibiting prostaglandins


Prostaglandins dilate the afferent arteriole NSAIDs constrict the afferent arteriole and decrease renal perfusion p Asymptomatic in healthy patients When patients are older and have underlying renal insufficiency from diabetes and/or hypertension, NSAIDs can tip them over into clinically apparent renal insufficiency due to direct toxicity to tubules
MTBS2CK p.316

Sloughing off renal papillae Etiology NSAIDs or sudden vascular insufficiency


Death of cells in papillae and their dropping off the internal structure of kidney

Occurs with underlying kidney damage:


Sickle Cell DM Urinary obstruction Chronic pyelonephritis
MTBS2CK p.316

PapillaryNecrosis

Clinical presentation:
Fever, hematuria, and sudden onset flank pain Looks like pyelonephritis

DifferencesbetweenPyelonephritis andPapillaryNecrosis Pyelonephritis Onset Symptoms Urineculture CTscan Treatment Fewdays Dysuria Positive Diffuse swollenkidney Antibiotics,suchas ampicillin/gentamicin orfluoroquinolones Papillarynecrosis Fewhours Necroticmaterial inurine Negative Bumpy contour ofkidneyinterior Notreatment

Best initial test:


UA

Most accurate test:


CT scan

Treatment:
No specific therapy
MTBS2CK p.317 MTBS2CK p.317

TubularDisease

GlomerularDiseases
Chronic Immune mediated All nephrotic Need biopsy Often steroids, cyclophosphamide, mycophenolate

GlomerularDiseases
Goodpasture Syndrome IgANephropathy Postinfectious Glomerulonephritis Alport Syndrome Polyarteritis Nodosa LupusNephritis Amyloidosis Nephrotic Syndrome

Acute Caused by toxins Not nephrotic No biopsy No steroids or immunosuppressives

MTBS2CK p.318

GlomerularDiseases/DiagnosticTests

Goodpasture Syndrome

All forms of glomerulonephritis have: UA with hematuria Dysmorphic red cells (deformed as they squeeze through an abnormal glomerulus) Red cell casts Urine sodium and FENa are low Proteinuria The amount of
proteinuria is the main difference between glomerulonephritis and nephrotic syndrome.
MTBS2CK p.318

Lung + kidney involvement No upper respiratory tract involvement Diagnosis Best initial test:
Antiglomerular basement membrane antibodies Kidney biopsy in

Most accurate test: Lung or kidney biopsy

Goodpasture syndrome shows linear deposits.

MTBS2CK p.319

Goodpasture Syndrome

Goodpasture Syndrome

Diagnosis Anemia from hemoptysis CXR abnormal, but not diagnostic Treatment Plasmapheresis Steroids Cyclophosphamide
Commons.wikimedia. Used with permission

MTBS2CK p.319

MTBS2CK p.319

10

IgAnephropathy(Bergerdisease)

PostinfectiousGlomerulonephritis

MCC of acute glomerulonephritis in U.S. Presentation: Gross hematuria 1 to 2 days after an upper respiratory tract infection IgA levels in 50% Most accurate test = kidney y biopsy p y No treatment: will resolve, progress to ESRD Steroids and ACE-inhibitors may help severe proteinuria
MTBS2CK p.319320

Most common infection is Streptococcus Follows throat infection or skin infection (impetigo) by 1 to 3 weeks Presentation

Cola-colored urine Edema (periorbital) HTN Oliguria

MTBS2CK p.320

Postinfectious Glomerulonephritis

Diagnostic Tests 1st - UA glomerulonephritis 2nd - Antistreptolysin O (ASO) titers and anti-DNAse antibody titers Most M t accurate t - Biopsy Bi

Commons.wikimedia, James Heilman, MD. Used with permission

MTBS2CK p.320

Postinfectious Glomerulonephritis

Postinfectious Glomerulonephritis

Treatment Management of strep infection does not reverse glomerulonephritis Use supportive therapies such as: Antibiotics A tibi ti Diuretics to control fluid overload
<5% of those with PSGN will progress.
MTBS2CK p.320
Commons.wikimedia. Used with permission

MTBS2CK p.320

11

AlportSyndrome

Polyarteritis Nodosa

Congenital defect of collagen Results in glomerular disease combined with:


Sensorineural hearing loss Visual disturbance from loss of collagen fibers that hold lens of eye in place

Definition Systemic vasculitis of small and medium-sized arteries Spares lungs Associated A i t d with ith hepatitis h titi B
PAN is nonspecific. Theres no single finding that allows you to answer the most likely diagnosis question.
MTBS2CK p.321

Theres no specific therapy to reverse this defect of type IV collagen


MTBS2CK p.320

PolyarteritisNodosa/Presentation The most common organ systems involved are: GI


Abdominal pain, bleeding, nausea, and vomiting occur

PolyarteritisNodosa/DiagnosticTests Blood tests will show: Anemia and leukocytosis ESR and C-reactive protein ANCA: Not present in most cases ANA and rheumatoid factor: Sometimes present in low titer Angiography Renal, mesenteric, or hepatic artery showing aneurysmal dilation Biopsy Most accurate if at symptomatic site
MTBS2CK p.321

Neuro
Peripheral neuropathy or mononeuritis mononeuritis multiplex. multiplex.

Skin
Petechiae, purpura, ulcers, livedo reticularis

Cardiac
Stroke or MI, particularly in young person

MTBS2CK p.321

Polyarteritis Nodosa

LupusNephritis SLE can give any degree of renal involvement Normal Mild, asymptomatic proteinuria Membranous glomerulonephritis Glomerulosclerosis scars kidneys without inflammation leading to ESRD requiring dialysis Diagnosis Biopsy (tells severity) Treatment Steroids, cyclophosphamide, mycophenolyate
MTBS2CK p.322

Treatment Prednisone and cyclophosphamide mortality Treat hepatitis B when its found

MTBS2CK p.321322

12

Amyloidosis

Amyloidosis

Amyloid is an abnormal protein produced in association with: Myeloma Chronic inflammatory diseases Rheumatoid arthritis Inflammatory bowel disease Chronic infections Amyloid, HIV
nephropathy, polycystic kidneys, and diabetes give large kidneys on sonogram and CT scan.
MTBS2CK p.322

Most accurate test = biopsy Apple-green birefringence with Congo red staining Best treatment = control underlying disease 2nd line Treatment = melphalan and prednisone

MTBS2CK p.322

Amyloidosis

NephroticSyndrome/Definition Measure of the severity of proteinuria in association with any form of glomerular disease
> 3.5 g/24 hrs

Liver can no longer increase the production of albumin to compensate for urinary losses Massive proteinuria leads to:
Edema (periorbital) Hyperlipidemia Thrombosis:
From urinary loss of natural anticoagulants protein C, protein S, and antithrombin
MTBS2CK p.322
Katsumi M. Miyai, MD, PhD. Regents of the University of California. Used with permission

MTBS2CK p.322

Nephrotic Syndrome/Etiology

NephroticSyndrome/DiagnosticTests

MCC = diabetes and hypertension Any of the glomerular diseases just described Other associations are: Cancer (solid organ): membranous Children: minimal change disease Injection drug use and AIDS: focal-segmental NSAIDs: minimal change disease and membranous SLE: Any of them

Best initial test = urinalysis Next best test = albumin/creatinine ratio or 24-hour urine protein collection Most accurate test = biopsy:
Focal-segmental Focal segmental Membranous Membranoproliferative Minimal change Mesangial
MTBS2CK p.323

MTBS2CK p.323

13

NephroticSyndrome/DiagnosticTests

NephroticSyndrome/Treatment

By definition, nephrotic syndrome is: Hyperproteinuria (> 3.5 g/24 hours) Hypoproteinemia Hyperlipidemia Edema

Treatment Best 1st therapy = glucocorticoids 2nd line = cyclophosphamide ACE inhibitors or ARBs to control proteinuria Edema managed with salt restriction and diuretics Hyperlipidemia is managed with statins as you would any form of hyperlipidemia

MTBS2CK p.324

MTBS2CK p.324

EndStageRenalDisease/Definition

EndStageRenalDisease
Etiology Presentation Manifestations Treatment KidneyTransplantation

Not defined as a particular BUN or creatinine Loss of renal function symptoms and laboratory abnormalities known as uremia Uremia is defined as the presence of:
Metabolic acidosis Fluid overload Encephalopathy Hyperkalemia Pericarditis
Peritoneal dialysis and hemodialysis are equally effective at removing wastes from the body.

Acute indications for dialysis


MTBS2CK p.324325

EndStageRenalDisease

EndStageRenalDisease

Manifestations of Renal Failure Anemia: Normocytic, normochromic


Loss of EPO

Manifestations of Renal Failure (contd) Bleeding:


No degranulation cant aggregate

Hypocalcemia: Cant absorb Ca from GI tract


25-hydroxy-vitamin D 1,25-dihydroxy-vitamin D

Infection:
No degranulation cant fight infection

Osteodystophy: Demineralized bones, soft/weak


Secondary hyperparathyroidism

Accelerated atherosclerosis:
Abnormal WBCs cannot clear lipid accumulation from arteries

Hyperphosphatemia: Cant excrete


PTH release of PO4 from bones

Pruritus:
Unclear reasoning; urea accumulating in skin causes itching
MTBS2CK p.325

Hypermagnesemia: Cant excrete


MTBS2CK p.325

14

EndStageRenalDisease

TreatmentofESRDManifestations Manifestation
Anemia Hypocalcemia Bleeding Pruritus Hyperphosphatemia Hypermagnesemia Atherosclerosis Endocrinopathy

Manifestations of Renal Failure (contd) Endocrinopathy:


Women are anovulatory Men have testosterone erectile dysfunction Hyperinsulinemia and insulin resistance
Either hyper- or hypoglycemic

Treatment
Erythropoietinreplacementand ironsupplementation ReplacevitaminDandcalcium DDAVPincreasesplateletfunction Dialysisandultravioletlight Oralbinders,seeHyperphos Rx RestrictionofhighMgfoods,laxatives, andantacids Dialysis Dialysis,estrogen&testosterone replacement

MTBS2CK p.325

MTBS2CK p.325

EndStageRenalDisease

EndStageRenalDisease

Treatment of Hyperphosphatemia Medications: Calcium acetate Calcium carbonate Use sevelamer and Sevelamer lanthanum when calcium level is high. Lanthanum
Never use aluminum containing phosphate binders. Aluminum causes dementia.
MTBS2CK p.326

Kidney Transplantation Only 50% of ESRD patients will be suitable for transplantation The donor doesnt have to be alive or related although these are both better related,
HLA-identical, related donor kidneys last 24 years on average.

MTBS2CK p.326

EndStageRenalDisease

SurvivalbyMethod
1year 3years 5years

Thromboticthromobocytopenicpurpura andHemolyticSyndrome

Living,relateddonor Deceaseddonor Dialysisalone Diabeticsondialysis


MTBS2CK p.326

95% 90% Variable Variable

88% 78% Variable Variable

72% 58% 3040% 20%

Different variants, same disease TTP is associated with HIV, cancer, and drugs (e.g., cyclosporine, ticlopidine, and clopidogrel) HUS is MC in children and the most frequently tested association is E. coli 0157:H7 and Shigella

MTBS2CK p.326

15

Thromboticthromobocytopenicpurpura andHemolyticSyndrome

Thromboticthromobocytopenicpurpura andHemolyticSyndrome

Both TTP and HUS are associated with:


Intravascular hemolysis Renal insufficiency Thrombocytopenia

TTP also l is i associated i d with: ih


Neurological symptoms Fever

The hemolysis is visible on smear with schistocytes, helmet cells, and fragmented red cells
MTBS2CK p.326 MTBS2CK p.327
Source: Abhay Vakil, MD.

Thromboticthromobocytopenicpurpuraand HemolyticSyndrome Most cases of HUS from E. coli will resolve spontaneously Plasmapheresis is generally urgent in TTP Severe HUS also needs urgent plasmapheresis If plasmapheresis is not one of the choices choices, use infusions of fresh frozen plasma (FFP) Steroids dont help Platelet transfusion is never the correct choice for TTP or HUS
MTBS2CK p.327

CysticDisease
SimpleversusComplexCysts Polycystic y y Kidney yDisease

Benign(Simple)vs.Malignant(Complex)Cysts SimpleCyst Echogenicity Walls Demarcation Transmission Echofree Smooth,thin Sharp Goodthrough toback ComplexCysts Mixedechogenicity Irregular,thick Lowerdensity onbackwall Debrisincyst

PolycysticKidneyDisease

Polycystic kidney disease (PCKD) presents with: Pain Hematuria Stones No therapy exists to Infection prevent or reverse cysts of any type. HTN

MTBS2CK p.326

MTBS2CK p.328

16

What is the MCC of death from PCKD?


a. b. c. d d. e. Intracerebral hemorrhage aneurysms Stones Can occur but do not lead to death Infection Pyelo is more common but rarely fatal potential. Doesn Doesnt t progress Malignancy No malignant potential Renal failure to RCC
Only 10-15% have cerebral

Sodium&Potassium Disorders
Hypernatremia Hyponatremia Hyperkalemia Hypokalemia

MTBS2CK p.328

Hypernatremia/Etiology

Hypernatremia/Etiology

Loss of free water Examples are:


Sweating Burns Fever F Pneumonia (insensible losses from hyperventilation) Diarrhea Diuretics
MTBS2CK p.328

Diabetes Insipidus (DI) High urinary volume water loss Insufficient or ineffective ADH Symptoms
Confusion Disorientation Lethargy Seizures

Sodium disorders cause CNS problems

If uncorrected, severe hypernatremia causes coma and irreversible brain damage


MTBS2CK p.328

DiabetesInsipidus(DI)
CentralDI NephrogenicDI

Hypernatremia/DiagnosticTests

Definition

LossofADH production CNSdisorders: Stroke Tumor Trauma Hypoxia Infection

LossofADH effect Lithium Demeclocycline Chronickidney disease Hypokalemia Hypercalcemia

Etiology

To distinguish DI from other causes of hypernatremia look for: Increased urine volume Decreased urine osmolality Decreased D d urine i sodium di
Increased urine volume despite dehydration and hyperosmolality of blood suggests DI
MTBS2CK p.329

MTBS2CK p.328329

17

DiagnosingDI
Best initial test? Continued high volume dilute urine

CentralDIvs.NephrogenicDI
Water deprivation test
urine volume osmolality

CDI

NDI

Polyuriaandnocturia Urineosmolalityandsodium Positivewaterdeprivationtest

Yes Low Yes Yes Low

Yes Low Yes No High

Diabetes Insipidus
Next best test?

Psychogenic Polydipsia

ADH administration test


urine volume osmolality Continued high volume dilute urine

Central DI
MTBS2CK p.329

Nephrogenic DI

**Most accurate test = ADH level NDI CDI

ResponsetoADH ADHlevel
MTBS2CK p.329

Hypernatremia/Treatment

Hypernatremia/Treatment

Fluid loss:
Correct underlying cause of fluid loss

CDI:
Replace ADH (vasopressin also known as DDAVP)

NDI:
Correct potassium and calcium Stop lithium or demeclocycline Give hydrochlorothiazide or NSAIDs for those still having NDI despite these interventions
MTBS2CK p.330

Complications of Therapy Cerebral edema: sodium levels brought down too rapidly Cerebral edema presents with worsening confusion and seizures

MTBS2CK p.330

Hyponatremia/Etiology

Hyponatremia/Etiology Intravascular volume depletion CHF, nephrotic syndrome, cirrhosis

Hypervolemia MCC of hyponatremia with hypervolemic state are:


CHF Nephrotic syndrome Cirrhosis

Increased ADH levels Stimulated by baroreceptors in atria and carotids

Free water reabsorption Sodium concentration drops


MTBS2CK p.330 MTBS2CK p.330

18

Hyponatremia/Etiology

Hyponatremia/Etiology

Hypovolemia Sweating Burns Fever Pneumonia Diarrhea Diuretics

Addisons disease Loss of adrenal function loss of aldosterone Aldosterone causes Na+ reabsorption If the body y loses aldosterone, , it loses Na+

MTBS2CK p.330

MTBS2CK p.331

Hyponatremia/Etiology

Hyponatremia/Presentation

Euvolemic Hyponatremia MCCs:


Pseudohyponatremia (hyperglycemia) Psychogenic polydipsia Hypothyroidism H th idi Syndrome of inappropriate ADH (SIADH) release

Hyponatremia presents entirely with CNS symptoms: Confusion Lethargy Disorientation Seizures Symptoms of hyponatremia Coma
are dependent on how fast it occurs.

MTBS2CK p.331

MTBS2CK p.331

Hyponatremia/DiagnosticTests

ResponsetoHyponatremia
Normallevels SIADH

SIADH High urine osmolality High urine sodium Low uric acid level and BUN Most accurate test is a high ADH level

Urine osmolality Urine sodium

Low (<100mOsm/kg) Low(<20mEq/L)

High

High(>40mEq/L)

MTBS2CK p.332

MTBS2CK p.332

19

Hyponatremia/Treatment

ClinicalManifestationsofHyponatremiabySeverity Degreeof hyponatremia Specific manifestation Management

Mild hyponatremia

Nosymptoms

Restrictfluids

The treatment answer isnt based on sodium level; ; its based on the symptoms.

Moderate

Minimalconfusion

Salineandloop diuretics Hypertonicsaline, conivaptan, tolvaptan

Severe

Lethargy,seizures, coma

MTBS2CK p.332

MTBS2CK p.332

Hyponatremia/Treatment

CentralPontineMyelinolysis

Complications of Treatment Goal: increase in Na is 0.5 to 1 mEq/hour (12 to 24 mEq/day) If the sodium level is brought up to normal too rapidly central pontine myelinolysis (osmotic demyelinization occurs)

MTBS2CK p.332

MTBS2CK p.332

Hyperkalemia

Hyperkalemia/Etiology
K > 5mEq/L

Typically defined as [K+] > 5mEq/L


Pseudohyperkalemia

Severe hyperkalemia can stop the heart in seconds if the level is high enough.

Decreased Excretion

Increased Release from Tissues

Hemolysis Leukocytosis Thrombocytosis


Repeat the blood sample

Renal failure Acute or chronic Low aldosterone state ACE inhibitors/ARBs RTA IV Drugs Addison disease

Cell lysis Low insulin Acidosis Drugs Beta blockers Digoxin Heparin

MTBS2CK p.332

MTBS2CK p.333

20

Hyperkalemia/Presentation

Hyperkalemia/DiagnosticTests

Potassium disorders interfere with muscle contraction and cardiac conductance Look for:
Weakness Paralysis when severe Ileus (paralyzes gut muscles) Cardiac rhythm disorders

If hyperkalemia is suspected Most urgent test:


EKG

The EKG in severe hyperkalemia shows:


Peaked T waves Wide QRS PR interval prolongation

Hyperkalemia does not cause seizures.


MTBS2CK p.333 MTBS2CK p.333334

Hyperkalemia/Treatment
K > 5mEq/L

Hyperkalemia/Treatment When theres hyperkalemia and an abnormal EKG, the most appropriate next step is clearly calcium chloride or gluconate.
No
Kayexalate Loop diuretics

Pseudohyperkalemia?
Yes
No treatment needed

EKG Changes?

Calcium chloride or calcium gluconate Insulin and glucose, inhaled beta agonist Give bicarbonate if acidosis is the cause Consider hemodialysis

Sodium = CNS symptoms Hyperkalemia = muscular and cardiac symptoms

MTBS2CK p.334

MTBS2CK p.334

Hypokalemia/Etiology
K < 3.5 mEq/L

Hypokalemia/Presentation

Decreased Intake

Shift into cells

Renal Losses

GI Losses

Very Rare Kidney can adjust excretion

Alkalosis Insulin agonists (stimulate Na/K ATPase pump)

aldosterone Conns Volume Cushings Bartters Licorice Hypomagnesemia RTA I and II

Hypokalemia leads to problems with muscular contraction and cardiac conduction Potassium is essential for proper neuromuscular contraction Hypokalemia presents with:
Weakness Paralysis Loss of reflexes
MTBS2CK p.335

Vomiting Diarrhea Laxatives

Muscular abnormalities may be so severe as to cause rhabdomyolysis.

MTBS2CK p.334335

21

Hypokalemia/Presentation

Hypokalemia/Treatment

EKG findings U waves are the most characteristic finding of hypokalemia Other findings are ventricular ectopy (PVCs), flattened T waves, and ST depression

No maximum rate of oral potassium replacement


GI tract cannot absorb potassium faster than the kidneys can excrete it

IV potassium replacement, however, can cause a fatal arrhythmia if its it s done too fast
You must allow time for potassium to equilibrate into cells

Hypokalemia does not cause seizures.

IV potassium replacement must be very slow.


MTBS2CK p.335

MTBS2CK p.335

Patient is admitted with vomiting and diarrhea from gastroenteritis. His volume status is corrected with IV fluids and diarrhea resolves. His pH is 7.40 and serum bicarbonate has normalized. Despite vigorous oral and IV replacement, his potassium level fails to rise. What should you do?

Woman with ESRD and G6PD deficiency skips dialysis for a few weeks. She experiences a crush injury during a MVA. She is taking dapsone and has recently eaten fava beans. What is the most urgent step?

a. b. c. d. e.

Consult nephrology Dont consult except for procedures! Magnesium level Parathyroid hormone level Mg needed for PTH release Intracellular pH level Not MCC 24-hour urine potassium level Doesnt tell you
underlying cause or change tx

a. b b. c. d. e. f. g. h.

Initiate dialysis Must look for EKG changes 1st EKG Bicarbonate administration Take 15-20min to work Insulin administration Kayexalate Take hours to work Urine dipstick Doesnt address life threatening CPK levels complications Urine myoglobin

MTBS2CK p.335336

MTBS2CK p.336

Nonaniongapmetabolicacidosis(NAGMA)

AcidBaseDisturbances
RenalTubularAcidosis UrineAnionGap p MetabolicAcidosis MetabolicAlkalosis RespiratoryAcidosisandAlkalosis

The anion gap is: Na+ - (Cl- + HCO3- ) Normal anion gap is 6 12 The difference in + & is due to negative charges g on albumin Elevated gaps means increased acids (negatively changed) present

MTBS2CK p.336

22

Nonaniongapmetabolicacidosis

RenalTubularAcidosis(RTA)

What causes NAGMA?


RTA Diarrhea

Distal RTA (Type I) Pathology in distal tubule Inability to generate enough bicarbonate
Inadequate bicarb acid cannot be excreted p pH of urine is high, g ,p pH of blood is low

Why is the gap normal in these?


They y have elevated Cl ( (hyperchloremic yp metabolic acidosis)

The anion gap increases from ingested substances (e.g., ethylene glycol or methanol), or organic acids (e.g., lactate) that are anionic and drives chloride levels down
MTBS2CK p.336337

Caused by factors damaging distal tubule


Amphotericin SLE or Sjgren syndrome

MTBS2CK p.337

RenalTubularAcidosis

DistalRTA(TypeI)/DiagnosticTests

Distal RTA (Type I) Symptoms to look out for?


Calcium oxalate kidney stones Due to alkaline urine

Best initial test


Urine pH (> 5.5)

Most accurate test


Infuse ammonium chloride (acid) Normal person excretes acid urine pH decreases Distal RTA cannot excrete acid urine pH remains high

Distal RTA calcifies kidney parenchyma (nephrocalcinosis)

No acid into the tubule makes the urine basic

Treatment
Replace HCO3
MTBS2CK p.337

MTBS2CK p.337

RenalTubularAcidosis

RTATypeII/DiagnosticTests

Proximal RTA (Type II) Pathology in proximal tubule Inability to absorb filtered bicarb Caused by factors damaging proximal tubule:
Amyloidosis Myeloma Fanconi syndrome Acetazolamide Heavy metals
MTBS2CK p.337

Proximal RTA (Type II) Urine pH: Low (< 5.5)


Because distal tubule absorbs bicarb normally If you give bicarb: urine pH increases

Osteomalacia
Chronic metabolic acidosis leaches calcium out of the bones and become soft

MTBS2CK p.337

23

RTATypeII/Treatment

RTATypeIV/Definition/DiagnosticTests

Treatment: MASSIVE doses of bicarbonate


Because proximal tubule doesnt absorb it normally

Thiazide diuretics
Cause volume depletion bicarb reabsorption
Both proximal and distal RTA are hypokalemic. Potassium is lost in the urine.

Hyporeninemic, Hypoaldosteronism (Type IV RTA) Pathology in distal nephron Problem is decreased amount or effect of aldosterone
Loss of Na+, retention of K+ and H+

Caused by diabetes Best test High urine Na even after Na-restricted diet Hyperkalemia
MTBS2CK p.338

MTBS2CK p.338

RTATypeIV/Treatment

TypesofRenalTubularAcidosis(RTA)
Proximal(TypeII) Distal(TypeI) UrinepH Variable High>5.5 Low TypeIV <5.5 High

Treatment: Fludrocortisone
To promote Na+ reabsorption, K+ and H+ secretion
Fludrocortisone is the steroid with the highest mineralocorticoid or aldosterone-like effect.
MTBS2CK p.338

Bloodpotassium Low level Nephrolithiasis Diagnostictest No Administer bicarbonate Thiazides

Yes

No

Administeracid Urinesalt loss Bicarbonate Fludrocortisone

Treatment
MTBS2CK p.338

UrineAnionGap/Definition

MetabolicAcidosis

NAGMA UAG Positive RTA Negative Diarrhea

Normal anion gap (612):


RTA and diarrhea

Elevated anion gap (>12):


The anion gap is increased if there are unmeasured anions driving bicarbonate level down
Respiratory alkalosis from hyperventilation compensates for all forms of metabolic acidosis.
MTBS2CK p.339

24

CausesofMetabolicAcidosiswith anIncreasedAnionGap
Cause Lactate Hypotensionor hypoperfusion DKA,starvation Ethyleneglycol overdose Test Bloodlactate level Acetonelevel CrystalsonUA Treatment Correct hypoperfusion Insulin&fluids Fomepizole, dialysis Formic acid

CausesofMetabolicAcidosiswith anIncreasedAnionGap
Cause Methanol overdose Renalfailure Test Inflamedretina Treatment Fomepizole, dialysis

Ketoacids Oxalicacid

Uremia Salicylates

BUN,creatinine Dialysis Alkalinizeurine

Aspirinoverdose Aspirinlevel

MTBS2CK p.339

MTBS2CK p.339

MetabolicAcidosis Arterial Blood Gas in Metabolic Acidosis The arterial blood gas (ABG) in metabolic acidosis will always have:
Decreased pH < 7.4 Decreased pCO2 indicating respiratory alkalosis as compensation p Decreased bicarbonate

You cannot determine the etiology of metabolic acidosis from the ABG. Metabolic problems always show compensation.
MTBS2CK p.339

MetabolicAlkalosis Key laboratory finding? Elevated serum bicarbonate level Respiratory compensation? Decreased respiratory rate CO2 retention respiratory acidosis Etiology GI loss: vomiting or nasogastric suction aldosterone: primary, Cushing, ACTH, volume contraction, licorice Diuretics Milk-alkali syndrome Hypokalemia
MTBS2CK p.339340

MetabolicAlkalosis/Etiology

RespiratoryAcidosisandAlkalosis

Arterial Blood Gas in Metabolic Alkalosis The ABG in metabolic alkalosis will always have:
Increased pH > 7.40 Increased pCO2 (compensatory respiratory acidosis) Increased bicarbonate
Metabolic derangements kill patients with cardiac arrhythmia. They also alter potassium levels.
MTBS2CK p.340

Minute ventilation is a more precise measure of respiratory status than respiratory rate Minute ventilation = RR x TV
Hyperventilation may occur with a tiny tidal volume.This does not increase minute ventilation.

MTBS2CK p.340

25

CausesofRespiratoryAcidosisandAlkalosis
Respiratoryalkalosis DecreasedpCO2 Increasedminuteventilation Metabolicacidosisas Compensation Anemia Anxiety Pain Fever Interstitiallungdisease Pulmonaryemboli
MTBS2CK p.340

Respiratoryacidosis IncreasedpCO2 Decreasedminuteventilation Metabolicalkalosisas Compensation COPD/emphysema Drowning Opiateoverdose 1antitrypsindeficiency Kyphoscoliosis Sleepapnea/morbidobesity

Nephrolithiasis
Etiology Treatment LongtermManagement MetabolicAcidosisandStoneFormation

Nephrolithiasis/Pearls

Most common stone?


Calcium oxalate

Forms more frequently in?


Alkaline urine

46-year-old man comes to the ED with excruciating left flank pain radiating to groin. He has some blood in urine. What is the next step?

Most M t common risk i kf factor? t ?


Overexcretion of calcium in urine

a. b. c. d. e.

Ketorolac X-ray Sonography Provide pain relief before diagnostic tests Urinalysis Serum calcium level

MTBS2CK p.340

MTBS2CK p.341

Nephrolithiasis/Treatment What is the most accurate diagnostic test for nephrolithiasis?


a. b. c. d. e. CT scan X-ray 10-20% false negative. Misses uric acid stones Sonography Less accurate Urinalysis y ay s show o hematuria, e a u a, not o spec specific c to o cause May Intravenous pyelogram Requires contrast, takes hours

The best initial therapy for acute renal colic is with: Analgesics Hydration Imaging I i CT noncontrast t t
Cystine stones are managed with surgical removal, alkalinizing urine.

Uric acid stones are not detectable on X-ray, but visualized on CT.
MTBS2CK p.341 MTBS2CK p.341

26

Nephrolithiasis/Treatment

Etiology of the stone determined with: Stone analysis Serum calcium, sodium, uric acid, PTH, magnesium, and phosphate levels 24-hour 24 h urine i f for volume, l calcium, l i oxalate, citrate, cystine, pH, uric acid, phosphate, and magnesium
Fat malabsorption increases stone formation.
MTBS2CK p.341

Woman with her first episode of renal colic has a 1.8 cm stone in the left renal pelvis. No obstruction. Normal BUN and creatinine. What is the next step in management?

a. a b. c. d. e.

Wait for it to pass; hydrate and observe If < 5mm Lithotripsy Surgical removal If > 2cm Hydrochlorothiazide Decreases calcium in urine Stent placement Only releives hydronephrosis

MTBS2CK p.342

Nephrolithiasis

50% with kidney stones will recur over the next 5 years
UTI gives struvite stones (magnesium/ammonium/pho sphate). Remove them surgically.

A man with a calcium oxalate stone is managed with lithotripsy. Stone is destroyed and passes. Urinary calcium level is increased. Besides increasing hydration, which is most likely to benefit this patient?

a. b. c. d. e.

Calcium restriction Can rate of oxalate stones Hydrochlorothiazide Furosemide Increases urinary calcium Stent placement Only relieves obstruction Increased dietary oxalate Can rate of oxalate stones

MTBS2CK p.342

MTBS2CK p.342

Nephrolithiasis

UrinaryIncontinence
Stressincontinence Urgeincontinence

Metabolic acidosis removes calcium from bones and increases stone formation In addition, metabolic acidosis decreases citrate levels
Citrate binds calcium, making it unavailable for stone formation

Symptoms

Olderwomanwith painlessleakagewith coughing,laughing,or liftingheavyobjects Havepatientstand andcough;observe forleakage

Suddenpaininbladder followedimmediatelyby theoverwhelmingurge tourinate Pressuremeasurement inhalffullbladder; manometry

Test

MTBS2CK p.342

MTBS2CK p.343

27

UrinaryIncontinence
Stressincontinence Urgeincontinence

Treatment

1. Kegel exercises 2. Localestrogen cream

3. Surgical tighteningof urethra


MTBS2CK p.343

1. Bladdertraining exercises 2. Localanticholinergic therapy 1. Oxybutynin y y 2. Tolterodine 3. Solifenacin 4. Dariferancin 3. Surgicaltighteningof urethra

Hypertension
Definition Etiology gy Presentation DiagnosticTests Treatment HypertensiveCrisis

Hypertension/Definition

Hypertension/Etiology Most common etiology? (95%)


Essential hypertension

Systolic pressure > 140 mmHg Diastolic pressure > 90 mmHg Special population with lower threshold?
Diabetics

Common secondary causes?


Renal artery stenosis Glomerulonephritis Coarctation of aorta Acromegaly Pheochromocytoma Hyperaldosteronism Cushing syndrome or any cause of hypercortisolism including therapeutic use of glucocorticoids Congenital adrenal hyperplasia
MTBS2CK p.343344

Definition of hypertension? > 130/80

MTBS2CK p.343

Hypertension/Presentation

Hypertension/Presentation

Typically asymptomatic Symptoms of end organ damage:


Coronary artery disease Cerebrovascular disease CHF Visual disturbance Renal insufficiency Peripheral artery disease
MTBS2CK p.344

Presentation of Secondary Hypertension Renal artery stenosis:


Bruit is auscultated at flank and is continuous throughout systole and diastole

Glomerulonephritis p Coarctation of aorta:


Upper extremity > lower extremity BP

MTBS2CK p.344

28

Hypertension/Presentation

Hypertension/DiagnosticTests

Acromegaly Pheochromocytoma: Episodic hypertension with flushing Hyperaldosteronism: Weakness from hypokalemia

Other tests to perform in hypertensive patients EKG Urinalysis Glucose Gl measurements t to t exclude l d concomitant diabetes Cholesterol screening

MTBS2CK p.344

MTBS2CK p.344

Hypertension/Treatment

Hypertension/DrugTherapy

Best initial therapy? Lifestyle modifications:


Weight loss (*most effective*) Sodium restriction Dietary modification Exercise Tobacco cessation
Lifestyle modifications are tried for 3 to 6 months before medications are started.
MTBS2CK p.344345

Best first drug? (all-comers)


Hydrochlorothiazide

If BP > 160/100?
Defines Stage 2 HTN Start 2 medications immediately

MTBS2CK p.345

Hypertension

Hypertension

If diuretics dont control BP, the most appropriate next step in management is:
ACE inhibitor Angiotensin receptor blocker (ARB) Beta blocker (BB) Calcium-channel blocker (CCB)

Medications for refractory hypertension: Central-acting alpha agonists


Alpha methyldopa, clonidine

Peripheral-acting alpha antagonists


Prazosin, terazosin, doxazosin

Direct-acting vasodilators
Hydralazine, minoxidil

MTBS2CK p.345

MTBS2CK p.345

29

Hypertension/CompellingIndications
Ifthisisinthehistory Thisisthebestinitialtherapy

HypertensiveCrisis

Coronaryarterydisease Diabetesmellitus Benignprostatic hypertrophy Depressionandasthma Hyperthyroidism Osteoporosis


MTBS2CK p.345

BB,ACE,ARB ACE,ARB Alphablockers

Defined as HTN with SYMPTOMS Confusion Blurry vision Dyspnea Chest Ch t pain i

AvoidBBs BBfirst Thiazides


MTBS2CK p.345346

HypertensiveCrisis

The best initial therapy? Labetolol or nitroprusside 2nd line agents? Enalapril CCBs:
Diltiazem Verapamil

Esmolol
MTBS2CK p.346

30

Stroke Neurology
ConradFischer,MD
AssociateProfessorofMedicine TouroCollegeofMedicine NewYorkCity

Definition Etiology Presentation DiagnosticTests Treatment

Stroke/Definition

Stroke/Etiology

Sudden onset neurological deficit Death of brain tissue 3rd most common cause of death in the USA Risk Ri k f factors t
Hypertension Diabetes Hyperlipidemia Tobacco
MTBS2CK p.273

Bleeding (15%) Blockage of flow (85%)


Thrombosis Embolus Heart H t
Atrial fibrillation Valvular heart disease DVT via Patent foramen ovale

Carotid stenosis
MTBS2CK p.273

Stroke/Presentation

Stroke/Presentation

Middle cerebral artery (MCA) >90% of all strokes Weakness or sensory loss
Opposite (contralateral) side from stroke

Contralateral homonymous hemianopsia Eyes look toward the side of lesion Aphasia If stroke on same side as speech center On left in 90%
MTBS2CK p.273
Source:commons.wikimedia.org

MTBS2CK p.273

Stroke/Presentation

Stroke/DiagnosticTests

Anterior cerebral artery (ACA)


Personality/cognitive defects (e.g., confusion) Urinary incontinence Leg > arm weakness

Posterior cerebral artery y (PCA) ( ) Loss of consciousness


Ipsilateral (same side) sensory loss of face, 9th, and 10th CNs Contralateral sensory loss of limbs Limb ataxia
MTBS2CK p.274

Best initial test for stroke is... CT scan of the head With or without contrast?... Without contrast Most accurate test for stroke is... MRI

MTBS2CK p.274

Stroke/DiagnosticTests

Stroke/Treatment Nonhemorrhagic Best initial therapy when LESS than 3 hours since onset...
Thrombolytics

CT Done first Excludes hemorrhage Prior to treatment

More than 3 hours since onset...


Aspirin

If the patient is already on aspirin...


ADD dipyridamole OR SWITCH to clopidogrel
Source:MohammadMaruf,MD

MTBS2CK p.274

MTBS2CK p.274 275

Stroke/Treatment

Stroke/EvaluationofCauses&Treatment

Hemorrhagic Best initial treatment... Nothing

Echocardiogram Damaged valves? Surgical replacement Thrombi? Heparin followed by warfarin to INR of 2-3

MTBS2CK p.275

MTBS2CK p.275

Stroke/EvaluationofCauses&Treatment

Stroke/EvaluationofCauses&Treatment

Electrocardiogram Atrial fibrillation or atrial flutter Warfarin as long as arrhythmia persists


noPwave Afib Pwave NSR
Source:JHeuser Source:KjetilLenes

Stroke/EvaluationofCauses&Treatment

Stroke/EvaluationofCauses&Treatment

Holter monitor
Detect arrhythmias If initial EKG is normal: Do Holter monitor Detect atrial arrhythmias G t sensitivity Greater iti it than th EKG

Holter monitor

MTBS2CK p.275

Source:Macro987

Stroke/EvaluationofCauses&Treatment

Stroke
Patient presents with: Sudden onset unilateral weakness Facial droop +/- Speech deficits

Carotid duplex ultrasound Carotid stenosis is a frequent cause of emboli Surgery? Symptomatic S t ti and d >70% 70% stenosis t i Endarterectomy superior to carotid angioplasty

Best Initial Test: Head CT without contrast

No acute process

Acute hemorrhage

Likely ischemic stroke


MTBS2CK p.275

Likely hemorrhagic stroke

Stroke

Likely ischemic stroke

Stroke
Likely hemorrhagic stroke Symptoms last < 24 hours, then resolve

< 3 hours from onset of symptoms

> 3 hours from onset of symptoms

Treatment: Consider TPA, if no hx of bleeding

Treatment: Aspirin ADD dipyridamole or switch to clopidogrel if already on aspirin

Transient ischemic attack

Treatment: Control blood pressure Optimal systolic BP is between 140-160 mmHg If > 170 mmHg, mmHg use nicardipine, nicardipine enalaprilat, enalaprilat or labetalol Reverse anticoagulation If patient is on warfarin fresh frozen plasma, vitamin K If patient is on heparin protamine sulfate

MTBS2CK p.274 275

Perform further studies to asses etiology

Stroke

Goals!
Diabetes
Hemoglobin A1C <7%

Headache
Types Physical y Examination DiagnosticTests Treatment Trigeminal&PostherpeticNeuralgia

Hypertension
BP <140/90 mmHg

LDL
<100 mg/dL if carotid stenosis is the cause

Tobacco smoking
Must stop!
MTBS2CK p.276

Headache

Headache/TensionHeadache Constant pressure Mild to moderate pain, mainly bilateral Lasts 4-6 hours Physical exam Nothing! Diagnostic tests None (all normal) Treatment NSAIDs and other analgesics
MTBS2CK p.276

Types Tension Migraine Cluster Giant cell (temporal) arteritis Pseudotumor cerebri

MTBS2CK p.276

Headache/Migraine
Visual disturbance Photophobia aura May be related to: Food/menses Precipitated by emotions Ph i l exam Physical Rare cases: aphasia, numbness, dysarthria, weakness Diagnostic tests All normal Scan head the first time, then stop
MTBS2CK p.276

Headache/Migraine
Treatment Abortive
Triptans or ergotamine

Prophylactic (preventive)
> 3 migraines/month Propranolol

Other preventive medications: Calcium-channel blockers Tricyclic antidepressants SSRIs Botulinum toxin injections
MTBS2CK p.277

Headache/ClusterHeadache Symptoms
Frequent, short duration, high intensity

Headache/ClusterHeadache Treatment Abortive Triptans Ergotamine 100% oxygen Preventive Verapamil Lithium Prednisone
MTBS2CK p.277

Men 10X MORE than women!!! Physical examination


Red, tearing g eye y with rhinorrhea Horner syndrome occasionally

Diagnostic tests
Scan head first time No need for subsequent imaging with recurrences
MTBS2CK p.276

Headache/GiantCell(Temporal)Arteritis Symptoms
Visual disturbance, jaw claudication Muscle pain, fatigue, and weakness

Headache
Headache
Episodic pain Unilateral periorbital intense pain Lacrimation Eye-reddening Nasal stuffiness Lid ptosis +/- Aura, photophobia Related to food/emotions/menses Rare: aphasia, numbness, dysarthria

Physical exam
Visual loss, temporal area tenderness

Bilateral band-like pressure Lasts 4-6 hours Normal P/E

Diagnostic tests
Elevated ESR Most accurate test? Biopsy!

Tension Headache H d h
Treatment NSAIDs Acetaminophen

Migraine
Treatment Avoid triggers NSAIDs 5-HT1 agonists 3 attacks/month Prophylaxis Propranolol Sodium valproate

Cluster Headache
Acute Treatment Sumitriptan Octreotide Oxygen Prophylaxis Verapamil Prednisone Sodium valproate

Treatment
Prednisone
MTBS2CK p.276 MTBS2CK p.389.4

Headache/PseudotumorCerebri

CranialNerve6Palsy

Associated with Obesity Oral contraceptives Vitamin A toxicity Mimics brain tumor: nausea & vomiting Physical exam Papilledema Diplopia: 6th CN (abducens) palsy
MTBS2CK p.276

Headache/PseudotumorCerebri

Headache/PseudotumorCerebri

Diagnostic tests CT or MRI Normal Done to exclude intracranial mass Lumbar puncture Increased pressure CSF Normal
MTBS2CK p.277

Treatment Acetazolamide +/- furosemide Weight loss Steroids Repeated lumbar puncture Ventriculoperitoneal shunt

MTBS2CK p.277

Headache/TrigeminalNeuralgia Idiopathic 5th CN Severe, overwhelming knife-like facial pain Precipitated by Chewing Touching the face Pronouncing words in which the tongue strikes the back of front teeth No specific diagnostic test
MTBS2CK p.278

Headache/TrigeminalNeuralgia

Treatment Carbamazepine or oxcarbazepine Baclofen Lamotrigine Surgical decompression when failing medications

MTBS2CK p.278

Headache/PostherpeticNeuralgia

Headache/PostherpeticNeuralgia

Residual pain following resolution of herpes zoster (shingles) vesicular lesions Shingles Shi l i is a painful i f l dermatomal rash that occurs in 15% with prior varicella zoster (chickenpox) infection
MTBS2CK p.278
Shinglesrashcausedbyherpeszostervirus.Source:NIAID

Acute Treatment: Acyclovir, famciclovir, or valacyclovir reduce the incidence of pain Steroids do not help

MTBS2CK p.278

Headache/PostherpeticNeuralgia Treatment
Tricyclic antidepressants (amitriptyline) Gabapentin or pregabalin Carbamazepine Phenytoin Topical capsaicin Antiepileptic medications
Have some beneficial effect in neuropathic pain (e.g., postherpetic neuralgia or peripheral neuropathy)

PreventionofHerpesZoster(Shingles)

Zoster vaccine Everyone > 60 High dose varicella vaccine Decreases reactivation of varicella into zoster t

None work in > 50% to 70%


MTBS2CK p.278 MTBS2CK p.278

ClassificationofSeizures

Seizures
Classification DiagnosticTests Treatment Management

Partial Absence (petit mal) Generalized (tonic-clonic) Status S a us epilepticus ep ep cus

MTBS2CK p.279

PartialSeizures

AbsenceSeizures

Focal to one part of the body


Limited to arm or leg

Partial seizures can be: Simple


Intact consciousness

Complex
Loss of consciousness

Also referred to as petit mal Consciousness briefly impaired Patient often remains upright and often appears normal or stares into space Absence Ab seizures i occur more often ft i in children

MTBS2CK p.279

MTBS2CK p.280

GeneralizedSeizures

GeneralizedSeizures/Causes

Also referred to as tonic-clonic Generalized seizure Varying phases Muscular rigidity (tonic) Followed by jerking of muscles for several minutes (clonic)

Hyponatremia or hypernatremia Hypoxia Hypoglycemia Any CNS infection Encephalitis, meningitis, abscess Any CNS anatomic abnormality Trauma, stroke, tumor

MTBS2CK p.280

MTBS2CK p.278

GeneralizedSeizures/Causes

Seizures/DiagnosticTests

Hypocalcemia Uremia (elevated creatinine) Hepatic failure Withdrawal Alcohol, barbiturate, and benzodiazepine Cocaine toxicity Hypomagnesemia (rare)
MTBS2CK p.278

Electroencephalogram (EEG) The right answer after the other tests are done If CT or MRI are normal No N point i ti in EEG if th there i is a clear l metabolic, toxic, or anatomic defect causing the seizure

MTBS2CK p.279

Delerium,Stupor,andComa Altered consciousness Unresponsiveness to stimuli From metabolic, toxic, and CNS infections Also called Confusion Difficulty Diffi lt with ith arousal l Obtundation When severe enough, a seizure occurs Confusion is to coma and seizure, as angina is to myocardial infarction
MTBS2CK p.279

TreatmentofStatusEpilepticus

The only seizure treatment that is clear Best initial therapy... Benzodiazepine Lorazepam Diazepam intravenously

MTBS2CK p.279

TreatmentofStatusEpilepticus If seizure persists... Phenytoin or fosphenytoin Fosphenytoin = phenytoin efficacy Fosphenytoin has fewer adverse effects Ph Phenytoin t i Hypotension and AV block Fosphenytoin No BP or cardiac effect Can be given more rapidly
MTBS2CK p.279

TreatmentofStatusEpilepticus

1. Benzodiazepines did NOT work 2. Fosphenytoin did NOT work 3. Use phenobarbital

MTBS2CK p.279

TreatmentofStatusEpilepticus

TreatmentofStatusEpilepticus

What if barbiturates do NOT work?


1. Neuromuscular blocking agents allow intubation: Succinylcholine Vecuronium Pancuronium

2. General anesthesia
Midazolam or propofol Place on ventilator before propofol, which can stop breathing
MTBS2CK p.278

To summarize... 1. Benzodiazepine 2. Fosphenytoin 3. Phenobarbital 4. General anesthesia

MTBS2CK p.280

EpilepsyTreatment/Indications

ChoiceofAntiepilepticDrugs Status epilepticus treatment is clear Epilepsy long-term treatment is not clear No medication is clearly superior to the others Phenytoin, valproic acid, and carbamazepine all have nearly equal efficacy Gabapentin, Gabapentin topiramate, topiramate lamotrigine, lamotrigine oxcarbazepine, or levetiracetam Ethosuximide is the best therapy for absence seizures You cannot be asked to choose between them based on efficacy
MTBS2CK p.280

Antiepileptic drugs Not long term for single seizure When should you start after a single seizure? Presentation P t ti in i status t t epilepticus il ti Abnormal EEG Family history of seizures

MTBS2CK p.280

ChoiceofAntiepilepticDrugs

DiscontinuanceofAntiepilepticMedication

If not controlled with single agent, an alternate medication should be tried If still not controlled, add a second drug If multiple medications do not work: Surgery!

Wait until seizure-free for 2 years Sleep deprivation EEG: The best way to detect possibility of recurrence Elicits abnormal activity with more sensitivity This is NOT a 100% sensitive test!
MTBS2CK p.281

MTBS2CK p.280

38-year-old man evaluated for seizures achieves partial control with second medication. He drives to work daily. What do you do about his ability to drive?
a. Confiscate his license Rules vary state to state Cannot b. Allow him to drive if he is seizure-free for 1 year prevent c. Allow him to drive as long as his seizure history is noted t d on hi his li license Rules vary state to state d. Recommend that he find an alternate means of transportation e. Do not let him leave the office unless he is picked up by someone; no further driving Cannot incarcerate f. Allow him to drive as long as he is accompanied Being accompanied does not prevent seizures

Subarachnoid Hemorrhage
Definition/Etiology DiagnosticTests Treatment

MTBS2CK p.281

10

SubarachnoidHemorrhage

CircleofWillis

Rupture of aneurysm Usually in Circle of Willis (anterior) Aneurysms found in 2% of autopsies Majority never rupture Cause of rupture not clear
MTBS2CK p.281 MTBS2CK p.281

SubarachnoidHemorrhage/Diagnosis

SubarachnoidHemorrhage/Diagnosis

What is the most likely diagnosis? Look for...


Sudden onset of severe headache with meningeal irritation (stiff neck, photophobia) and fever!

How does SAH differs from meningitis? Very sudden in onset Loss of consciousness

Loss of consciousness in 50% Sudden increase in intracranial pressure Focal neurological complications occur in 30%

MTBS2CK p.281

MTBS2CK p.281

SubarachnoidHemorrhage/Diagnosis Best initial test is... CT without contrast (95% sensitive) Most accurate test is... Lumbar puncture showing blood

SubarachnoidHemorrhage/Diagnosis

LP necessary only for 5% with falsely negative CT CSF in SAH has both increased WBCs and RBCs WBC can mimic meningitis Ratio of WBCs to RBCs will be normal in SAH

Source:SabaAnsari,MD

MTBS2CK p.282

MTBS2CK p.282

11

SubarachnoidHemorrhage/Diagnosis

SubarachnoidHemorrhage/Diagnosis

WBC count > normal 1:500?


Suspect meningitis

Xanthochromia
Yellow CSF From breakdown red cells in CSF

EKG
May show large or inverted T waves Suggestive of myocardial ischemia Cerebral T waves From excessive sympathetic activity
MTBS2CK p.282

Angiography Determines site of aneurysm Guides lesion repair MRA Diagnosis based on... CT and sometimes LP Only angiography can tell location

MTBS2CK p.282

SubarachnoidHemorrhage/Diagnosis

SubarachnoidHemorrhage/Treatment

Nothing reverses hemorrhage Nimodipine (calcium-channel blocker) prevents subsequent ischemic stroke

MTBS2CK p.282

MTBS2CK p.283

SubarachnoidHemorrhage/Treatment

SubarachnoidHemorrhage/Treatment

Embolization (coiling) Catheter clogs up site of bleeding Prevents repeated hemorrhage Interventional neuroradiologist places platinum wire p Embolization superior to surgical clipping for survival and complications

Ventriculoperitoneal shunt SAH associated with hydrocephalus Shunt only if hydrocephalus develops

MTBS2CK p.283

MTBS2CK p.283

12

SubarachnoidHemorrhage/Treatment

SubarachnoidHemorrhage/Treatment

Seizure prophylaxis Phenytoin is not routine If the question asks Which of the following is most likely to decrease mortality? ... mortality?
Phenytoin is not the answer

Consultation is only right when you want to do a procedure that isnt given as a choice.

MTBS2CK p.283

MTBS2CK p.284

A woman in the ED with a severe headache one day prior to admission. Temp 103F, nuchal rigidity & photophobia. Head CT is normal. LP CSF WBCs: 1,250 RBCs: 50,000 Whats the next step?

SpineDisorders
AnteriorSpinalArteryInfarction SubacuteCombinedDegenerationoftheCord SpinalTrauma BrownSquardSyndrome Syringomyelia

Angiography g g p y For location of SAH, this is not SAH 50 000 RBC should 50,000 h ld only l Ceftriaxone and vancomycin give 50-100 WBCs Nimodipine Prevent SAH stroke 1,250 WBCs is infection Embolization Permanent SAH fix Surgical clipping Worse alternative to embolization Useless for blood Repeat CT scan with contrast Neurosurgical consultation Dont consult unless you REALLY want a procedure MTBS2CK p.283 not listed a. b. c. d. e. f. g.

AnteriorSpinalArteryInfarction
Loss of all function except posterior column Position and vibratory sensation intact Flaccid paralysis below the level of infarction Loss of deep tendon reflexes (DTRs) at level of infarction Evolves into spastic paraplegia several weeks later Loss of pain and temperature Extensor plantar response

SubacuteCombinedDegeneration oftheCord

B12 deficiency or neurosyphilis Position & vibratory sensation lost Everything else intact

No specific therapy

MTBS2CK p.284

MTBS2CK p.284

13

SpinalTrauma Acute limb weakness and/or sensory disturbance Below level of injury Severity in proportion to degree of injury Sphincter p function impaired p Loss of DTRs at level of injury Followed by hyperreflexia below level of trauma Treat with glucocorticoids
MTBS2CK p.284

BrownSquardSyndrome Unilateral hemisection of spinal cord Due to injury Knife cutting half the cord On physical exam...
Ipsilateral Motor Position Vibration Contralateral Pain Temperature

No treatment
MTBS2CK p.284

Syringomyelia Fluid-filled, dilated central canal of spinal cord Widening bubble or cavitation damages neural fibers passing near center of spine Caused by tumor or severe trauma to spine

Syringomyelia What is the most likely diagnosis? Look for: Loss of pain & temperature bilaterally Across the upper back and both arms Look for the phrase capelike capelike distribution distribution Loss of reflexes Muscle atrophy

MTBS2CK p.285

MTBS2CK p.285

Syringomyelia The most accurate test is... MRI The best treatment is... Surgical Removal of tumor Drain cavity
Source:MohammadMaruf,MD.

CNSAbscess&Disease
Brainabscess Neurocutaneousdiseases

MTBS2CK p.285

14

BrainAbscess/Definition&Etiology Collection of infected material in brain parenchyma Acts as space-occupying lesion Spreads from contiguous infection Starts in sinuses, , mastoid air cells, , or otitis media Any bacteremia
Pneumonia and endocarditis cause bacteremia, which can lead to a brain abscess

BrainAbscess/Presentation

Headache, nausea, vomiting, fever, seizures, and focal neurological findings No way to distinguish brain abscess from cancer without a biopsy Cancer gives fever fever, too

MTBS2CK p.285

MTBS2CK p.286

BrainAbscess/DiagnosticTests
Best initial test is... Head CT or MRI Most accurate test is... Brain biopsy

BrainAbscess/DiagnosticTests
Scan shows a ring or contrast-enhancing lesion Surrounding edema Mass effect Cancer and infection are indistinguishable based on imaging study alone
Source:NishithPatel Source:NishithPatel

MTBS2CK p.286

MTBS2CK p.286

BrainAbscess/Microbiology
Biopsy is essential Only biopsy distinguishes abscess from cancer Only way to know sensitivity of organism Abscesses can be... Staphylococci, streptococci, Gram-negative bacilli, and anaerobes Frequently mixed (polymicrobial) Treat for... 6 to 8 weeks intravenously Followed by 2 to 3 more months orally
MTBS2CK p.286

BrainAbscess/Treatment
Empiric therapy Penicillin + metronidazole + ceftriaxone (or cefepime) Vancomycin (alternative to penicillin) Use vancomycin if theres been recent neurosurgery

MTBS2CK p.287

15

NeurocutaneousDiseases

TuberousSclerosis
Neurological abnormalities
Seizures, slowly progressive mental deterioration

Tuberous sclerosis Neurofibromatosis (von Recklinghausen Disease) Sturge-Weber Syndrome

Skin
Adenoma sebaceum (reddened facial nodules) Shagreen patches (leathery plaques on trunk) Ash leaf (hypopigmented) patches


MTBS2CK p.287

Retinal lesions Cardiac rhabdomyomas No specific treatment Control seizures


MTBS2CK p.287

Neurofibromatosis (vonRecklinghausenDisease)
Neurofibromas: soft, flesh-colored lesions attached to peripheral nerves 8th CN tumors Cutaneous hyperpigmented lesions (caf au lait spots) Meningioma and gliomas
Source:MohammadMaruf,MD.

Neurofibromatosis (vonRecklinghausenDisease)

No specific treatment Lesions affecting 8th CN may require surgical decompression to preserve hearing

MTBS2CK p.287

MTBS2CK p.287

SturgeWeberSyndrome Port-wine stain of face Seizures CNS


Homonymous hemianopsia Hemiparesis Mental subnormality

MovementDisorders Part1
EssentialTremor Parkinsonism RestlessLegSyndrome Huntington Disease TouretteDisorder MultipleSclerosis

Skull X-ray
Calcification of angiomas

No treatment Control seizures


MTBS2CK p.288

16

EssentialTremor
Occurs at both rest and intention (i.e., reaching for things) Tremor greatest in hands, but can affect head Affects some manual skills
Handwriting or use of computer keyboard

EssentialTremor

Caffeine makes it worse Treat with propranolol

Tremor at rest and exertion improved with a drink of alcohol is key to diagnosis
MTBS2CK p.288

Parkinsonism/Definition Loss of substantia nigra Decrease in dopamine Movement disorder Tremor and gait disturbance

Parkinsonism/Etiology Head trauma from boxing Use of antipsychotic medications


Thorazine

Encephalitis Reserpine Metoclopromide

No test for parkinsonism Diagnosis is entirely on clinical presentation

MTBS2CK p.288

MTBS2CK p.288

Parkinsonism/Presentation
Look for 50-60 (or older) with... Tremor Muscular rigidity Bradykinesia (slow movements) Shuffling gait with unsteadiness on turning Tendency T d to t fall f ll Cogwheel rigidity (slowing of movement on passive flexion or extension of extremity) Facial expression limited (hypomimia) Writing is small (micrographia)
MTBS2CK p.289

PillRollingTremor

17

ParkinsonFacies

Parkinsonism/Presentation Postural instability or orthostatic hypotension Inability of pulse and BP to reset Lightheadedness when getting up from seated position

The most frequent Parkinsonism question is: Treatment


Know the drugs!
MTBS2CK p.289

Parkinsonism/Treatment Mild disease Anticholinergic medications relieve tremor and rigidity Benztropine and trihexyphenidyl Adverse effects Dry mouth Worsening prostate hypertrophy Constipation More frequent in older patients
MTBS2CK p.289

Parkinsonism/Treatment Mild disease Amantadine Increases release of dopamine from substantia nigra Definitely y the answer in older patients p ( (> 60) ) intolerant of anticholinergic medications

MTBS2CK p.289

Parkinsonism/Treatment Severe disease Inability to care for themselves, orthostatic Dopamine agonists Pramipexole Ropinirole Bromocriptine and cabergoline are older agents Infrequent use because of adverse effects

Parkinsonism/Treatment Severe disease Levodopa/carbidopa Single most effective medication Associated with on/off phenomena Episodes of insufficient dopamine (off) ( off ) characterized by bradykinesia The on effect is too much dopamine resulting in dyskinesia

MTBS2CK p.289

MTBS2CK p.289

18

Parkinsonism/Treatment Severe disease COMT inhibitors (tolcapone, entacapone) Extends duration of levodopa/carbidopa Blocks metabolism of dopamine Used only with levodopa/carbidopa Use when there are on/off phenomena to even out dopamine level When response to therapy is inadequate

Parkinsonism/Treatment
Severe disease MAO inhibitors (rasagiline, selegiline)
As single agent or an adjunct to levodopa/carbidopa Block metabolism of dopamine

Deep brain stimulation


Electrical stimulation Highly effective in some patients

MTBS2CK p.289

MTBS2CK p.289

Spasticity
70-year-old man with extremely severe parkinsonism comes to ED with psychosis and confusion developing at home. Hes maintained on levodopa/carbidopa, ropinirole, and tolcapone. Whats the most appropriate next step in management?

a. b. c. d. e.

Stop levodopa/carbidopa Severe Parkinsonism Start clozapine Stop ropinirole Dont stop meds severe parkinsonism Stop tolcapone Dont stop meds severe parkinsonism Start haloperidol More adverse effects than clozapine

Dont Don t stop meds!

Painful, contracted muscles From damage to CNS Often associated with MS No single treatment always effective Baclofen and tizanadine (central-acting alpha agonist) may work

MTBS2CK p.290

MTBS2CK p.290

RestlessLegsSyndrome

HuntingtonDisease(HD)

Uncomfortable sensation in legs Creepy and crawly at night Bed partner who is being kicked at night Discomfort worsened by caffeine Relieved by moving legs Treat with dopamine agonists (e.g., pramipexole)

Hereditary disease CAG trinucleotide repeat sequences on chromosome 4

MTBS2CK p.290

MTBS2CK p.291

19

HuntingtonDisease/Presentation What is the most likely diagnosis? HD is the answer when you see... Choreaform movement disorder (dyskinesia) Dementia Behavior changes Irritability, moodiness, antisocial behavior Onset between 30 - 50 Often with family history of HD

HuntingtonDisease

Diagnosis Symptom triad Movement Memory Mood

Genetic testing >99% accurate

MTBS2CK p.291

MTBS2CK p.291

HuntingtonDisease

TouretteDisorder Idiopathic disorder Vocal tics, grunts, and coprolalia Motor tics (sniffing, blinking, frowning) Obsessive-compulsive behavior No specific diagnostic test Treat with neuroleptics e.g. Fluphenazine, clonazepam, pimozide ADHD drugs Methylphenidate
MTBS2CK p.291

Treatment No treatment can reverse HD Dyskinesia treated with tetrabenazine Psychosis treated with haloperidol or quetiapine ti i

MTBS2CK p.291

MultipleSclerosis

MultipleSclerosis/Presentation
What is the most likely diagnosis? Look for: Multiple neurological deficits of CNS Affects any aspect of CNS Most common presentation:
Blurry vision Visual disturbance from optic neuritis

Idiopathic disorder Exclusively CNS (brain and spinal cord) More common in white women living in colder climates

MTBS2CK p.291

MTBS2CK p.291

NEUR_02_19

20

MultipleSclerosis/Presentation
After optic neuritis, the most common abnormalities are motor and sensory Other findings
Fatigue Spasticity and hyperreflexia Cerebellar deficits

MultipleSclerosis/DiagnosticTests
MRI: Best initial AND Most accurate test

Least common findings Cognitive defects Dementia Sexual dysfunction


Source:SabaAnsari,MD

MTBS2CK p.292

MTBS2CK p.292

MultipleSclerosis/DiagnosticTests
Lumbar puncture: CSF Mild elevation in protein < 50 - 100 WBCs
Oligoclonal bands: Found in 85% of MS Oligoclonal bands: Not specific to MS

MultipleSclerosis/Treatment

The best initial therapy for acute exacerbations of MS... High-dose steroids

Oligoclonal bands are the answer in 3% - 5% with equivocal (nondiagnostic) MRI

MTBS2CK p.292

MTBS2CK p.292

MultipleSclerosis/Treatment Drugs that prevent relapse and progression Glatiramer (copolymer1) Beta-interferon Natalizumab Mitoxantrone Fingolimod Dalfampridine Azathioprine and cyclophosphamide (rarely used)
MTBS2CK p.292

MultipleSclerosis/Treatment A new MS treatment is started. MRI shows new, multiple white matter hypodense lesions. What is it? Progressive g multifocal leukoencephalopathy (PML) What is the cause? Natalizumab

MTBS2CK p.293

21

AmyotrophicLateralSclerosis(ALS)

MovementDisorders Part2
AmyotrophicLateralSclerosis CharcotMarieTooth Disease PeripheralNeuropathy FacialNervePalsy GuillainBarrSyndrome MyastheniaGravis

Exclusively a loss of upper and lower motor neurons

Idiopathic
MTBS2CK p.293

AmyotrophicLateralSclerosis(ALS)
What is the most likely diagnosis? Look for... Weakness starting in 20s to 40s Combination of upper and lower motor neuron loss Initial presentation... Difficulty Diffi lt chewing h i and d swallowing ll i Decrease in gag reflex Pooling of saliva in pharynx and weak cough Frequent episodes of aspiration Death in patients within 3 to 5 years after diagnosis Most commonly due to respiratory failure
MTBS2CK p.293

PresentationofAmyotrophicLateralSclerosis UpperMotorNeurons LowerMotorNeurons

Weakness Weakness Spasticity Wasting Hyperreflexia Fasciculations Extensorplantarresponses

MTBS2CK p.293

KineticTremor

Fasciculations

22

AmyotrophicLateralSclerosis/DiagnosticTests

AmyotrophicLateralSclerosis/Treatment
Riluzole Reduces glutamate buildup in neurons Delays progression Baclofen Treats spasticity CPAP and d BiPAP Help respiratory difficulties secondary to muscle weakness Tracheostomy Maintenance on ventilator necessary when disease advances
MTBS2CK p.293

Electromyography

Loss of neural innervation in multiple muscle groups CPK levels

MTBS2CK p.293

CharcotMarieToothDisease
Genetic disorder Lose both motor and sensory innervation
Distal weakness & sensory loss Wasting in legs Decreased DTRs Tremor

PeripheralNeuropathy
MCC: Diabetes mellitus Other causes: uremia, alcoholism, and paraproteinemias (e.g., MGUS) Best initial therapy
Pregabalin, gabapentin

Foot deformity with high arch common (pes cavus) Legs look like inverted champagne bottles Most accurate test: Electromyography No treatment
MTBS2CK p.294

Tricyclic antidepressants Most seizure medications effective in some people


Phenytoin Carbamazepine Lamotrigine

MTBS2CK p.294

SpecificPeripheralNerveNeuropathies
Nerve Ulnarnerve Precipitatingevent Biker,pressureonpalmsof hands,medialelbowtrauma Presentation Wastingof hypothenar eminence, 4th/5th digitpain p Wristdrop

SpecificPeripheralNerveNeuropathies
Nerve Tibial nerve Peroneal nerve Precipitatingevent Worsenswithwalking Presentation Pain/numbnessin ankle&soleoffoot Weakfootwith decreased dorsiflexionand eversion Thenar eminince, pain/numbness in1st 3fingers

Radialnerve

pp arm, Pressureoninner/upper useofcrutches,Saturdaynight palsy (fallingasleeponarm) Obesity,pregnancy,sittingwith crossedlegs

Highboots,pressureonthe backofknee

Lateral cutaneousn. ofthethigh


MTBS2CK p.294

Pain/numbness ofouteraspect ofthigh

Median nerve

Typists,carpenters, workingwithhands

MTBS2CK p.294

23

Facial(SeventhCranial)NervePalsy

Facial(SeventhCranial)NervePalsy
Paralysis of entire side of face Stroke paralyzes only lower half of face Upper half of face innervate from both cerebral hemispheres Difficulty closing eye If they can wrinkle forehead on affected side worry about stroke

Also known as Bells palsy Mostly idiopathic Some identified causes


Lyme, sarcoidosis, herpes zoster, and tumors

Look for statement the face feels stiff or pulled to one side
MTBS2CK p.294

Cannot wrinkle forehead = Bell's palsy!

MTBS2CK p.295

Facial(SeventhCranial)NervePalsy Hyperacusis Sounds are extra loud 7th CN supplies stapedius muscle Acts as a shock absorber on ossicles of middle ear Taste disturbance 7th CN supplies sensation of taste to anterior two-thirds of tongue
MTBS2CK p.295

Facial(SeventhCranial)NervePalsy

The most accurate test is... Electromyography and nerve conduction studies But...no test is usually done

MTBS2CK p.295

Facial(SeventhCranial)NervePalsy

The best initial therapy is... Prednisone But...


60% of patients have full recovery even without treatment

38-year-old with pain near his ear followed by weakness of one side of his face. Both upper and lower parts of face are weak. Sensation is intact. What is the most common complication?

Acyclovir does not help

a. b. c. d. e. f.

Corneal ulceration Aspiration pneumonia Gag reflex and cough are normal Sinusitis Nasal discharge & face pain Otitis media Ear pain, decreased hearing Deafness Sounds are actually extra loud Dental caries Cavities do not paralyze your face!

MTBS2CK p.278

MTBS2CK p.295

24

GuillainBarrSyndrome
Acute inflammatory polyneuropathy Autoimmune damage of multiple peripheral nerves

GuillainBarrSyndrome
What is the most likely diagnosis? Look for: Weakness of legs that ascends from feet Moves toward chest Loss of DTRs Some S have h mild ild sensory di disturbance t b Respiratory muscle weakness

No CNS involvement
Circulating antibody attacks myelin sheaths of peripheral nerves Associated with Campylobacter jejuni

GBS hits diaphragm!


Autonomic dysfunction with hypotension, HTN, or tachycardia can occur
MTBS2CK p.296 MTBS2CK p.296

GuillainBarrSyndrome
CSF = increased protein + normal cell count Most specific diagnostic test Nerve conduction studies/electromyography
Decrease in propagation of electrical impulses along nerves

GuillainBarrSyndrome Tests of respiratory muscle involvement Inspiration is the active part of breathing and the patient loses the strength to inhale
Decrease in FVC Decrease in peak inspiratory pressure

PFTs tell who might die from GBS

MTBS2CK p.296

MTBS2CK p.296

GuillainBarrSyndrome

Treatment Intravenous immunoglobulin (IVIG) or plasmapheresis are equal in efficacy

Woman comes with bilateral leg weakness over last few days. No knee jerk or ankle jerk reflexes. Weakness started in feet and progressed up to calves and thighs. Which of the following is the most urgent step?

Prednisone is a wrong answer for GBS, doesnt help Combining IVIG and plasmapheresis is incorrect
MTBS2CK p.296

a. b. c. d. e.

Pulmonary function testing Arterial blood gas Dont wait for inc CO2 on ABG Nerve conduction study Most accurate, but not next Lumbar puncture For Dx, not severity assessment Peak flow meter Assesses expiratory flow

MTBS2CK p.296

25

MyastheniaGravis

MyastheniaGravis/Presentation
What is the most likely diagnosis? Look for...
Double vision and difficulty chewing Weakness of limb muscles worse at end of the day Ptosis Weakness with sustained activity Normal pupillary responses

Muscular weakness from antibodies against acetylcholine receptors at the NMJ

Extraocular muscles & mastication (masseter) are often the only 2 muscular activities universally done by people (e.g., watching TV and eating)

MTBS2CK p.297

MTBS2CK p.297

MyastheniaGravis/DiagnosticTests Best initial test is... Acetylcholine receptor antibodies (80% 90% sensitive) Better answer than edrophonium testing Edrophonium
Short-acting inhibitor of acetylcholinesterase Temporary bump up in acetylcholine levels Associated with a clear improvement in motor function that lasts for a few minutes

MyastheniaGravis/DiagnosticTests
Most accurate test is... Electromyography
Shows decreased strength with repetitive stimulation

Questions often ask What What imaging test should be done? Answer... Chest something!
Chest X-ray, CT, or MRI are done to look for thymoma or thymic hyperplasia
MTBS2CK p.297

MTBS2CK p.297

MyastheniaGravis/Treatment

MyastheniaGravis/Treatment If meds dont control disease, the most appropriate next step in management is... Thymectomy if < 60 If > 60 prednisone is used Azothioprine, p , cyclophosphamide, y p p , or mycophenolate are used in order to get patient off steroids

Best initial treatment is... Neostigmine or pyridostigmine Longer acting versions of edrophonium

MTBS2CK p.297

MTBS2CK p.298

26

MyastheniaGravis/Treatment

Acute myasthenic crisis Severe, overwhelming disease Profound weakness Respiratory involvement Treated with IVIG or plasmapheresis

MTBS2CK p.298

27

Obstetricsand Gynecology
JasonM.Franasiak,MD ChiefResidentPhysician Obstetrics&Gynecology UniversityofNorthCarolina

Pregnancy
Definitions SignsandDiagnosisofPregnancy Ph i l i Ch Physiologic Changesi inP Pregnancy

Pregnancy Pregnancy Symptoms


Amenorrhea Breast tenderness Nausea and vomiting Fatigue
27-year-old woman with nausea and vomiting for 2 weeks. Symptoms worsen in the morning, but occur at any time during the day. She has a decrease in appetite. Her last menstrual period (LMP) was 6 weeks ago. Physical examination is unremarkable. Which is the best next step?

The surge in estrogen, progesterone, and betahuman chorionic gonadotropin (beta-HCG) leads to many symptoms of pregnancy

p for anemia, , but a. Complete p blood count Helpful not next step b. Beta-HCG c. HIDA scan Not initial work-up for nausea/vomitting d. Comprehensive metabolic panel Important, but must diagnose cause e. Urinalysis Important, but must diagnose cause

MTBS2CK p.441

MTBS2CK p.441

Definitions

DatingMethods Developmental age (DA): Days since fertilization Gestational age (GA): Days/weeks since the LMP
Fetus Infant

Embryo

On average, 2 weeks longer than DA

Ngele rule: Estimate day of delivery


Take the first day of LMP, subtract 3 months then add 7 days
1 year

Bi th Birth 8 weeks gestation Fertilization

Example: LMP = 7 / 1 / 2010 -3 +7 EDD = 4 / 8 / 2011

MTBS2CK p.441

MTBS2CK p.442

TrimesterBreakdown
1st Trimester Fertilization 12 weeks DA 14 weeks GA
FIRST screening Fetal heart tones with doppler

TermLengths
2nd Trimester 24 weeks DA 26 weeks GA 3rd Trimester Delivery
Frequent visits Monitoring for labor

Previable Fertilization

Preterm

Term

Postterm 42 weeks GA

Genetic triple or quad screen Fetal movement at 16-20 weeks GA Anatomic ultrasound at 18-20 weeks GA

24 weeks GA

37 weeks GA

MTBS2CK p.442

MTBS2CK p.442

Gravidity/Parity

G6P2124
Gravity Number of Pregnancies Parity Full-term birth Living Children Abortions

20-year-old woman presents to the office because she believes shes pregnant. Her sexual partner usually wears a condom, but didnt 2 weeks ago. She is now 4 weeks late for her menstruation. Which of the following is a first sign of pregnancy?

Preterm birth

F-PAL = Full-term (F); Preterm (P); Abortions (A); Living Children (L)
MTBS2CK p.442443

a. Quickening b. Goodell sign c. Ladin sign d. Linea nigra e. Chloasma

Not felt until second trimester Seen at 6 weeks gestation Seen in second trimester Seen at 16 weeks gestation

MTBS2CK p.443

SignsofPregnancy
Sign Goodellsign Ladinsign Chadwicksign Telangiectasias/ palmarerythema Chloasma Lineanigra PhysicalFinding Softeningofcervix Softeningofuterine midline Bluediscolorationof vaginaandcervix Smallbloodvessels/ reddeningofpalms Hyperpigmentationof theface,worsewithsun Hyperpigmentation downabdomenmidline TimefromConception 46weeks 6weeks 68weeks 1st trimester 16weeks 2nd trimester

DiagnosticEvaluation Beta-hCG Urine and serum testing all highly sensitive Produced by cytotrophoblast or syncytiotrophoblast in placenta First trimester Doubling every 48 hours for first 4 weeks Urine pregnancy tests are positive 4 weeks following the first day of LMP Peak levels at 10 weeks gestation Levels drop in 2nd trimester

MTBS2CK p.444

MTBS2CK p.444

DiagnosticEvaluation

PhysiologicChangesinPregnancy Cardiovascular changes Cardiac output increases 30-50% Lower blood pressure
Decreased afterload Increased blood volume

Step2KeyPoints
Ultrasound confirms intrauterine pregnancy At 5 weeks, or a beta-HCG of 1500 IU/L, a gestational sac should be seen on ultrasound

Gestational Sac
Yolk Sac Image: X.Compagnion , commons.wikimedia.org

MTBS2CK p.444

MTBS2CK p.444

PhysiologicChangesinPregnancy

PhysiologicChangesinPregnancy

Gastrointestinal changes Morning sickness


Caused by increase in estrogen, progesterone, and HCG made by placenta

GenitourinaryandRenalChanges
Increase in the size of kidney and ureters
Increased risk of pyelonephritis Increased incidence of stress urinary incontinence

Physiologic hydronephrosis in pregnancy

Gastroesophageal reflux
Lower L esophageal h l sphincter hi t h has d decreased dt tone Displacement of stomach by uterus

Constipation
Motility in large intestine decreased

Increase in GFR
Decrease in BUN/creatinine
Image: Nevit Dilmen, WikiCommons

MTBS2CK p.445

MTBS2CK p.445

PhysiologicChangesinPregnancy Hematology Changes Increased...


RBCs Plasma Coagulation factors

PhysiologicChangesinPregnancy Hematology Changes Hypercoagulable state


Fibrinogen (Factor I) increases 50% along with fibrin split products (Factors VII, VIII, IX, X) Protein C and S (inhibitors of coagulation) decrease No increase in PT, , PTT, , or INR Recall: Virchow triad
Hypercoagulation

Anemia
Plasma volume > RBCs

Endothelial Damage
MTBS2CK p.445 MTBS2CK p.445

Stasis

PrenatalCare/FirstTrimester Visits every 4 to 6 weeks Between 11 and 14 weeks


Ultrasound confirms gestational age and check for nuchal translucency

PrenatalCare
FirstTrimester SecondTrimester ThirdTrimester OtherScreeningTests

A thickened or enlarged nuchal translucency is an indication of Down syndrome.

MTBS2CK p.445

Image: Dr. Wolfgang Moroder WikiCommons

PrenatalCare/SecondTrimester 17-year-old woman presents for routine prenatal checkup at 12 weeks. Which of the following is the most accurate method to establish gestational age? Genetic Screening: Risk of chromosomal anomalies 15-20 weeks gestation Triple screen
Maternal serum alpha-fetoprotein (MSAFP) beta-HCG Estriol

a. Ultrasound b. Beta-HCG c. Pelvic exam d. Fundal height e. LMP

Levels vary y widely y May change with multiple gestations May change with multiple gestations May be unreliable

Quad screen
Inhibin A added to triple screen
Banana Sign

MTBS2CK p.445446

MTBS2CK p.446

PrenatalCare/SecondTrimester Increased MSAFP may indicate dating error, neural tube defect, or abdominal wall defect
Banana Sign

PrenatalCare/SecondTrimester Landmarks in 2nd trimester: Auscultation of fetal heart rate 16 to 20 weeks: Quickening Fetal movement first detected by mother Multiparous M lti may experience i earlier li Anatomic ultrasound
18 to 20 weeks

Ultrasound at 16 weeks showing banana sign created by compression of cerebellum in posterior fossa due to neural tube defect.

Image: Dr. W. Moroder, WikiCommons

MTBS2CK p.446

MTBS2CK p.446

PrenatalCare/ThirdTrimester Visits are every 2 to 3 weeks until 36 weeks Weekly visits after 36 weeks Braxton-Hicks Contractions
Oft Often occur during d i 3rd trimester ti t Sporadic and dont cause cervical dilation If they become regular, the cervix should be checked to rule out preterm labor before 37 weeks

ThirdTrimesterTesting Week
27

Test
Completeblood count

Action
Ifhemoglobin<11, replaceironorally *withstoolsoftener Ifglucose>140atone hour performoral hour, glucosechallengetest Treatifpositive

2428 Glucosechallenge

36

Cervicalculturesfor Chlamydia andgonorrhea Prophylacticantibiotics Rectovaginal culturefor duringlabor groupBStreptococcus

MTBS2CK p.446

MTBS2CK p.446

PrenatalCare/ThirdTrimester Glucose challenge test: Fasting or nonfasting ingestion of 50 g of glucose and serum glucose check 1 hour later

PrenatalCare/OtherScreeningTests Chorionic Villus Sampling Done at 10 to 13 weeks Obtains fetal karyotype Catheter into intrauterine cavity to aspirate chorionic villi from placenta

Glucose challenge test: Fasting serum glucose, ingestion of 100 g of glucose Serum glucose checks at 1, 2, and 3 hours Elevated glucose during any two of these tests is abnormal
MTBS2CK p.447 MTBS2CK p.447
Image: National Human Genome Research Institute, WikiCommons

PrenatalCare/OtherScreeningTests

PrenatalCare/OtherScreeningTests

Amniocentesis Done after 15-20 weeks Obtains fetal karyotype Needle placed transabdominally into amniotic i ti sac and d withdraw ithd amniotic i ti fluid

Percutaneous umbilical blood sample In cases of Rh isoimmunization and when a fetal CBC is needed Needle placed transabdominally into uterus to get blood from umbilical cord

MTBS2CK p.447

MTBS2CK p.447

EctopicPregnancy
RiskFactors Presentation Diagnosis Management

29-year-old woman with history of chlamydia and abnormal vaginal bleeding presents with left lower quadrant abdominal pain for the past eight hours. Her LMP was 6 weeks ago. Her temperature is 99F, heart rate is 100 bpm, blood pressure is 130/80 mmHg, and respiratory rate is 13 breaths/minute. Which of the following is the most likely diagnosis? a. Ectopic pregnancy b. Menstrual cramps Altered menstrual pattern present c. Diverticulitis Does not cause vaginal bleeding d. Ovarian torsion e. Ovarian cyst

MTBS2CK p.447448

EctopicPregnancy Pregnancy implants outside the uterus Most commonly occurs in ampulla of fallopian tube

EctopicPregnancy/RiskFactors Risk Factors Previous ectopic pregnancies (strongest risk factor) Pelvic inflammatory disease (PID) Intrauterine devices (IUD)

70-80%

Source: Kaplan Lecture Notes

Source: Nevit Dilmen, wikimedia commons

MTBS2CK p.448

MTBS2CK p.448

Source: Gloecknerd, wikimedia commons

EctopicPregnancy

EctopicPregnancy

Presentation

Period of amenorrhea Unilateral lower abdominal or pelvic pain Vaginal bleeding If ruptured ruptured, can be hypotensive with peritoneal irritation

Differential diagnosis Abortion Acute appendicitis Adnexal torsion Corpus luteum cyst rupture

Diverticulitis Endometriosis Gastroenteritis PID UTI

MTBS2CK p.448

EctopicPregnancy

EctopicPregnancy/TransvaginalUltrasound

Diagnostic Tests Beta-hCG


Confirms pregnancy
Uterus Bladder

Ultrasound
Locates implantation p site

Laparoscopy:
Invasive test and treatment to visualize and remove the ectopic pregnancy
Ectopic pregnancy

MTBS2CK p.448

Image: Courtesy of Jason Franasiak, MD

EctopicPregnancy/Treatment
Ectopic Confirmed Ruptured Medical Treatment Stable Unstable IV fluids, blood products, dopamine Surgical Treatment Not Ruptured

EctopicPregnancy/MedicalTreatment
Baseline labs: CBC LFTs Kidney function -hCG Methotrexate given, -hCG checked 4 & 7 days later

< 15% drop in -hCG

> 15% drop in -hCG

2nd dose of methotrexate, -hCG checked day 4 & 7 > 15% drop in -hCG Surgical treatment

Surgery (Laparoscopy)

Persistently high -hCG

Observe for side effects, no other Rx necessary

MTBS2CK p.448

MTBS2CK p.449

EctopicPregnancy Exclusion Criteria for Methotrexate


Hemodynamically unstable patients Signs of impending or ongoing ectopic mass rupture Clinically important abnormalities in baseline hematologic, renal, or hepatic laboratory values Immunodeficiency, active pulmonary disease, or peptic ulcer disease Hypersensitivity to Methotrexate Coexistent viable intrauterine pregnancy Breastfeeding Unwilling or unable to be compliant with post-therapeutic monitoring Do not have timely access to a medical institution
MTBS2CK p.449

EctopicPregnancy/SurgicalTreatment
Surgical Treatment Salpingostomy Salpingectomy ostomy = cut ectomy = remove

Images: Courtesy of Jason Franasiak, MD

MTBS2CK p.449450

EctopicPregnancy Mothers who are Rh negative should receive anti-D Rh immunoglobulin (RhoGAM) to prevent hemolytic disease

Abortion

MTBS2CK p.450

Abortion
20-year-old woman presents to ED for vaginal bleeding and lower abdominal pain for one day. She states that shes 15 weeks pregnant. T 99.0F, HR 100 bpm, BP 110/75 mmHg, and RR 12/min. Pelvic exam, blood present in vault. Ultrasound shows intrauterine bleeding, products of conception, and dilated cervix. Which is the most likely diagnosis?

A pregnancy that ends < 20 weeks gestation or fetus weighs 500g 80% of spontaneous abortions occur prior to 12 weeks gestation

a. Complete abortion b. Incomplete abortion c. Inevitable abortion d. Threatened abortion e. Septic abortion

We are going to review the definitions in this section

MTBS2CK p.450

MTBS2CK p.450

Abortion Maternal factors that increase risk of abortion Maternal age Anatomic abnormalities

Abortion

MTBS2CK p.450

Source: Ed Uthman, MD., commons.wikimedia.org

Source: RadsWiki, commons.wikimedia.org

Abortion

Abortion

Maternal factors that increase risk of abortion


5 6 3 2


1. 2. 3. 4. 5. 6. Dye injector Cervix Uterus Adhesions Right Tube Left Tube

Maternal Age Anatomic abnormalities Infections Immunological factors (e.g. Anti-phospholipid syndrome or SLE) Endocrinological factors Malnutrition Trauma
MTBS2CK p.450

Source: Floranerolia, commons.wikimedia.org

Abortion/Presentation

Abortion/DiagnosticTests

Signs/Symptoms Cramping abdominal pain Vaginal bleeding Hypotension Tachycardia

CBC Blood type and Rh screen Pelvic ultrasound

You cannot answer the most likely diagnosis question about abortion without an ultrasound
MTBS2CK p.451

MTBS2CK p.450

Abortion/Types
Complete Incomplete Inevitable Threatened Missed Septic

Abortion

No products of conception Follow up in office

Products of conception intact, but intrauterine bleeding present and dilation of cervix D&C/Medical

Death of fetus, but all products of conception present in the uterus D&C/M di l D&C/Medical Infection of uterus and surrounding areas D&C and IV Antibiotics (levofloxacin & metronidazole)

Mothers whore Rh negative should also receive anti-D Rh immunoglobulin at this time

Some products of conception Dilation & Curettage (D&C) / Medical


MTBS2CK p.451

Products of conception intact, intrauterine bleeding, No dilation of cervix Bed rest, pelvic rest

MTBS2CK p.451

MultipleGestations/Presentation

MultipleGestations

Signs/Symptoms Exponential growth of uterus Rapid weight gain by mother Elevated beta-HCG and Maternal S Serum Al Alpha-Fetoprotein h F t t i (MSAFP) Fertility drugs increase multiple gestations
MTBS2CK p.451

MultipleGestations/DiagnosticTests Ultrasound visualizes fetus Monozygotic 1eggand Identicaltwins 1sperm Samegender thatsplits Samephysicalcharacteristics Samebloodtype Fingerprintsdiffer Dizygotic 2eggsand Fraternaltwins 2sperm Differentorsamesex Theyresembleeachother Asanysiblingswould

MultipleGestations

MTBS2CK p.452

Source:Trlkly , commons.wikimedia.org

MultipleGestations/DiagnosticTests

MultipleGestations

For monozygotic twins, amnionicity and chorionicity depend on timing of cleavage

10

MultipleGestations

MultipleGestations

Complications

Spontaneous abortion of one fetus Premature labor and delivery Placenta previa Anemia

Source: Kevin Dufendach, commons.wikimedia.org

MTBS2CK p.452

PretermLabor

28-year-old woman 28th week of pregnancy with severe lower back pain. The pain is cyclical and is increasing in intensity. On physical examination she seems to be in pain. T 98.9F, HR 104 bpm, BP 135/80 mmHg, RR 15/min. Cervix is 3 cm dilated. Which of the following is the most likely diagnosis?

a. Premature rupture of membranes Membranes intact b. Preterm labor c. Cervical incompetence Presents with contractions d. Preterm contractions Cervical dilation present

MTBS2CK p.452

PretermLabor

PretermLabor/RiskFactors

Preterm births occur < 37 weeks gestation


Spontaneous: preterm labor Indicated: maternal or fetal

Preterm labor
Activation of hypothalamic-pituitary-adrenal yp p y (HPA) axis Decidual hemorrhage Inflammation Uterine distension

Premature rupture of membranes Multiple gestation Previous history of preterm labor Placental abruption Maternal factors
Uterine anatomical abnormalities Infections

MTBS2CK p.452453

11

PretermLabor/RiskFactors

PretermLabor/RiskFactors

MTBS2CK p.453

Source: RadsWiki, commons.wikimedia.org

MTBS2CK p.453

Source: Ed Uthman, , commons.wikimedia.org

PretermLabor/Presentation

PretermLabor/Evaluation

Contractions

Cervical change

Initial evaluation Gestational age Fetal weight Presenting fetal part

Occurring between 20 and 37 weeks gestation

MTBS2CK p.453

MTS2CK p.453

PretermLabor/Evaluation Indications for delivery


Maternal severe HTN Maternal cardiac disease Maternal cervical dilation > 4 cm Maternal hemorrhage (abruptio placenta, DIC) Fetal death Chorioamnionitis Preterm rupture of membranes (34 weeks) Fetal distress Intrauterine growth restriction with reverse diastolic flow

PretermLabor

Preterm labor Stop delivery if Delivery if EGA 24-33 wks EGA 34-37 wks EFW 600-2,500 600-2 500 g EFW > 2,500 2 500 g Betamethasone & Tocolytics
MTS2CK p.453

When any of these is present, answer delivery


MTS2CK p.453

12

PretermLabor/Corticosteroids Corticosteroids Betamethasone or dexamethasone Effects of betamethasone begin within 24 hours and peak at 48 hours Goal Decrease risk of RDS and neonatal mortality

PretermLabor/Tocolytics

Goal Delay preterm labor Allows time for steroids to work Transport to specialist unit Agents Magnesium sulfate

Corticosteriods increase surfactant and thus mature fetal lungs


MTS2CK p.453 MTS2CK p.454

PretermLabor

PretermLabor/Tocolytics

Magnesium toxicity leads to respiratory depression & cardiac arrest, check deep tendon reflexes often!

Goal Decrease contractions Given to allow time for steroids to work Agents Magnesium sulfate Calcium-channel blockers -adrenergic agents Prostaglandin synthetase inhibitors
MTS2CK p.454

Toxicity: Give calcium gluconate

MTS2CK p.454

PretermLabor/Tocolytics

PrematureRuptureof Membranes
Ductus Arteriosus

Source: commons.wikimedia.org

13

PrematureRuptureofMembranes(PROM)

PrematureRuptureofMembranes

The rupture of chorioamniotic membrane before onset of labor

Diagnosis Sterile speculum examination


Fluid pools in posterior fornix Fluid turns nitrazine paper blue When dry, fluid has ferning pattern
Source: Elizabeth August, MD

MTS2CK p.454

MTS2CK p.454

PrematureRuptureofMembranes

PrematureRuptureofMembranes/Treatment
Before 32 weeks gestation
Corticosteroids Antibiotics

PROM can lead to Preterm labor Cord prolapse Placental ace a ab abruption up o Chorioamnionitis
PROM = Do fewer exams!
MTS2CK p.454455

>37 weeks gestation, unknown GBS, > 18 hours rupture


Penicillin administered for prophylaxis

34-37 34 37 weeks gestation, unknown GBS


Initiate Penicillin

Known GBS negative


NO Antibiotics

<34 weeks gestation, unknown GBS


Erythromycin and Penicillin initiated
MTS2CK p.455

ThirdTrimesterBleeding

DDx

ThirdTrimesterBleeding

Vulva (trauma, varicose veins) Vagina (lacerations) Cervix (polyp, cervicitis, carcinoma) Ut i Uterine Uterine rupture Placenta previa Vasa previa Placental abruption

14

ThirdTrimesterBleeding/Previa

Placenta previa Abnormal implantation of placenta over internal cevical os Risk Ri k F Factors t
Previous uterine scar Multiple gestations Previous placenta previa
MTS2CK p.456

24-year-old woman in her 32nd week of pregnancy presents to ED. She woke up in bed in a pool of blood. Denies contractions or pain. HR 105 bpm, BP 110/70 mmHg. Which is the next step?

a. Digital vaginal exam knowing placental location b. Transabdominal ultrasound c. Immediate vaginal delivery Gather information first d. Immediate cesarean delivery e. Transvaginal ultrasound Abdominal US is safer

Never do digital exam w/o

MTS2CK p.455

ThirdTrimesterBleeding/Previa

ThirdTrimesterBleeding/Previa

Digital vaginal exam contraindicated in third trimester vaginal bleeding Can cause Separation between placenta and uterus Severe hemorrhage

Source: Jason Franasiak, MD

MTS2CK p.455

ThirdTrimesterBleeding/Previa

ThirdTrimesterBleeding/PreviaTypes
Partial Complete Marginal Fetal vessel present over cervical os Vasa Previa Lowlying Placenta

Presentation PAINLESS vaginal bleeding Usually presents > 28 weeks Diagnosis Transabdominal ultrasound
Placenta location

Partial covering of the internal cervical os, but covers more than th the marginal Complete covering of the internal cervical os
MTS2CK p.456

Marginal covering of the internal cervical os

Placenta thats implanted in lower segments of uterus but not covering internal cervical os

MTS2CK p.456

15

ThirdTrimesterBleeding/PreviaTypes Full moon Half moon Crescent moon

ThirdTrimesterBleeding/PreviaTypes

Umbilical cord Placenta Fetal vessels Internal os


Source: Sigrid de Rooij, comons.wikimedia.org

MTS2CK p.456

MTS2CK p.457

ThirdTrimesterBleeding/Treatment

ThirdTrimesterBleeding/Accreta

Treatment of placenta previa Strict pelvic rest Type and screen, CBC Fetal considerations Delivery by C-section
Labor Severe hemorrhage Fetal distress
MTS2CK p.457

Placental invasion (based on depth) Accreta Increta Percreta

MTS2CK p.457458

ThirdTrimesterBleeding

ThirdTrimesterBleeding/Accreta

Placental invasion (based on depth) Accreta Increta Percreta Risk Factors Placenta previa Prior uterine scars
MTS2CK p.458 MTS2CK p.457

16

ThirdTrimesterBleeding/Abruption Placental abruption Abnormal, premature separation of placenta from uterus Effects Complete Partial
Life-threatening g bleeding Premature delivery Uterine tetany DIC And Hypovolemic shock
MTS2CK p.458

PlacentalAbruption

Minor bleed No clinical signs/symptoms

PlacentalAbruption/Etiology

PlacentalAbruption/Presentation

Risk factors Maternal HTN (chronic, preeclampsia, eclampsia) Prior placental abruption Tobacco T b and/or d/ cocaine i use Trauma

Clinical presentation Vaginal bleeding Severe abdominal PAIN (uterine tenderness) Contractions C t ti Possible fetal distress

MTS2CK p.458

MTS2CK p.458

PlacentalAbruption/Presentation
Late Decelerations

PlacentalAbruption/DiagnosticTests

Diagnosis Transabdominal ultrasound Clinical scenario

Placenta previa = painLESS vaginal bleeding


Source: Jason Franasiak, MD

Placental abruption = painFUL vaginal bleeding


MTS2CK p.459

17

PlacentalAbruption/DiagnosticTests

PlacentalAbruption
Type Concealed Description Bloodwithinuterine cavity Placentamorelikely tobecompletely detached Complications Seriouscomplications: DIC Uterinetetany Fetalhypoxia Fetaldeath Sheehansyndrome (postpartum hypopituitarism)

External

Source: Nevit Dilman commons.wikimedia.org

Blooddrainsthrough Usuallysmallerwithminimal complications cervix Placentamorelikely tobepartially detached

MTS2CK p.459

PlacentalAbruption/Treatment

PlacentalAbruption/Treatment

Delivery plan Cesarean delivery


Uncontrollable hemorrhage Fetal distress

Mothers who are Rh negative should also receive anti-D Rh immunoglobulin at this time

Vaginal delivery
Placental separation is limited Fetal heart tracing is assuring Fetal death prior to presentation

MTS2CK p.459

MTBS2CK p.451

UterineRupture Spontaneous complete transection of uterus from endometrium to serosa Usually occurs during labor

UterineRupture

Life threatening to mother and baby Life-threatening

Immediate delivery!

MTS2CK p.459

18

UterineRupture/RiskFactors Risk factors Previous C-section


Classical: higher risk of uterine rupture Low transverse

UterineRupture

Trauma Uterine myomectomy Uterine overdistention


Polyhydramnios Multiple gestations
Types of cesarean scars. Source: Elizabeth August, MD.

Placenta percreta
MTS2CK p.460 MTS2CK p.460

UterineRupture/Presentation

UterineRupture/Treatment Treatment Emergent laparotomy and delivery Repair of uterus or hysterectomy Future management Early delivery via C C-section section Uterine rupture requires immediate laparotomy and delivery of the fetus

Clinical presentation: Extreme abdominal pain Abnormal bump in abdomen Lack of uterine contractions Regression of fetus

MTS2CK p.460

MTS2CK p.461

RhIncompatibility

Rh incompatibility

RhIncompatability

Mother RhD negative Baby RhD positive Leads to isoimmunization Rh isoimmunization Fetal RBCs cross placenta Maternal antibodies to RhD antigen are made
MTS2CK p.461

19

RhIncompatibility Clinical significance 1st pregnancy: mild anemia/hyperbilirubinemia 2nd pregnancy: maternal antibodies attack the second Rh positive baby

RhIncompatibility Hemolytic Disease of Newborn Fetal anemia Extramedullary production of fetal RBCs

Hemolysis heme and bilirubin levels


Neurotoxicity from hyperbilirubinemia High output cardiac failure Hydrops fetalis

Leads to hemolysis of fetal RBCs Hemolytic disease of newborn

MTS2CK p.461

MTS2CK p.461

RhIncompatibility Rh Antibody Screening Rh Negative Antibody titer Sensitized Further Monitoring


MTS2CK p.461

RhIncompatibility

Rh Positive No further screening

Antibody screen: if mother is Rh or Rh+ Antibody titer: how many maternal antibodies to Rh+ blood

Unsensitized
Repeat at 28 weeks and give Rhogam as indicated

MTS2CK p.461

RhIncompatibility

RhIncompatibility

Rhogam indications for Rh unsensitized patients 28 weeks gestation Delivery Procedures (amniocentesis) ( ) Bleeding (abortion, abruption) Unsensitized = no anti-Rh antibodies present

Antibody Screen Positive Antibody titer Titer 1:4 Sensitized Titer 1:16 consider treatment algorithm l ith

MTS2CK p.461462

MTS2CK p.462

20

RhIncompatibility
Antibody titer 1:16

RhIncompatibility
Antibody titer 1:16 Fetus is Rh Antigen Positive Middle Cerebral Artery Dopplers

Determine paternal Rhesus type

Homozygote Positive Treatment algorithm

Heterozygote or unsure paternity

Homozygote g Negative

Peak MCA Velocity is >1 5 MOM >1.5 (Multiples of median)

No Treatment Fetal genotyping performed on samples of chorionic villi, amniocytes or fetal blood Cordocentesis with transfusion if fetal Hct is <30%
MTS2CK p.462.3

MTS2CK p.462

RhIncompatibility

RhIncompatibility

Source: Jason Franasiak, M.D. Source: Jason Franasiak, M.D.

RhIncompatibility
Antibody titer 1:16 No MCA Doppler Capability

Amniocentesis for fetal cells to be evaluated under spectrophotometer (evaluated bilirubin) Indeterminate Affected Transfusion zone Fetus probably is anemic Do percutaneous umbilical blood sample (fetal hematocrit) Fetal Hct is <30% Perform an intrauterine transfusion
MTS2CK p.462.3

Hypertension

Repeat amniocentesis 2-3 weeks

Repeat amniocentesis 1-2 weeks

21

Hypertension/Chronic
29-year-old woman G2P1 in her 30th week of pregnancy presents for routine prenatal visit. Her wedding ring is getting too tight. BP 150/100 mmHg, HR 92 bpm, RR 12, T 99F. Urine 1+ protein. LFTs: normal. Which is the most likely diagnosis? a. Chronic hypertension b. Gestational hypertension No proteinuria c. HELLP syndrome No laboratory abnormalities d. Preeclampsia e. Eclampsia Pre-eclampsia + seizures

Chronic HTN BP 140/90 mmHg before 20 weeks gestation Treatment


Labetalol, nifedipine, or methyldopa ACE inhibitors and ARBs cause fetal malformations Dont use during pregnancy

MTS2CK p.463

MTS2CK p.463

Hypertension/Gestational

Gestational HTN BP 140/90 mmHg after 20 weeks gestation No proteinuria Treatment T t t Labetalol, nifedipine, or methyldopa
Only during pregnancy

Hypertension/Preeclampsia Mildpreeclampsia Severepreeclampsia Hypertension Proteinuria >140/90 Dipstick1+to 2+;24hour urine>300mg Hands,feet,face No No No >160/110 Dipstick3+;24hour urine>5g Generalized Yes Yes Yes

Edema Mental lstatus changes Visionchanges Impairedliver function

MTS2CK p.463

MTS2CK p.463

Hypertension/Preeclampsia

Hypertension/Preeclampsia
Mild BP > 140/90 mmHg Proteinuria 1+ to 2 + At term Induce I d delivery Preterm 1. Betamethasone a. Mature fetus lungs 2. Magnesium sulfate a. Seizure prophylaxis 1. Prevent Eclampsia p a. Magnesium sulfate 2. Control BP a. Hydralazine 3. Delivery a. Preterm b. Term Preeclampsia Severe BP > 160/110 Proteinuria 3+ to 4+

Preeclampsia Risk Factors Nulliparity Multiple gestation Advanced maternal age Chronic HTN Renal disease History of preeclampsia

MTS2CK p.463

MTS2CK p.464

22

Hypertension/Preeclampsia

Hypertension/Eclampsia

The only definitive treatment in preeclampsia is delivery

Eclampsia Tonic-clonic seizures occurring in patient with preeclampsia Treatment Stabilize St bili mother th
Seizure control: magnesium BP control: hydralazine and labetalol

Deliver baby
MTS2CK p.464 MTS2CK p.464

Hypertension HELLP

HELLP Syndrome Hemolysis, Elevated Liver enzymes, Low Platelets Treatment Stabilize mother
BP control: hydralazine and labetalol

GestationalDiabetes

Deliver baby
MTS2CK p.464

PregestationalDiabetes
28-year-old woman in her 27th week of gestation presents for routine prenatal visit. No complaints. T 99F, BP 120/80 mmHg, HR 87 bpm. The patient is given 50 mg of glucose. An hour later, blood glucose 145 mg/dL. Which is the best next step? a. Treat with insulin The 1h GTT is a screening test b. Treat with sulfonylurea c. Do a fasting blood glucose level A 3h GTT is needed d. Perform oral glucose tolerance test

Pregestational diabetes Diabetes before pregnancy Either Type 1 or Type 2 DM

MTS2CK p.464465

MTS2CK p.465

23

PregestationalDiabetes

PregestationalDiabetes

Maternal Complications Preeclampsia Spontaneous abortion Increased rate of infection Increased postpartum hemorrhage

Fetal Complications Congenital anomalies Macrosomia


Shoulder dystocia

Preterm P t labor l b

MTS2CK p.465

MTS2CK p.465

PregestationalDiabetes/Evaluation

PregestationalDiabetes DMtype Type1 Type2 Routeofadministration Insulinpump Subcutaneousinsulin Insulintype NPH NPH,lispro

Additional prenatal testing EKG 24-hour urine for baseline renal function HbA1C Ophthalmological exam

Oral Medications Metformin and glyburide

MTS2CK p.465

MTS2CK p.465

PregestationalDiabetes/FetalTesting Age (weeks) 32-36 >36 37 38-39


Weekly nonstress test (NST) and ultrasound NST: fetal well-being Ultrasound: fetal size Lecithin/sphingomyelin ratio (L/S ratio) L/S ratio: assess fetal lung maturity test (if mat mature re delivery) deli er )

GestationalDiabetes/Complications Gestational diabetes (GDM) Glucose intolerance identified during pregnancy Complications Preterm birth Fetal F t l macrosomia i Birth injuries from fetal macrosomia Neonatal hypoglycemia Development of overt Type 2 DM postpartum

Twice-weekly testing; one NST and one biophysical prole (BPP) NST: fetal well-being BPP: amount of amniotic uid and fetal well- being
MTS2CK p.466

Continue testing, IOL at 39 weeks

MTS2CK p.466

24

GestationalDiabetes/Complications

GestationalDiabetes/Evaluation

Erbs Palsy

Diagnosis Routine screening between 24 and 28 weeks GA 1-hour glucose tolerance test
Positive Positi e if > 140 mg/dL

3-hour glucose tolerance test


Klumpkes Palsy

MTS2CK p.466

GestationalDiabetes
Glucose load test Non-fasting ingestion of 50g glucose followed by No serum measurement one hour later < 140 mg/dL NO gestational diabetes > 140 mg/dL Oral glucose tolerance test Fasting ingestion of 100 mg of glucose followed by serum glucose measurements at 1, 2, and 3 hours after ingestion Elevation of serum glucose at 1, 2, or 3 hours Gestational diabetes
MTS2CK p.467

Diabetes/Treatment Treatment Diabetic diet and exercise (walking) Medical management


Insulin Oral hypoglycemics

All glucose levels are normal

No further test

Dont tell pregnant patients to lose weight. Its the most common wrong answer. Once patients are put on insulin they should follow fetal testing schedule starting at 32 weeks.
MTS2CK p.467

Diabetes/Treatment

Gestational diabetes needs formal 2-hour GTT postpartum to screen Type 2 DM

FetalGrowthAbnormalities
IntrauterineGrowthRestriction Macrosomia

MTS2CK p.467

25

IntrauterineGrowthRestriction Fetuses with intrauterine growth restriction (IUGR) weigh in bottom 10% for gestational age TypesofIUGR Type Symmetric Characteristic Braininproportionwithrestofbody Occurs<20weeksgestation

IntrauterineGrowthRestriction/Etiology

Chromosomal abnormalities Neural tube defects Infections (viral, protozoans) Multiple gestations Maternal disease
HTN or renal disease Malnutrition Substance abuse Hemoglobinopathies

Asymmetric Brainweightisntdecreased Abdomenissmallerthanhead Occurs>20weeks


MTS2CK p.467468

MTS2CK p.468

IntrauterineGrowthRestriction Physical examination Fundal height = gestational age in weeks Example: a patients fundal height at 28 weeks should be 28 cm

IntrauterineGrowthRestriction

Diagnosis Ultrasound done to confirm gestational age and fetal weight


MTS2CK p.468
Source: Jason Franasiak, M.D.

IntrauterineGrowthRestriction

IntrauterineGrowthRestriction

Source: Jason Franasiak, M.D.

Source: Jason Franasiak, M.D.

26

IntrauterineGrowthRestriction

IntrauterineGrowthRestriction

Complications Premature labor Stillbirth Fetal hypoxia Lower IQ Seizures Mental retardation

Treatment/Prevention Quit smoking Prevent maternal infection with immunizations Determine D t i optimal ti l d delivery li ti time

MTS2CK p.468

MTS2CK p.468

Macrosomia/RiskFactors

Macrosomia/DiagnosticTests

Macrosomia Estimated birth weight > 4500 g Risk factors Maternal M t l diabetes di b t or obesity b it Advanced maternal age Post term pregnancy Fetal genetic syndromes
MTS2CK p.468

Physical examination
Fundal height = gestational age in weeks Example: a patients fundal height at 28 weeks should be 28 cm

Macrosomia
Fundal height 3 cm greater than GA
MTS2CK p.469

Macrosomia/DiagnosticTests

IntrauterineGrowthRestriction

Physical Examination Fundal height 3 cm than GA Ultrasound Ultrasound confirms the estimated weight by Femur length Abdominal circumference Head circumference and biparietal diameter

MTS2CK p.469

Source: Jason Franasiak, M.D.

27

IntrauterineGrowthRestriction

IntrauterineGrowthRestriction

Source: Jason Franasiak, M.D.

Source: Jason Franasiak, M.D.

Macrosomia

ShoulderDystocia/ClavicleFracture

Complications Shoulder dystocia Birth injuries Low Apgar scores Hypoglycemia

MTS2CK p.469

Source: Nevit Dilman, commons.wikimedia.org

GestationalDiabetes/Complications

Macrosomia

Treatment Induction of labor


Erbs Palsy

Lungs mature EFW < 4500 grams

Cesarean deli delivery er


EFW > 5000 grams

Klumpkes Palsy

MTS2CK p.469

28

NonstressTest Allows for evaluation of fetal well-being in utero

LaborandDelivery
FetalTesting ElectronicFetalMonitoring g PhysiologicChangesBeforeLabor InductionofLabor

Source: Jason Franasiak, MD

MTS2CK p.469

NonstressTest

BiophysicalProfile

Biophysical profile (BPP) Test NST g FetalBreathing FetalMovement Tone AmnioticFluid


Source: Jason Franasiak, MD

Score 0or2 0or2 0or2 0or2 0or2

MTS2CK p.469

MTS2CK p.470

ElectronicFetalMonitoring

ElectronicFetalMonitoring

Source: Jason Franasiak, MD

Source: Jason Franasiak, MD

MTS2CK p.470

MTS2CK p.470

29

ElectronicFetalMonitoring
Type Early decelerations Variable decelerations Description Decreaseinheartratethat occurswithcontractions Decreaseinheartrateand returntobaselinewith no relationshiptocontractions Decreaseinheartrateafter contractionstarted.No returntobaselineuntil contractionends Cause Head compression Umbilicalcord compression

ElectronicFetalMonitoring

Late decelerations (mostserious anddangerous)

Fetalhypoxia

MTS2CK p.470

MTS2CK p.471

ElectronicFetalMonitoring

ElectronicFetalMonitoring

PhysiologicalChangesBeforeLabor

Labor&Delivery Stages of Labor


Stage 1 Onset of labor full dilation of cervix Primipara: 618 h p 210 h Multipara: Latent phase Onset of labor 4 cm dilation Primipara: 67 h Multipara: 45 h Stage 2 Full dilation of cervix delivery of neonate Primipara: p 2h Multipara: 1 h Active phase 4 cm dilation full dilation Primipara: 1.2 cm/h (minimum) Multipara: 1.5 cm/h (minimum) Stage 3 Delivery of neonate delivery of placenta 30 min

Early Changes Lightening Braxton-Hicks contractions Bloody show

MTS2CK p.471

MTS2CK p.471

30

Labor&Delivery

Labor&Delivery

Monitoring (Stage 1) Maternal BP and pulse Electronic fetal monitor: fetal HR and uterine contractions Examine E i cervix i ( (every 2 h hours) )
Cervical dilation Cervical effacement Fetal station
MTS2CK p.471

NOT effaced NO dilation

Fully effaced 1 cm dilated

5 cm dilation

Source: Fred the Oyster, commons.sikimedia.org

Fully dilated at 10 cm

Labor&Delivery

Labor&Delivery

Stage 2 Cervix fully dilated to delivery


Internal vs. External Fetal Monitoring

MTS2CK p.472

Stages of fetal head descent. Source: Elizabeth August, MD.

Labor&Delivery Internal vs. External Fetal Monitoring

Labor&Delivery Internal vs. External Fetal Monitoring

Source: Jason Franasiak, MD Source: Jason Franasiak, MD Source: Jason Franasiak, MD

31

Labor&Delivery

Labor&Delivery

Stage 2 Cervix fully dilated to delivery


Internal vs. External Fetal Monitoring Rupture of amniotic membranes Spontaneous or artificial Meconium

Stage 2: Cervix is fully dilated to delivery Cardinal Movements of Labor


1. 2. 3 3. 4. 5. 6. 7. Engagement Descent Flexion Internal Rotation Extension External Rotation Expulsion

MTS2CK p.472473

Labor&Delivery Stage 3 From delivery of neonate to placenta Immediately after delivery Repair lacerations of vagina Signs of placental separation include: Fresh bleeding from vagina Umbilical cord lengthening Uterine fundus rising Uterus becoming firm
MTS2CK p.473

OperativeDelivery

Operative Vaginal Delivery Forceps Vacuum C-section

OperativeDelivery

InductionofLabor/Medications

Induction of labor Initiating labor via medical means Medications PGE2 used for cervical ripening Oxytocin
Types of cesarean scars. Source: Elizabeth August, MD.

MTS2CK p.460

MTS2CK p.473

32

InductionofLabor/Medications

Mechanical means Amniotomy Foley Balloon


Inspect for a prolapsed umbilical cord after puncturing amniotic sac

ComplicationsofLabor andDelivery
ProlongedLatentStage ProtractedCervicalDilation ArrestDisorders Malpresentation ShoulderDystocia PostpartumHemorrhage

MTS2CK p.473

ProlongedLatentStage
22-year-old primipara in 39th week of pregnancy, with intense abdominal pain thats intermittent. Gush of fluid felt 3 hours ago. Cervix 3 cm dilated, 50% effaced, and fetus head is felt at 2 station. The next 3 hours she progresses, her cervix is 5 cm dilated, 60% effaced, and fetal head at 1 station. Six hours after presentation, her cervix is 5 cm dilated and 60% effaced, fetal head at 0 station. Which is the most likely diagnosis? a. Prolonged latent stage Cervix has not dilated b. Protracted cervical dilation Cervix has not dilated c. Arrest of descent Fetal head has descended d. Arrest of cervical dilation

Prolonged latent stage Latent phase 20 hours for primipara Latent phase 14 hours for multipara Etiology Sedation Unfavorable cervix Uterine dysfunction with irregular/weak contractions
MTS2CK p.474

MTS2CK p.474

ProlongedLatentStage

ProtractedCervicalDilation Stage 1 of labor < 1.2 cm/hour in primipara < 1.5 cm/hour in multipara Etiology: 3 Ps Power: strength and frequency of contractions Passenger: size and position of fetus Passage: size of pelvis

Treatment of Prolonged Latent Phase Rest Hydration


Most will convert to spontaneous delivery in 6 to 12 hours

MTS2CK p.474

MTS2CK p.474475

33

ProtractedCervicalDilation

ArrestDisorders/Types

Treatment of protracted cervical dilation Power


Pitocin Amniotomy

Arrest of Cervical Dilation No cervical dilation for 2 hours Arrest of Fetal Descent No fetal descent for 1 hour

Passenger/Pelvis
C-section
MTS2CK p.475

MTS2CK p.475

ArrestDisorders/Etiology

Etiology Cephalopelvic disproportion


Cesarean delivery

Excessive sedation/anesthesia
Rest or reversal

Malposition
Time Operative delivery (forceps) Cesarean delivery

25-year-old woman in 35th week of gestation presents for routine prenatal checkup. T 98F, BP 130/90, HR 87, and RR 12. Her abdomen is gravid. A hard circular surface is felt in proximal part of the uterus. Which is the next step? a. External cephalic version Perform ultrasound first b. Ultrasound c. CT scan Ultrasound is used to determine position d. X-ray

MTS2CK p.475

MTS2CK p.475

Malpresentation/Presentation

Malpresentation/Presentation

Presenting Part Part of fetal body thats closest to vaginal canal and will be engaged when labor starts Cephalic
Head

Diagnosis Physical Examination


Leopold maneuvers Vaginal exam

Ultrasound Ultraso nd Should always confirm suspected diagnosis on physical examination with ultrasound
MTS2CK p.476.

Malpresentation
Foot or buttock
MTS2CK p.476

34

TypesofBreechPresentation

Malpresentation/Presentation

Type

Description

Frankbreech Fetushipsareflexedwithextended kneesbilaterally Complete b breech h Footling breech Fetushipsandkneesareflexed bil bilaterally ll Fetusfeetarefirst oneleg(singlefootling) or bothlegs(doublefootling)
MTS2CK p.476
Source: Elizabeth August, MD.

MTS2CK p.476

Malpresentation/Presentation

Malpresentation/Presentation

MTS2CK p.477

Source: Elizabeth August, MD.

MTS2CK p.477

Source: Elizabeth August, MD.

Malpresentation/Treatment

ShoulderDystocia

Treatment External cephalic version C-section

Shoulder dystocia Entrapment of anterior shoulder behind pubic symphisis after delivery of fetal head

MTS2CK p.477

MTS2CK p.477

35

ShoulderDystocia

ShoulderDystocia/RiskFactors

Risk Factors Maternal diabetes Maternal obesity Post-term pregnancy History of prior shoulder dystocia Any factor that indicates fetus is too big or the pelvis is too small is a risk factor for shoulder dystocia
Source: Elizabeth August, MD.

MTS2CK p.478

MTS2CK p.478

ShoulderDystocia/Treatment

ShoulderDystocia/Treatment

Treatment 1. McRoberts Maneuver

McRoberts maneuver. Source: Elizabeth August, MD.

MTS2CK p.478479

MTS2CK p.478

ShoulderDystocia/Treatment

PostpartumHemorrhage/Etiology

Treatment
1. McRoberts maneuver 2. Rubin maneuver 3. Woods maneuver 4. Delivery of posterior arm 5. Deliberate fracture of fetal clavicle 6. Zavanelli maneuver

Postpartum hemorrhage More than 500 mL after delivery Early vs. late postpartum bleeding Etiology gy a = without Uterine atony tony = contractions Laceration Retained products of conception Coagulopathy
MTS2CK p.479

MTS2CK p.478

36

PostpartumHemorrhage/RiskFactors

Risk factors for atony Anesthesia Uterine overdistention Prolonged/rapid labor Augmented labor Uterine leiomyoma Preeclampsia with magnesium therapy

PostpartumHemorrhage/Treatment Evaluation/Treatment
Examination of perineum, vagina, and cervix Bimanual examination of uterus (+/- compression/massage) Administer uterotonics (oxytocin, methylergonovine maleate, 15 methyl-PGF2alpha/Hemabate, misoprostol) Operative management
Uterine artery y embolization D&C Bakri balloon placement
Collapsed silicone ballon inserted in uterus, later filled with fluid

B-Lynch stitch Uterine artery ligation Hysterectomy


MTS2CK p.479

MTS2CK p.479

PostpartumHemorrhage/Treatment B-Lynch Suture Uterine Artery Ligation

PostpartumHemorrhage/Treatment

Evaluation/Treatment Examination of perineum, vagina, and cervix Bimanual examination of uterus Administer uterotonics Operative management Blood products
PRBCs, FFP, Cryoprecipitate

Source: Niels Olson, commons.wikimedia.org Modified: Jason Franasiak, MD

PostpartumHemorrhage/Treatment Product PackedRBCs Contents Effect

RBCs,WBCs,plasma Hct3% Hgb1g/dL Solubleplasma proteins p fibrinogen10 mg/dL g/ fibrinogen10 mg/dL 5000 10,000/mm3 per unit

TheUterus
PremenstrualSyndrome Menopause AUB/DUB Contraception

FFP

Cryoprecipitate FactorsVIIIandXIII, fibrinogen, fibronectin,vWF Platelets Platelets,RBCs, WBCs,plasma

37

PMS/PMDD

PMS/PMDD

Premenstrual syndrome (PMS) Physical, mood-related, and behavioral changes

Over 200 symptoms attributed to PMS Common symptoms


Headache Breast B t tenderness t d (mastodynia) ( t d i ) Pelvic pain, bloating Irritability, lack of energy

Premenstrual Dysphoric Disorder (PMDD) Severe form of PMS

MTBS2CK p.481

MTBS2CK p.481

PMS/PMDD

PMS/PMDD

Diagnostic criteria Present for 2 consecutive cycles Symptom-free in first part of cycle Symptoms present in second half of cycle Dysfunction in social or economic performance

Treatment of PMS/PMDD Lifestyle


Limit caffeine, alcohol, cigarettes, and chocolate Aerobic exercise Increase calcium and magnesium

Pharmacologic
NSAIDs Severe: SSRIs

MTBS2CK p.481

MTBS2CK p.481

Menopause

Menopause

Menopause Permanent cessation of menses Due to permanent cessation of estrogen production Median M di age of f onset: t 51

Physiology Early
The oocytes produce less estrogen and progesterone t LH and FSH start to rise Shortening of menstrual cycles

Late
Changes in sex hormones Testosterone Androstenedione Estrone Estradiol

MTBS2CK p.482

38

Menopause

Menopause

Symptoms Menstrual irregularity Sweats and hot flashes Mood changes Dyspareunia (pain during sexual intercourse) Women are symptomatic for an average of 12 months, but some experience symptoms for years
MTBS2CK p.482

Physical Examination Decrease in breast size Vaginal/cervical atrophy Uterovaginal prolapse Atrophic vaginitis: vaginal epithelium is estrogen dependent. The absence of estrogen causes thinning, presents as itching, burning, and/or dyspareunia.

MTBS2CK p.482

Menopause/Osteoporosis

Menopause Hormone replacement therapy (HRT) Estrogen +/- progesterone Contraindications


Estrogen-dependent carcinoma History of PE or DVT

HRT associated with endometrial hyperplasia, can lead to endometrial carcinoma

Vertebral body fractures. Source: Dr Robert CARLIER, CHU Raymond Poincar, Garches, France, commons.wikimedia.org

MTBS2CK p.482

AbnormalUterineBleeding
Menorrhagia
Heavy, prolonged menstrual bleeding Gushing of blood Clots may be seen Endometrial hyperplasia Uterine fibroids Dysfunctional uterine bleeding Intrauterine device Light menstrual flow May only have spotting Obstruction (hymen, cervical stenosis) OCPs

AbnormalUterineBleeding
Metrorrhagia
Intermenstrual bleeding Endometrial polyps Endometrial/cervical cancer Exogenous estrogen administration d i i t ti Menometrorrhagia Oligomenorrhea Menstrual cycles > 35 days long Pregnancy Menopause Significant weight loss (anorexia) Tumor T secreting ti estrogen t Postcoital bleeding

Hypomenorrhea

Irregular bleeding Time intervals Duration u at o Amount of bleeding

Endometrial polyps Endometrial/cervical cancer Exogenous estrogen administration Malignant tumors

Bleeding after intercourse Cervical cancer Cervical polyps Atrophic vaginitis

MTBS2CK p.482483

MTBS2CK p.483

39

AbnormalUterineBleeding Uterine fibroid

AbnormalUterineBleeding

Postcoital bleeding is cervical cancer until proven otherwise

Any patient > 35 with abnormal bleeding should undergo endometrial biopsy to rule out endometrial carcinoma
Source: James Heilman, MD, commons.wikimedia.org

MTBS2CK p.482483

AbnormalUterineBleeding Normal squamous cells HPV infects cells causing mild dysplasia

AbnormalUterineBleeding

Anovulatory Uterine Bleeding Estrogen produced No progesterone


No ovulation no corpus luteum no progesterone Prevents withdrawal bleeding

Source: Ed Uthman MD, commons.wikimedia.org

AbnormalUterineBleeding

DysfunctionalUterineBleeding

Diagnosis/Evaluation CBC Pregnancy test PT/PTT Pelvic ultrasound Endometrial biopsy Pap smear Thyroid studies Prolactin levels

Dysfunctional uterine bleeding (DUB)


Unexplained abnormal bleeding

Theres no specific test for DUB Diagnosis by exclusion


MTBS2CK p.483

40

DysfunctionalUterineBleeding

DysfunctionalUterineBleeding

Treatment of DUB Oral contraceptive pills (OCP) Cyclic progesterone Acute hemorrhage D&C IV estrogen

Long-term Endometrial ablation Hysterectomy

MTBS2CK p.484

Source: Hic et nunc, commons.wikimedia.org

DysfunctionalUterineBleeding

Contraception/FemaleCondoms

Advantages Offer some protection against HIV and STDs Under female control Disadvantages Not as effective as other methods They are larger and bulkier than male condoms

Source: Hic et nunc, commons.wikimedia.org

MTBS2CK p.484

Contraception/FemaleCondoms

Contraception/VaginalDiaphragm

Source: : ka Grzywacz, commos.wikimedia.org

Source: Axefan2, commons.wikimedia.org

MTBS2CK p.484

MTBS2CK p.484

41

Contraception/CervicalCap

Contraception/VaginalDiaphragm

Advantages Under female control Disadvantages Need to be fitted properly Requires advance preparation Improper placement or dislodging reduces efficacy
Source: Dake, commons.wikimedia.org

Source: Ceridwen, commons.wikimedia.org

MTBS2CK p.484

Contraception/OralContraceptivePills

Contraception/OralContraceptivePills

Hormones Combination of both estrogen and progestin Progestin only Use 21 days of active pill 7 days placebo Menses occurs during 7 days of placebo pills
Source; Matthew Bowden, commons.wikimedia.org

MTBS2CK p.484

MTBS2CK p.484

Contraception/OralContraceptivePills

Contraception/VaginalRing

Advantages Effective with perfect use Reduces rates of ovarian and endometrial cancer Easily y reversible Disadvantages User dependent Risk of thromboembolism
MTBS2CK p.484

A flexible vaginal ring inserted in vagina


Releases both estrogen and progesterone Remains in place for 3 weeks

The vaginal aginal ring has similar side effects and efficacy to OCPs

MTBS2CK p.485

42

Contraception/TransdermalPatch

Contraception/TransdermalPatch

Transdermal patch
Combination of estrogen and progesterone Placed on skin for 7 days

Patches shouldnt sho ldnt be placed on breast Side effects and efficacy = OCPs
Source : Keitei, commons.wikimedia.org

MTBS2CK p.485

MTBS2CK p.485

Contraception/Injectable

Contraception/Implantable

Depo-Medroxyprogesterone Acetate (DMPA)


IM injection Every 3 months

Implant contraceptive system Releases progestin daily Inhibits ovulation and thickens cervical mucus

Contraception/IntrauterineDevice

Contraception/IntrauterineDevice

Intrauterine device (IUD)


Copper device Levonorgestrel device

Most commonly used worldwide Most effective aside from sterilization

Source: Gloecknerd, commons.wikimedia.org

MTBS2CK p.485

MTBS2CK p.485

43

Contraception/IntrauterineDevice

Contraception/Sterilization

Tubal Ligation/Occlusion Vasectomy

Source: Nevit Dilmen ,commons.wikimedia.org

MTBS2CK p.485

VulvaandVagina/LabialFusion Fusion Occurs when excess androgens are present


Exogenous Endogenous

VulvaandVagina

MCC of labial fusion is 21-B hydroxylase deficiency Treatment Conservative Reconstructive surgery
MTBS2CK p.485

VulvaandVagina/EpithelialAbnormalities
Abnormality Lichen sclerosus Agegroup affected Anyage Ifpostmenopausal, increasedconcern forcancer Description White,thinskin fromlabiato perianalarea, parchment Treatment Topical steroids

VulvaandVagina/BartholinGlandCyst

Bartholin Glands Location: lateral sides of vulva Function: secrete mucus Bartholin Gland Cyst/Abscess y Presents with pain, tenderness, and dyspareunia Edema and inflammation with deep fluctuant mass
MTBS2CK p.486

q Any yage; g p patients Squamous Patientswith cell whovehadchronic chronicirritation hyperplasiavulvarpruritus develophyper keratosis(raised whitelesion) Lichen planus 30s60s

Sitzbathsor lubricants (relieve pruritus)

Pruritic,polygonal, Topical papularand steroids Purple

MTBS2CK p.486

44

VulvaandVagina/BartholinGlandCyst
Clitoris Labia Minora Urethra Skenes glands g Vagina Bartholins glands

VulvaandVagina/BartholinGlandCyst Bartholin Abscess Simple incision and drainage (I&D) Word catheter placement 4-6 weeks duration Small rubber catheter, inflatable balloon tip is inserted into cyst incision, incision after the contents of the cyst have been drained Recurrence: marsupialization or excision During I&D, fluid released should be cultured for STDs (e.g., Neisseria gonorrhea and Chlamydia trachomatis)
MTBS2CK p.486

Source: Nicholasolan, commons.wikimedia.org

Vaginitis/RiskFactors 19-year-old woman with vaginal pruritus and discharge for one week. Discharge is green and profuse. Shes had multiple sexual partners in past 2 months. LMP 2 weeks ago. Wet mount: motile flagellates. Which is the most likely diagnosis? a. Chlamydia Dx w/ culture or DNA probe b. Bacterial vaginosis Clue cells on wet mount c. Neisseria gonorrhoeae d. Candidiasis Hyphae on wet mount e. Trichomonas vaginalis

Vaginitis: spectrum of conditions Risk factors


Antibiotic use (Lactobacillus normally keeps vaginal pH < 4 4.5) 5) Diabetes Overgrowth of normal flora

MTBS2CK p.486487

MTBS2CK p.487

TypesofVaginitis
Bacterial vaginosis Candidiasis Trichomonas (most common nonviral STD)

Vaginitis

Gardnerella Vaginal discharge with fishy odor; gray white Saline prep: clue cells Metronidazole or clindamycin

Candida albicans White, cheesy vaginal discharge Potassium hydroxide (KOH): pseudohyphae; vaginal culture is most specific Miconazole or clotrimazole, econazole, or nystatin

Trichomonas vaginalis Profuse, green, frothy vaginal discharge Saline prep: motile flagellates Treat both patient and partner with metronidazole

Trichomonas: partners need to be treated

MTBS2CK p.487

MTBS2CK p.487

45

Vaginitis/Gardnerella

Vaginitis/Candidaalbicans

Clue cells. Source: Per Grinsted, commos.wikimedia.org

Source: Nephron, commons.wikimedia.org

Vaginitis/Trichomonasvaginalis

Vulva/MalignantDisorders

Paget Disease Intraepithelial neoplasia Most common in postmenopausal Caucasians Presentation Vulvar soreness and pruritus Appears as a red lesion with superficial white coating
Trichomonas. Source: cdc.gov Trichomonas. Source: Alex Brollo, commons.wikimedia.org

MTBS2CK p.487

Vulva/MalignantDisorders
Large cells with clear cytoplasm in epidermis

Vulva/MalignantDisorders

Biopsy is needed for definitive diagnosis Treatment Wide local excision or vulvectomy 2 cm margin is optimal

Source: Nephron, commons.wikimedia.org

MTBS2CK p.487

46

Vulva/MalignantDisorders Squamous Cell Carcinoma Most common type of vulvar cancer Presentation Pruritus, bloody vaginal discharge, and postmenopausal bleeding Exam: ranges from a small ulcerated lesion to large cauliflower-like lesion A biopsy is essential for diagnosis

StagingofSquamousCellCarcinoma

Stage 0 I II III IV IVa

Findings Carcinomainsitu Limitedtovaginalwall<2cm Limitedtovulvaorperineum>2cm Tumorspreadingtolowerurethraoranus, unilateral il llymph l hnodes d present Tumorinvasionintobladder,rectum,or bilaterallymphnodes Distantmetastasis

MTBS2CK p.488

MTBS2CK p.488

Vulva/MalignantDisorders

Treatment Unilateral: modified radical vulvectomy Bilateral: radical vulvectomy Lymph nodes thatre involved must undergo lymphadenectomy

UterineAbnormalities

MTBS2CK p.488

UterineAbnormalities/Adenomyosis Adenomyosis Invasion of endometrial glands into myometrium Typically between ages of 35 and 50 Risk factors Endometriosis Uterine fibroids Presentation Dysmenorrhea and menorrhagia
MTBS2CK p.488

UterineAbnormalities/Adenomyosis

Diagnosis Clinical diagnosis Physical examination


Uterus is large, globular, boggy

MRI Treatment Hysterectomy - only definitive treatment

MTBS2CK p.488

47

UterineAbnormalities/Adenomyosis

UterineAbnormalities/Adenomyosis

Source: Hic et nunc, commons.wikimedia.org

Source: Hic et nunc, commons.wikimedia.org

UterineAbnormalities/Endometriosis Endometriosis Endometrial tissue outside of endometrial cavity Most common sites are ovary and pelvic peritoneum Endometriosis occurs in women of reproductive age More common if first-degree relative has endometriosis

UterineAbnormalities/Endometriosis

Presentation Cyclical pelvic pain Abnormal bleeding Infertility

Physical examination Nodular uterus Adnexal mass

Dysmenorrhea and dyspareunia are common in endometriosis

MTBS2CK p.488

MTBS2CK p.488489

UterineAbnormalities/Endometriosis

UterineAbnormalities/Treatment

Mild disease NSAIDs Combined OCPs Severe disease Danazole Leuprolide acetate (leupron) Surgery

Source: Julie M. Hastings, Asgerally T. Fazleabas, commons.wikimedia.org

MTBS2CK p.489490

MTBS2CK p.489

48

UterineAbnormalities/Treatment

OvarianAbnormalities

Source: Hic et nunc, commons.wikimedia.org

PolycysticOvarianSyndrome/Symptoms

PolycysticOvarianSyndrome/Diagnosis

Presentation Amenorrhea or irregular menses Hirsutism and obesity Acne Insulin resistance MCC of androgen excess and hirsutism
MTBS2CK p.489

Diagnostic test Pelvic ultrasound Elevated free testosterone LH to FSH ratio > 3:1

MTBS2CK p.489490

PolycysticOvarianSyndrome/Diagnosis

PolycysticOvarianSyndrome/Diagnosis

Uterus

Ovary

Source: Schomyny, commons.wikimedia.org

49

PolycysticOvarianSyndrome/Treatment

Treatment Weight loss OCPs Spironolactone (hirsutism) Metformin (insulin resistance) Clomiphene (infertility)

MTBS2CK p.490

50

Oncology
EmmaHolliday, Holliday MD ResidentPhysician RadiationOncology UniversityofTexasMDAndersonCancerCenter

BreastCancer
Presentation Diagnosis g GeneticTests Treatment

BreastCancer/Presentation

BreastCancer/Presentation Painless lump Skin changes Nipple retraction Nipple discharge

Most often found by patient Found in asymptomatic women


- On screening mammography - By palpation of a mass Hard immobile Hard, immobile, fixed to the chest wall

Source:http://www.4woman.gov/faq/cancerillustrations-with-t.gif

MTBS2CK p.347

MTBS2CK p.347

BreastCancer/DiagnosticTests Biopsy is the best initial test Different methods are: Fine needle aspiration (FNA)
Best initial biopsy

BreastCancer/DiagnosticTests

Mammography Screen the general population starting at age 50

Core needle biopsy


Tells receptor status

Open surgical biopsy


Most accurate test

MTBS2CK p.347

MTBS2CK p.347

WhenIsUltrasoundtheAnswer? A woman finds a hard, nontender breast mass on self-examination. There is no alteration of the mass with menstruation. She is scheduled to undergo a FNA biopsy. Which of the following is most likely to benefit the patient?
a. Mammography b. BRCA testing c. Ultrasound d. Bone scan e. PET scan
MTBS2CK p.348

Clinically indeterminate mass lesions. It tells cysts versus solid lesions Answer ultrasound if the lesion: Is painful Varies in size or pain with menstruation

Confirms an extra risk of cancer Tells cystic vs solid To exclude bone metastases

To look for metastatic disease


MTBS2CK p.348

WhenIsPETScantheAnswer? Determines content of abnormal lymph nodes that are not easily accessible to biopsy. Cancer increases uptake on PET scan.

WhenIsPETScantheAnswer? For example: 80-year-old woman with biopsy-proven breast cancer has no nodes with cancer in the axilla. The primary lesion is small and the woman may not need adjuvant j chemotherapy. py Chest CT shows an abnormal hilar lymph node.

How do you tell the content of an abnormal, inaccessible lesion without biopsy? Try PET scan.

MTBS2CK p.348

MTBS2CK p.348

WhenIsBRCATestingtheAnswer? BRCA associated with increased risk of breast cancer, particularly within families BRCA associated with ovarian cancer What is not clear is what to do when BRCA is positive BRCA not shown to add mortality benefit positive.

WhenIsSentinelLymphNodeBiopsy theAnswer? When you want to know how likely it is that the breast cancer has spread

The precise utility of MRI for breast cancer is not yet clear.
Source:http://www.cancer.gov/ncicancerbulletin/022211/page2

MTBS2CK p.348

MTBS2CK p.349

WhenAreEstrogenandProgesterone ReceptorsTested? Estrogen receptor (ER) and progesterone receptor (PR) testing is routine for all patients Hormone manipulation therapy is done if either test is positive

BreastCancer/Treatment Surgery Options Lumpectomy Modified Radical Mastectomy Radical Mastectomy

Radical mastectomy is always the wrong answer Lumpectomy + Radiation is JUST AS GOOD as modified radical mastectomy
MTBS2CK p.349 MTBS2CK p.349

HormonalManipulation All ER or PR positive patients should receive:


Tamoxifen Raloxifene Aromatase inhibitors (anastrazole, letrozole, exemestane) If all are among the answer choices, aromatase inhibitors are the answer to the most likely to benefit the patient question

BreastCancer/Treatment Know the differences in side effects:

Tamoxifen rarely gives endometrial cancer and clots (tamoxifen is a selective ER modifier). ) Aromatase inhibitors give osteoporosis (aromatase inhibitors inhibit estrogen effect everywhere, even the good effects, like on bone density).
MTBS2CK p.349

MTBS2CK p.349

WhenIsTrastuzumab theAnswer? All breast cancers should be tested for HER 2/neu. This is an abnormal estrogen receptor Those who are positive should receive antiHER 2/neu antibodies known as trastuzumab Trastuzumab decreases the risk of recurrent disease

WhenIsChemotherapy theAnswer?

Neoadjuvant means therapy given BEFORE the definitive treatment. Goal is to decrease cancer burden Adjuvant means an additional therapy to clean up presumed microscopic cancer cells too small in amount to be detected

MTBS2CK p.350

MTBS2CK p.350

WhenIsAdjuvantChemotherapy theAnswer? Adjuvant chemotherapy is the answer when: Lesions >1 cm Positive axillary lymph nodes found Use tamoxifen when multiple first-degree relatives have breast cancer. It lowers the risk of breast cancer.

BreastCancer/Treatment

We do not know what to do about BRCA when it is positive.

All of these definitely lower mortality: Mammography ER/PR testing, then tamoxifen/raloxifene Aromatase inhibitors Adjuvant chemotherapy Lumpectomy and radiation Modified radical mastectomy Trastuzumab (anti-Her 2/neu) Prophylaxis with tamoxifen

MTBS2CK p.350

MTBS2CK p.350

ProstateCancer/Presentation

ProstateCancer
Presentation Treatment Screening

Obstructive symptoms Palpable mass Elevated or rising PSA

Source:http://www.cancer.gov/cancertopics/pdq/treatment/prostate/Patient/page2

MTBS2CK p.350

ProstateCancer

ProstateCancer/Treatment

Biopsy is the best initial test and the most accurate test.

Prostatectomy Radiation therapy Brachytherapy Hormonal therapy Watchful waiting

Half of men above age 80 have prostate cancer on autopsy.


MTBS2CK p.350
Source:http://kidney.niddk.nih.gov/kudiseases/pubs/i magingut/

MTBS2CK p.350351

ProstateCancer/Treatment

GleasonGrading Gleason Score = Tumor Grade High Gleason grade suggests greater g benefit of surgical removal of the prostate
Source:http://www.training.seer.cancer.gov/prostate/abstract-codestage/morphology.html

Prostatectomy has slight benefit over radiation in terms of survival Most common complications of prostatectomy are:
Erectile dysfunction Urinary incontinence

Get it out before it metastasizes if the Gleason grade is high


MTBS2CK p.350351 MTBS2CK p.351

HormonalManipulationinProstateCancer

ManagementThatIsDefinitely NotBeneficialinProstateCancer These answers are ALWAYS wrong: Screening imaging study like ultrasound Lumpectomy Chemotherapy Hormonal manipulation to prevent recurrences

Flutamide
Competitive inhibitor of testosterone & DHT

Leuprolide, goserelin
GNRH agonists: downregulates LH & FSH

Ketoconazole K t l
Suppresses testosterone

Orchiectomy
Stops endogenous production

MTBS2CK p.351

MTBS2CK p.351

ProstateSpecificAntigen(PSA) PSA is controversial: No mortality benefit with PSA PSA is not to be routinely offered Normal PSA does not exclude prostate cancer High PSA doesnt always mean prostate cancer

ProstateSpecificAntigen(PSA)

If question specifically says, The patient is requesting PSA to screen for cancer, then the answer is do the test

The higher the PSA, the greater the risk of cancer. PSA corresponds to the volume of cancer.
MTBS2CK p.351 MTBS2CK p.351

ElevatedPSAAlgorithm Elevated PSA Palpable mass Biopsy p y the mass No palpable mass Transrectal ultrasound

LungCancer

Mass seen Biopsy the mass

No mass seen Multiple blind biopsies

MTBS2CK p.351

LungCancer

Surgery

Who can get surgery? Size of the tumor alone does not determine whether or not it can be resected
Large tumors can be resected with
Demonstration of suffficient residual lung volume by PFTs It is solitary and surrounding tissue is healthy

Small tumors cannot be resected if PFTs indicate poor lung function


MTBS2CK p.352

Source:http://www.cancer.gov/cancertopics/pdq/treatment/non- Source:http://www.cancer.gov/cancertopics/pdq/treatment/non small-cell-lung/Patient/page4 -small-cell-lung/Patient/page4

Source:http://www.cancer.gov/cancertopics/pdq/treatment/n on-small-cell-lung/Patient/page4

LungCancer

Surgery is not possible in these cases: Bilateral disease Malignant pleural effusion Heart, carina, aorta, or vena cava is involved Small cell cancer is considered unresectable in 95% of cases because it is metastatic or spread outside one lung

OvarianCancer
Screening Diagnosis g Treatment

MTBS2CK p.352

OvarianCancer

OvarianCancer/Diagnosis

No screening test for ovarian cancer Presentation:


Woman >50 years old Increasing abdominal girth
No history of liver disease

The initial test is an ultrasound or CT scan

Weight loss, fatigue


Source:http://www.cancer.gov/cancertopics/pdq/treatment/ovaria nepithelial/Patient/page1 Source: James Heilman, MD, http://en.wikipedia.org/wiki/File:POvarianCA.png

MTBS2CK p.352

MTBS2CK p.352

OvarianCancer/Diagnosis

OvarianCancer/Treatment

Most accurate test is biopsy

The only cancer in which removing large amounts of locally metastatic disease benefits patient Remove all visible tumor and pelvic organs and give chemotherapy

Source: http://en.wikipedia.org/wiki/File:Ovarian_carcinoma.JPG

MTBS2CK p.352

MTBS2CK p.352

TesticularCancer/Presentation Young men: 20-40 years old Painless lump in the scrotum
Next best step?
Transillumination Scrotal Ultrasound

TesticularCancer
Presentation Diagnosis g Treatment

Differential ee a d diagnosis? ag os s
Epididymitis Hematocele Varicocele

MTBS2CK p.352

TesticularCancer/Diagnosis

TesticularCancer/Diagnosis Seminoma Non-seminoma


Yolk sac or Endodermal sinus
AFP

Diagnostic Testing Remove the whole testicle with inguinal orchiectomy Do not cut the scrotum, which hi h can spread d th the disease Needle biopsy of the testicle is always a wrong answer

Has NORMAL AFP Can have elevated bHCG LDH correlated with disease burden

Embryonal
AFP and bHCG

Choriocarcinoma
AFP and bHCG

Teratoma
AFP and bHCG

MTBS2CK p.352

Source:Ed Uthman, MD. http://en.wikipedia.org/wiki/File:Seminoma_of_the_Testis.jpg

TesticularCancer/Staging

TesticularCancer/Treatment

Staging is performed with:


CT scan of the abdomen, pelvis, and chest Lymphatic channels in the retroperitoneum spread testicular cancer into the chest

1st- orchiectomy 2nd- radiation 3rd- chemotherapy Testicular cancer is one of the only malignancies in which chemotherapy can cure widely metastatic disease, including spread into the brain

MTBS2CK p.353

http://emedicine.medscape.com/article/437966-clinical#a0218

MTBS2CK p.353

PreventionofInvasiveCervicalCancer Human papillomavirus (HPV) vaccine is given to all women between ages 11 and 26 Pap smear is performed starting at age 21 Every y3y years, , with cytology, until 30 years old After age 30, every 5 years, if HPV testing added Stop at age 65 with adequate screening history and low risk
MTBS2CK p.353

CervicalCancer
Prevention/earlydetection ClinicalPresentation Treatment

Source:http://www.cdc.gov/cancer/dcpc/prevention/vaccination.htm

Source: Nephron, http://en.wikipedia.org/wiki/File:Adenocarcinoma_on_pap_test_1.jpg

DetectionofCervicalCancer Atypical squamous cells of undetermined significance (ASCUS)


HPV testing
If positive colposcopy and biopsy If negative repeat Pap in 6 mo

DetectionofCervicalCancer

Low-grade g and high-grade g g dysplasia y p


Colposcopy and biopsy

Pap smear does not lower mortality as much as mammography or colonoscopy.

MTBS2CK p.353

MTBS2CK p.353

CervicalCancer/Presentation Asymptomatic
Detected on Pap

CervicalCancer Symptomatic
Abnormal vaginal bleeding Post coital bleeding Abnormal discharge Pelvic p pain or fullness Dysuria

The management of early cervical cancer = hysterectomy

Source: Hic et nunc, http://en.wikipedia.org/wiki/File:Scheme_hysterectomy-en.svg

Ophthalmology Ophthalmology
ConradFischer,MD AssociateProfessorofMedicine TouroCollegeofMedicine NewYorkCity Conjunctivitis TheRedEye(OphthalmologicEmergencies) Cataracts DiabeticRetinopathy RetinalArtery&VeinOcclusion RetinalDetachment MaculaDegeneration

ComparisonofViralandBacterial Conjunctivitis Viralconjunctivitis Bacterialconjunctivitis Bilateral Waterydischarge Easilytransmissible Normalvision Itchy Preauricular adenopathy Nospecifictherapy Unilateral Purulent,thickdischarge Poorlytransmissible Normalvision Notitchy Noadenopathy Topicalantibiotics

Conjunctivitis Viral conjunctivitis Bacterial conjunctivitis

David C Cogan Ophthalmic Pathology Collection

Source: phil.cdc.gov

MTBS2CK p.497

Conjunctivitis

EtiologiesofTheRedEye
Conjunctivitis
Presentation

Uveitis Autoimmune diseases Photophobia

Glaucoma Pain Fixed midpoint pupil Tonometry Acetazolamide Mannitol Pilocarpine Laser trabeculoplasty

Abrasion Trauma Feels like sand in eyes Fluorescein stain No specific therapy Patch not clearly beneficial

The must know subjects in ophthalmology are: The red eye (emergencies) Diabetic retinopathy Artery and vein occlusion Retinal detachment

Discharge
Eyefindings

Normal pupils
Most accuratetest

Clinical diagnosis
Bestinitial therapy

Slit lamp examination

Topical antibiotics
MTBS2CK p.497 MTBS2CK p.498

Topical steroids

Glaucoma/ChronicGlaucoma

Most often asymptomatic Diagnosed by screening Confirmation with tonometry Elevated intraocular pressure

Glaucoma/ChronicGlaucoma Treattodecreaseproductionofaqueoushumoror increasedrainage Prostaglandin analogues Topical carbonic Latanaprost anhydrase Travoprost inhibitors
Bimatoprost

Topical p beta blockers


Timolol Carteolol Metipranolol Betaxolol Or Levobunolol

Dorzolamide Brinzolamide

Alpha-2 agonists
Apraclonidine

Pilocarpine Laser

MTBS2CK p.498

MTBS2CK p.498

Glaucoma/AcuteAngleClosureGlaucoma

Glaucoma/AcuteAngleClosureGlaucoma
Conjunctival vessels dilated at corneal edge

Look for Sudden onset Extremely painful, red eye hard to palpation Walking into dark room precipitates pain because of p pupillary p y dilation Pupil doesnt react to light because its stuck Cup-to-disc ratio > 0.3

Hazy cornea
Author:Jonathan Trobe, M.D. Source: commons.wikimedia.org

MTBS2CK p.498

Glaucoma/AcuteAngleClosureGlaucoma

HerpesKeratitis

Diagnosis Confirmed with tonometry Treatment IV acetazolamide IV mannitol (osmotically draws of fluid out) Pilocarpine & beta blockers constrict pupil & enhance drainage Laser iridotomy
MTBS2CK p.498

Infection of cornea Eye is red, swollen, and painful, but dont use steroids Steroids markedly increase production of virus

Fluorescein stain confirms dendritic pattern

MTBS2CK p.499

HerpesKeratitis

Cataracts

Treatment Oral acyclovir, famciclovir, or valacyclovir Topical trifluridine or idoxuridine

Beware of steroid use for herpes keratitis Steroids worsen condition

No medical therapy for cataracts Surgically remove lens & replace with new intraocular lens New lens may automatically have bifocal capability Early cataracts are diagnosed with an ophthalmoscope or slit-lamp exam Advanced cataracts visible on exam

MTBS2CK p.499

MTBS2CK p.499

Cataracts

DiabeticRetinopathy Annual screening exams INDISPENSIBLE!!!! Detects retinopathy before visual loss occurs Nonproliferative or background retinopathy is managed by controlling glucose level Most accurate test is
Fluorescein angiography

Proliferative: treated with laser photocoagulation Vascular endothelial growth factor inhibitors (VEGF) injected in some patients to control neovascularization

nih.gov

MTBS2CK p.499

DiabeticRetinopathy Vitrectomy may be necessary to remove a vitreal hemorrhage obstructing vision

RetinalArteryandVeinOcclusion Both present with Sudden onset monocular visual loss Cant make diagnosis without retinal examination No conclusive therapy for either condition

MTBS2CK p.499

New blood vessel formation obscures vision. Source: Conrad Fischer, MD.

MTBS2CK p.500

RetinalArteryOcclusion

RetinalVeinOcclusion

Retinal artery occlusion presents with sudden loss of vision and a pale retina and dark macula. Source: Conrad Fischer, MD.

Retinal vein occlusion leads to extravasation of blood into the retina. Source: Conrad Fischer, MD.

MTBS2CK p.500

MTBS2CK p.500

RetinalArteryandVeinOcclusion Treatment of artery occlusion attempted with... 100% oxygen Acetazolamide to intraocular pressure Thrombolytics

RetinalDetachment

Macula Mac la is described as cherry redin artery occlusion because the rest of retina is pale
MTBS2CK p.500

Caused by Trauma Extreme myopia (changes shape of eye) Diabetic retinopathy Anything that pulls on retina can detach it Presents with Sudden onset of painless, unilateral loss of vision Described as curtain coming down
MTBS2CK p.501

RetinalDetachment Reattachment by mechanical methods Surgery, laser, cryotherapy Injection of expansile gas pushes retina back up against globe of eye

MacularDegeneration MCC of blindness in older persons in U.S. Idiopathic Atrophic (dry) type and neovascular (wet) type Far more common in older patients Bilateral N Normal l external t l appearance of f eye Central vision lost

MTBS2CK p.501

Sudden painless loss of vision like a courtain coming down. Source: Conrad Fischer, MD.

MTBS2CK p.501

MacularDegeneration Neovascular disease More rapid More severe New vessels grow between retina and underlying Bruchs membrane Neovascular or wet type causes 90% of permanent blindness from macular degeneration Atrophic macular degeneration has no proven effective therapy
MTBS2CK p.501

MacularDegeneration

MTBS2CK p.502

Macular degeneration can be diagnosed only by visualization of the retina. Source: Conrad Fischer, MD.

MacularDegeneration

Best initial therapy for neovascular disease VEGF inhibitors


Ranibizumab or bevacizumab

Injected directly into vitreous chamber every few weeks Over 90% stop progression 1/3 improve vision

MTBS2CK p.502

Pediatrics
RyanClose,MDMPH ResidentPhysician InternalMedicinePediatrics HospitaloftheUniversityofPennsylvania

RoutineManagement oftheNewborn
PhysicalExam ApgarScore EyeCare RoutineScreeningandPrevention

PhysicalExam

ApgarScore

Adults Newborns Physical exams start with vital signs...


HeartRate(HR) RespiratoryRate (RR) SystolicBlood Pressure(SBP) DiastolicBlood Pressure(DBP)
MTBS2CK p.403

60100BPM 1224BrPM

120160BPM 4060BrPM

Important part of physical exam Translates physical exam into a score


Used for management decisions

Provides both short- and long-term prognostic indicators

120mmHg

65mmHg

80mmHg

50mmHg
MTBS2CK p.404

ApgarScore

ApgarScore 0points
Appearance Skincolor Pulse Rate

Scores calculated at 1 minute and 5 minutes Score at 1 minute Represents conditions during labor and delivery Indicates need for resuscitation Score at 5 minutes Represents effectiveness of resuscitation efforts Prognostic of survival
MTBS2CK p.404

1point

2points
Pinkallover >100bpm S Sneeze, cough, h loudcry Active movement Strong

Blueallover Pinktorso/blue extremities <60bpm or 60100bpm asystole

Grimace G i N response Grimace/ No Gi / Reflex&irritability feeblecry Activity Muscletone Respiration Breathing


MTBS2CK p.404

None Absent

Someflexion Weak,irregular

ApgarScore
A 28-year-old G1PO woman delivers a 3.9 kg male infant whose Apgar scores are 9 and 10 at 1 and 5 minutes, respectively. The delivery was uncomplicated, and both mother and child are in no acute distress. Whats the most appropriate first step upon delivery of this patient? a. Intubate the child No signs of respiratory distress / high Apgar score b. Send cord blood for arterial blood gas (ABG) Patient is stable c. Suction the mouth and nose d. Nasogastric tube (NGT) placement No need for decompression e. Give prophylactic antibiotics No evidence of infection or sepsis
What we are not told: - Is this child term, or pre-term? - Vital signs ?

Low Apgar score isnt predictive of cerebral palsy

Knowing differences in management if Apgar score is low

MTBS2CK p.404

MTBS2CK p.403

Prevention/EyeCare A 3.9 kg male infant whose Apgar scores were 9 and 10 at 1 and 5 minutes, respectively, presents five days after delivery because of red eyes. The delivery was without any complications. What is the most likely diagnosis at day 5 of life? a. Chemical irritation b. Neisseria gonorrhoeae c. Chlamydia trachomatis d. Group-B Streptococci e. Herpes simplex

- Erythromycin ointment - Tetracycline ointment - Silver nitrate drops

Prevention/EyeCare

Chemical Irritation Due to silver nitrate Developing countries Not an allergy

Neisseria gonorrhea Gram Gramnegative diplococci


Prevent with ointments

Chlamydia trachomatis Not effectively prevented by prophylaxis ointments Treat with oral erythromycin

Herpes Simplex Treat with systemic acyclovir and topical vidarabine

Treat with ceftriaxone

MTBS2CK p.404405

Prevention/EyeCare A 3.9 kg male infant whose Apgar scores were 9 and 10 at 1 and 5 minutes, respectively, presents five days after delivery because of red eyes. The delivery was without any complications. What is the most likely diagnosis at day 5 of life? a. Chemical irritation b. Neisseria gonorrhoeae c. Chlamydia trachomatis d. Group-B Streptococci e. Herpes simplex

Prevention&Screening/VitaminKDeficiency Vitamin K deficient bleeding = hemorrhagic disease of newborn Neonate colon lacks normal bacterial flora that produces vitamin K Vitamin K is responsible for clotting factors II, VII, IX, , and X as well as proteins p C and S
Vit K
Inactive Clotting Factors Active Clotting Factors

Treatment: one IM injection of vitamin K


Oral doses will not work
MTBS2CK p.405

Prevention&Screening/VitaminKDeficiency
Intrinsic Pathway
XIIa XI IX VIII XIa IXa VIIIa Xa V Va IIa Thrombin Fibrin Fibrinogen X TF VIIa II VII

Prevention&Screening/Testing Newborns should be screened for: Phenylketonuria (PKU) Congenital adrenal hyperplasia (CAH) Biotinidase Beta-thalassemia Galactosemia G l t i Hypothyroidism Homocysteinuria G6PD deficiency Hearing test
MTBS2CK p.405 406

Extrinsic Pathway

TransientConditionsoftheNewborn

The Big Transient Three

MedicalConditions oftheNewborn
Transientconditions Polycythemia Tachypnea Hyperbilirubinemia Deliveryassociatedinjuries Newborninfections

Polycythemia

Tachypnea

Hyperbilirubinemia
MTBS2CK p.406407

TransientPolycythemiaoftheNewborn

TransientPolycythemiaoftheNewborn

Polycythemia
Increased RBCs
Epo!

Transient Polycythemia

Primary vs. Secondary


Primary: Normal or low EPO levels Secondary: High EPO levels
Source:JamesVanRhee

Benign Most often related to cord clamping


Hypoxia EPO!

Polycythemia of the Newborn


Can be very harmful
Hyperviscosity, decreased perfusion, thromboses LGA, SGA, and IDM
MTBS2CK p.406 MTBS2CK p.406

TransientTachypneaoftheNewborn(TTN)

TransientTachypneaoftheNewborn(TTN)

Tachypnea
60 BrPM (40 60 is normal)
TTN or Respiratory Distress Syndrome II (RDS II)

Benign condition 1% to 2% of newborns


Causes
Excess remaining lung fluid Pulmonary immaturity Surfactant deficiency/insufficiency

The Takeaway: Benign condition Term infants Delivered via C-section Oxygen, antibiotics, and watch closely Watch for
Sepsis

Management
Oxygen and antibiotics Work-Up: CBC and chest radiograph
MTBS2CK p.406407

MTBS2CK p.406407

TransientHyperbilirubinemia Hyperbilirubinemia Production > Elimination increased TSB


From increased production, decreased elimination, or both

TransientHyperbilirubinemia

Any hyperbilirubinemia < 24 hr of life Evaluation Any conjugated hyperbilirubinemia Evaluation

Indirect = Unconjugated bilirubin Direct = Conjugated bilirubin


(Total Bili) (Direct Bili) = Indirect Bili

Transient Hyperbilirubinemia
Benign and very common (~60% newborns) Peaks at 2-3 days of life Increased production of unconjugated bilirubin
MTBS2CK p.407 MTBS2CK p.407

TransientConditionsoftheNewborn

DeliveryAssociatedInjuries Types of injuries Subconjunctival hemorrhage Skull fracture Scalp injuries Brachial palsies Clavicular Cl i l fracture f t Facial nerve palsy
Microhemorrhages Benign Caput succedaneum Cephalohematoma

The Big Transient Three


Polycythemia

Tachypnea

Brief paralysis of facial nerve

Hyperbilirubinemia
MTBS2CK p.406407 MTBS2CK p.407

BrachialPlexusInjuries General Macrosomic infants (e.g., IDM) Shoulder dystocia Duchenne-Erb Paralysis 90% of brachial palsies C5 C6 Waiters tip Cannot abduct or externally rotate Klumpke Paralysis C7 T1 Claw hand +/- Horner syndrome
MTBS2CK p.407408

BrachialPlexusInjuries

ClavicularFracture

PhysicalExam Physical exams start with vital signs...


Heart Rate (HR) Respiration Rate (RR) Systolic Blood Pressure (SBP) Diastolic Blood Pressure (DBP)

Most common newborn fracture Usually result of:


Shoulder dystocia

Age Group

Best diagnostic tool:


Radiograph g p
Adult Newborns 60-100 BPM 16-24 BrPM 120-160 BPM 40-60 BrPM

120 mmHg 65 mmHg

80 mmHg 50 mmHg

Most appropriate management:


Immobilize

MTBS2CK p.408

NeonatalSepsis
Early Late

ToRCHInfections
Type Presentation Diagnostictests Treatment
Initial:IgM Mostaccurate:PCR Initial:VDRL/RPR Mostaccurate:FTA ABS/DarkField Elevatedmaternal rubellaIgM with clinicalpicture Initial:Urine/saliva viraltiters Most accurate:PCR Initial:Tzanck smear Most accurate:PCR Chorioretinitis,hydrocephalus, Toxo plasmosis ringenhancing lesion

Pyrimethamine andsulfadiazine Penicillin IV

Syphilis
GBS E. Coli Listeria IVF Cultures Antibiotics Ampicillin Gentamicin Cefotaxime*
MTBS2CK p.433 Staphylococci

Rashonpalms/soles,snuffles, frontalbossing,Hutchinson 8th n.palsy,saddlenose PDA,cataracts,deafness, hepatosplenomegaly, lowplts, elevatedbilirubins Periventricularcalcifications, microcephaly,chorioretinitis, hearingloss Week1:ShockandDIC Week2:Vesicular skinlesions Week3:Encephalitis

E. Coli GBS

Rubella CMV

Supportive

Ganciclovir

Ampicillin Gentamicin Cefotaxime*

Herpes

Acyclovirand supportivecare

MTBS2CK p.434

AmnioticFluidAbnormalities

CommonAbnormalities oftheNewborn
AmnioticFluidAbnormalities AbdominalAbnormalities GenitourinaryAbnormalities

Amniotic Fluid 80% from mother 20% from infant Problems Polyhydramnios (too much) Oligohydramnios (too little)

MTBS2CK p.408

AmnioticFluidAbnormalities Polyhydramnios Too much fluid Overproduction / Decreased resorption CNS malformations

AmnioticFluidAbnormalities Polyhydramnios GI malformations


Esophageal atresia

MTBS2CK p.408

MTBS2CK p.408

AmnioticFluidAbnormalities Oligohydramnios Too little fluid (Low AFI) Under production Causes
Post-term pregnancies Renal agenesis and renal failure

AbdominalAbnormalities A premature infant born at 28 weeks in respiratory distress with grunting, nasal flaring, and use of accessory muscles. Bowel sounds heard upon auscultation of the back and chest X-ray shows air fluid levels in the chest. Which of the following is the most likely diagnosis? a. Hydrocele b. Gastroschisis c. Diaphragmatic Hernia d. Hiatal Hernia e. Omphalocele
MTBS2CK p.409

Cord Compression ACE-inhibitors Potters syndrome


MTBS2CK p.408

AbdominalAbnormalities

AbdominalAbnormalities Which of the following is the most likely diagnosis? a. b. c. d d. e.


Urological issue not respiratory issue Hydrocele Abdominal wall defect Gastroschisis Diaphragmatic Hernia M More common i in GERD seen in i adults d lt Hi t l H Hiatal Hernia i Omphalocele Abdominal wall defect

Source: Niket Sonpal, MD

MTBS2CK p.409

MTBS2CK p.409

AbdominalAbnormalities Diaphragmatic Hernia Congenital defect in diaphragm Two types: Bochdalek and Morgagni
LEFT side most common

AbdominalAbnormalities Omphalocele
Midline wall defect With sac covering Associations
Imperforateanus Congenitalheartdefects(50%) Conjoinedtwins Trisomy 18(Edwards syndrome) BeckwithWiedemann syndrome

Gastroschisis
Lateral wall defect No sac covering Atresias Surgery is necessary

Key findings:
Respiratory distress Scaphoid abdomen Bowel sounds in chest Abnormal chest radiograph

First step in management


Intubation
MTBS2CK p.409

Surgery is necessary

MTBS2CK p.410

AbdominalAbnormalities

AbdominalAbnormalities/Tumors Wilms Tumor Most common primary renal malignancy in peds Presents with

An elevated AFP in abdominal wall defects The MCC of elevated AFP is incorrect dating

Asymptomatic flank mass Hematuria Hypertension and aniridia

Umbilical hernias: Similar to omphaloceles

First step in evaluation: Abdominal ultrasound MC diagnostic test: Computed tomography Relationships with syndromes
WAGR, Denys-Drash, Beckwith-Wiedemann

Treatment: Surgery + chemotherapy + radiation


MTBS2CK p.410 MTBS2CK p.410

AbdominalAbnormalities/Tumors Neuroblastoma Very common among children Often involves adrenal gland Presents with
Painful abdominal mass Neurological findings
Opso(myo)clonus

GenitourinaryAbnormalities

Diarrhea

Diagnostic keys
Urine catecholamines and their metabolites
Vanillyl Mandelic Acid (VMA) Homovanillic acid (HVA)
MTBS2CK p.411

Hydrocele Painless, benign, fluid-filled Cryptorchidism Undescended testis, increased cancer risk Surgical correction after 6 months Hypospadias Ventral surface opening, surgery Epispadias Dorsal surface opening, surgery
MTBS2CK p.411

CyanoticHeartDefects

CyanoticHeartDefects

Tetralogy of Fallot Transposition of the Great Vessels Hypoplastic Left Heart Syndrome Truncus Arteriosus Total Anomalous Pulmonary Venous Return

TetralogyofFallot

TetralogyofFallot

The most common cyanotic heart defect in children Four aspects p Pulmonary stenosis VSD Overriding aorta Right ventricular hypertrophy
MTBS2CK p.412

TetralogyofFallot/Signs&Symptoms Cyanosis Lips and extremities Squatting Increases systemic pressure Shunts blood to pulmonary circulation Holosystolic H l t li murmur VSD There are 3 holosystolic murmurs Mitral regurgitation (MR) Tricuspid regurgitation (TR) Ventricular septal defect (VSD)
MTBS2CK p.412413

TetralogyofFallot

TetralogyofFallot

TranspositionoftheGreatVessels

Two separate circulations Right heart/systemic circulation Left heart/pulmonary circulation Defect dependent Patent ductus arteriosus (PDA) VSD Atrial septal defect (ASD)
Most common cyanotic lesion of neonates
MTBS2CK p.413

TranspositionoftheGreatVessels

TranspositionoftheGreatVessels

Treatment Prostaglandin E1 Surgery

MTBS2CK p.413

HypoplasticLeftHeartSyndrome

HypoplasticLeftHeartSyndrome

Syndrome Absent pulses Right ventricular heave Mild cyanosis/gray

MTBS2CK p.414

TruncusArteriosus

TruncusArteriosus

One Great Vessel The less common heart defect (2%) NOT dependent on PDA Mild cyanosis Big problem: Pulmonary hypertension Treat with Surgery!

MTBS2CK p.414

TotalAnomalousPulmonaryVenousReturn

TAPVR 2
Sign/Symptoms TAPVRwith obstruction Earlyinlifewith respiratory distressand severecyanosis Presentslater Age12years withheart failure Tests CXRshows pulmonary edema Echoistestof choice Treatment Surgery

Abnormal pulmonary venous return Pulmonary veins Right atrium PFO dependent Right heart overload Two types: With obstruction Without obstruction

TAPVRwithout obstruction

S Surgery CXRshows snowman sign Echoistestof choice

MTBS2CK p.414415

MTBS2CK p.415

10

CyanoticHeartDefects
R LShunt TOF TGV Hypoplastic LH Truncus Art TAPVR PDAdep VSD Surgery

AcyanoticHeartDefects

AcyanoticHeartDefects

Heartmurmurinachild
3-year old female brought in because her parents say she wont eat anymore. Upon physical examination, a loud pansystolic murmur is appreciated. The child appears small for her age, but her records dont show any maternal or delivery complications.
Which of the following is the most likely finding on EKG?

VSD ASD PDA Coarctation of the aorta

a. b. c. d. e.

Right ventricular hypertrophy RBBB ST-segment elevation QT-prolongation P-wave inversion

MTBS2CK p.415

VentricularSeptalDefect

VSD,Murmurs,andAuscultation

There are 3 holosystolic murmurs... MR TR VSD


VSD

VSD is the most common congenital heart defect


MTBS2CK p.415

11

VentricularSeptalDefect Pathophysiology
Left-to-Right shunt (opposite of cyanotic defects) Pulmonary hypertension

VentricularSeptalDefect

Presentation
Dyspnea with distress Loud pulmonic S2 High-pitched holosystolic murmur

Tests
CXR: Findings are not diagnostically helpful Best initial test is an echocardiogram Most diagnostic (definitive) test is a cardiac catheterization
MTBS2CK p.416

Heartmurmurinachild
3-year old female brought in because her parents say she wont eat anymore. Upon physical examination, a loud pansystolic murmur is appreciated. The child appears small for her age, but her records dont show any maternal or delivery complications.
Which of the following is the most likely finding on EKG?

VSD

a. b. c. d. e.

Right ventricular hypertrophy RBBB ST-segment elevation QT-prolongation P-wave inversion

MTBS2CK p.415

VSD &Eisenmenger Syndrome


3-year old female brought in because her parents say she wont eat anymore. Upon physical examination, a loud pansystolic murmur is appreciated. The child appears small for her age, but her records dont show any maternal or delivery complications.
Which of the following is the most likely finding on EKG?

a. b. c. d. e.

Right ventricular hypertrophy RBBB ASD or ischemic disease ST-segment elevation Myocardial infarction Electrolyte disturbances QT-prolongation P-wave inversion Atrial arrhythmias

MTBS2CK p.415

12

AtrialSeptalDefect Types of ASDs


Primum, secundum, and sinus venosus 2xs common in men

AtrialSeptalDefect

Usually asymptomatic Fixed wide-splitting S2 vs. Physiologic splitting Tests


Best initial test is an echocardiogram (bubble study) Most diagnostic test is a cardiac catheterization

Prognosis
Most close without intervention

MTBS2CK p.416417

AtrialSeptalDefect Without closure


Atrial enlargement Dysrhythmias Embolic Risk

PatentDuctusArteriosus Failure of closure after 12 hours Left-to-Right shunt Presentation


Machinery-like murmur Wide pulse pressures Bounding pulses

Treatment
Surgical

Tests
Best initial test is an echocardiogram Most diagnostic test is a cardiac catheterization

Treatment
NSAIDS (indomethacin)
MTBS2CK p.417

PatentDuctusArteriosus

PatentDuctusArteriosus

13

CoarctationoftheAorta Congenital narrowing of aorta


Usually after cephalic branches

CoarctationoftheAorta

Turners Syndrome Presentation


Differential blood pressure
Between upper and lower body Hypertension

CHF and respiratory distress

Tests
Best initial tests are an echo or CXR Most diagnostic test is a cardiac catheterization
MTBS2CK p.418419

Volvulus What is it? How is it managed?


Decompression of gut + IVF Surgery Malrotation of gut

Gastroenterology Part2
Volvulus&Intussusception MeckelsDiverticulum Diarrhea&Gastroenteritis

What causes it?


Non-rotation during development

If decompensating...
Proceed immediately y to OR Get antibiotics on board

How does it present?


90% within first year of life Bilious vomiting Colicky abdominal pain
MTBS2CK p.424425

Intussusception Telescoping bowel Presentation


Colicky abdominal pain Bilious vomiting Currant-jelly stool Sausage-like Sausage-like mass
This triad is commonly used on S2!

Intussusception Testing and Treatment


Best initial test: Ultrasound Air contrast/barium enema
Diagnostic and therapeutic

First steps in management:


IVF and correction of electrolytes NG tube for decompression

Signs of peritonitis/perforation Su Surgery ge y Recurrence ~10%

Source: Brad L. Kocher

MTBS2CK p.425426

MTBS2CK p.426

Source: Boma O. Afiesimama

14

BiliousVomiting
Bilious Vomiting
Duodenal Atresia Volvulus Intussusception

MeckelsDiverticulum Rule of 2s
2% prevalence 2 years old 2 ft proximal to ileocecal valve 2 inches long 2 types of ectopic tissue Males 2x more affected 2% symptomatic

Within 1st day Initial test: AXR Double

Within 1st year Initial test: AXR

Within 1st year Initial test: US Doughnut First Step: IVF! Treatment Air Enema

bubble
First Step IVF! Treament Surgery
MTBS2CK p.424 426

First Step IVF! Treament Surgery

Painless rectal bleeding


Katsumi M. Miyai, M.D., Ph.D., Regents of the University of California. Used with permission.

MTBS2CK p.426427

DiarrheaandGastroenteritis

DiarrheaandGastroenteritis
Infectious Diarrhea

The most important questions to ask: Is this chronic or acute? Infectious or not? Bloody or not? How sick is the child?

Viral

Parasites

Fungal

Bacterial

Giardia Cryptosporidia

Candida spp Histoplasma

Salmonella Shigella C difficile C. E. Coli Campylobacter Yersinia

MTBS2CK p.427

DiarrheaandGastroenteritis
Infectious Diarrhea

DiarrheaandGastroenteritis

Viral

Rotavirus
Most M tcommon Winter Symptoms: Fever,emesis NOblood <7days Viralprodrome Vaccine

Adenovirus
Endemic E d i Yearround Symptoms: Fever,emesis NOblood >7days Viralprodrome

Small,round
Norwalk N lk EPIdemic Symptoms: Explosive Cramping,pain Shortlived 12days

Management Hydration is key Almost always the answer Antibiotics for suspected bacterial i f ti infection WBC or blood in stool NEVER use antidiarrheal meds in these patients
MTBS2CK p.428

15

LargeforGestationalAge/Macrosomia
A 10.5-pound infant is born to a mother with Type I diabetes. Upon examination of newborn, he is shaking, and a holosystolic murmur is heard over precordium. The babys right arm is adducted and internally rotated. His lab findings show an elevated bilirubin. Which of the following is the most appropriate next step in management?

Endocrinology
InfantsofDiabeticMothers Congenital g AdrenalHyperplasia yp p (CAH) ( ) VitaminDDeficiencies

a. b. c. d. e.

IV insulin Blood sugar level Serum calcium levels Serum TSH CT head and neck

MTBS2CK p.429

InfantsofDiabeticMothers(IDM)

InfantsofDiabeticMothers(IDM)

Maternal Hyperglycemia

Fetal / Infant Hyperglycemia

Infant Hyperinsulinemia

Four primary issues... Macrosomia Hypoglycemia Electrolyte abnormalities Jaundice

MTBS2CK p.430

MTBS2CK p.430

IDM/Macrosomia Macrosomia = weight 4500 g Large for gestational age (LGA) = top 90th percentile Causes in IDM Oversupply of AAs, glucose, etc. Insulin is a growth factor Consequences C Trauma Risk of C-section Treatment None Prevention!
MTBS2CK p.430

IDM/Hypoglycemia

Maternal Hyperglycemia

Infant Hyperglycemia

Infant Hyperinsulinemia

Birth

MTBS2CK p.430

16

IDM/Hypoglycemia Direct consequence of maternal hyperglycemia Hypertrophied pancreatic beta-cells Insulin overproduction Hyperresponsive After birth: No N changes h i in i insulin li production/activity d ti / ti it Consequences Severe hypoglycemia Seizures Treatment Monitoring + Glucose
MTBS2CK p.430

IDM/Electroyteabnormalities Hypocalcemia Hypomagnesemia / Hyperphosphotemia Twitching and Tremulousness Cardiac arrhythmias Calcium and Magnesium levels linked Always Al check h kb both th Correct together

MTBS2CK p.430

IDM/Jaundice One-third of IDMs will develop jaundice Unconjugated/Indirect hyperbilirubinemia Overproduction of bilirubin Resolving hematomas Polycythemia Treatment T t t Phototherapy
A 10.5-pound infant is born to a mother with Type I diabetes. Upon examination of newborn, he is shaking, and a holosystolic murmur is heard over precordium. The babys right arm is adducted and internally rotated. His lab findings show an elevated bilirubin. Which of the following is the most appropriate next step in management?

a. b. c. d. e.

IV insulin Blood sugar level Serum calcium levels Serum TSH CT head and neck

IDMs have hyperinsulinemia

Check, but not most immed. Good test, but not relevant No concern trauma or bleed

MTBS2CK p.430

MTBS2CK p.429

IDM/Other Other abnormalities Small left colon syndrome Respiratory distress syndrome (RDS) Cardiac abnormalities

CongenitalAdrenalHyperplasia(CAH)
Cholesterol
Pregnenolone

17-OHg Pregnenolone

17-OHProgesterone

Deoxycortisol

Cortisol

DHEA Androstenediol

Androstenedione Testosterone

Estrone Estradiol

17-hydroxylase deficiency 21-hydroxylase deficiency


MTBS2CK p.430

11-hydroxylase deficiency

17

CAH:17HydroxylaseDeficiency
Cholesterol
Aldosterone

CongenitalAdrenalHyperplasia
17hydroxylasedef 21hyroxylasedef 11hydroxylasedef

Cortisol

Aldosterone Cortisol Sexhormones HYPERtensive

SexDevelopment p
Testosterone Estradiol
Girls:nml atbirth Boys:pseudo hermaphroditism

Classic S2CK: teenage girl presents with delayed puberty and incidentally found to have elevated BP

Electrolytes
Hypokalemia

17-hydroxylase deficiency

MTBS2CK p.430

MTBS2CK p.430

21HydroxylaseDeficiency
Cholesterol
Aldosterone

CongenitalAdrenalHyperplasia
17hydroxylasedef 21hyroxylasedef 11hydroxylasedef

Cortisol

Aldosterone Cortisol Sexhormones HYPERtensive

Aldosterone Cortisol Sexhormones HYPOtensive


Saltwasting Shock

SexDevelopment p
Testosterone Estradiol
Girls:nml atbirth Boys:pseudo hermaphroditism

SexDevelopment
Girls:virilized Boys:nml atbirth

Electrolytes
Hypokalemia

Electrolytes
Hyponatremia Hypochloremia Hyperkalemia

21-hydroxylase deficiency
MTBS2CK p.430 MTBS2CK p.430

11HydroxylaseDeficiency
Cholesterol
11-DOC Aldosterone

CongenitalAdrenalHyperplasia
17hydroxylasedef 21hyroxylasedef 11hydroxylasedef

Cortisol

Aldosterone Cortisol Sexhormones HYPERtensive

Aldosterone Cortisol Sexhormones HYPOtensive


Saltwasting Shock

SexDevelopment p
Testosterone Estradiol
Girls:nml atbirth Boys:pseudo hermaphroditism

Aldosterone Cortisol Sexhormones 11DOC HYPERtensive


SexDevelopment
Girls:virilized Boys:nml atbirth

SexDevelopment
Girls:virilized Boys:nml atbirth

Electrolytes
Hypokalemia

Electrolytes
Hyponatremia Hypochloremia Hyperkalemia

11-hydroxylase deficiency

Fewelectrolytes abnormalities

MTBS2CK p.430

MTBS2CK p.430

18

CongenitalAdrenalHyperplasia
Cholesterol
Pregnenolone

Rickets Disorder of children Soft and weak bones fractures Vitamin D, calcium, and phosphate Children are particularly susceptible Rapidly growing bones Breast B t milk ilk d deficiency fi i i in vitamin it i D Prophylaxis with vitamin D supplements

17-OHg Pregnenolone

17-OHProgesterone

Deoxycortisol

Cortisol

DHEA Androstenediol

Androstenedione Testosterone

Estrone Estradiol

17-hydroxylase deficiency 21-hydroxylase deficiency

11-hydroxylase deficiency

Rickets

BoneMetabolism/Simplified
Vitamin D PTH

Calcium

Phosphate

BoneMetabolism/Simplified
Vitamin D PTH

Rickets

Calcium

Phosphate

Source:PaulMMichaud

MTBS2CK p.432

19

Rickets Disorder of children Soft and weak bones fractures Vitamin D, calcium, and phosphate Children are particularly susceptible Rapidly growing bones Breast B t milk ilk d deficiency fi i i in vitamin it i D Prophylaxis with vitamin D supplements Treat with vitamin D and calcium supplements

Pulmonary
Croup Epiglottitis pg WhoopingCough Asthma

MTBS2CK p.431

Croup
2-year-old brought in by daycare provider for severe cough, fever, and runny nose. The childs cough sounds like a bark and shes in obvious respiratory distress. Upon physical examination, the child refuses to lie flat. A chest X-ray shows a positive steeple sign. What is the most appropriate next step in management?

Anatomyofupperairwaydisease

a. b. c. d. e.

Intubate Racemic epinephrine Empiric antibiotics Acetaminophen CT-scan of neck

Not necessary in croup Not first step in improving airway Not first step in improving airway Not first step in improving airway

Croup
MTBS2CK p.435
commons.wikimedia.org. Used with permission

Croup
Laryngotracheitis or Laryngotracheobronchitis Infection of upper airway Subglottic space Viral Parainfluenza or RSV Presentation Triad: barking cough, coryza, and stridor Respiratory distress: accessory muscle use CXR: Steeple sign
MTBS2CK p.435

Epiglottitis
Treatment Moderate severity: Steroids Severe: Racemic epinephrine and steroids
Think bacterial causes: (1) older kids or (2) those unresponsive to racemic epinephrine

4-year-old child brought in by daycare provider because hes extremely irritable and refuses to eat. He refuses to lean back, speaks in muffled words, looks extremely ill, and is drooling. Chest radiograph shows a positive thumb-print sign. What is the most appropriate next step in management?

The MOST common cause of stridor in children

a. b. c. d. e.

Intubate Racemic epinephrine Empiric antibiotics Acetaminophen CT-scan of the neck

Only for croup Good not most immeidate Fever is not a concern Further imaging not warranted

MTBS2CK p.436

20

Anatomyofupperairwaydisease

Epiglottitis
Upper-airway infection and emergency MCC: Bacterial Non-vaccinated: H. influenzae type-B Vaccinated: Streptococcus species and nontypeable H. influenzae Presentation Fever, F drooling, d li respiratory i t di distress t NO coryza, NO prodrome, NO cough Management Transfer to O.R. INTUBATE Start empiric antibiotics: ceftriaxone or cefuroxime No imaging required
commons.wikimedia.org. Used with permission

Epiglottitis

Croup

MTBS2CK p.436

Epiglottitis

Pertussis
Bordetella pertussis Whooping Cough Gram-negative, non-invasive Causes ciliary paralysis Three Stages:

Source: MS-4 USU

MTBS2CK p.437

Pertussis/Stages
~ 14 days 14 30 days ~ 14 days

Pertussis
Bordetella pertussis Whooping Cough Gram-negative, noninvasive Causes ciliary paralysis Three Stages: Catarrhal Stage: 14 days Runny R nose, congestion, ti URI S Symptoms t Most contagious time period Paroxysmal Stage: 14-30 days Severe coughing, posttussive emesis Convalescent Stage: 14 days Resolution of symptoms
MTBS2CK p.437

Catarrhal Rhinorrhea, congestion, cold sxs sx s cold Most contagious time period Only time abx helpful to patient
MTBS2CK p.437

Paroxysmal Severe coughing Post-tussive emesis Usually no fever Low requirement for admission Abx prescribed to reduce transmissibility

Convalescent Prolonged resolution of symptoms Coughing fits remain less respiratory distress

21

Pertussis
Diagnosis This is a clinical diagnosis CXR is helpful PCR useful where available Management Patient: P ti t Primarily Pi il supportive ti Catarrhal stage azithromycin/erythromycin Everybody gets treated Close contacts: Macrolides The community: Vaccination campaigns
MTBS2CK p.437

ViralInfectionsofChildhood
Virus
Varicella

Presentation
Multiplehighlypruritic vesicularrash;startsonface; fever/malaise Cough,Coryza,and Conjunctivitis;Koplik spots FeverandURIsymptoms; l slapped dcheeks h k rash h

Diagnostictests Treatment
Initial:Tzanck smear Mostaccurate: culture

Supportive

Rubeola (Measles) Fifths d disease

Initial:Clinical Supportive Mostaccurate:IgM

Clinicaldiagnosis

Supportive

Roseola Mumps

FeverandURIprogressing Clinicaldiagnosis todiffuserash


Feverprecedesclassic parotidglandswellingwith possibleorchitis

Supportive Supportive

Clinicaldiagnosis

MTBS2CK p.434

Asthma
The most common chronic disease in children Pathophysiology Reversible obstruction Hyperresponsiveness Inflammation Remodeling Diagnosis Decreased FEV1 and No single available test FEV1/FVC ratio! Clinical Pulmonary Function Tests (PFTs) Bronchoprovocation CXR
MTBS2CK p.129130

Asthma
Treatment Avoidance of triggers Short-acting 2-agonists (SABA) Inhaled corticosteroids (ICS) Long-acting 2-agonists (LABA) Leukotriene L k ti antagonists t i t (modifiers) ( difi ) (LTRA)

MTBS2CK p.131132

Asthma/Treatment

SABA as needed

ADD Low-dose ICS

ADD LABA, LTRA or LTRA, move to mediumdose ICS

High-dose ICS and LABA

High-dose ICS and LABA and Oral steroids

Severity of Symptoms
MTBS2CK p.131132

22

ChildhoodDisorders Psychiatry
SamAsgarian,MD/MBA Classof2012 TulaneUniversity
MentalRetardation PervasiveDevelopmentalDisorders AttentionDeficitHyperactivityDisorder TouretteDisorder

MentalRetardation To determine the level of retardation patients must exhibit deficits in both Intellectual functioning
Cognitive abilities

TypesofMentalRetardation

Mild
IQ range
5055 to 70

Moderate
IQ range
3040 to 5055

Level of functioning
6th grade level Can work and live independently Needs help in difficult or stressful situations

Level of functioning
2nd grade level May work with supervision and support Needs help in mildly stressful situations

Social adaptive functioning


Perform daily activities

More frequent in boys Highest incidence being in school-age children

MTBS2CK p.503

MTBS2CK p.503

TypesofMentalRetardation

MentalRetardation/Treatment

Severe
IQ range
2025 to 3540

Profound
IQ range
< 20

Level of functioning
Little or no speech, y limited abilities to very manage self care

Level of functioning
Needs continuous p care and supervision

Treatment Genetic counseling, prenatal care, and safe environments for expectant mothers If due to medical condition (e.g., PKU) treat disorder Special education to improve level of functioning Behavioral therapy to reduce negative behaviors

MTBS2CK p.503

MTBS2CK p.503504

PervasiveDevelopmentalDisorders/Definition Characterized by Social, behavioral, and language problems


Occurs before age 3

AutisticDisorder
Incidence: boys > girls Lacks peer relationships, poor eye contact, and social smile Absent or bizarre speech Repetitive behaviors
Stacking Injurious behavior to self or others

Childhood Developmental Disorders Autistic disorder Rett disorder Childhood disintegrative disorder Asperger disorder

~18 mo boy with autism, obsessively stacking cans Source: Andwhatsnext , commons.wikimedia.org

MTBS2CK p.504

MTBS2CK p.504

AutisticTreatment Improve ability to develop relationships, attend school, and achieve independent living May benefit from behavioral modification programs If aggressive, use antipsychotic medications

Rettdisorder Greater incidence in girls Progressive encephalopathy Microcephaly Hand-wringing Loss of speech Ataxia At i Psychomotor retardation Treatment Symptomatic Behavior therapy for self-injurious behavior Physiotherapy for muscular dysfunction
MTBS2CK p.504

MTBS2CK p.504

ChildhoodDisintegrativeDisorder

ChildhoodDisintegrativeDisorder/Treatment

Greater incidence in boys Normal development for 2 years, followed by marked functional regression in
Loss of language Social interaction Motor function Bladder function

Improve ability to develop relationships, attend school, and achieve independent living May benefit from behavioral modification programs If aggressive, use antipsychotic medications

Repetitive/stereotyped behaviors

MTBS2CK p.504

MTBS2CK p.504

AspergerDisorder Greater incidence in boys Social and behavioral problems No language or intellectual deficits Preoccupied with rules
Often display intense interests. Source: Poindexter Propellerhead at the English language, commons.wikimedia.org

Gabriel is a healthy 2-year-old boy whose parents have taken him to the pediatrician. His problems started at 18 months of age, when he did not speak much. He does not have much attachment to his parents and seems aggressive toward other children. What is the most likely diagnosis?
a. Deafness
Ruled out Impaired judgment,

b. Schizophrenia, childhood onset behavior, and reality

Treatment Improve relationships with others


MTBS2CK p.504

c. Rett disorder speech, ataxia, and psychomotor d. Autism


retardation

Progressive encephalopathy, loss of

e. Learning decit No evidence of learning deficit


MTBS2CK p.504505

AttentionDeficitHyperactivityDisorder Characterized by Inattention Short attention span Or Hyperactivity that interferes with daily functioning in school, home, or work Must be present for > 6 months and usually appears before age 7 The symptoms may persist into adulthood
MTBS2CK p.505

ADHD Symptomsmustbepresentinatleast2areas,suchas: School Home


Interrupt others Fidget in chairs Run or climb excessively y Unable to engage in leisure activities Talk excessively Unable to pay attention Make careless mistakes in schoolwork Do not follow through with instructions Difficulties organizing tasks Easily distracted

Source: cdc.gov

MTBS2CK p.505

Treatment First-line: methylphenidate and dextroamphetamine


Side effects: insomnia, decreased appetite, and headache

DisruptiveBehavioralDisorders

Second-line: atomoxetine (norepinephrine reuptake inhibitor)

Oppositional Defiant Disorder Epidemiology Usually noted by age 8 Boys > girls before puberty
But equal incidence after puberty

On the USMLE Step 2 CK, atomoxetine is usually chosen over the first-line treatment, given the side effect proles of those treatments

MTBS2CK p.505

MTBS2CK p.506

DisruptiveBehavioralDisorders/ OppositionalDefiantDisorder
Features

DisruptiveBehavioralDisorders/ OppositionalDefiantDisorder

Argues often Loses temper Easily annoyed Blames others for their mistakes Tends to have problems with authority figures

Justifies behavior as response to others actions

Treatment Teach parents appropriate child management skills and how to lessen oppositional iti lb behavior h i

Source: Chris Willia, commons.wikimedia.org

MTBS2CK p.506

MTBS2CK p.506

DisruptiveBehavioralDisorders

Conduct Disorder Epidemiology Seen more frequently in


Boys whose parents have antisocial personality disorder and alcohol dependence

DisruptiveBehavioralDisorders/ ConductDisorder
Features Persistent behavior where rules are broken Behaviors include aggression toward others:
Bullying Cruelty to animals Fighting Or Using weapons

Source: Diego Grez, commons.wikimedia.org

Vandalize and destroy property; set fires Steal items from others or lie to obtain goods from others Violate rules (truancy, running away from home, breaking curfew)
MTBS2CK p.506 MTBS2CK p.506

DisruptiveBehavioralDisorders/ ConductDisorder

Treatment Behavioral intervention using rewards for prosocial and nonaggressive behavior If aggressive
Antipsychotic medications have been used

A 10-year-old boy is seen by the school counselor after teachers complained of his behavior. He frequently becomes angry toward others and loses his temper. His parents report that he refuses to comply with house rules, stays up past his bedtime, and frequently talks back to them. What is the most likely diagnosis? a. Conduct disorder Breaks rules of society or commits crimes b. Tourette disorder Multiple tics

c. Adjustment disorder Maladaptive reaction to identifiable stressor d. Oppositional Deant Disorder e. Learning disorder, not otherwise specied Needs evidence of a learning problem

MTBS2CK p.506

MTBS2CK p.506

TouretteDisorder

TouretteDisorder/Treatment

Characterized by onset of multiple tics


Lasting > 1 yr Occurs before age 18

Seen more frequently in


Boys Begins at age 7

Motor tics most commonly involve


Facial and neck muscles (e.g., head shaking and blinking)

Dopamine p antagonists g
Antipsychotic medications (e.g., risperidone)

The vocal tics include


Grunting, coughing, and throat clearing

MTBS2CK p.506

MTBS2CK p.507

MoodDisorders Psychiatry
SamAsgarian,MD/MBA Classof2012 TulaneUniversity
MajorDepression BipolarDisorder Dysthymia Cyclothymia AtypicalDepression SeasonalAffectiveDisorder Bereavement(Grief)

MajorDepression Mood disorders present with at least a 2-week course of symptoms thats a change from the previous level of functioning

MajorDepression Symptoms Depressed mood or anhedonia (absence of pleasure) And 4 others including
Depressed mood most of day Weight changes Sleep changes Psychomotor disturbances
MTBS2CK p.507

Fatigue Poor concentration Thoughts of death and worthlessness

Vincent van Gogh's 1890 painting Sorrowing old man ('At Eternity's Gate') Source: The Yorck Project, commons.wikimedia.org

MTBS2CK p.507

MajorDepression/DiagnosisandTreatment Diagnosis Rule out medical causes


Most common (hypothyroidism)

MajorDepression/Treatment
SSRIs Effective and relatively mild side effects Less toxic in overdose than other antidepressants If some improvement, but not full response Increase dose of SSRI Psychotherapy (e.g., cognitive therapy) proven to be effective Goal of cognitive therapy is Reduce depression by teaching patients to identify negative cognitions and develop positive ways of thinking

Most common neurological associations are


Parkinson disease and dementia

First-line treatment: SSRIs Fluoxetine Paroxetine Sertraline Citalopram Escitalopram


MTBS2CK p.507

MTBS2CK p.507

MajorDepression

ExceptionstoSSRIUse
Varietyofdepression SpecificalternativetoSSRIs Usedesvenlafaxine Approvedforboth depression&neuropathy Bupropionhasfewer sexual lside id effects ff and dless l weightgainthanSSRIs Alsousedasadjunct treatmentforSSRI inducedsexualsideeffects

SSRIs should NOT be taken with MAO inhibitors as they will cause a dramatic increase in serotonin.

Patientwithdepressionand neuropathicpain

USMLE Step 2 CK wont give you 2 SSRIs from which to choose

Patientwithdepressionwho isf i fearful f lof fweight i h gain i or sexualsideeffects

MTBS2CK p.507508

MTBS2CK p.507

MajorDepression
45-year-old woman was seen by her PCP due to complaints of depressed mood, lack of pleasure, sleep problems, decreased appetite and weight, decreased energy, and problems with concentration. She states that these symptoms started when she was fired from her job about 4 weeks ago, and that since then, she has been unable to function. What is the most indicated treatment at this time? a. Alprazolam Anxiolytic b. Paroxetine c. Bupropion Not 1st line d. Venlafaxine When initial therapy doesnt e. Trazodone work, or depression more f. Electroconvulsive therapy severe and associated with psychotic features

Choices on USMLE may include an SSRI and another antidepressant medication Pick the cleanest: SSRI

MTBS2CK p.508

MTBS2CK p.508

BipolarDisorder

BipolarDisorder

Mood disorder Patient experiences


Manic symptoms that last at least 1 week Cause significant distress in level of functioning
Source: commons.wikimedia.org

Manicsymptoms
Elevated mood Increased self-esteem Distractibility Pressured speech Decreased need for sleep Increase in goaldirected activity Racing thoughts Excessive involvement pleasurable in p activities

Source: commons.wikimedia.org

Source: commons.wikimedia.org

MTBS2CK p.508

MTBS2CK p.508

BipolarDisorder

BipolarDisorder The difference between mania and hypomania Severity of symptoms Level of functioning Duration Mania M i
> 1 week Affect functioning Warrant hospitalization

Typically starts with depression Diagnosis Exclude drug use Cocaine/amphetamine Obtain history and urine drug screen

Hypomania H i
< 1week Dont severely affect functioning Dont warrant hospitalization

MTBS2CK p.508

MTBS2CK p.508

TypesofBipolarDisorders
BipolardisordertypeI BipolardisordertypeII Maniaanddepression Hypomaniaanddepression 21-year-old college student was taken to the university clinic after she was noted to be acting bizarrely in class. She is talking fast and reported that she has not slept for over 4 days. She appears to be giggling and not paying attention in class. Her roommate reported that she has been drinking alcohol excessively over the last few days and has had many sexual contacts with unknown men. Wh t is What i th the most t lik likely l di diagnosis? i ? a. Alcohol-induced mood disorder No history of alcoholism

b. c. d. e.

MTBS2CK p.509

Bipolar disorder type I Bipolar disorder type II Hypomania Major depression with psychosis Thoughts of death, preoccupation with Cyclothymia Hypomanic worthlessness, episodes and psychomotor mild depression MTBS2CK p.509 retardation, psychosis

BipolarDisorder/Treatment You must distinguish whether Acute mania 1st line


Lithium Valproic acid Atypical antipsychotics

BipolarDisorder

If kidneys are compromised, dont use lithium

Bipolar depression

Severe symptoms, consider

Lithium Lamotrigine

Lithi Lithium i is th the correct t answer t to most t bipolar questions

Atypical antipsychotics
Shorter onset of action
MTBS2CK p.509 MTBS2CK p.509

Dysthymia
33-year-old man was taken to emergency room by police after neighbors complained about his behavior. His family informed the doctor hes been diagnosed with bipolar disorder and was recently started on lithium. While in the emergency room, he became combative and punched a nurse on the mouth. What is the next step in the management of this patient? a. Obtain lithium level Symptoms are acute b. Admit to psychiatric unit > important to treat c. Refer to psychiatry Never refer 1st line is an antipsychotic d. Add valproic acid e. Olanzapine

Characterized by Depressed mood that lasts most of the day and is present almost continuously Symptoms must be present for: > 2 years Treatment
Antidepressant medications and psychotherapy
MTBS2CK p.510

MTBS2CK p.509510

Cyclothymia

AtypicalDepression

Characterized by Hypomanic episodes and mild depression Symptoms must be present for: > 2 years Treatment
Lithium, valproic acid, or carbamazepine Psychotherapy

Characterized by

Reverse vegetative changes Increased sleep Increased weight Increased appetite Mood tends to be worse in evenings and patients may complain of extremities feeling heavy
MTBS2CK p.510

MTBS2CK p.510

AtypicalDepression/Treatment

SeasonalAffectiveDisorder Characterized by Seasonal changes in mood during fall and winter Symptoms
Weight gain Increased sleep Lethargy

SSRIs or MAOIs SSRIs have better side-effect profile If MAOIs and SSRIs are in the same question, choose SSRIs because of sideeffect profile Usually MAOIs are answer on Step 2 for atypical depression

Treat with
Phototherapy and bupropion

MTBS2CK p.510

MTBS2CK p.510

PostpartumDisorders
Disorder Onset Symptoms Mothersfeelings towardbaby Treatment Postpartumbluesorbabyblues Immediatelyafterbirthupto2weeks Sadness,labilemood,tearfulness Nonegativefeelings Supportive,usuallyselflimited

PostpartumDisorders
Disorder Onset Symptoms Mothersfeelings towardbaby Treatment Postpartumdepression Within13monthsafterbirth Depressedmood,weightchanges,sleep disturbances,andexcessiveanxiety Mayhavenegativefeelingstowardbaby Antidepressantmedications

Source: commons.wikimedia.org

MTBS2CK p.511

MTBS2CK p.511

Source: commons.wikimedia.org

PostpartumDisorders
Disorder Onset Symptoms Postpartumpsychosis Within23weeksafterbirth Depression,delusions,andthoughts ofharm Mothersfeelingstowardbaby Mayhavethoughtsofharmingbaby Treatment Antipsychoticmedication,lithium, andpossiblyantidepressants

Bereavement(Grief)
Bereavement Majordepression (greaterseveritythanbereavement)

Beginsafterdeathof lovedone Feelingsof Sadness Worrying y g Irritability Sleepdifficulties Poorconcentration Tearfulness <6months,butcango onlonger
MTBS2CK p.511

Thoughtsofdeath Morbidpreoccupationwith worthlessness Markedpsychomotor retardation Psychosis Prolongedfunctional impairment >2monthsandadversely affectfunctioning

MTBS2CK p.511

Source: commons.wikimedia.org

Bereavement(Grief) Treatment
Supportive psychotherapy
A 65-year-old man brought to office by daughter. He has been hopeless and helpless since his wife died 3 months ago. Daughter is worried about his isolative behavior and lack of appetite. He lost over 30 pounds. He doesnt seem interested in getting better and believes he should have died with his wife. What is the most likely diagnosis? a. Bereavement Severe symptoms just to be bereavement b. Dysthymia Depressed mood, lasts most of the day and continuously c. Major depression Anxiety, depression or d. Adjustment disorder disturbances of conduct

Medical therapy is wrong answer

Source: Bundesarchiv, Bild 146-1996-036-01 / Unknown / CC-BY-SA, commons.wikimedia.org

e. Bipolar disorder
MTBS2CK p.511 MTBS2CK p.511

Mood disorder with manic symptoms, at least 1 week

Antidepressants,MoodStabilizers, ElectroconvulsiveTherapy
Typeofmedication Adverseeffects

Antidepressants,MoodStabilizers, ElectroconvulsiveTherapy
Typeofmedication Adverseeffects

Tricyclic antidepressants
Amitriptyline Nortriptyline Imipramine

Hypotension Drymouth Constipation Confusion Arrhythmias Sexualsideeffects Weightgain GIdisturbances

Monoamine oxidaseinhibitors
- Phenelzine - Isocarboxazid - Tranylcypromine

Monitordiet,giventhat foodrichintyraminewill producehypertension


Safe foods include white wine and processed cheese Unsafe foods include red wine, aged cheese, and chocolate

MTBS2CK p.512

MTBS2CK p.512

Source: Vincent van Gogh, Still life with bottle, two glasses, cheese and bread. commons.wikimedia.org

Antidepressants,MoodStabilizers, ElectroconvulsiveTherapy
Typeofmedication Adverseeffects

Antidepressants,MoodStabilizers, ElectroconvulsiveTherapy
Typeofmedication Adverseeffects

Serotoninselective Headaches reuptakeinhibitors Weightchanges - Fluoxetine Sexualsideeffects - Paroxetine GIdisturbances


Sertraline Citalopram Escitalopram Fluvoxamine

Serotoninnorepinephrine reuptakeinhibitors - Venlafaxine - Duloxetine - Desvenlafaxine

Hypertension Blurryvision Weightchanges Sexualsideeffects GIdisturbances

MTBS2CK p.512

MTBS2CK p.512

10

Antidepressants,MoodStabilizers, ElectroconvulsiveTherapy
Typeofmedication Adverseeffects

Antidepressants,MoodStabilizers, ElectroconvulsiveTherapy
Typeofmedication Adverseeffects

Others
Bupropion Trazodone Mirtazapine

Bupropion:seizures Trazodone:priapism Mirtazapine:weightgain andsedation

Lithium

Tremors,weightgain,GI disturbance,nephrotoxic, teratogenic,leukocytosis, diabetesinsipidus Severetoxicitygives confusion,ataxia,lethargy, andabnormalreflexes

MTBS2CK p.512

MTBS2CK p.512

Antidepressants,MoodStabilizers, ElectroconvulsiveTherapy
Typeofmedication Adverseeffects

Antidepressants,MoodStabilizers, ElectroconvulsiveTherapy
Typeofmedication Lamotrigine Electroconvulsive therapy Adverseeffects StevensJohnsonsyndrome Headaches,transientmemoryloss

Valproicacid

Tremors,weightgain,GI disturbances,alopecia, teratogenic,hepatotoxic Mustmonitorlevels;toxicity causes


Hyponatremia Coma Death

MTBS2CK p.512

MTBS2CK p.512

Source: commons.wikimedia.org

SerotoninSyndrome
What is the single most effective treatment for depression? a. Electroconvulsive therapy b. Fluoxetine Equally efficacious, but the SSRIs c. Venlafaxine are used more frequently due to side-effect profiles d. Imipramine

Potentially life-threatening From use of SSRIs, often with interactions between drugs, overdose, or recreational use of drugs that are serotonergic in origin Symptoms Cognitive effects
Agitation, confusion, hallucinations, hypomania

e. Phenelzine

MTBS2CK p.513

MTBS2CK p.513

11

SerotoninSyndrome Autonomic effects


Sweating, hyperthermia, tachycardia, nausea, diarrhea, shivering

PsychoticDisorders
Schizophrenia DelusionalDisorder

Somatic effects
Tremors, myoclonus

Treatment Stop SSRI medication Symptomatic treatment of fever, diarrhea, hypertension Cyproheptadine (serotonin antagonist)
MTBS2CK p.513

ClassificationofPsychoticDisorders

ClassificationofPsychoticDisorders

BriefPsychoticDisorder
Duration of symptoms > 1 day, but < 1 month Symptoms Delusions Hallucinations Disorganized speech Grossly disorganized Catatonic behavior Treatment Antipsychotic medication

SchizophreniformDisorders
Duration of symptoms > 1 month, but < 6 months Symptoms Delusions Hallucinations Disorganized speech Grossly disorganized Catatonic behavior
MTBS2CK p.513

Negative symptoms
Flat affect Poor grooming Social withdrawal

Treatment Antipsychotic medication

MTBS2CK p.513

ClassificationofPsychoticDisorders

PsychoticDisorders Duration of symptoms distinguishes brief psychosis, schizophreniform, and schizophrenia If no time is mentioned, always choose schizophrenia as the correct answer to the What is the most likely y diagnosis? g question q

Schizophrenia
Duration of symptoms > 6 months Symptoms Delusions Hallucinations Disorganized speech Grossly disorganized Catatonic behavior Negative symptoms
MTBS2CK p.513

Severely affects level of functioning Treatment Antipsychotic medication

MTBS2CK p.514

12

Schizophrenia/Definition

Schizophrenia/Definition

Thought disorder: impairs judgment, behavior, and ability to interpret reality At least 6 months and must affect functioning Incidence: men = women
Affects men at young age

Urine drug screen


Cocaine Amphetamine
A schizophrenic patient at the Glore Psychiatric Museum made this piece of cloth and it gives us a peek into her mind. Source: Cometstarmoon, commons.wikimedia.org

MTBS2CK p.514

TypesofSchizophrenia/Diagnosticcriteria Paranoid Delusions or hallucinations, mostly persecutory or grandiose type Most common type of schizophrenia Later age of onset Catatonic Psychomotor disturbances from retardation to excitation
Stupor, rigidity, excitement, or posturing

TypesofSchizophrenia/Diagnosticcriteria

Disorganized Marked regression to disinhibited behavior with little contact with reality Typically appear disheveled and have bizarre emotional responses Worst prognosis and earliest age of onset

Mutism is common

MTBS2CK p.514

MTBS2CK p.514

TypesofSchizophrenia/Diagnosticcriteria Residual Lack of positive symptoms


Hallucinations Delusion

Schizophrenia/Treatment

Hospitalize: acutely psychotic Ensure patient safety, administer atypical antipsychotic such as
Risperidone, olanzapine, quetiapine, ziprasidone, aripiprazole, or paliperidone

Presence of negative symptoms


Flat affect Poor grooming Social withdrawal

In emergency situation where intramuscular medication is needed consider


Olanzapine or ziprazidone Haloperidol may be used, but has more side effects
MTBS2CK p.514515

Undifferentiated Characterized by not meeting criteria for other types


MTBS2CK p.514

13

Schizophrenia/Treatment

Schizophrenia/Treatment

If noncompliant with medication


Long-acting antipsychotic
Risperidone: first-line

Haloperidol may be used, more side effects

Clozapine Use when NO response to a trial of typical


or atypical antipsychotics
Never used as first-line

Know the differences in the side effect profiles of the atypical antipsychotics Its common to have 2 appear on the test; you need to pick the best based on side effects

MTBS2CK p.515

MTBS2CK p.515

AdverseEffectsof AtypicalAntipsychoticMedications
Olanzapine Greater incidence of diabetes and weight gain; avoid in diabetic & obese Risperidone Greater incidence of movement disorders Quetiapine Less incidence of movement disorders

22-year-old woman with schizophrenia. Shes 30 pounds overweight and suffers from type 2 DM. She is concerned about her medications and asks for advice. Which of the following would be most indicated in this patient?

Ziprasidone Increased risk of prolongation of QT interval; avoid in conduction defects


MTBS2CK p.515

Clozapine High risk of agranulocytosis; monitor CBC on regular basis; never used as firstline treatment given sideeffect profile

a. b. c. d. e.

Aripiprazole Olanzapine Highest risk of metabolic abnormalities Quetiapine Have medium risk Clozapine Risperidone

MTBS2CK p.515

ManagementofAdverseEffectsof AtypicalAntipsychoticMedications
Onset of symptoms Acute dystonia Hours to days Symptoms Muscle spasms
Torticollis Laryngeal spasms Occulogyric crisis

Acutedystonia
Treatment
Benztropine Trihexyphenidyl Diphenhydramine

Muscle spasms

Akathisia

Weeks

Generalized restlessness Pacing Rocking Inability to relax

Reduce dose Beta blockers Switch to atypical


Source: James Heilman, MD, commons.wikimedia.org

MTBS2CK p.516

14

ManagementofAdverseEffectsof AtypicalAntipsychoticMedications
Onset of symptoms Symptoms Abnormal involuntary movements of
Head Limb Trunk Perioral, most common Muscular rigidity Fever Autonomic changes Agitation Obtundation

Treatment

Tardive dyskinesia

Rare before 6 months

Switch to atypical antipsychotic Clozapine has least risk

Neuroleptic malignant syndrome

Not time limited

Dantrolene or bromocriptine

23-year-old man recently diagnosed with schizophrenia is started on haloperidol. Within a few hours he develops muscle stiffness. His eyes roll upward and he cannot move them down. What is the most likely diagnosis? a Tardive dyskinesia 6 mo. involuntary, perioral movements a. No time, muscular b. Neuroleptic malignant syndrome rigidity, fever Weeks. Generalized restlessness, c. Akathisia pacing, rocking, inability to relax d. Serotonin syndrome SSRIs e. Acute dystonic reaction
MTBS2CK p.516

MTBS2CK p.516

DelusionalDisorder Characterized by Non-bizarre delusions for > 1 month and NO impairment in level of functioning The p patient may y believe the country is about to be invaded, but he or she still obeys the law, goes to work, and pays bills
Source: commons.wikimedia.org

DelusionalDisorder

Hallucinations arent present Treatment


Atypical antipsychotic: first-line therapy Psychotherapy to help promote reality testing

MTBS2CK p.516

MTBS2CK p.516

PanicDisorder/Definition Is the experience of intense anxiety along with feelings of dread and doom

AnxietyDisorders
PanicDisorder Phobias

Source: commons.wikimedia.org

MTBS2CK p.517

15

PanicDisorder/Definition

PanicDisorder/Definition

Thisisaccompaniedbyatleast4symptoms ofautonomichyperactivitysuchas
Diaphoresis Trembling Chest pain Fear of dying Chills Palpitations SOB Nausea Dizziness Dissociative symptoms Paresthesias

Last < 30 minutes and may be accompanied by agoraphobia


Fear of places where escape is felt to be difficult

Typically in women, can occur at any time, and usually has no specific stressor R/O thyroid disease, hypoglycemia, and cardiac disease

MTBS2CK p.517

MTBS2CK p.517

PanicDisorder/Treatment SSRIs, typically


Fluoxetine, paroxetine, and sertraline are indicated for this disorder

Which is the first-line treatment for panic disorder?

Along with SSRIs, patients may benefit from benzodiazepines (e.g., alprazolam)
Begin g with both Then taper the benzodiazepine (potential abuse)

Behavioral and individual therapy


Helpful But Not sufficient as only treatment
MTBS2CK p.517

a. b. c c. d. e.

Alprazolam Panic attack Buspirone Generalized anxiety Sertraline Imipramine Major depression, enuresis Fluvoxamine Not 1st-line

MTBS2CK p.517

PhobiasDefinition

TypesofPhobias

Fear of an object or situation and the need to avoid it

Specific phobia
Fear of an object (e.g., animals, heights, or cars)

Social phobia Fear of a situation


Public restrooms Eating in public Public speaking

May be learned, involves 2 main types

Situations where something potentially embarrassing may happen


Source: Kilarin, commons.wikimedia.org

Source: Alex.Ramek, commons.wikimedia.org

MTBS2CK p.517518

MTBS2CK p.518

16

Phobias/DiagnosisandTreatment

Phobias/Treatment Relaxation techniques such as


Breathing Or Guided imagery
Beta blockers (e (e.g., g atenolol or propanolol) are used for performance anxiety such as stage fright. They are given 30 minutes to 1 hour before performance.

Diagnosis History of
Anxiety symptoms in specific situations Or When in contact with feared objects

Treatment Behavioral modification techniques such as


Systematic desensitization:
Expose individuals to their feared objects moving from least most anxiety-provoking
MTBS2CK p.518

MTBS2CK p.518

40-year-old man referred to psychiatrist because he is too shy. He has problems going to parties, feels anxious about getting close to others, and stays at home in fear that others would laugh at him. When confronted by others, he develops severe anxiety as well as hyperventilation and increased sweating. Which is the most likely diagnosis?

ObsessiveCompulsive Disorder

a. Panic disorder autonomic hyperactivity symptoms b. Social anxiety Chronic worrying about things c. Generalized anxiety disorder that dont merit concern d. Specific phobia Fear of an object e. Acute stress disorder Stressor, 2 days to 1 month, relive
the event
MTBS2CK p.518

Intense anxiety, dread and doom, 4

ObsessiveCompulsiveDisorder(OCD)/ Definition

DifferencebetweenObsessionsand Compulsions

Obsessions alone Or, most commonly Combination of obsessions and compulsions l i Typically affect the individuals level of functioning
MTBS2CK p.518

Obsessions
Thoughts that are intrusive, senseless, and distressing, thus increasing anxiety Includes fear of contamination

Compulsions
Rituals such as counting and checking done to neutralize thoughts Time consuming and tends to lower anxiety

Source: Lars Klintwall Malmqvist, commons.wikimedia.org

MTBS2CK p.519

Source: commons.wikimedia.org

17

OCD/DiagnosisandTreatment
Diagnosis More frequent in young patients Incidence: men = women OCD can coexist with Tourette disorder Treatment SSRIs: treatment of choice Most common, first-line: fluoxetine, paroxetine, sertraline, citalopram, or fluvoxamine Main behavioral therapy used Exposure and response prevention

PosttraumaticStressDisorder andAcuteStressDisorder

MTBS2CK p.519

PosttraumaticStressDisorder(PTSD)and AcuteStressDisorder/Definition In both, individuals have Usually overwhelming been exposed to a (e.g., war, rape, stressor to which they hurricanes, or react with fear and earthquakes)
helplessness Patients continually relive li the h event and d avoid anything that reminds them of event

PTSDandAcuteStressDisorder/Definition

Affect the patients level of functioning Other symptoms Increased startle response Hypervigilance Sleep disturbances Anger outbursts Concentration difficulties

Source: Department of Defense visual information (DVIC) Author: US Army, Office of War Information (OWI)

MTBS2CK p.519

MTBS2CK p.519

DifferencebetweenPTSDand AcuteStressDisorder

PTSDandAcuteStressDisorder/Diagnosis

PTSD Symptoms last for > 1 month

Acutestressdisorder Symptoms last for > 2 days and a maximum of 1 month th They occur within 1 month of traumatic event

Main feature
Determine the time period when traumatic events occurred in relationship to symptoms

Rule out
Depression and substance abuse Both worsen diagnosis

MTBS2CK p.519

MTBS2CK p.519520

18

PTSDandAcuteStressDisorder/Treatment

First-line treatment
Paroxetine and sertraline

Relaxation techniques and hypnosis, proven helpful Psychotherapy after traumatic events will allow
Development of coping techniques and acceptance of event

35-year-old woman with palpitations, dizziness, and increased sweating for 8 months. Has visited numerous physicians, none have been helpful. Her husband is concerned because she cannot relax and worries about everything. She worries about her parents health even though they are healthy. She worries about her finances, although her husband assures her theyre financially secure. Wh t is What i th the most t lik likely l di diagnosis? i ? a. Generalized anxiety disorder b. Phobias Fear of an object or a situation Intense anxiety, dread and doom, 4 autonomic c. Panic disorder hyperactivity symptoms d. Adjustment disorder Maladaptive reaction to a stressor e. Social anxiety Anxiety to social interactions, situations
MTBS2CK p.520

MTBS2CK p.520

GeneralizedAnxietyDisorder/Definition Excessive anxiety and worry about most things, lasting > 6 months Typically, anxiety is out of proportion to event Accompanied by
Fatigue Concentration difficulties Sleep problems Muscle tension Restlessness

GeneralizedAnxietyDisorder

Usually women, who complain of feeling anxious as long as they can remember
MTBS2CK p.520

GeneralizedAnxietyDisorder/Treatment

SSRIs
Fluoxetine, paroxetine, sertraline, or citalopram

AntianxietyMedicationsand TheirAdverseEffects

Antianxietymedication
Benzodiazepines Diazepam Lorazepam Clonazepam Alprazolam Oxazepam Chlordiazepoxide Temazepam Flurazepam Buspirone

Adverseeffects
Sedation Confusion Memory deficits Respiratory depression And Addiction potential

Venlafaxine and buspirone have also been used Psychotherapy and behavioral therapy
Beneficial Not considered first-line

Headaches Nausea Dizziness

MTBS2CK p.520

MTBS2CK p.521

19

AntianxietyMedicationsand TheirSpecificIndications

AntianxietyMedicationsand TheirSpecificIndications

Lorazepam Used frequently in emergency situations because it can be given intramuscularly Clonazepam May be used if addiction is a concern concern, longer half-life Chlordiazepoxide, oxazepam Used frequently in treatment of alcohol withdrawal
MTBS2CK p.521

Alprazolam Used frequently in panic disorder Flurazepam, temazepam, Flurazepam temazepam triazolam Approved as hypnotics (rarely used)

MTBS2CK p.521

GeneralizedAnxietyDisorder
Flumazenil is a benzodiazepine antagonist used only when The overdose is acute And: Youre certain that theres no chronic dependence

SubstanceRelatedDisorders

Flumazenil can cause seizures in benzodiazepinedependent patients. It causes acute withdrawal: tremor or seizures similar to delirium tremens (alcohol withdrawal).

MTBS2CK p.521

SpecificSubstanceAbuse/Definition Intoxication Reversible experience - substance leads to either psychological or physiological changes Withdrawal Cessation or reduction of a substance leading to either psychological or physiological changes

SubstanceRelatedDisorders/Definition

Abuse Maladaptive pattern - substances that leads to


Engaging in hazardous situations Legal problems Inability to fulfill obligations Continued use despite adverse consequences

MTBS2CK p.521

Source: commons.wikimedia.org

MTBS2CK p.521

20

SubstanceRelatedDisorders/Definition Dependence Maladaptive pattern, substances leads to


Tolerance

IntoxicationandWithdrawal/Alcohol
Alcohol Intoxication
Signs & Symptoms

Withdrawal
Source: GeographBot, commons.wikimedia.org

Withdrawal when trying to cut down Patients spend a great deal of time engaging in drug use Continued use despite adverse consequences
Source: Carlos t . commons.wikimedia.org

Talkative Sullen Gregarious Moody Disinhibited

Tremors Hallucinations Seizures Delirium tremens

Treatment

Mechanical ventilation, if severe


MTBS2CK p.522

MTBS2CK p.521

Benzodiazepines Thiamine Multivitamins Folic acid

IntoxicationandWithdrawal/ Amphetamines&Cocaine
Intoxication Amphetamines & Cocaine Withdrawal
Signs & Symptoms Signs & Symptoms

IntoxicationandWithdrawal/Cannabis
Cannabis Intoxication
Signs & Symptoms

Withdrawal
Signs & Symptoms

Euphoria Hypervigilance Autonomic hyperactivity Weight loss Pupillary dilatation Perceptual disturbances
Treatment

Anxiety Tremulousness Headache Increased Appetite Depression Risk of suicide


Treatment

Impaired motor coordination Slowed sense of time Social withdrawal Increased appetite Conjunctival injection
Treatment

None
Treatment

Source: JonRichfield, commons.wikimedia.org

None

Antipsychotics and/or benzodiazepines and/or antihypertensives


MTBS2CK p.522

Bupropion and/or bromocriptine

None
MTBS2CK p.522

IntoxicationandWithdrawal/Hallucinogens
Hallucinogens Intoxication
Signs & Symptoms

IntoxicationandWithdrawal/Inhalants Hallucinogens Intoxication


Signs & Symptoms

Withdrawal
Signs & Symptoms

Withdrawal
Signs & Symptoms

Ideas of reference Perceptual disturbances Impaired judgment Tremors Incoordination Dissociative symptoms
Treatment

None
Treatment
Source: Ksd5 Ksd5, commons commons.wikimedia.org wikimedia org

None

Belligerence Apathy Aggression Impaired judgment Stupor Coma


Treatment

None
Treatment

None

Antipsychotics and/or benzodiazepines and/or talking down


MTBS2CK p.522

Antipsychotics
MTBS2CK p.522

21

IntoxicationandWithdrawal/Opiates
Opiates Intoxication
Signs & Symptoms

IntoxicationandWithdrawal/ Phencyclidine(PCP)
Intoxication
Signs & Symptoms

Withdrawal
Signs & Symptoms

PCP

Apathy Dysphoria Pupillary constriction Drowsiness Slurred speech Coma Death


Treatment

Fever Chills Lacrimation Abdominal cramps Muscle spasms Diarrhea


Treatment

Source: Eleassar, commons.wikimedia.org

Belligerence Psychomotor agitation Violence Nystagmus HTN Seizures


Treatment

Withdrawal
Signs & Symptoms
Source: Jara172, commons.wikimedia.org

None
Treatment

Naloxone
MTBS2CK p.522

Clonidine Methadone Or Buprenorphine

Antipsychotics and/or benzodiazepines and/ or talking down


MTBS2CK p.522

None

IntoxicationandWithdrawal/AnabolicSteroids
Anabolic Steroids Intoxication
Signs & Symptoms

SubstanceRelatedDisorders/Treatment
If you suspect someone is an alcoholic, do CAGE test. Two positive responses to four questions are considered a positive test and indicate that further assessment is warranted.

Withdrawal
Signs & Symptoms

Irritability Aggression Mania Psychosis


Treatment

Depression Headaches Anxiety Increased concern over bodys physical state


Treatment

Source: Cristian navas roman roman, commons.wikimedia.org

Antipsychotics

C - Have you ever felt you should cut down on your drinking? A - Have people annoyed you by criticizing your drinking? G - Have you ever felt bad or guilty about your drinking? E - Eye opener: Have you ever had a drink first thing in the morning to steady your nerves or to get rid of a hangover?

SSRIs
MTBS2CK p.522 MTBS2CK p.523

SubstanceRelatedDisorders/Treatment
Detoxification
Usually 5-10 days Mostly in hospital settings to assure safe detoxification 65-year-old engineer suffered femur fracture and some cuts and bruises after being involved in an MVA. Hes admitted to the medicine floor and started on oxycodone. The day after admission, he appears confused with observable tremors in both extremities. He becomes concerned about bugs on the walls in his room and asks for your help. What is the most likely explanation for his symptoms? a. Brain concussion Typically after injury b. Alcohol withdrawal c. Oxycodone intoxication Constipation, miosis
behavior >1 day but <1 month, delusions,

Rehabilitation
Usually 28 days or more with a focus on relapse prevention techniques

Alcoholics Anonymous Narcotics Anonymous Pharmacologic treatments often include:


Disulfram (acetaldehyde dehydrogenase inhibitor) Naltrexone (opioid receptor antagonist) Acamprosate
MTBS2CK p.523

d. Brief psychotic disorder hallucinations, disorganized speech & e. Schizophrenia >6 months, delusions, hallucinations, disorganized
MTBS2CK p.523

speech & behavior

22

SomatoformDisorders Characterized by
Physical symptoms with no medical explanation

SomatoformDisorder, FactitiousDisorder,and Malingering

Symptoms severe enough to adversely affect level of functioning More frequent in young women Usually has psychological component, which the patient is unaware Treatment of choice: psychotherapy
Source of symptoms is psychological

MTBS2CK p.523

TypesofSomatoformDisorders/ Diagnosticcriteria Somatization disorder Hypochondriasis Patients must have at Patients believe that least they have some specific disease 4 Pain despite constant 2 GI reassurance 1 Se Sexual al
And 1 Pseudoneurological symptom

TypesofSomatoformDisorders/ Diagnosticcriteria

Conversion Affects voluntary motor or sensory functions, indicative of a medical condition Usually caused by psychological factors Associated with la belle indifference
Unconcerned about impairment

MTBS2CK p.524

MTBS2CK p.524

TypesofSomatoformDisorders/ Diagnosticcriteria Body Dysmorphic Disorder Patients believe that some part of the body is abnormal, defective, or misshapen

Pain Disorder Main complaint: presence of pain And must have Psychological f t factors associated i t d with pain

35-year-old married mother of 3 has frequent complaints of dizziness, nausea, diarrhea, vomiting, pain during intercourse, paresthesias, leg pain, stomach pain, food intolerance, and headaches. She has tried numerous medications, but none have been beneficial. Neurological examination: normal. What is the next step in the management of this patient? a. Lorazepam Anxiety disorder b. Sertraline Fibromyalgia & depression c. Individual psychotherapy d. Lithium Bipolar disorder e. Risperidone Psychosis

MTBS2CK p.524

Source: elovedfreak, commons.wikimedia.org

MTBS2CK p.524

23

FactitiousDisorder

FactitiousDisorder Inflict life-threatening injuries on themselves Behavior may be compulsive at times Formerly known as Mnchausen syndrome Factitious disorder by proxy, caretaker fakes signs and symptoms in another person
Usually a child child, in order to assume the sick role by proxy as the caregiver

Individual fakes an illness to get attention and emotional support in patient role Either psychological or physical illness Psychological symptoms
Hallucinations, delusions, depression, and bizarre behavior

Physical symptoms
Abdominal pain, fever, nausea, vomiting, or hematomas

MTBS2CK p.524

MTBS2CK p.525

FactitiousDisorder Typically, women with a history of being employed in healthcare Men more often have physical symptoms Patients ultimate goal: admission to hospital Always exclude any medical disorder with similar symptoms
MTBS2CK p.525

FactitiousDisorder/Treatment

No specific therapy has been proven to be effective When a child is involved in factitious disorder by proxy
Contact child protective services to ensure childs safety
Factitious disorder cannot be diagnosed without first ruling out medical illness
MTBS2CK p.525

Malingering Characterized by Conscious production of signs and symptoms for an obvious gain, such as
Avoiding work Evading criminal prosecution Or Achieving financial gain

Malingering/Diagnosis

Diagnosis More frequently in prisoners and military personnel Typically diagnosed when theres a discrepancy between
Patients complaints And Actual physical or laboratory findings

Not a mental illness

MTBS2CK p.525

MTBS2CK p.525

24

Malingering

If medical evaluation reveals malingering Confront patient with the outcome

AdjustmentDisorder

A lack of cooperation from patients is characteristic of malingering

MTBS2CK p.525

AdjustmentDisorder
Characterized by Maladaptive reaction to an identifiable stressor
Loss of job Divorce Or Failure in school

AdjustmentDisorder/Treatment
Symptoms
Anxiety Depression Disturbances of conduct

Severe enough to cause impairment in functioning

Treatment of choice Psychotherapy Both individual and group therapy have been used effectively

Usually occur within 3 months of stressor and must remit within 6 months of removal of stressor
MTBS2CK p.525 MTBS2CK p.525

PersonalityDisorders
Characterized by Personality patterns that are:
Pervasive Inflexible Maladaptive
Cl t A Cluster Cl t B Cluster Cl t C Cluster

PersonalityDisorders

Paranoid Schizoid Schizotypal

Histrionic Antisocial Borderline Narcissistic

Avoidant Dependent Obsessive compulsive

MTBS2CK p.525526

25

TypesofPersonalityDisorders
Paranoid Schizoid Schizotypal

TypesofPersonalityDisorders
Histrionic

Suspicious Mistrustful Secretive Isolated A d And Questioning loyalty of family & friends

Choice of solitary activities Lack of close friends Emotional E i l coldness No desire for or enjoyment of close relationships

Ideas of reference Magical thinking Odd thinking thi ki Eccentric behavior Increased social anxiety Brief psychotic episodes
MTBS2CK p.526

Must be center of attention Inappropriate sexual behavior Self-dramatization Use physical appearance to draw attention to self

MTBS2CK p.526

TypesofPersonalityDisorders
Antisocial Failure to conform to social rules Deceitful Lack of remorse Impulsive I l i Aggressive towards others Irresponsible Must be 18+ Borderline Unstable relationships Impulsive Recurrent suicidal behaviors Chronic Ch i f feelings li of emptiness Inappropriate anger Dissociative symptoms when severely stressed Brief psychotic episodes Narcissistic Grandiose sense of self Belief that they are special Lack empathy Sense S of f entitlement Require excessive admiration

TypesofPersonalityDisorders
Avoidant Dependent Obsessive compulsive Preoccupied with details Rigid y Orderly Perfectionists Excessively devoted to work Inflexible

Unwilling to get involved with people Views self as socially inept Reluctant to take risks Feelings of inadequacy
MTBS2CK p.526

Difficulty making day-to-day decisions Unable to assume responsibility Unable to express disagreement Fear of being alone Seeks relationship as source of care

MTBS2CK p.526

PersonalityDisorders/Treatment

Individual psychotherapy Medications if mood or anxiety symptoms are present

Which of the following personality disorders has been associated with positive psychotic symptoms? a. Borderline & Schizotypal b Histrionic b. Not c. Schizoid associated d. Paranoid e. Antisocial

MTBS2CK p.526

MTBS2CK p.527

26

EatingDisorders
AnorexiaNervosa BulimiaNervosa EatingDisorderNot OtherwiseSpecified

15-year-old girl brought to clinic by her mother, who found her vomiting in the bathroom. She vomits daily after each meal and exercises excessively. She has numerous calluses on her hands as well as cavities. She is 55 and weighs 90 pounds. What is her most likely diagnosis? a. Bulimia nervosa Frequent binge eating, normal weight, obesity history Neither b. Anorexia nervosa anorexia c. Eating disorder not otherwise specified nor bulimia d. Obesity Increased weight e. Atypical depression Increased apetite & weight

MTBS2CK p.527

AnorexiaNervosa
Characterized by
Failure to maintain a normal body weight Fear and preoccupation of gaining weight Body image disturbance

AnorexiaNervosa/Diagnosis
Lose weight by
Strict caloric control Excessive exercise Purging
Laxative & diuretic

Source: http://www.womenshealth.gov/bodyimage/eating-disorders/

And Fasting F ti

Bradycardia Lanugo hair And Edema EKG changes


Potassium deficiency y

Unrealistic self-evaluation as overweight Amenorrhea is common from low body weight Deny their emaciated condition Great concern with appearance and frequently examine and weigh themselves
MTBS2CK p.527

More frequently, teenage girls 14 -18 years of age Severe weight loss Hypotension

Arrhythmia MCC of death

MTBS2CK p.527

AnorexiaNervosa/Treatment

BulimiaNervosa
Characterized by

Hospitalization to prevent
Dehydration Starvation Electrolyte imbalances Death

Frequent binge eating in a discrete amount of time Lack of control of overeating episodes

Accompanied by compensatory behavior to prevent weight gain


Purging
Laxatives & diuretics

Psychotherapy Behavioral therapy SSRIs


To promote weight gain
Source: girlshealth.gov

Fasting And Excessive exercise

Self-evaluation influenced by body shape and weight

MTBS2CK p.528

MTBS2CK p.528

27

BulimiaNervosa Diagnosis More frequently women and later in adolescence than anorexia nervosa Most normal weight, but history of obesity Treatment Doesnt require hospitalization, unless
Severe electrolyte abnormality

EatingDisorderNotOtherwiseSpecified

Doesnt meet criteria for either anorexia nervosa or bulimia nervosa Examples
Criteria for anorexia present in girls, but menstruation is normal Anorexic patient with normal weight Use of compensatory behavior after eating normal amounts of food

Psychotherapy SSRIs
MTBS2CK p.528

MTBS2CK p.528

Narcolepsy Characterized by
Excessive daytime sleepiness And Abnormalities of REM sleep

SleepDisorders
Narcolepsy Insomnia

Most frequently begins in young adulthood Sleep studies are usually indicated in diagnosis
MTBS2CK p.528

Source:http://www.cdc.gov/sleep/

Narcolepsy

No curative therapy Forced daytime naps Modafinil


To maintain alertness

Psychiatric&PhysicalSymptoms ofNarcolepsy(SleepDisorder)
Sleep attacks Specific feature of narcolepsy Cataplexy Episodic irresistible sleepiness And Feeling refreshed upon awakening Sudden muscle tone loss: pathognomonic And May be precipitated by loud noise or emotions Sleep paralysis

Hypnogogic and hypnopompic hallucinations

MTBS2CK p.528529

MTBS2CK p.529

Hallucinations occur as patient is going to sleep and waking up

Awake but unable to move Typically upon awakening

28

Insomnia Characterized by
Inability to initiate or maintain sleep

Insomnia Sleep hygiene techniques


Going to bed and waking up at the same time Avoiding caffeinated beverages And Avoiding daytime naps

May be due to anxiety and depression g to affect Severe enough level of functioning Typically women who complain of feeling tired or have increased appetite and yawning
MTBS2CK p.529
Source: nih.gov

Behavioral modification techniques include


Using the bed only for sleeping (not for reading, watching TV, or eating)

Medical therapy
Zolpidem, eszopiclone, or zaleplon

MTBS2CK p.529

TerminologyofHumanSexuality Sexual identity Based on a persons secondary sexual characteristics Gender identity Based on a persons sense of maleness or femaleness, established by age 3 Gender role Based on external patterns of behavior that reflect inner sense of gender identity Sexual orientation Based on persons choice of love object; may be heterosexual, homosexual, bisexual, or asexual

HumanSexuality

MTBS2CK p.529

HumanSexuality Masturbation Normal precursor of object-related sexual behavior All men and women masturbate Problematic if it interferes with daily functioning Homosexuality H lit Not considered a mental illness unless it is egodystonic (person not happy with sexual orientation) May be considered normal experimentation in teenagers

TypesofSexualDysfunction
Impotence Persistent or recurrent inability to attain or maintain an erection until completion of sexual se ua act
Treatment

Premature ejaculation Ejaculation before penetration or just after penetration, usually due to anxiety
Treatment

R/O medical causes or medication Psychotherapy, couples sexual therapy

Psychotherapy, behavioral modification techniques (stop and go, squeeze), SSRI medication

MTBS2CK p.530

MTBS2CK p.530

29

TypesofSexualDysfunction
Dyspareunia Pain associated with sexual intercourse, not diagnosed if due to medical condition
Treatment

Paraphilias
Vaginismus Prolonged and painful contraction or spasm of the vagina Usually severe enough to prevent intercourse
Treatment

Group of disorders that are recurrent, sexually arousing, and seen more frequently in men Usually focus on humiliation, nonconsenting partners, or use of nonliving objects Must occur
> 6 months and cause distress Affect level of functioning

Psychotherapy

Psychotherapy Dilator therapy

Dont diagnose if experimentation

MTBS2CK p.530

MTBS2CK p.530

TypesofParaphilias Exhibitionism Recurrent urge to expose oneself to strangers Pedophilia Recurrent urges or arousal toward prepubescent children Masochism Recurrent urge or behavior involving the act of humiliation

TypesofParaphilias

Sadism
Recurrent urge or behavior involving acts in which physical or psychological suffering of victim is exciting

Transvestic fetishism
Fetishism Recurrent use of nonliving objects to achieve sexual pleasure
MTBS2CK p.531

Recurrent urge or behavior involving cross dressing for sexual gratification; usually in heterosexual males

MTBS2CK p.531

TypesofParaphilias Frotteurism Rubbing ones pelvis or erect penis against a nonconsenting person for sexual gratification

Paraphilias/Treatment

Individual psychotherapy Behavioral modification techniques


Aversive conditioning

Antiandrogens or SSRIs to reduce sexual drive

MTBS2CK p.531

Source: Dschwen, commons.wikimedia.org

MTBS2CK p.531

30

GenderIdentityDisorder Characterized by
Persistent discomfort and sense of inappropriateness regarding the patients assigned sex

GenderIdentityDisorder Patients will take hormones when older to deepen voice, if female, or soften voice, if male Women may bind their breasts and men may hide their penis and testicles Its seen more frequently in young men Treatment Sexual reassignment surgery if approved Individual psychotherapy

Diagnosis Wearing g opposite pp g genders clothes Using toys assigned to opposite sex Play with opposite-sex children when young And Feeling unhappy about ones own sexual assignment
MTBS2CK p.531

MTBS2CK p.531

Suicide Recent suicide attempt Complaints of suicidal thoughts Admission of suicidal thoughts Demonstration of suicidal behaviors
Buying weapons Giving away possessions Writing a will
MTBS2CK p.532
Source: samhsa.gov

Suicide

Source: samhsa.gov

Suicide Risk Factors Men Older adults Social isolation Presence of psychiatric illness/drug abuse Perceived hopelessness Previous attempts Treatment Hospitalize patient Take all threats seriously

MTBS2CK p.532

31

Asthma/Definition Asthma or reactive airway disease Abnormal bronchoconstriction of airways Characterized by Reversible airway obstruction secondary to bronchial smooth muscle hyperactivity Airway inflammation, mucus plugging, and smooth muscle hypertrophy
Can lead to chronic, irreversible airway obstruction

Asthma
StephenBagley,MD ResidentPhysician InternalMedicine HospitaloftheUniversityofPennsylvania

MTBS2CK p.129

Asthma/Etiology

Asthma/Etiology Causes acute exacerbations of symptoms Allergens such as pollen, dust mites, cockroaches, and cat dander Infection Changes in weather (especially cold air) Exercise Catamenial (related to menstrual cycle) Aspirin, NSAIDs, beta blockers, tobacco smoke Gastroesophageal reflux disease (GERD)

Extremely common Etiology unknown Associated with atopic disorders and obesity Asthma prevalence, incidence, and hospitalization rates are increasing
MTBS2CK p.129

MTBS2CK p.129

Asthma/Presentation Most often presents with


Wheezing Acute onset of SOB Cough And Chest tightness

Asthma/Presentation

Whichofthefollowingismostlikelytobefound inanasthmaticpatient?
Symptoms worse at night Nasal polyps Eczema or atopic physical y dermatitis on p examination Increased length of expiratory phase of respiration
I/E ratio decreases (normal is 1:2)
MTBS2CK p.129130

Increased sputum production common Fever is often not present Remember all that wheezes is not asthma!
MTBS2CK p.129

Increased use of accessory respiratory muscles (e.g., intercostals) Hyperresonance to percussion, pulsus paradoxus

Asthma/Presentation

Asthma/DiagnosticTests

The answer to the best initial test question in asthma is based on the severity of presentation

The best initial test in an acute exacerbation Arterial blood gas (ABG) or peak expiratory flow (PEF)
ABG if mild, mild early earl e exacerbation: acerbation mild hypoxia, respiratory alkalosis ABG if severe, late exacerbation: severe hypoxia, respiratory acidosis

MTBS2CK p.130

MTBS2CK p.130

Asthma/DiagnosticTests

Asthma/DiagnosticTests

Chest X-ray
Most often normal in asthma, but may show hyperinflation Useful for excluding other disease processes
Pneumonia, CHF, pneumothorax

Most accurate diagnostic test is pulmonary function testing (PFTs) You must understand lung volumes!

MTBS2CK p.130

MTBS2CK p.130

Asthma/DiagnosticTests

Asthma/DiagnosticTests PFTs in asthma show OBSTRUCTION: FEV1 and FVC with a FEV1/FVC ratio HYPERINFLATION: total lung capacity (TLC) AIR TRAPPING: in residual volume REVERSIBILITY: in FEV1 > 12% with use of albuterol BRONCHIAL HYPERRESONIVENESS: FEV1 > 20% with use of methacholine

MTBS2CK p.131

Asthma/DiagnosticTests
15-year-old boy with occasional SOB every few weeks. Currently feels well. No medications and denies any other medical problems. Pulse is 70 and RR is 12. Chest examination is normal. Which is the single most accurate diagnostic test at this time? Acute exacerbation a. Peak expiratory flow Less likely in asymptomatic b. Increase in FEV1 with albuterol Asymptomatic c. Diffusion capacity of carbon monoxide d. > 20% decrease in FEV1 with use of methacholine e. Increased alveolar-arterial oxygen difference Asymptomatic (A-a gradient) f. Increase in FVC with albuterol Less likely in asymptomatic g. Flow-volume loop on spirometry Best for fixed obstructions h. Chest CT scan Shows nothing or hyperinflation i. Increased pCO2 on ABG Acute exacerbation
MTBS2CK p.130

Acetylcholine and histamine provoke bronchoconstriction and increase in bronchial secretions Methacholine is artificial form of acetylcholine used in diagnostic testing

MTBS2CK p.131

Asthma/DiagnosticTests

Asthma/Treatment

Additional testing options include CBC may show an increased eosinophil count Skin testing to identify specific allergens that provoke bronchoconstriction Increased IgE levels suggest allergic etiology
Also seen in allergic bronchopulmonary aspergillosis

Chronic management Stepwise fashion of progressively adding more types of treatment if no response

MTBS2CK p.131

MTBS2CK p.131

Asthma/ChronicManagement
Step 1 Step 2 Step 3 Step 4

Asthma/Treatment
Moderate persistent: daily, or > 1 night/week Add a longg beta acting agonist (LABA) to a SABA & ICS Severe persistent: continual, frequent Increase d dose of f ICS to maximum in addition to LABA & SABA

Mild intermittent: < 2 days/week < 2 nights/ month Inhaled shortacting ti beta b t agonist SABA Albuterol Levalbuterol

Mild persistent: > 2 days/week > 2 nights/month

Adverse effects of inhaled steroids are dysphonia and oral candidiasis Alternate long-term control agents include
C Cromolyn l and d nedocromil: d il i inhibit hibit mast t cell ll mediator di t release useful for exercise Theophylline: phosphodiesterase inhibitor increasing cAMP levels (cardio- and neurotoxicity) Leukotriene modifiers: montelukast, zafirlukast, or zileuton best with atopic patients
MTBS2CK p.131

Add a long-term control agent: Low-dose inhaled corticosteroids (ICS) for daily use Beclomethasone Budesonide Flunisolide Fluticasone Mometasone Triamcinolone

Or Increase dose of ICS LABA Salmeterol Or Formoterol

MTBS2CK p.131132

Asthma/Treatment

Asthma/DiagnosticTests

Zafirlukast is hepatotoxic and has been associated with Churg-Strauss syndrome

Management of Acute Exacerbation

Step#1: determine severity of asthma exacerbation, quantified by Decreased peak expiratory flow (PEF) ABG with evidence of increased A-a Aa gradient or CO2 retention

MTBS2CK p.131

MTBS2CK p.133

Asthma/DiagnosticTests

Asthma/Treatment Management of Acute Exacerbation Oxygen Albuterol (often nebulized to ensure adequate delivery to lungs) +/- inhaled anticholinergic (e.g., ipratropium) Corticosteroids: need 4-6 4 6 hours to begin to work work, so give right away Epinephrine injections are no more effective than albuterol and have more adverse systemic effects

PEF is an approximation of FVC No precise normal value Based predominantly on height and age, not weight PEF is used in acute assessment by y seeing g how much difference theres from patients usual PEF when stable

MTBS2CK p.133

MTBS2CK p.133

Asthma/Treatment Anticholinergics Role of ipratropium and tiotropium in asthma management unclear In general, ipratropium should be used, but doesnt work as rapidly as albuterol Epinephrine Rarely used and only as a drug of last resort

Asthma/Treatment Magnesium Some modest effect in bronchodilation Not as effective as albuterol, ipratropium, or steroids, but it does help Magnesium g helps p bronchospasm. p Magnesium is used only in acute, severe asthma exacerbation not responsive to several rounds of albuterol while waiting for steroids to take effect.

MTBS2CK p.132133

MTBS2CK p.133

Asthma/Treatment Adverse Effects of Chronic Systemic Corticosteroids Should be a last resort because of very harsh adverse effects such as Osteoporosis Cataracts Adrenal suppression and fat redistribution Hyperlipidemia, hyperglycemia, acne, and hirsutism (particularly in women) Thinning of skin, striae, and easy bruising

Asthma/Treatment

High-dose inhaled steroids rarely lead to adverse effects associated with prednisone

MTBS2CK p.132

MTBS2CK p.132

Asthma/Treatment 47-year-old man with history of asthma comes to ED with several days of increasing SOB, cough, and sputum production. RR is 34. He has diffuse expiratory wheezing and prolonged expiratory phase. Which would you use as the best indication of the severity of his asthma? a. Respiratory rate b. Use of accessory muscles Subjective c. Pulse oximetry Hypoxia, until imminent respiratory failure d. Pulmonary function testing Short of breath e. Pulse rate Adds nothing

The following are not effective in acute exacerbations Theophylline Cromolyn and nedocromil (best with ith extrinsic t i i allergies ll i lik like h hay fever) Leukotriene modifiers Omalizumab Salmeterol
MTBS2CK p.133

MTBS2CK p.132.4

Asthma/Treatment

If no response to oxygen, albuterol, and steroids or develops respiratory acidosis (increased pCO2)
Consider endotracheal intubation and mechanical ventilation

ChronicObstructive PulmonaryDisease

MTBS2CK p.133

COPD/Definition

COPD/Definition

Characterized by airflow obstruction secondary to emphysema and/or chronic bronchitis Emphysema


Terminal airway destruction resulting in decreased elastic recoil of lungs
Normally, elastin fibers allow passive exhalation
MTBS2CK p.134

Chronic bronchitis
Productive cough for > 3 months/year for 2 consecutive years

Both
Part of COPD spectrum Results in decrease in FEV1 and FVC with increase in TLC

MTBS2CK p.134

COPD/Etiology

COPD/Presentation

Tobacco smoking Leads to almost all COPD Destroys elastin fibers Young and Y d a nonsmoker k , answer alpha-1 antitrypsin deficiency as most likely cause

SOB worsened by exertion Intermittent exacerbations with cough, sputum, and SOB often brought on by infection Barrel chest from air trapping Muscle wasting and cachexia
COPD now thought to have a component of systemic inflammation

MTBS2CK p.134

MTBS2CK p.134

COPD/DiagnosticTests

COPD/DiagnosticTests

Best initial test: chest X-ray Increased anteriorposterior (AP) di diameter t Air trapping and flattened diaphragms

Most accurate diagnostic test: PFT Decreased FEV1, FVC, and FEV1/FVC ratio (< 70%) Increased TLC due to RV Decreased DLCO in emphysema p y (destruction of alveolar septae where capillaries are found) Incomplete improvement with albuterol Little or no worsening with methacholine
Source: James Heilman, MD, commons.wikimedia.org

MTBS2CK p.134

MTBS2CK p.134

COPD/DiagnosticTests Reversibility with Inhaled Bronchodilators COPD generally associated with IRREVERSIBLE airway obstruction But COPD has a broad response to inhaled bronchodilators (e (e.g., g albuterol) Ranges from no reversibility to complete reversibility Reversibility in response to bronchodilators is: > 12% increase and 200 mL increase in FEV1
MTBS2CK p.134

COPD/DiagnosticTests

Arterial blood gas (ABG) Acute exacerbations of COPD are associated with pCO2 and hypoxia
Respiratory acidosis may be present if theres there s insufficient metabolic compensation and bicarbonate level will be elevated to compensate In between exacerbations, not all those with COPD will retain CO2
MTBS2CK p.134

COPD/DiagnosticTests

COPD/Treatment Improves Mortality and Delays Progression of Disease: only 2 interventions! Smoking cessation Oxygen therapy for hypoxia
PaO2 <55 or SaO2 <90% PaO2 <60 or SaO2 <90%
If patient also has pulmonary hypertension, cor pulmonale or polycythemia

EKG Right atrial hypertrophy and right ventricular hypertrophy A-fib or multifocal atrial tachycardia (MAT) Echocardiography Right atrial and right ventricular hypertrophy Pulmonary hypertension

Mortality benefit from oxygen is directly proportional to the number of hours that oxygen is used

MTBS2CK p.135

MTBS2CK p.135

COPD/Treatment Definitely Improves Symptoms (But Does Not Decrease Disease Progression) Short-acting beta agonists (e.g., albuterol) Anticholinergic agents: tiotropium, ipratropium Inhaled steroids Long-acting beta agonists (e.g., salmeterol) Pulmonary rehabilitation

COPD/Treatment Asthmatics not controlled with albuterol Inhaled steroid COPD not controlled with albuterol Anticholinergic (e.g., tiotropium) Inhaled steroid Inhaled anticholinergic agents are most effective in COPD

MTBS2CK p.135

MTBS2CK p.135

COPD/Treatment Possibly Improves Symptoms Theophylline Lung volume reduction surgery No Benefit Cromolyn C l Leukotriene modifiers (e.g., montelukast) When all medical therapy is insufficient, the answer is refer for transplantation
MTBS2CK p.135

COPDAcuteExacerbations/Treatment

Combination of bronchodilators and corticosteroid therapy

COPD exacerbation treatment is identical to asthma exacerbation treatment, but with less proven benefit

MTBS2CK p.136

COPDAcuteExacerbations/Treatment

Antibiotics For exacerbation of moderate to severe COPD


Defined as requiring hospitalization or having at least two of the three cardinal symptoms of a COPD flare Dyspnea Sputum production Sputum purulence

ChronicBronchitisAcuteExacerbations/ Treatment Antibiotics Although viruses cause 20-50% of episodes, coverage should be provided against Streptococcus pneumoniae, H. influenzae, and Moraxella catarrhalis
Macrolides: azithromycin, clarithromycin Cephalosporins: cefuroxime, cefixime Amoxicillin/clavulanic acid Quinolones: levofloxacin, moxifloxacin

Second-line Agents Doxycycline, TMP/SMX


MTBS2CK p.136

MTBS2CK p.136

COPDAcuteExacerbations/Treatment Criteria for Oxygen Use in COPD Although the hypoxic drive elimination concept is incorrect Avoid reflexively placing a patient with COPD on very high-flow 100% nonrebreather mask Use only as much oxygen necessary to raise pO2 above 90% saturation The idea of eliminating hypoxic drive is not accurate. Dyspneic, hypoxic patients with COPD must get oxygen
MTBS2CK p.136

Bronchiectasis & AssociatedConditions

Bronchiectasis/Definition Uncommon disease from chronic destruction, remodeling, and dilation of the large bronchi Permanent anatomic abnormality that cannot be reversed or cured

Bronchiectasis/Etiology

Single MCC Cystic fibrosis: 50% of cases Other causes are Infections
TB ( (and non-tuberculous mycobacterium) y ) Pneumonia (staph or repeated aspiration)

Panhypogammaglobulinemia and immune deficiency

MTBS2CK p.136

Source: Yale Rosen, commons.wikimedia.org

MTBS2CK p.137

Bronchiectasis/Etiology

Bronchiectasis/DiagnosticTests

Foreign body or tumors Allergic bronchopulmonary aspergillosis (ABPA) Collagen vascular such as RA Immotile cilia syndrome y ( (Kartagener g syndrome) These conditions result in repeated and persistent lung infections
MTBS2CK p.137

Best initial test Chest X-ray shows


Dilated, thickened bronchi, sometimes with tram-tracks, which is thickening of bronchi

Most accurate test High-resolution CT, general rule of thumb


Sizable airways are larger in diameter than corresponding adjacent bronchial arteries

MTBS2CK p.137

Bronchiectasis/DiagnosticTests

Bronchiectasis/DiagnosticTests

Impossible to diagnose bronchiectasis without imaging study such as CT

Sputum culture
Only way to determine specific bacterial etiology

MTBS2CK p.137

Source: Thomas P. Eberle

MTBS2CK p.137

Bronchiectasis/Treatment

1. Chest physiotherapy (cupping and clapping) and postural drainage


Essential for dislodging plugged-up bronchi

AllergicBronchopulmonaryAspergillosis (ABPA)

2. Treat each episode of infection as it arises


Same antibiotics as for COPD exacerbations Only difference is inhaled antibiotics seem to have some efficacy

Hypersensitivity to fungal antigens that colonize bronchial tree Almost exclusively with asthma and atopic disorders Look for
Asthmatic with recurrent episodes of brown-flecked sputum and transient infiltrates on X-ray

3. Surgical resection
Focal lesions
MTBS2CK p.138

Cough, wheezing, hemoptysis, bronchiectasis


MTBS2CK p.138

AllergicBronchopulmonaryAspergillosis (ABPA)/DiagnosticTests

AllergicBronchopulmonaryAspergillosis (ABPA)/Treatment

Peripheral eosinophilia Skin test reactivity to aspergillus antigens Precipitating p g antibodies to aspergillus p g on blood test Elevated serum IgE levels Pulmonary infiltrates on chest X-ray or CT
MTBS2CK p.138

1. Oral steroids (prednisone) for severe cases


Inhaled steroids arent effective for ABPA

2. Itraconazole or voriconazole orally for recurrent episodes i d Inhaler cant deliver high enough dose steroids to be effective in ABPA

MTBS2CK p.138

CysticFibrosis(CF)/Etiology Autosomal recessive from mutation that codes for chloride transport
Known as CF transmembrane conductance regulator (CFTR)

CysticFibrosis/Etiology Leads to abnormally thick mucus in


General Growth failure (malabsorption) Vitamin deficiency states Vitamins A,D,E,K Nose & Sinuses Nasal polyps Sinusitis Liver Hepatic steatosis Portal hypertension Gallbladder Biliary cirrhosis Neonatal obstructive jaundice Cholelithiasis Lungs Bronchiectasis Bronchitis Broncholitis Pneumonia Atelectasis Hemoptysis Pneumothorax Reactive airway disease Cor pulmonale Respiratory failure Mucoid impaction of the bronchi Allergic bronchopulmonary aspergillosis

Mutations in CFTR g gene damage g chloride and water transport across apical surface of epithelial cells in exocrine glands throughout the body

MTBS2CK p.138139

Source: Mucoviscidose.PNG: Mirmillon, commons.wikimedia.org

Heart Right ventricular hypertrophy Pulmonary artery dilation


Source: Maen K Abu Househ, commons .wikimedia.org

MTBS2CK p.139

10

CysticFibrosis/Etiology
Bone Hypertrophic osteoarthropathy Clubbing Arthritis Osteoporosis Spleen Hypersplenism Stomach GERD Pancreas Pancreatitis Insulin I li d deficiency fi i Symptomatic hyperglycemia Diabetes Reproductive Infertility Aspermia, absence of vas deferens Amenorrhea Delayed puberty
Source: Maen K Abu Househ, commons .wikimedia.org

CysticFibrosis/Etiology Damaged mucus clearance decreases ability to get rid of inhaled bacteria

Intestines Meconium ileus Meconium peritonitis Rectal prolapse Intussusception Volvulus Fibrosing colonopathy (strictures) Appendicitis Intestinal atresia Distal intestinal obstruction syndrome Inguinal hernia MTBS2CK p.139

Neutrophils in CF dump tons of DNA into airway secretions clogging them up

MTBS2CK p.139

CysticFibrosis/Etiology

CysticFibrosis/Presentation Over 1/3 CF patients are adults Look for


Young adult with chronic lung disease
Cough Sputum py Hemoptysis Bronchiectasis Wheezing Dyspnea

And Recurrent episodes of infection

Sinus pain and polyps are common


Source: nih.gov

MTBS2CK p.139

CysticFibrosis/Presentation

CysticFibrosis/Presentation Recurrent pancreatitis Distal intestinal obstruction Biliary y cirrhosis

GastrointestinalInvolvement
Meconium ileus: infants with abdominal distention Pancreatic insufficiency (in 90%) with steatorrhea and vitamin A, D, E, and K malabsorption

Genitourinary Involvement Men are often infertile


95% have azoospermia, vas deferens missing in 20%

Women are infertile because


Chronic lung disease alters menstrual cycle And Thick cervical mucus blocks sperm entry
MTBS2CK p.139

Islets spared, beta cell function is normal until much later in life

MTBS2CK p.139

11

CysticFibrosis/DiagnosticTests Most accurate test: increased sweat chloride test Pilocarpine increases acetylcholine levels increases sweat production
Chloride levels in sweat > 60 mEq/L in CF on repeated testing establishes diagnosis

CysticFibrosis/DiagnosticTests

Sputum Culture
Nontypable Haemophilus influenzae Pseudomonas aeruginosa Staphylococcus aureus Burkholderia cepacia

Genotyping, not as accurate as increased sweat chloride level


Because there are so many different types of mutations leading to CF

MTBS2CK p.139

MTBS2CK p.140

CysticFibrosis/Treatment
1. Antibiotics are routine
Eliminating colonization: difficult Sputum culture: essential to guide therapy Inhaled aminoglycosides: almost exclusively limited to CF

CysticFibrosis/Treatment Gross hemoptysis Rigid bronchoscopy


Unsuccessful

2. Inhaled recombinant human deoxyribonuclease ( hDN (rhDNase) )


Breaks down massive amounts of DNA in respiratory mucus that clogs up airways

Embolization Interventional radiology Place patient bad lung-side down


Helps prevent bleeding into healthier lung, which is providing most of oxygen exchange

3. Inhaled bronchodilators
Albuterol

4. Lung transplantation
MTBS2CK p.140

CommunityAcquiredPneumonia/Definition

Pneumonia Part1

Community-acquired pneumonia (CAP)


Pneumonia occurring before hospitalization Or Within 48 hours of hospital admission

MTBS2CK p.140

12

CAP/Definition

CAP/Etiology
Common Pathogens in CAP & Their Associations
Haemophilus influenzae COPD Klebsiella pneumoniae Alcoholism Diabetes Legionella Contaminated water sources Air conditioning Ventilation systems Coxiella burnetii Animals at time of giving birth Veterinarians Farmers

Most common infectious cause of death in U.S. MCC: Streptococcus pneumoniae

Mycoplasma pneumoniae Young Healthy patients

Staphylococcus aureus Recent viral infection (influenza)

Anaerobes

Poor dentition Aspiration

Chlamydophila pneumoniae

Chlamydia psittaci Birds

Hoarseness MTBS2CK p.140 MTBS2CK p.141

CAP/Presentation All forms of pneumonia present with Fever and cough Severe infection Associated with dyspnea Distinguished by abnormalities of
Vital signs (tachycardia, hypotension, tachypnea) Or Mental status

CAP/Presentation Abdominal pain or diarrhea can occur with infection in lower lobes irritating intestines through diaphragm
Legionella is particularly known for causing diarrhea

Chills or rigors are sometimes a sign of bacteremia


S. pneumo would be most common

Dyspnea, high fever, and abnormal chest X-ray are main ways to distinguish pneumonia from bronchitis
MTBS2CK p.141

Chest pain (often pleuritic) occurs from inflammation of pleura Hypothermia is as bad as fever in terms of pathologic significance
MTBS2CK p.141

CAP/Presentation

CAP/Presentation Organism-specific Associations on Presentation

USMLE S2 may play abnormal breath sounds as part of multimedia and ask you to recognize them Dullness to percussion if theres effusion Bronchial breath sounds and egophony consolidation of air spaces
MTBS2CK p.141

Klebsiella pneumoniae Hemoptysis from necrotizing disease disease, currant jelly sputum

Mycoplasma pneumoniae Dry cough, rarely severe bullous severe, myringitis

Pneumocystis AIDS with <200 CD4 cells

Legionella Anaerobes Foul-smelling sputum, rotten eggs


MTBS2CK p.141

Gastrointestinal symptoms Abdominal pain, diarrhea, or CNS symptoms Headache & confusion

13

CAP/Presentation

CAP/Presentation

Dry or non-productive cough Mycoplasma Viruses Coxiella Pneumocystis Chlamydia

Preferentially involve interstitial space Air spaces of alveoli empty


Thats why theres less sputum production! Specific sputum colors are useless in determining etiology
MTBS2CK p.142

MTBS2CK p.142

CAP/DiagnosticTests

CAP/DiagnosticTests

Best initial test for all respiratory infections


Chest X-ray: but cant determine specific etiology

Atypical pneumonia Organism not visible on Gram stain and not culturable on standard blood agar
Mycoplasma Chlamydophila Legionella Coxiella And Viruses
MTBS2CK p.142

Sputum Gram stain and sputum culture


Best ways to first determine specific etiology Many organisms wont be detected
MTBS2CK p.142

30-50% of cases of CAP

CAP/DiagnosticTests

CAP/DiagnosticTests

Chest X-ray Bilateral interstitial Nonproductive infiltrates with cough Mycoplasma X-rays lag behind Viruses clinical findings
Coxiella Pneumocystis Chlamydia
Right middle lobe infiltrate characteristic of bacterial pneumonia. Source: Nirav Thakar, MD

1st chest X-ray can be falsely negative in 10-20%

MTBS2CK p.142

MTBS2CK p.143

14

CAP/DiagnosticTests

CAP/DiagnosticTests

Blood Cultures are positive 5-15% of CAP Particularly with S. pneumoniae! Sputum Gram stain is adequate if: > 25 WBCs and < 10 epithelial cells

MTBS2CK p.143

Interstitial infiltrates leave the air space empty. This chest x-ray can be consistent with PCP, Mycoplasma, viruses, and Chlamydia. Source: Craig Thurm, MD

MTBS2CK p.143

CAP/DiagnosticTests

CAP/DiagnosticTests Tests Done in Severe Disease with an Unclear Etiology, or Those Not Responding to Treatment Thoracentesis
Any new large effusion should be analyzed Empyema
Infected pleural effusion Acts like abscess Only improves if drained with chest tube
MTBS2CK p.143

Chest CT and MRI show Greater definition of abnormalities found on chest X-ray But Wont W t determine d t i specific ifi microbiologic i bi l i etiology In infectious diseases radiologic test is never the most accurate test
MTBS2CK p.143

Source: nih.gov

CAP/DiagnosticTests Etiology of pleural effusions Transudative

CAP/DiagnosticTests

Empyema
LDH > 60% of serum OR Protein > 50% of serum pH < 7.2, 7 2 +gram stain stain, +culture +culture, or frank pus

Exudative

Secondary to Increased PCWP Or Decreased intravascular oncotic pressure

Secondar to Secondary Increased vascular permeability

New, large effusions secondary to pneumonia should be tapped


MTBS2CK p.143

15

CAP/DiagnosticTests

CAP/DiagnosticTests S. pneumo can be tested via urine antigen


Specific Diagnostic Tests by Organism Mycoplasma pneumoniae Pneumocystis jiroveci (PCP) Bronchoalveolar lavage (BAL) Legionella Rising serologic titers Urine antigen, Culture on charcoalyeast extract

Bronchoscopy
Only needed if sputum stain and culture and blood cultures dont yield an organism and patients condition is worsening despite empiric therapy Exception is pneumocystis pneumonia

PCR Cold agglutinins Serology Special culture media


MTBS2CK p.144

Chlamydophila pneumoniae, Chlamydia psittaci & Coxiella burnetii

MTBS2CK p.143144

CAP/Treatment

CAP/Treatment

Rare to have specific organism identified at time treatment is initiated If case describes organism on Gram stain treatment is directed towards that organism Most important step is determining severity of disease.
Determines location in which to place patient
MTBS2CK p.144

Severity of disease, not the etiology drives initial therapy

Mycoplasma & Chlamydophila, rarely confirmed because they are simply treated empirically

MTBS2CK p.144

CAP/Treatment
Outpatient Treatment Previously healthy or no antibiotics in past 3 months and mild symptoms Comorbidities or antibiotics in past 3 months

CAP/Treatment Inpatient Treatment Respiratory fluoroquinolone: levofloxacin or moxifloxacin Or Ceftriaxone and azithromycin

Macrolide - Azithromycin or clarithromycin Or Doxycycline


MTBS2CK p.144

Respiratory fluoroquinolone Levofloxacin Or Moxifloxacin

Almost all infectious diseases are initially treated empiricallythat is, without a specific etiology

MTBS2CK p.144

16

CAP/Treatment Reasons to Hospitalize 80% safely treated outpatient with oral antibiotics Severe disease is defined as a combination of
Hypotension (systolic < 90 mmHg) Respiratory rate > 30/min pO2 < 60 mmHg, pH < 7.35 Elevated BUN > 30 mg/dL, Sodium <130 mmol/L
MTBS2CK p.145

CAP/Treatment

Glucose >250 mg/dL Pulse > 125/min Confusion Temperature > 104F 65 or comorbidities such as cancer, COPD, CHF, renal failure, or liver disease

Hypoxia and hypotension as single factors are a reason to hospitalize a patient

Chest X-ray does not guide admission cannot tell severity of hypoxia

MTBS2CK p.145

CAP

CURB 65 Confusion Uremia Respiratory Distress BP low

65-year-old woman is hospitalized with CAP. Sputum Gram stain: Gram-positive diplococci. Sputum culture doesnt grow a specific organism. Chest X-ray: lobar infiltrate and large effusion. Shes placed on ceftriaxone and azithromycin. Thoracentesis: marked elevated LDH and protein level with 17,000 WBC/L. Blood cultures grow Streptococcus pneumoniae with minimal inhibitory concentration (MIC) to penicillin i illi < 0 0.1 1 g/mL. / L Oxygen O saturation t ti 96% on room air. i BP 110/70, T 102F, Pulse 112. What is the next step in management? a. Repeated thoracentesis Will add nothing b. Placement of chest tube for suction c. Add ampicillin to treatment No benefit d. Place patient in ICU No need for chest tube e. Consult pulmonary Will add nothing

MTBS2CK p.145

CAP/Treatment

CAP/Treatment

MTBS2CK p.146

Pleural effusion with a large meniscus sign. Only a fluid sample from thoracentesis can determine the specific cause. Source: Craig Thurm, MD

MTBS2CK p.146

Effusion should be freely mobile and from a layer when the patient lies on her side. Source: Nishith Patel.

17

CAP/Treatment

CAP/Treatment Pneumococcal Vaccination Everyone > 65 should receive vaccination with 23 polyvalent vaccine Chronic heart Liver Vaccinated as soon as Kidney their underlying disease is apparent, regardless Or of age Lung disease
Including asthma

MTBS2CK p.146

Hydropneumothorax is both abnormal air and fluid (effusion) in the pleural space. Chest tube drainage is the most effective way to remove this condition. Source: Albert Takem, MD.

MTBS2CK p.146

CAP/Treatment

CAP/Treatment

Pneumococcal Vaccination Other reasons to vaccinate


Functional or anatomic asplenia (e.g., sickle cell disease) Hematologic malignancy (leukemia, lymphoma) Immunosuppression: diabetes mellitus, alcoholics, corticosteroid users, AIDS or HIV positive CSF leak and cochlear implantation recipients

Pneumococcal Vaccination Generally healthy: single dose at 65 If first vaccination was given before 65 or with other conditions previously described a second dose should also described, be given 5 years after first dose Healthcare workers do not need pneumococcal vaccine
MTBS2CK p.147

MTBS2CK p.146147

HealthcareAssociatedPneumoniaor HospitalAcquiredPneumonia(HAP)/Definition

Pneumonia Part2

Pneumonia developing > 48 hours after admission g incidence of Gram Much higher negative bacilli such as E. coli or Pseudomonas

MTBS2CK p.147

18

HAP/Treatment Main difference in management is


Macrolides (azithromycin or clarithromycin) are unacceptable as empiric therapy

HAP/Treatment

Centered around therapy for Gram-negative bacilli


Antipseudomonal cephalosporins: cefepime or ceftazidime Or Antipseudomonal penicillin: piperacillin/tazobactam Or Carbapenems: imipenem, meropenem, or doripenem
MTBS2CK p.147

Piperacillin and ticarcillin always used in combination with beta-lactamase inhibitor (e.g., ( g tazobactam or clavulanic acid)

MTBS2CK p.147

VentilatorAssociatedPneumonia/Definition

VAP/Diagnosis

Mechanical ventilation interferes with normal mucociliary clearance of respiratory tract (e.g., ability to cough) Positive pressure is tremendously damaging to normal ability to clear colonization Ventilator-associated pneumonia (VAP) has an incidence as high as 5% per day in first few days on ventilator

Because of multiple countercurrent illnesses (e.g., CHF) even a diagnosis of VAP can be hard to establish Look for
Fever and/or rising WBC count New infiltrate on chest X-ray Purulent secretions coming from endotracheal tube

MTBS2CK p.147

MTBS2CK p.147

VAP/DiagnosticTests

VAP/DiagnosticTests
Least accurate but easiest to do Tracheal aspirate Bronchoalveolar lavage (BAL) Most accurate but dangerous Protected brush specimen Tip of p bronchoscope covered when passed through nasopharynx, then uncovered only inside lungs Much more specific, decreased contamination

Sputum culture: nearly worthless


Due to colonization of endotracheal tube (ET)

Diagnosis of specific etiology is extremely difficult on ventilator

Suction catheter placed l d into i t ET and d aspirates contents below trachea when catheter is past end of tube

Bronchoscope placed deeper in lungs where there arent supposed to be organisms Can be contaminated when passed through the nasopharynx

MTBS2CK p.148

MTBS2CK p.148

19

VAP/DiagnosticTests

VAP/Treatment
Combine 3 different drugs Cephalosporin (ceftazidime or cefepime) Or Penicillin (piperacillin/tazobactam) Or Carbapenem (imipenem, meropenem, or doripenem) d i ) Aminoglycoside (gentamicin or tobramycin or amikacin) Vancomycin Or Linezolid

1. Antipseudomonal beta-lactam PLUS 2. Second antipseudomonal agent PLUS

MTBS2CK p.148

Subcutaneous emphysema is air abnormally leaking into soft tissue of the chest wall. Chest tube placement may cause air to leak into soft tissues of the chest wall. Source: Birju Shah, MD.

3. Methicillin-resistant antistaphylococcal agent


MTBS2CK p.148149

VAP/Treatment
Patient hospitalized for head trauma and subdural hematoma is intubated for hyperventilation and subsequent craniotomy. Several days after admission hematemesis ensues and stomach stress ulcers are found. Lansoprazole is started. VAP develops, placed on imipenem, linezolid, and gentamicin. Phenytoin, started prophylactically. Three days later creatinine rises followed by seizures . Repeat head CT shows no changes. h What is the most appropriate next step in management? a. Switch phenytoin to carbamazepine Less likely to cause renal failure (RF) b. Stop lansoprazole Not likely to cause RF c. Stop imipenem d. Stop linezolid Not likely to cause RF e. Perform an electroencephalogram Will add nothing

Change initial therapy for VAP if specific etiology is identified

MTBS2CK p.149

MTBS2CK p.149

LungAbscess/Etiology

LungAbscess/Etiology

Rare because of prompt treatment of aspiration pneumonia Occurs only with


Large-volume Large ol me aspiration of oral/phar oral/pharyngeal ngeal contents, usually with poor dentition, who isnt adequately treated

Large-volume aspiration occurs from


Stroke with loss of gag reflex Seizures Intoxication Endotracheal intubation

Aspiration pneumonia happens in the right upper lobe when lying flat

MTBS2CK p.149

MTBS2CK p.149

20

LungAbscess/Presentation

LungAbscess/DiagnosticTests

Look for
Person with one of these risk factors presenting a chronic infection developing over several weeks with large-volume sputum that is foul smelling because of anaerobes

Best initial test: chest X-ray


Cavity, possibly with air-fluid level

Chest CT is more accurate Only lung biopsy can establish specific microbiologic etiology Sputum culture is the wrong answer for diagnosing a lung abscess. Everyones sputum has anaerobes from mouth flora.
MTBS2CK p.150

Weight loss is common

MTBS2CK p.150

LungAbscess/DiagnosticTests

LungAbscess/Treatment

Clindamycin is best to cover lung abscess

Cavity consistent with an abscess with a thick wall and an air-fluid level. Source: Alejandro de la Cruz, MD.

MTBS2CK p.150

MTBS2CK p.150

PneumocystisPneumonia(PCP)/Etiology

PCP/Presentation

Agent causing PCP has been renamed P. jiroveci instead of P. carinii Almost exclusively in patients with AIDS whose CD4 cell count is < 200/L and who arent aren t on prophylactic therapy
Also occurs in chronically immunosuppressed patients, especially those on long term high dose steroids

Look for
Patient with AIDS presenting with dyspnea on exertion, dry cough, and fever

Question will often suggest or directly state that CD4 count is low (< 200/L) and that patient isnt on prophylaxis

MTBS2CK p.150

MTBS2CK p.150

21

PCP/DiagnosticTests

PCP/DiagnosticTests

Best initial test


Chest X-ray showing bilateral interstitial infiltrates Or ABG looking for hypoxia or an increased A-a gradient

Sputum stain: quite specific if its positive


If positive, there is no need to do further testing If negative, bronchoscopy as the best diagnostic test

LDH levels are always elevated Most accurate test


Bronchoalveolar lavage
MTBS2CK p.150

Normal LDH means DONT answer PCP as the most likely diagnosis

MTBS2CK p.150151

PCP/DiagnosticTests

PCP/Treatment
Best initial therapy for both treatment and prophylaxis
Trimethoprim/sulfamethoxazole (TMP/SMX)

S2CK asks what is the most likely diagnosis, not what is the for sure diagnosis

Add steroids to decrease mortality if PCP is severe Severe PCP: pO2 < 70 or A-a gradient > 35 Atovoquone
Alternative to TMP/SMX if mild PCP, meaning g there is only mild hypoxia

Cannot distinguish PCP from Mycoplasma, Chlamydophila, or viruses by X-ray alone. However, in HIV, PCP is most likely with interstitial infiltrates

If toxicity from TMP/SMX switch to either


Clindamycin and primaquine Or Pentamidine

MTBS2CK p.151

MTBS2CK p.151

PCP/Treatment HIV-positive African American man is admitted with dyspnea, dry cough, high LDH, and pO2 of 63 mmHg. He is started on TMP/SMX and prednisone. On 3rd hospital day he develops severe neutropenia and rash. He has anemia and smear shows bite cells. What is the most appropriate next step in management?

a. Stop TMP/SMX Need to treat Will not help acute b. Begin antiretroviral medications opportunistic infection c. Switch TMP/SMX to intravenous pentamidine IV for active d. Switch TMP/SMX to aerosol pentamidine disease e. Switch TMP/SMX to clindamycin and primaquine
MTBS2CK p.151

Often students will see 2 correct treatments and think theres a mistake in the question. question If there are 2 correct treatments, look for a contraindication to one of them.

Contraindicated due to G6PD

MTBS2CK p.152

22

PCP/Prophylaxis AIDS with CD4 count < 200/L Rash Or Neutropenia

PCP/Treatment

1. TMP/SMX

2. Atovoquone or Dapsone

Always choose therapy based first on efficacy, not adverse effects

Aerosol pentamidine: not used as second-line therapy for prophylaxis


Due to less efficacy than either atovoquone or dapsone

Dapsone is contraindicated in those with G6PD deficiency

MTBS2CK p.152

MTBS2CK p.152

HIV-positive woman with 22 CD4 cells/L is admitted with PCP and is treated successfully with TMP/SMX. Prophylactic TMP/SMX and azithromycin are started. She is then started on antiretroviral medication and her CD4 rises to 420 cells for last 6 months. What is the most appropriate next step in management? p TMP/SMX And azithromycin a. Stop y because CD4 b. Stop both TMP/SMX and azithromycin Cannot stop antiretrovirals; c. Stop all medications and observe CD4 will d. Stop all medications if the PCR-RNA viral load is undetectable CD4 will ; antiretrovirals maintain CD4 e. Continue all the medications No prophylaxis needed f. Stop the azithromycin And TMP/SMX because CD4
MTBS2CK p.152

Tuberculosis

Tuberculosis(TB)/Etiology

Tuberculosis/Etiology Almost all patients with TB have 1+ established risk factors


Recent immigrants (in past 5 years) Prisoners HIV positive Healthcare workers Close contact of someone with TB Steroid use Hematologic malignancy Alcoholics Diabetes mellitus

Due to Mycobacterium tuberculosis Initial infection most commonly leads to latent TB Most symptomatic cases due to
Reactivation R ti ti of f latent l t t infection i f ti rather th than primary exposure

Pulmonary TB most common, but can affect any organ system


MTBS2CK p.152

MTBS2CK p.153

23

Tuberculosis/Presentation Look for


Previously listed risk factors presenting with fever, cough, sputum, weight loss, hemoptysis, and night sweats

Tuberculosis/DiagnosticTests Best initial test: chest X-ray as with all respiratory infections Sputum stain and culture specifically for acid-fast bacilli (mycobacteria) must be done 3 times to fully exclude TB g acid-fast stains, , but clinical suspicion p If 3 negative high
Bronchoscopy with BAL or pleural biopsy

Symptoms are almost always > 3 weeks in duration You cannot answer TB as diagnosis without a clear risk factor, a cavity on chest X-ray or a positive smear

PPD skin testing is never best test for TB in symptomatic patient


MTBS2CK p.153154

MTBS2CK p.153

Tuberculosis/DiagnosticTests Most common finding: cavitary upper lobe infiltrate

Tuberculosis/Treatment Smear positive: begin therapy with 4 drugs


Rifampin, isoniazid, pyrazinamide, and ethambutol (RIPE)

Ethambutol not needed if known from start of therapy that organism is sensitive to all TB medications
Ethambutol: given as part of 4-drug empiric therapy prior to knowing sensitivity of organism

After using RIPE for first 2 months


Stop ethambutol and pyrazinamide And Continue rifampin and isoniazid for next 4 months
MTBS2CK p.153
Chest x-ray with upper lobe disease consistent with TB. Source: Craig Thurm, MD.

MTBS2CK p.154

Tuberculosis/Treatment

Tuberculosis/Treatment

Standard of care: 6 months total of therapy Treatment is extended to 9 months for


Osteomyelitis Miliary tuberculosis Meningitis Pregnancy or any other time pyrazinamide isnt used

Toxicity of Therapy All TB medications cause hepatoxicity Dont stop them unless transaminases rise i 3 3-5 5 ti times upper limit li it of f normal l

MTBS2CK p.154

MTBS2CK p.154

24

AdverseEffectsofAntituberculosisTherapy Rifampin
Toxicity

Tuberculosis/Treatment Ethambutol Optic neuritis/ color vision i i Use of Steroids Glucocorticoids decrease risk of constrictive pericarditis in those with pericardial involvement Decrease neurologic complication in TB meningitis

Isoniazid

Pyrazinamide

Red color to body secretions


Treatment

Peripheral neuropathy

Hyperuricemia

Pregnant patients shouldnt receive pyrazinamide Use pyridoxine to prevent No treatment unless symptomatic Decrease dose in renal failure
MTBS2CK p.154

None, benign finding


MTBS2CK p.154

LatentTuberculosis/PPDTesting

LatentTuberculosis/PPDTesting What Is Considered a Positive Test? Only induration is counted towards a positive test Erythema is irrelevant Induration > 5 mm
HIV-positive patients Glucocorticoid users Close contact with active TB patients Abnormal calcifications on chest X-ray Organ transplant recipients

Indications for PPD Testing Not a general screening test for whole population Only those in risk groups previously described should be screened PPD testing isnt useful in those who are symptomatic or those with abnormal chest X-rays
MTBS2CK p.154

MTBS2CK p.155

LatentTuberculosis/PPDTesting Induration > 10 mm


Recent immigrants (past 5 years) Prisoners Healthcare workers Close contacts with TB patients Hematologic malignancy malignancy, alcoholics alcoholics, DM

LatentTuberculosis/PPDTesting Two-Stage Testing If patient never had a PPD skin test before, a second test is indicated within 1-2 weeks if first test is negative
1st test maybe falsely negative

Induration > 15 mm
Those with no risk factors

If 2nd test is negative: truly negative If 2nd test is positive: first test was false negative If the first test is positive, a second test isnt necessary
MTBS2CK p.155

Everyone with a reactive PPD test should have a chest X-ray to exclude active disease
MTBS2CK p.155

25

LatentTuberculosis/PPDTreatment

LatentTuberculosis/PPDTreatment

Treatment for a Positive PPD After active TB has been excluded with a chest X-ray, patients should receive 9 months of isoniazid Positive PPD confers a 10% lifetime risk of TB Isoniazid results in 90% reduction in risk; after isoniazid lifetime risk of TB goes from 10% to 1%

Those at high risk (e.g., healthcare workers), should have a PPD annually to screen for conversion Majority of risk for developing active TB lies within first 2 years after conversion Once PPD is positive, itll always be positive in future

MTBS2CK p.155

MTBS2CK p.155

LatentTuberculosis/PPDTreatment PPD testing is one of the hardest and most misunderstood tests on USMLE S2 CK. Reread preceding section and forget what youve learned in the past.

SolitaryPulmonaryNodule& InterstitialLungDisease

Previous BCG has no effect on these recommendations. If PPD is positive, the patient must take isoniazid for 9 months even if he or she had BCG.
MTBS2CK p.155

SolitaryPulmonaryNodule For Step 2, be sure to know when biopsy is appropriate!


QualitiesofBenignandMalignantPulmonaryNodules Benign <30yearsold Nochangeinsize Nonsmoker Smoothborder Small,<1cm Normallung Adenopathy() Dense,centralcalcification NormalPETscan
MTBS2CK p.156

SolitaryPulmonaryNodule Best initial step in all lung lesions is to compare size with old X-rays Biopsy all enlarging lung lesions, particularly p y if they y are rapidly p y enlarging g g However, doubling in size < 30 days more likely to be infectious than malignant (think about clinical scenario to decide)

Malignant >40yearsold Enlarging Smoker Spiculated(spikes) Large,>2cm Atelectasis Adenopathy(+) Sparse,eccentriccalcification AbnormalPETscan

MTBS2CK p.156

26

SolitaryPulmonaryNodule

HighProbabilityLesions/Management

When many features are described under malignant


Resect (remove) lesion Sputum cytology, needle biopsy, and PET scanning shouldnt be done because a negative ti t test ti is lik likely l false f l negative ti

If resection is one of the choices, then its the answer

Source: Lange123 at the German language Wikipedia, commons.wikimedia.org

MTBS2CK p.156

IntermediateProbabilityLesions/ Management

IntermediateProbabilityLesions/ Management Sputum cytology


If positive, this is highly specific, the most appropriate next step in management is resection of lesion If negative cytology doesnt exclude malignancy

There are some gray or inconclusive aspects of solitary pulmonary nodule such as
Age range gap (between 30 and 40) Or Size ( (between 1 cm and 2 cm) )

Definition of intermediate probability

Bronchoscopy or transthoracic needle biopsy


Each is the most appropriate next step in most patients with intermediate probability of malignancy Bronchoscopy for central lesions Transthoracic biopsy for peripheral lesions
MTBS2CK p.156

MTBS2CK p.156

IntermediateProbabilityLesions/ Management

IntermediateProbabilityLesions/ Management

Diagnostic test question in intermediate lesions; clear answer must be present. Choice of test may not be clear, but adverse effects are always clear. clear Most common adverse effect of a transthoracic biopsy is pneumothorax.

Positron emission tomography (PET scan) Tells whether content of lesion is malignant without biopsy Malignancy has increased uptake of tagged glucose Sensitivity of PET scan is 85 - 95% Negative scan points away from malignancy

MTBS2CK p.157

MTBS2CK p.157

27

IntermediateProbabilityLesions/ Management

SolitaryPulmonaryNodule

Video-assisted thoracic surgery (VATS) VATS is both more sensitive and more specific than all other forms of testing Frozen section in operating room allows for immediate conversion to an open thoracoscopy and lobectomy if malignancy is found
MTBS2CK p.157

Benign pulmonary nodules often represent scar from previous infection


Immigrant, think TB SW United States, think coccidioidomycosis y Ohio River Valley, think histoplasmosis

InterstitialLungDisease/Definition Diseases characterized by inflammation and/or fibrosis of interalveolar septum Fibrosis causes Impaired gas exchange Increased lung stiffness And Decreased lung compliance & expansion
MTBS2CK p.157
Source: Ed Uthman. commons.wikimedia.org

InterstitialLungDisease/Etiology Specific Causes of Interstitial Lung Disease Idiopathic pulmonary fibrosis (IPF) and other idiopathic interstitial pneumonias Collagen vascular disease Granulomatous disorders Hypersensitivity disorders Pneumoconiosis Radiation Drugs: bleomycin, busulfan, amiodarone, methylsergide, nitrofurantoin, cyclophosphamide

MTBS2CK p.157

InterstitialLungDisease/Etiology TypesofPneumoconioses Exposure Coal Sandblasting,rock mining,tunneling Shipyard py workers, ,p pipe p fitting,insulators Cotton Electronicmanufacture Moldysugarcane Disease Coalworkerspneumoconiosis Silicosis Asbestosis Byssinosis Berylliosis Bagassosis

InterstitialLungDisease/Presentation All forms of pulmonary fibrosis, regardless of etiology, present with


Dyspnea worsening on exertion Hypoxia worsening on exertion Fine rales or crackles on examination Loud P2 heart sound (if pulmonary HTN present) Clubbed fingers

Inflammatory infiltration with white cells is reversible (treatable), whereas fibrosis is irreversible

Biopsy shows granulomas in berylliosis


MTBS2CK p.157 MTBS2CK p.158

28

InterstitialLungDisease/DiagnosticTests

InterstitialLungDisease/Diagnostictests

Best initial test: always chest X-ray High resolution CT scan is more accurate than chest X-ray Most accurate test: lung biopsy Echocardiography E h di h will ill often ft show h pulmonary HTN and possibly right ventricular hypertrophy

MTBS2CK p.158

PCP. Source: commons.wikimedia.org

InterstitialLungDisease/Diagnostictests

InterstitialLungDisease/DiagnosticTests PFTs Restrictive lung disease with proportional decrease FEV1, FVC, TLC, and RV all , but since everything is decreased, the FEV1/FVC ratio will be normal DLCO Decreased in proportion to severity of alveolar septal thickening

MTBS2CK p.158

Severe, long-standing interstitial fibrosis produces thick walls between alveoli that give the appearance of honeycombing. Source: Craig Thurm, MD.

MTBS2CK p.158

InterstitialLungDisease/Treatment Most types of interstitial lung diseases are untreatable Biopsy shows
White cell or inflammatory infiltrate; prednisone should be used Mostly y fibrosis ( (e.g., g IPF), ) steroids typically yp y NOT effective

InterstitialLungDisease/ HypersensitivityPneumonitis

Specific type of interstitial lung disease caused by pulmonary hypersensitivity reaction to certain environmental antigens Inflammation from hypersensitivity reaction results in alveolar thickening and granuloma formation
MTBS2CK p.158

Of all causes of pneumoconioses, berylliosis is most likely to respond to steroids


Due to granulomas (sign of inflammation)

MTBS2CK p.158

29

InterstitialLungDisease/ HypersensitivityPneumonitis

InterstitialLungDisease/ HypersensitivityPneumonitis Most common antigens causing hypersensitivity pneumonitis include


Antigens from feathers (bird fanciers lung) MAI antigen (hot tub lung) Spores of actinomyces from moldy hay (farmers l lung) ) Spores of actinomyces from compost (mushroom workers lung) Spores of actinomyces from air conditioners (air conditioner lung)

Presentation Acutely
Fever, dyspnea, severe cough Within 4-6 hours of antigen exposure

Or Chronically
Progressive dyspnea on exertion, fine rales, pulmonary fibrosis

InterstitialLungDisease/ HypersensitivityPneumonitis

Treatment Corticosteroids help to reduce inflammation AVOID O ONGOING O GO G EXPOSURE OSU to inciting agent

Sarcoidosis& ThromboembolicDisease

Sarcoidosis/Definition/Etiology

Sarcoidosis/Presentation Look for


Young African American woman with SOB on exertion and occasional fine rales on lung exam, but without wheezing of asthma

More common in African American women Idiopathic inflammatory disorder predominantly of lungs, lungs but can affect most of the body Characterized by noncaseating granulomas
MTBS2CK p.159

Erythema y nodosum and lymphadenopathy y p p y on chest X-ray hands you diagnosis question

MTBS2CK p.159

30

Sarcoidosis/Presentation

Sarcoidosis/Presentation
Brain complications Eye problems (burning, itching, tearing or pain) Salivary glands Enlarged lymph nodes in neck & chest Heart complications Granulomas (inflamed lumps in lungs) Liver enlargement Spleen enlargement Enlarged lymph nodes in chest near windpipe and lungs Scarring and granulomas in lung Lupus pernio (painful skin sores on face) Skin lesions on back, arms, neck, face, and scalp

Also presents with


Constitutional symptoms: fever, malaise, weight loss Arthritis Parotid gland enlargement Facial pals palsy Heart block and restrictive cardiomyopathy CNS involvement Iritis and uveitis

Erythema nodosum (itchy and painful rashes) on the lower legs and ankles

MTBS2CK p.159

Source: National Heart, Lung, and Blood Institute, commons.wikimedia.org

Sarcoidosis/Presentation

Sarcoidosis/DiagnosticTests Chest X-ray: best initial test Hilar lymphadenopathy > 95% with sarcoidosis Parenchymal involvement can also be present in combination with lymphadenopathy
CXR can reveal bilateral hilar lymphadenopathy only Hilar lymphadenopathy with parenchymal disease Or Parenchymal disease alone (depending on stage of disease)

Answer sarcoidosis when chest X-ray or CT shows hilar adenopathy in generally healthy African American woman Although liver and kidney granulomas are very common on autopsy, they are rarely symptomatic

MTBS2CK p.159

MTBS2CK p.159

Sarcoidosis/DiagnosticTests

Sarcoidosis/DiagnosticTests

Lymph node biopsy: most accurate test


Granulomas are noncaseating

Elevated ACE level: 60% Hypercalciuria: 20% Hypercalcemia: yp 5%


Granulomas in sarcoidosis make vitamin D

PFTs: restrictive lung disease


Decreased FEV1, FVC, and TLC with a normal FEV1/FVC ratio
Source: Jmh649, commons.wikimedia.org

MTBS2CK p.159

31

Sarcoidosis/Treatment

ThromboembolicDisease/Definition

Prednisone: drug of choice


Few patients fail to respond

Pulmonary emboli (PE) and deep venous thrombosis (DVT) are treated as same disease PE From DVT vessels of legs in 70% of cases and pelvic veins in 30%, but since risks and treatment are the same they can be discussed simultaneously

Asymptomatic hilar adenopathy doesnt need to be treated

MTBS2CK p.159

MTBS2CK p.160

ThromboembolicDisease/Etiology Virchows triad outlines conditions that predispose to venous TE


Immobility CHF Recent surgery Trauma T Surgery Recent fracture

ThromboembolicDisease/Presentation Look for Sudden onset SOB with clear lungs on examination and normal chest X-ray Other findings in PE
Tachypnea, tachycardia, cough, and hemoptysis Leg pain from DVT Pleuritic chest pain from lung infarction Fever can arise from any cause of clot or hematoma Extremely severe emboli will produce hypotension

MTBS2CK p.160

Factor V Leiden mutation Any malignancy leads to DVT

MTBS2CK p.160

ThromboembolicDisease/DiagnosticTests
Most questions about PE concern diagnostic testing and treatment

ThromboembolicDisease/DiagnosticTests Chest X-ray Usually normal in PE Most common abnormality is atelectasis Wedge Wedge-shaped shaped infarction (Hampton hump), and oligemia of one lobe (Westermark sign) are much less common than simple atelectasis
MTBS2CK p.160161

There is no single, uncomplicated diagnostic test for PE y, EKG, , and ABG are best initial tests Chest X-ray, In PE, the main issue is to know Whats the most common finding? And Whats the most common abnormality when theres an abnormality?
MTBS2CK p.160

EKG Usually shows sinus tachycardia Most common abnormality is nonspecific p ST-T wave changes Only 5% will show right axis deviation, RV hypertrophy or RBBB

32

ThromboembolicDisease/DiagnosticTests ABG Hypoxia and respiratory alkalosis ( pH and p CO2) with normal chest X-ray is extremely suggestive of PE
65-year-old woman with recent hip replacement has acute onset of SOB and tachycardia. Chest X-ray normal. Hypoxia on ABG, increased A-a gradient, EKG with sinus tachycardia. What is the most appropriate next step in management? a. Intravenous unfractionated heparin b. Thrombolytics Hemodynamically unstable & acute RV failure Anticoagulants contraindicated, c. Inferior I f i vena cava filter filt recurrent emboli & RV dysfunction d. Embolectomy If heparin ineffective & persistent e. Spiral CT scan hypotension, hypoxia, tachycardia f. Ventilation/perfusion (V/Q) scan If initial labs suggestive, treat! g. Lower-extremity Doppler studies Dont wait to confirm PE h. D-dimer Poor specificity
MTBS2CK p.161

Frequent wrong answer is to choose S1, Q3, T3 as the most common abnormality on EKG

MTBS2CK p.161

ThromboembolicDisease/DiagnosticTests

ThromboembolicDisease/DiagnosticTests

Spiral CT scan Also called CT angiogram Standard of care in diagnostic testing to confirm presence of PE after X-ray, EKG and ABG EKG, Specificity is excellent (> 95%) However, sensitivity is 85%, it can miss 15% of clots
MTBS2CK p.161
Chest spiral CT scan with radiocontrast agent showing multiple filling defects both at the bifurcation and in the pulmonary arteries. Source: James Heilman, MD, commons.wikimedia.org

ThromboembolicDisease/DiagnosticTests Ventilation/Perfusion (V/Q) scan May reveal segmental areas of mismatch High probability scans have no clot (false positive) in 15% Low probability scans have a clot (false negative) in 15% Interpret results in combination with clinical suspicion Chest X-ray must be normal for V/Q scan to have any degree of accuracy. Do a spiral CT if chest X-ray is abnormal.
MTBS2CK p.161162

ThromboembolicDisease/DiagnosticTests

V/Q or ventilation perfusion scanning is still very useful in evaluating pulmonary emboli. A positive test is an area that is ventilated with decreased perfusion. Source: Nishith Patel.

MTBS2CK p.161

33

ThromboembolicDisease/DiagnosticTests

ThromboembolicDisease/DiagnosticTests

D-dimer Test is very sensitive (better than 97% negative predictive value) Specificity poor. Any clot or bleeding elevates D-dimer level Negative test excludes clot Positive test doesnt mean anything

D-dimer is answer when pretest probability of PE is low and you need a simple, noninvasive test to exclude thromboembolic disease

MTBS2CK p.162

MTBS2CK p.162

ThromboembolicDisease/DiagnosticTests Lower extremity (LE) Doppler study If LE Doppler positive, no further testing needed Only 70% of PEs originate in legs, so itll miss 30% of cases You dont need a spiral CT or V/Q scan to confirm a PE if theres there s a clot in legs because they wont won t change therapy Patient still needs heparin and 6 months of warfarin

ThromboembolicDisease/DiagnosticTests

Spiral CT negative V/Q or LE Doppler negative withhold therapy with heparin LE D Dopplers l are a good d test if V/Q and spiral CT do not give clear diagnosis

MTBS2CK p.162

MTBS2CK p.162

ThromboembolicDisease/DiagnosticTests Angiography Most accurate test Nearly 100% specificity and a false negative rate < 1% Unfortunately, theres 0.5% mortality, which is high if you consider the tens of thousands of tests a year that are needed to exclude PE in all cases

ThromboembolicDisease/DiagnosticTests

When testing for PE, angiography is rarely done


Thrombus (labeled A) causing a central obstruction in the left main pulmonary artery. ECG tracing shown at bottom. Source: Aung Myat and Arif Ahsan, commons.wikimedia.org

MTBS2CK p.162

34

ThromboembolicDisease/Treatment

ThromboembolicDisease/Treatment

Heparin Best initial therapy Warfarin should be started simultaneously with heparin to
Achieve Achie e therapeutic therape tic INR of 2 to 3 times normal as quickly as possible

When is an inferior vena cava (IVC) filter the right answer? Contraindication to the use of anticoagulants (e.g., melena, CNS bleeding) Recurrent emboli while on heparin or fully therapeutic warfarin (INR of 23) Right ventricular (RV) dysfunction with an enlarged RV on echo. In this case, the next embolus, even if seemingly small, could be potentially fatal
MTBS2CK p.162

MTBS2CK p.162

ThromboembolicDisease/Treatment

ThromboembolicDisease/Treatment

When are thrombolytics the right answer? Hemodynamically unstable patients (e.g., hypotension and tachycardia) Acute RV dysfunction Contraindicated in p patients with recent surgery or bleeding No specific time limit in which to use thrombolytics as there is in stroke or MI
MTBS2CK p.162

When are direct-acting thrombin inhibitors (argatroban, lepirudin) the answer? Heparin-induced thrombocytopenia When is aspirin the answer? Never

MTBS2CK p.162163

PulmonaryHypertension/Definition

PulmonaryHypertension, ObstructiveSleepApnea,& AcuteRespiratory DistressSyndrome

Lung or pulmonary circulation normally extremely low pressure


Systolic 25 mmHg Diastolic 8 mmHg Mean 15 mmHg

MTBS2CK p.163

35

PulmonaryHypertension/Definition

PulmonaryHypertension/Etiology

Classified as either Primary


Idiopathic dysfunction of pulmonary arteries

Secondary causes include


L sided-heart failure (MCC) ( ) Intracardiac L R shunting Hypoxic vasoconstriction from chronic lung disease (e.g., COPD) Thromboembolic disease
MTBS2CK p.163

Chronic hypoxemia leads to pulmonary HTN, which results in more hypoxemia

MTBS2CK p.163

PulmonaryHypertension/Presentation Dyspnea and fatigue Syncope (exertional!) Chest pain (exertional!) Wide splitting of S2 with loud P2 or tricuspid and pulmonary valve insufficiency Sign of right right-sided sided heart failure Its impossible to know that pulmonary HTN is causing dyspnea without tests

PulmonaryHypertension/DiagnosticTests

ChestXrayand CT
Best initial tests Showing dilation of proximal pulmonary arteries with narrowing or pruning of distal vessels

Source: tmcr.usuhs.mil

MTBS2CK p.163

MTBS2CK p.163

PulmonaryHypertension/DiagnosticTests

PulmonaryHypertension/DiagnosticTests

RightheartorSwanGanzcatheter Most accurate test And Most precise method to measure pressures

EKG Right axis deviation, right atrial and ventricular hypertrophy Echocardiography RA and RV hypertrophy; Doppler estimates pulmonary artery (PA) pressure
MTBS2CK p.163

V/Q scanning Identifies chronic PE as cause of pulmonary HTN CBC Shows polycythemia from chronic hypoxia

MTBS2CK p.163

Source: commons.wikimedia.org

36

PulmonaryHypertension/Treatment

PulmonaryHypertension/Treatment

1. Correct underlying cause when one is clear 2. Idiopathic disease is treated with
Inhaled or intravenous prostacyclin analogues (pulmonary arterial vasodilators): epoprostenol, treprostinil, iloprost, beraprost Endothelin antagonists: bosentan Phosphodiesterase inhibitors: sildenafil Calcium-channel blockers
Only if demonstrated to be responsive to them during right heart catheterization

Oxygen slows progression, particularly with COPD Only lung transplantation is curative for idiopathic pulmonary HTN

MTBS2CK p.164

MTBS2CK p.164

ObstructiveSleepApnea(OSA)/Definition

ObstructiveSleepApnea(OSA)/Presentation

Cessation of airflow due to upper airway obstruction during sleep Obesity Ob it i is most t commonly identified cause
Source: Habib M'henni, commons.wikimedia.org

Patients present with daytime somnolence and history of loud snoring Other symptoms include
Headache Impaired memory and judgment Depression HTN Erectile dysfunction

MTBS2CK p.164

MTBS2CK p.164

ObstructiveSleepApnea Risk factors include


Male gender Obesity Large uvula/tongue And Retrognathia (recession of mandible)

ObstructiveSleepApnea/Treatment

Arrhythmias and erythrocytosis are common Can lead to pulmonary HTN and RV failure Most accurate test is polysomnography (sleep study)
Shows multiple episodes of apnea
MTBS2CK p.164

1. Weight loss and avoidance of alcohol 2. Nasal continuous positive airway pressure (nasal CPAP) 3. Surgical widening of airway (uvuloplatopharyngoplasty) if this fails 4. Avoid use of sedatives

MTBS2CK p.164

37

AcuteRespiratoryDistressSyndrome(ARDS)/ Definition Acute respiratory failure from overwhelming lung injury or systemic disease Characterized by Severe hypoxia Poor lung compliance And Noncardiogenic pulmonary edema
ARDS is caused by endothelial injury at level of alveolus, making lung cells leaky so that alveoli fill up with fluid
MTBS2CK p.164165

ARDS/Etiology

ARDS is idiopathic Large number of illnesses and injuries are associated with alveolar epithelial cell and capillary endothelial cell damage Illnesses and injuries associated with developing ARDS include
Sepsis or aspiration Lung contusion/trauma Near-drowning Burns or pancreatitis
MTBS2CK p.165

ARDS/DiagnosticTests

ARDS/DiagnosticTests

Chest X-ray shows bilateral infiltrates quickly become confluent (white out) Air bronchograms are common

MTBS2CK p.165

Source: Samir , commons.wikimedia.org

ARDS/DiagnosticTests

ARDS/DiagnosticTests

Defined as having PaO2/FiO2 ratio < 200


The FIO2 is expressed as a decimal (e.g., room air with 21% oxygen = 0.21) If pO2 is 105 on room air (21% oxygen or 0.21), then the ratio of pO2/FIO2 is 500 (i.e., 105/.21) If pO2 (as measured on ABG) is 70 while breathing 50% oxygen, the ratio is 70/0.5 or 140

ARDS is associated with normal findings on right heart catheterization Wedge pressure is normal (< 18 mmHg)
Air bronchograms are a sign of dense consolidation of the lung air space. This is a case of pneumococcal pneumonia that left only the air space in the larger bronchi open or air bronchograms. Source: Omid Edrission, MD.

MTBS2CK p.165

MTBS2CK p.165

38

ARDS/Treatment

ARDS/Treatment Positive end-expiratory pressure (PEEP) is used when patient is undergoing mechanical ventilation to FIO2 Levels of FIO2 > 50% are toxic to lungs Maintain plateau pressure < 30 cm of water measured on ventilator No treatment is proven to reverse ARDS. Dont forget to treat the underlying cause.

Low tidal volume mechanical ventilation is the best support while waiting to see if lungs will recover
Use 6 mL/kg of tidal volume

Steroids aren arent t clearly beneficial in most cases


They may help in late-stage disease in which pulmonary fibrosis develops

MTBS2CK p.166

MTBS2CK p.166

39

CancerScreening PreventativeMedicine
ConradFischer,MD AssociateProfessorofMedicine TouroCollegeofMedicine NewYorkCity

BreastCancer CervicalCancer ColonCancer ProstateCancer LungCancer

BreastCancerScreening

Mammography screening tool of choice Start at 50 years old When reward is highest Starting at 40 is controversial MRI, MRI CT CT, and US are adjuvant screening
Self-examination not recommended. Never the correct answer on S2CK.

Which of the following is most likely to benefit an asymptomatic patient with multiple first-degree relatives with breast cancer? a. b. c c. d. e. f. Tamoxifen Most familial breast ca. NOT BRCA BRCA testing Aromatase inhibitors Unproven in prevention Effective but to lesser degree Dietary modification Performed in breast cancer patients HER-2/neu testing Estrogen/progesterone Performed in breast cancer patients receptor testing

MTBS2CK p.356

MTBS2CK p.356

TAMOXIFEN Greater risk: DVT/PE and endometrial cancer Less risk: Fractures Remember! Better to be ALIVE with a DVT than DEAD with normal legs

CanSTOPmammographyatage75

HER-2/neu positive patients treated with trastuzumab if you HAVE cancer! Not a prophylactic!

CervicalCancerScreening

ColonCancerScreening

Papanicolaou test = Pap Smear


Start at age 21 Done every 3 years, with cytology, until 30 years old After age 30, if HPV testing added, then every 5 years Stop at age 65 with adequate screening history and low risk
MTBS2CK p.357

Colonoscopy
The most accepted screening test Start at 50 years All other testing Repeat every 10 years modalities are inferior to Exceptions colonoscopy Family history 1st-degree relative 40 or ten years earlier Whichever is EARLIER! HNPCC: 3 relatives, 2 generations, 1 premature Start at 25 and do colonoscopy every 1-2 years
MTBS2CK p.357

Katsumi M. Miyai, M.D., Ph.D., Regents of the University of California. Used with permission.

ColonCancerScreening

ProstateCancerScreening

Familial Adenomatous Polyposis


Sigmoidoscopy at age 12, then every year

Personal History of Polyps


Repeat p every y 3-5 y years

Theres no test that reduces mortality Prostate Specific Antigen (PSA) Doesnt reduce mortality Useful in tracking current disease Digital Rectal Exam (DRE) Doesnt reduce mortality
PSAs require a conversation with the patient regarding pros and cons
MTBS2CK p.357358

LungCancerScreening

SummaryofPreventiveMedicineKeyPoints

Not recommended at this time Chest radiograph, CT scan, MRI, PET, etc. All have their purpose but NOT in screening
Any question asking who and when to screen for lung cancer is a trick

Mammography >50 to age 75 Tamoxifen for multiple 1st degree relatives BRCA is NOT clear! Pap smear interval to 5 years after age 30, if combined with HPV testing No lung or prostate screening clearly effective

MTBS2CK p.358

MTBS2CK p.358

LipidScreening Different by age and gender


Men start at 35 yo Women start at 45 yo Frequency depends on CVD status

NonCancerScreeningand Prevention
LipidScreening Hypertension:DiagnosisandControl DiabetesMellitus OtherScreenings

What is the test?


Cholesterol panel
Total CHL, HDL, LDL LDL guides therapy usually We do NOT know what to do with Triglycerides!

MTBS2CK p.358

LipidScreening Normal Values?


Determined by whats tolerable given risk

LipidScreening Goals of Therapy


HDL Cholesterol > 40

Interventions
Diet, Exercise, Pharmacology

0-1 Risk:
Diet at LDL >160 Drugs at LDL >190

Goals differ based on comorbidities


Diabetes HTN Coronary disease or equivalent
Coronary Artery Disease (CAD) equivalents: Carotid disease, peripheral arterial disease (PAD), diabetes mellitus, and aortic disease
MTBS2CK p.358

2 or more risks
Diet at LDL >130 Drugs at LDL >160

For CAD and Equivalents


LDL Cholesterol < 100 Both diet and drugs
Possibly < 70 for CAD combined with Diabetes!
MTBS2CK p.358

Hypertension All adults (>18) should be screened at every visit At least every two years HTN
Elevated BP at 2 separate occasions Not clear what to do about Pre-hypertension

DiabetesScreening Who is screened?


Those with CHD Hypertension is the clearest risk to screen for DM
Test
FastingBloodGlucose(FBG) OralGlucose ToleranceTest(OGTT) HemoglobinA1C

Diabetes 126mg/dL 200mg/dL 6.5%

MTBS2CK p.358

MTBS2CK p.359

Osteoporosis Bone density 2.5 SDs below average Recommendation


Every woman screened at 65 yo Dual-Energy X-ray Absorptiometry (DEXA) scan

OtherScreenings Abdominal Aortic Aneurysms


All men 65 -75 ANY smoking history Receive abdominal ultrasounds at least once

Osteoporosis Fractures Increased Mortality

Tobacco (traditional or smokeless)


Employ the Five-As Ask, Advise, Attempt, Assist, Arrange

Intimate Partner Violence


Not to be overlooked

MTBS2CK p.361

MTBS2CK p.361

AlcoholDependence Alcohol
Know difference between dependence and abuse CAGE Questions
Do they feel the need Cut down? Do they feel Annoyed with criticism of drinking? Have they ever felt Guilty by their drinking? Do they ever need an Eye-opener?

Vaccinations
InfluenzaandPneumococcal Varicella HepatitisA&B Tetanus Meningococcal

MTBS2CK p.361362

InfluenzaVaccination Influenza A and B


Strains circulating in previous year Inactivated (killed) injection

PneumococcalVaccine 23-valent Pneumococcal Polysaccharide Vaccine Protects against Streptococcal pneumoniae Who gets it?
Chronically ill
Respiratory, heart, kidney, and liver diseases

Who gets it?


Everybody Every year Healthcare workers, workers pregnant women women, elders
INFLUENZA VACCINE Every person Every year

Immunocompromised
Asplenia

Elderly: 65 yo

Revaccination after 5 years

MTBS2CK p.359

MTBS2CK p.359

Varicella&HerpesZosterVaccination Live attenuated vaccine against varicella virus Who gets Chicken Pox vaccination?
ALL adults seronegative for varicella infection

VaricellaVaccination

Who gets Herpes Zoster vaccination?


ALL adults 60 yo
Reduces incidence and severity

Herpes Zoster vaccine Chicken Pox vaccine


Different strengths and indications
Vaccines given to persons 60 to prevent zoster have 18 times viral antigens of pediatric vaccines
MTBS2CK p.360

Richard Usatine, M.D. Used with permission.

HepatitisVaccination
HepatitisA Fecaloraltransmission Infectionrelatedtopoor hygieneandcrowding Nocarrierstate Low mortality/morbidity HepatitisAVaccine Chronicliverdisease MSMorIVdrugusers Infectedclosecontacts Travelers
MTBS2CK p.360

HepatitisVaccination
HepatitisB Percutaneous,perinatal, andsexualroutes (+)Carrierstate 10%developchronic disease HepatitisBVaccine ChronicLiverdisease MSMorIVdrugusers Infectedclosecontacts Healthcareworkers Dialysispatients
MTBS2CK p.360

HepatitisBVaccine DIABETESisanindication!

TetanusVaccine Tetanus Toxoid booster every 10 years One Tdap (Tetanus Acellular Pertussus) should be one of them Five years after a dirty wound

MeningococcalVaccination Quadrivalent conjugate polysaccharide vaccine


A,C,Y,W

Very efficacious Who gets it?


ALL children age 11 Adults:
Asplenia or equivalent immunodeficiency Epidemic settings (military, college dormitories)

MTBS2CK p.360

MTBS2CK p.360

ImportantDefinitions
Prevalence: Incidence: Sensitivity (SN): Total number of diseased patients in the population. Number of NEW cases during a specified period of time. Probability that person with disease has a positive test. Probability that person without disease has a negative test. Of those with a positive test, what proportion have disease?

Epidemiology
Sensitivity&Specificity PositiveandNegativePredictiveValues ChangingtheCutoff? ImportantFormulas

Specificity (SP): Positive Predictive Value (PPV):

Negative Predictive Of those with a negative test, what proportion do not have disease? Value (NPV):

The2x2Table
(+) D (+) Test a (-) D b

The2x2Table:SensitivityandSpecificity
(+) D (+) Test a (-) D b

(-) ( ) Test est

(-) ( ) Test est

Sensitivity (SN):

Probability that person with disease has a positive test. = a / (a+c)

The2x2Table:SensitivityandSpecificity
(+) D (+) Test a (-) D b

The2x2Table:PPVandNPV
(+) D (+) Test a (-) D b

(-) ( ) Test est

(-) ( ) Test est Positive Predictive Value (PPV):

Specificity (SP):

Probability that person without disease has a negative test. = d / (b+d)

Of those with a positive test, what proportion have disease? = a / (a+b)

The2x2Table:PPVandNPV
(+) D (+) Test a (-) D b

The2x2Table:SN,SP,PPV,NPV
(+) D (+) Test a (-) D b PPV a / (a+b) NPV d / (c+d)

(-) ( ) Test

(-) ( ) Test

Negative Predictive Of those with a negative test, what proportion do not have disease? Value (NPV): = d / (c+d)

SN a / (a+c)

SP d / (b+d)

FormulasList
Sensitivity (SN): Specificity (SP): Positive Predictive Value (PPV): a / (a + c) d / (b + d) a / (a + b) a b

EpidemiologyinPractice

Disease c d

No disease

Negative g Predictive d / (c + d) Value (NPV): False Negative Ratio: False Positive Ratio: c / (a + c) or (1 SN) b / (b + d) or (1 SP) Test Cutoff

EpidemiologyinPractice

EpidemiologyinPractice

Positive Test Disease No disease Disease

Negative Test No disease

Test Cutoff

Test Cutoff

EpidemiologyinPractice

MovingtheCutoff

Positive Test Disease

Negative Test No disease Disease No disease

Test Cutoff

Test Cutoff

MovingtheCutoff

MovingtheCutoff

Positive Test Disease

Negative Test No disease

Positive Test Disease

Negative Test No disease

Test Cutoff

Test Cutoff

Howdoesthisaffect:SP,SN,PPV,NPV?
Positive Test Negative Test

ImportantEpidemiologyConcepts
(+) D (-) D b a

(+)T
Disease No disease

Sensitivity and specificity are independent of disease prevalence


Characteristics of test

(-)T
Positive Test Negative Test

PPV and NPV are dependent on disease prevalence

SN:
Disease No disease

SP: PPV: NPV:

ImportantEpidemiologyConcepts When using a particular test:


Raising the Cutoff
Increases Sensitivity and NPV Decreases Specificity and PPV

FormulasList
Sensitivity (SN): Specificity (SP): Positive Predictive Value (PPV): Negative Predictive Value (NPV): False Negative Ratio: False Positive Ratio: Positive Likelihood Ratio: Negative Likelihood Ratio: a / (a + c) d / (b + d) a / (a + b) d / (c + d) c / (a + c) or (1 SN) b / (b + d) or (1 SP) (SN) / (1 SP) (1 SN) / SP

Lowering the Cutoff


Increases Specificity and PPV Decreases Sensitivity and NPV

Epidemiology
Sensitivity&Specificity PositiveandNegativePredictiveValues Changingthecutoff? ImportantFormulas

Xrays RADIOLOGY
MatthewKinney,MD
OrthopedicResident UniversityofCaliforniaSanDiego

ChestXrayIndications Subjective pulmonary complaints


Cough SOB/dyspnea Pleuritic chest pain Hemoptysis

ChestXrayIndications Objective pulmonary findings


Rales (Crackles) Rhonchi Wheezing Hyperresonance/ yp dullness to percussion Chest wall tenderness Tracheal deviation SVC syndrome
JVD Plethora
Source: James Heilman, MD

MTBS2CK p.491

MTBS2CK p.491

ChestXrayViews Posterior/anterior (PA) films


Standard CXR positioning Patient must be capable of standing

ChestXrayViews Anterior/posterior (AP) films


Necessary for immobile patients
ICU Paralysis

Source: commons.wikimedia.org

Source: Mikael Hggstrm

MTBS2CK p.491

MTBS2CK p.491

ChestXrayViews Lateral
Useful to localize lesion found on PA films Most sensitive test for pleural effusion

ChestXrayViews Lateral decubitus


Patient lies on side Useful to evaluate pleural effusions

Source: Nicholas Lange

MTBS2CK p.492

Source: Zachary M Alexander

MTBS2CK p.491

ChestXrayFindings Lobar pneumonia

ChestXrayFindings Solitary pulmonary nodule

Source: Nicholas Lange

Source: Zachary M Alexander

MTBS2CK p.491

MTBS2CK p.491

ChestXrayFindings Pleural effusion

ChestXrayFindings Pneumothorax

Source: James Heilman, MD

Source: Magnus Manske Source: John Yasmer

MTBS2CK p.491

MTBS2CK p.491

ChestXrayFindings Free air under diaphragm

AbdominalXrayIndications Small bowel obstruction


Mechanical obstruction Ileus

Source: Brian Johnston

MTBS2CK p.491

MTBS2CK p.492

AbdominalXrayViews Abdominal plain film


Erect view of abdomen

AbdominalXrayViews Kidneys, ureters, and bladder (KUB)


Supine view of abdomen

Source: Nevit Dilmen

Source: Nevit Dilmen

MTBS2CK p.492

MTBS2CK p.492

AbdominalXrayFindings Small Bowel Obstruction


Mechanical obstruction

BoneXrayIndications Fracture Bone tumor evaluation Osteomyelitis Skeletal Survey


Trauma Child abuse

Air fluid levels


Source: Stephanie A Bernard Source: James Heilman, MD

MTBS2CK p.492

MTBS2CK p.492

BoneXrayFindings Osteomyelitis
Periosteal elevation Osteolysis
Surrounded by ring of sclerosis

ComputedTomography(CT)

Note: XR findings are delayed (2 weeks)


MRI is positive earlier

Source: commons.wikimedia.org

MTBS2CK p.492

CTBasics
Definition Computed Tomography Large series of 2D X-rays around a single axis of rotation to create 3D image
Increased detail clarity Windowing capability

NoncontrastHeadCTIndications Severe head trauma


Loss of consciousness Altered mental status

Stroke
Hemorrhagic Ischemic

Drawbacks High doses of radiation Often requires IV contrast


Anaphylaxis risk Renal damage
Consider 1-2 liters of fluids, NaHCO3, and/or

Intracranial bleeding
Subdural hematoma Epidural hematoma

NAC if mild renal insufficiency

Medication contraindications (metformin)


MTBS2CK p.493

NoncontrastHeadCT Bone - bright white Blood - bright white CSF - dark black

NoncontrastHeadCTFindings Ischemic Stroke


Darkening of brain parenchyma Blurred Gray-White Junction Mass effect

Source: Andrew Ciscel

Source: Lucien Monfils

MTBS2CK p.493

MTBS2CK p.493

NoncontrastHeadCTFindings Hemorrhagic Stroke


Bright, localized area corresponding to bleed Mass effect Gray-white junction intact

NoncontrastHeadCTFindings Subdural Hematoma Crescent-shaped, bright white region


Not confined by skull sutures Adjacent j to skull

Mass effect

Source: commons.wikimedia.org Source: James M. Grimson

MTBS2CK p.493

MTBS2CK p.493

NoncontrastHeadCTFindings Epidural Hematoma Biconvex, bright white region


Adjacent to skull Confined by skull sutures

ContrastHeadCTIndications Infection
Meningitis Abscess

Tumor
Primary Cancer Brain Metastasis

Mass effect

Source:Hellerhoff,commons.wikimedia.org

Source:commons.wikimedia.org

MTBS2CK p.493

MTBS2CK p.493

ContrastHeadCTFindings Tumor
Primary tumor: Single lesion
Bright white Mass effect Identical to abscess

ContrastHeadCTFindings Tumor
Metastasis: Often multiple lesions
Bright white Found mostly at gray-white junction Mass effect

Source: James Heilman, MD

Source: James Heilman, MD

MTBS2CK p.493

MTBS2CK p.493

AbdominalCTIndications Retroperitoneal structure pathology


Pancreatitis Adrenal gland tumors

AbdominalCTViews

Appendicitis Diverticulitis Nephrolithiasis Abdominal viscera masses


Liver tumors

Splenic laceration NOTE: Use both oral and IV contrast


MTBS2CK p.493 MTBS2CK p.493
Source: commons.wikimedia.org

AbdominalCTFindings Pancreatitis
Irregular pancreatic outline Pancreatic enlargement Retroperitoneal fluid

AbdominalCTFindings Appendicitis
Appendix enlargement (> 6mm) Wall thickening Fat stranding

Appendix dilatedto15 mmthat doesntfillwith contrast.


Source: James M Grimsom

MTBS2CK p.493

MTBS2CK p.493

Source:AndrewSellers

AbdominalCTFindings Diverticulitis Localized bowel wall thickening Fat stranding Evidence of diverticulae Note: If advanced, abscess may be present

AbdominalCTFindings Nephrolithiasis Evidence of hydronephrosis Visible calcification in (if obstructing) urinary tract Note: No oral or IV contrast

Source: Phil Kang

MTBS2CK p.493

Source:Hellerhoff,commons.wikimedia.org

MTBS2CK p.493494

ChestCTIndications Mass lesions


Lung Mediastinum

NormalChestCT

Lung pathology
Interstitial lung disease Sarcoidosis Bronchiectasis Cavitary lesions

Pulmonary embolism

Source: Brendan T Doherty

MTBS2CK p.494

MTBS2CK p.494

MRIBasics Definition Magnetic Resonance Imaging


1) Magnetic field aligns H20 molecular dipoles 2) RF waves are applied, which alters proton spin 3) When waves are stopped, the protons relax to their natural spin -- producing a measurable signal Upshot: Allows visualization of tissue based on water content Best test for evaluating soft tissue

MagneticResonanceImaging, Ultrasonography,&Nuclear Medicine

Drawbacks
Long scans requiring complete immobility Certain metal implants are contraindicated Small bore tubes claustrophobia + body mass issues
MTBS2CK p.494

MRIIndications CNS pathology


Brain/spinal cord tumors MS Specific regions of the head Sinuses Orbits

MRIFindings Primary Brain Tumor Contrast enhancing Solitary lesion Mass effect

Musculoskeletal disease
Osteomyelitis Soft tissue injury Nerve compression Herniated disc disease Brachial plexus injury
Source: Steven J. Goldstein

MTBS2CK p.494

MTBS2CK p.494

MRIFindings Multiple Sclerosis Contrast-enhanced Demyelinated plaques appear white

MRIFindings Spinal Disc Herniation Disc extruding from IV space Compression of spinal cord

MTBS2CK p.494

Source: Jeffery Hirsch

MTBS2CK p.494

USBasics Definition Ultrasonography


Sound waves of a certain frequency emitted Echoes return based on specific tissue properties Dense objects (e.g., stones, tissue) reflect waves better than fluid/air Scanner measures echoes and creates image based on differences in object density

Ultrasonography

Drawbacks
Poor at visualizing structures beyond bone Poor at visualizing structures beyond air Body habitus affects image quality

MTBS2CK p.494495

USIndications Gallstone disease Renal disease


PCKD Hydronephrosis Note: CT preferred for kidney stones

USFindings Cholecystitis
Presence of gallstone Stone in lumen Silhouette Gall bladder wall thickening Pericholestatic fluid Sonographic Murphys sign
Source: Joseph Lagrew

Gynecologic evaluation
PCOS Uterine evaluation Pregnancy evaluation Ectopic pregnancy

Pancreatic disease (via endoscopic US)


MTBS2CK p.494495 MTBS2CK p.494495

NuclearScansBasics Definition
Radiolabeled molecules localize to specific organs Emission detected and allow for visualization of organ function PET scan = Glucose Cholescintigraphy = HIDA V/Q scan
Source: Jeffrey Hirsch

NuclearMedicine

MTBS2CK p.495

NuclearScanTypes HIDA (Hepatobiliary) scan

NuclearScanTypes Ventilation/perfusion (V/Q) scanning

V
Source: Jonathan Sexton

Source: Kieran Maher

MTBS2CK p.495

MTBS2CK p.495

NuclearScanTypes Bone scan


Uptake into osteoblasts Useful for detecting area of high bone turnover Occult cancer metastasis

Indium scan
Uptake into WBCs Useful in detecting fever of unknown origin (FUO)

Gallium scan
Uptake with iron metabolism Useful for detecting FUO and some cancers

Nuclear Ventriculography
Used to measure cardiac ejection fraction
MTBS2CK p.495

RHEUMATOLOGY
NiketSonpal,MD
ChiefResident LenoxHillHospitalNSLIJ AssistantClinicalProfessor TouroCollegeofMedicine

Osteoarthritis,Gout,& Pseudogout

Osteoarthritis/Definition

Osteoarthritis/Etiology

Osteoarthritis or degenerative joint disease (DJD) Chronic, slowly progressive, erosive damage to joint surfaces Loss of articular cartilage

Incidence directly proportional to increasing age and trauma to joint


Contact sports with trauma

Obesity increases DJD


DJD is, by far, the MCC of joint disease.

MTBS2CK p.167

MTBS2CK p.167

Osteoarthritis/Presentation

Osteoarthritis/Presentation

Most commonly symptomatic in weightbearing joints Hand is affected, but isnt as great a cause of disability Distal interphalangeal (DIP) joints are > proximal interphalangeal joints (PIP) and metacarpophalangeal joints (MCP)

Crepitations of involved joints are common Effusion is rare Stiffness is of short duration (<15 min) DIP enlargement: l t Heberdens H b d nodes d PIP enlargement: Bouchards nodes

MTBS2CK p.167

MTBS2CK p.167

Osteoarthritis
Heberdens nodes (DIP joint)

Osteoarthritis/DiagnosticTests
Erythrocyte sedimentation rate
Bouchards nodes (PIP joint)

Complete blood count t

Normal Lab Tests

Antinuclear antibody tib d

Rheumatoid factor
Richard Usatine, M.D. Used with permission

MTBS2CK p.167

Osteoarthritis/DiagnosticTests
Dense subchondral bone

Osteoarthritis

Joint space narrowing

X-rays y Show:

Osteophytes

Bone cysts

MTBS2CK p.168

Source: Russell A. Patterson

Osteoarthritis/DiagnosticTests

Osteoarthritis/Treatment

Absence of inflammation, normal lab tests, and short duration of stiffness distinguishes DJD from RA

1. Weight loss and moderate exercise 2. Acetaminophen: best initial analgesic 3. NSAIDs: used if symptoms arent controlled with acetaminophen; toxicity - GI bleeding 4. Capsaicin p cream 5. Hyaluronic acid injections 6. Intra-articular steroids 7. Joint replacement Glucosamine and
chondroitin sulfate are no more effective than placebo.

MTBS2CK p.168

MTBS2CK p.168

Gout/Definition/Etiology

Gout/Etiology
Idiopathic
Overproduction

Defect in urate metabolism 90% of cases in men Can be from:


Overproduction or Underexcretion

Enzyme deficiency

Increased cell turnover

Renal Insufficiency
Underexcretion

Acidosis

Thiazides or ASA
MTBS2CK p.168 MTBS2CK p.168

Gout/Presentation

Gout

Man who develops sudden, excruciating pain, redness, and tenderness of big toe at night after binge drinking with beer Fever is common Hard to distinguish initial gouty attack from infection without arthrocentesis Metatarsal phalangeal (MTP) joint of great toe most frequently affected Also in ankles, feet, and knees
Richard Usatine, M.D. Used with permission.

MTBS2CK p.168

Gout/Presentation Chronic Gout Tophi: tissue deposits of urate crystals with foreign body reaction Most often tophi occur in cartilage, subcutaneous tissues, bone, and kidney Often take years to develop Uric acid kidney stones occur in 5% to 10% of patients Long asymptomatic periods between attacks are common

Gout/DiagnosticTests Most accurate test:


Aspiration of joint

Needle-shaped crystals with negative birefringence on polarized light microscopy


Commons.Wikimedia.org. used with permission

Tophi can occur anywhere in the body.

MTBS2CK p.169

MTBS2CK p.169

Gout/DiagnosticTests

Gout/DiagnosticTests

WBC joint fluid elevated 2,000 to 50,000/L Predominantly neutrophils Infected joint has redness, warmth, and tenderness Its essential to tap joint to exclude infection Protein and glucose levels in synovial fluid dont help answer the most likely diagnosis question

Uric acid levels: elevated at some point in 95% of patients Single level during an acute attack normal in 25% Acute attacks = ESR and leukocytosis

MTBS2CK p.169

MTBS2CK p.169

Gout/DiagnosticTests X-rays:

Gout/Treatment Acute Attack NSAIDs superior to colchicine as best initial therapy Corticosteroids injection: single joint , oral: multiple joints Steroids (e (e.g., g triamcinolone) is answer when:
No response to NSAIDs Contraindication to NSAIDs such as renal insufficiency

Normal in early disease Erosions of cortical bone happen later

Contraindication questions are always clear


Source: Wikimedia.org

MTBS2CK p.169

MTBS2CK p.169

ChronicManagement
Management 1) Diet
Decrease consumption of alcohol, particularly beer; lose weight Decrease high-purine foods

Gout/Treatment

2) Stop thiazides, aspirin, and niacin 3) Colchicine is effective at preventing second attack of gout 4) Probenecid and sulfinpyrazone increase the excretion of uric acid in kidney (uricosuric) 5) Allopurinol decreases production of uric acid
MTBS2CK p.169170

Colchicine gives diarrhea and bone marrow suppression (neutropenia). Probenecid, NSAIDs, and sulfinpyrazone are contraindicated in renal insufficiency. Allopurinol is safe with renal injury.
MTBS2CK p.170

Gout/Treatment Adverse Effects of Chronic Treatment Hypersensitivity (rash, hemolysis, allergic interstitial nephritis) occurs with uricosuric agents and allopurinol Colchicine can suppress white cell production Toxic epidermal necrolysis or Stevens-Johnson Stevens Johnson syndrome may occur from allopurinol Dont start uricosuric agents or allopurinol during acute attacks of gout. If the patient is already on allopurinol you can safely continue it.
MTBS2CK p.170

CalciumPyrophosphateDepositionDisease (Pseudogout)/Definition/Etiology

Calcium-containing salts depositing in articular cartilage Most common risk


Hemochromatosis and hyperparathyroidism

CPDD can occur:


Diabetes Hypothyroidism Wilson disease

MTBS2CK p.170

CalciumPyrophosphateDepositionDisease (Pseudogout)/Presentation

CalciumPyrophosphateDepositionDisease (Pseudogout)/DiagnosticTests

CPDD differs from gout in that large joints such as the knee and wrist are affected, but not particularly the first MCP of the foot It differs from DJD in that DIP and PIP arent affected

Uric acid levels normal X-ray: calcification of cartilaginous structures Most accurate test is arthrocentesis, which reveals p positively y birefringent g rhomboidshaped crystals Synovial fluid: elevated level of WBCs 2,000 to 50,000/L - nonspecific

MTBS2CK p.170

MTBS2CK p.170

Pseudogout

Pseudogout

Source Wikimedia Source: Boma O. Afiesimama

CalciumPyrophosphateDepositionDisease (Pseudogout)/DiagnosticTests

Disease DJD

Characteristic History Older,slow, worsewithuse

Physical Findings

SynovialFluid Analysis

You cannot confirm a diagnosis of CPDD without aspiration of the joint.


Gout

DIP,PIP,hip, <200WBCs, andknees osteophytesand jointspace narrowing 1st bigtoe 2,00050,000 WBCs,negatively birefringent needles

Men,acute, bingedrinking

MTBS2CK p.170

MTBS2CK p.171

Disease CPDD

Characteristic History

Physical Findings

SynovialFluid Analysis 2,00050,000 WBCs,positively birefringent rhomboids

Disease Septic arthritis

Characteristic History Highfever, veryacute

Physical Findings Singlehot joint

SynovialFluid Analysis >50,000 neutrophils, cultureoffluid

Hemochromatosis, Wristsand hyperparathyroidism knees

Rheumatoid Young,female, arthritis morningstiffness betterwithuse

Multiplejoints Anticyclic ofhandsand citrulinated feet peptide(anti CCP)

MTBS2CK p.171

MTBS2CK p.171

CalciumPyrophosphateDepositionDisease (Pseudogout)/Treatment

Best initial therapy: NSAIDs Severe disease not responsive to NSAIDs give intra-articular steroids (e g triamcinolone) (e.g., Colchicine helps prevent subsequent attacks as prophylaxis between attacks

LowBackPain&Lumbar SpinalStenosis

MTBS2CK p.171

LowBackPain/Etiology

LowBackPain

Low back pain is No. 1 complaint in US DJD on X-ray or MRI of the spine is nearly universal in those > 50 years totally nonspecific

What Is the Most Likely Diagnosis? If all diseases described in the following are excluded, patient has simple low back pain from lumbosacral strain (idiopathic) No imaging studies and no treatment beyond NSAIDs

Most frequently tested issue is who shouldnt get an imaging study


MTBS2CK p.171 MTBS2CK p.171

CompressionoftheSpinalCord

CompressionoftheSpinalCord

Malignancy or infection compressing spinal cord is a neurological emergency that needs urgent identification and treatment Look for a history of cancer with sudden onset of focal neurological deficits (e.g., sensory level)

Compression at 10th thoracic vertebra leads to sensory loss below the umbilicus Point tenderness at spine with percussion of vertebra is highly suggestive of cord compression Hyperreflexia is found below level of compression

MTBS2CK p.171

MTBS2CK p.172

CompressionoftheSpinalCord

DiskHerniation(Sciatica)

Epidural abscess is most often from Staphylococcus aureus Epidural abscess presents in same way as cord compression from cancer, but theres there s high fever and markedly elevated ESR

Herniations at L4/5 and L5/S1 level account for 95% of all disk herniations The straight leg raise (SLR) test is pain going into the buttock and below the knee when the leg is raised > 60 degrees
Although only 50% of those with a positive SLR actually have a herniated disk; sensitivity is 90% A negative SLR excludes herniation with 95% sensitivity
MTBS2CK p.172

MTBS2CK p.172

NerveRootInnervation
Nerveroot Motordeficit Reflex affected(lost) Sensoryarea affected

LowBackPain/DiagnosticTests

L4

Dorsiflexion offoot Dorsiflexion oftoe Eversionof foot

Kneejerk

Innercalf

L5

None

Inner forefoot Outerfoot

S1

Anklejerk

Imaging required for cord compression, epidural abscess, ankylosing spondylitis, and cauda equina syndrome Best initial test for cancer with compression, infection, and fractures is plain X-ray Most accurate test is MRI CT scan is used as most accurate test if theres a contraindication to MRI (e.g., pacemaker) Intrathecal contrast must be given to increase accuracy (CT myelogram)

MTBS2CK p.172

MTBS2CK p.172

LowBackPain/DiagnosticTests Imaging in disk herniation is controversial We recommend you answer no MRI for just low back pain and a positive SLR alone Neurological deficits = MRI

ClassificationofBackPain
Diagnosis Cord Compression Historytoanswer MostLikelyDiagnosis Historyofcancer PhysicalFindings Vertebraltenderness, sensorylevel, hyperreflexia Sameascord compression Bilaterallegweakness, saddleareaanesthesia

Epiduralabscess

Fever highESR Fever,

Caudaequina

Bowelandbladder incontinence, erectiledysfunction

MTBS2CK p.172173

Source: Nirav Thakur, MD.

MTBS2CK p.173

ClassificationofBackPain
Diagnosis Ankylosing spondylitis Historytoanswer MostLikelyDiagnosis Underage40,pain worsenswithrest andimproveswith activity Pain/numbnessof medialcalforfoot PhysicalFindings Decreasedchest mobility

LowBackPain/Treatment
Chemotherapy for Lymphoma

Systemic Glucocorticoids Gl ti id Lossofkneeandankle reflexes,positive straightlegraise

Cord Compression p

Diskherniation

Radiation for Solid Tumors

MTBS2CK p.173

MTBS2CK p.174

LowBackPain/Treatment

LowBackPain/Treatment

MRSA
Acute Neurologic Deficits

Vancomycin Linezolid

Beta-lactam antibiotics when organism is sensitive Gentamicin = synergy with staph Surgical drainage for larger collections
Thi k of Think f epidural id l abscess b lik like endocarditis d diti Use vancomycin as initial empiric therapy Switch to oxacillin if its sensitive Drain it if the infection is large enough to produce neurological deficits or it doesnt respond to antibiotics alone

Epidural p Abscess

Systemic Glucocorticoids

MSSA

Oxacillin Nafcillin Cefazolin

MTBS2CK p.174

MTBS2CK p.174

LowBackPain/Treatment

Cauda equina syndrome: surgical decompression Disk herniation (sciatica): NSAIDs with continuation of ordinary activities Steroid injection into epidural space achieves rapid id and dd dramatic ti b benefit fit f for th those with ith sciatica Surgery rarely needed The most common wrong answer for sciatica is bed rest.
MTBS2CK p.174

Man with a history of prostate cancer comes to the emergency department with severe back pain and leg weakness. He has tenderness of the spine, hyperreflexia, and decreased sensation below his umbilicus. What is the most appropriate next step? a. Dexamethasone b. MRI Save neurons before seeing them c. X-ray Low specificity d. Radiation Not fast enough e. Flutamide Not fast enough to save neurons f. Ketoconazole Not fast enough to stop androgens g. Finasteride Not fast enough to stop androgens h. Leuprolide Dangerous with peripheral blockade i. Biopsy Only if cause unclear j. Orchiectomy Best long-term treatment

MTBS2CK p.174175

LowBackPain/Treatment

LumbarSpinalStenosis/Presentation

Most commonly tested point: no imaging studies in patients without focal neurological abnormalities or with simple lumbosacral strain

Look for a person > 60 with back pain while walking, radiating into buttocks and thighs bilaterally Pain described as worse when walking downhill, and better when sitting, but the pedal pulses and ankle/brachial index are normal
Spinal stenosis can simulate peripheral arterial disease, but vascular studies are normal.

MTBS2CK p.175

MTBS2CK p.175

LumbarSpinalStenosis/Presentation

LumbarSpinalStenosis/Treatment

Unsteady gait and leg weakness when walking also occur About have diminished lower extremity reflexes Pain is less with activities in which patient is leaning forward (e.g., cycling)

The only test is MRI Weight loss and steroid injections into the lumbar epidural space improve 25% to 50% of cases Surgical correction to dilate the spinal canal is needed in 75% of patients

MTBS2CK p.175

MTBS2CK p.175

Fibromyalgia

Fibromyalgia,CarpalTunnel Syndrome,&Dupuytren Contracture

What Is the Most Likely Diagnosis? Look for young woman with chronic musculoskeletal pain and tenderness with trigger points of focal tenderness at trapezius, medial fat pad of knee, and lateral epicondyle Cause is unknown Pain occurs at many sites (neck, shoulders, back, and hips) with:
Stiffness, numbness, and fatigue Headaches Sleep disorder
MTBS2CK p.176

Fibromyalgia/DiagnosticTests

Fibromyalgia/Treatment

No test to confirm fibromyalgia Based on complex of symptoms, trigger points at predictable points All lab tests are normal (e.g., ESR, Creactive protein protein, rheumatoid factor (RF), (RF) and CPK levels)

Best initial therapy is amitriptyline Other treatments are milnacipran and pregabalin Milnacipran is an inhibitor of serotonin and norepinephrine reuptake and is approved specifically for the management of fibromyalgia Trigger point injections with local anesthetic are also sometimes used Steroids are the wrong answer for fibromyalgia.

MTBS2CK p.176

MTBS2CK p.176

10

CarpalTunnelSyndrome/Definition

CarpalTunnelSyndrome/Etiology
Amyloidosis
Acromegaly Hypothyroidism

Peripheral neuropathy from compression of median nerve as it passes under the flexor retinaculum Pressure on nerve interferes with its sensory and motor function

Median Nerve Compression

Pregnancy

Rheumatoid Arthritis

Diabetes

MTBS2CK p.176

MTBS2CK p.176

CarpalTunnelSyndrome

CarpalTunnelSyndrome/DiagnosticTests

What Is the Most Likely Diagnosis? Look for pain in hand affecting the palm, thumb, index finger, and radial half of ring finger, muscle atrophy of thenar eminence Pain is worse at night and more frequent in those whose work involves prolonged use of the hands (e.g., typing)

Tinel sign: reproduction of pain and tingling with tapping or percussion of the median nerve
IMC 2010 DxR Development Group, Inc. All Rights Reserved.

MTBS2CK p.177

MTBS2CK p.177

CarpalTunnelSyndrome/DiagnosticTests

CarpalTunnelSyndrome/DiagnosticTests

Phalen sign: reproduction of symptoms with flexion of wrists to 90 degrees

Most accurate diagnostic tests are electromyography and nerve conduction testing

Sensory symptoms happen before motor symptoms.


IMC 2010 DxR Development Group, Inc. All Rights Reserved.

MTBS2CK p.177

MTBS2CK p.177

11

CarpalTunnelSyndrome/Treatment
Management Best Initial
Wrist Splints Immobilization to relieve pressure NSAIDS

DupuytrenContracture Hyperplasia of palmar fascia leading to nodule formation and contracture of fourth and fifth fingers Genetic predisposition and association with alcoholism and cirrhosis Patients lose ability to extend fingers, which is more often cosmetic embarrassment than functional impairment Triamcinolone injection Surgical release when function is impaired

Avoid Manual Activity Steroid St id Injection is used if splints and NSAIDs dont control Surgery Can be curative by mechanically decompressing the tunnel such as with cutting open the flexor retinaculum
MTBS2CK p.177

MTBS2CK p.177

DupuytrenContracture

SportsMedicine& Osteoporosis

Commons.Wikimedia.org. used with permission

RotatorCuffInjury

RotatorCuffInjury

Damage to rotator cuff of muscles, tendons, and bursae around shoulder leads to inability to flex or abduct the shoulder It presents with pain in shoulder thats that s worse at night when lying on affected shoulder There can be severe tenderness at the insertion of the supraspinatus
MTBS2CK p.177

MRI is the most accurate test Treat with NSAIDs, rest, and physical therapy Steroids Surgery
Commons.Wikimedia.org. used with permission

MTBS2CK p.177

12

PatellofemoralSyndrome

PatellofemoralSyndrome

Cause of anterior knee pain secondary to trauma, imbalance of quadriceps strength, or meniscal tear Pain in front of knee or underneath patella Particularly bad when walking up or down stairs Worse just after starting to walk after having been seated for a prolonged period It improves after walking

Crepitus, joint locking, and instability X-rays: normal Most cases respond to physical therapy and strength training with cycling Knee K b braces d dont t help h l Theres nothing to fix surgically

MTBS2CK p.178

MTBS2CK p.178

PlantarFasciitis

PlantarFasciitis

Very severe pain in bottom of foot near calcaneus where fascia inserts Pain worst in the morning and improves with walking a few steps Point tenderness @ the fascia inserts at the calcaneus

Commons.Wikimedia.org. used with permission

MTBS2CK p.178

PlantarFasciitis

Osteoporosis

Treatment consists of stretching exercises, arch supports, and NSAIDs Steroid injection is performed if these dont solve problem Surgical release of plantar fascia is rarely necessary X-ray of the foot is not useful in plantar fasciitis. There is no correlation with the presence of heel spurs.
MTBS2CK p.178

Look for an older person, more often a woman, with vertebral fractures leading to loss of height or wrist fracture Asymptomatic, fractures are found on routine screening with bone densitometry, which is recommended for all women > 65
Osteoporosis gives spontaneous fractures of weight-bearing bones.
MTBS2CK p.198

13

Osteoporosis/DiagnosticTests

Osteoporosis/Treatment

Most accurate test is bone densitometry (DEXA) scanning Osteopenia: bone density (T-score) is between 1 and 2.5 standard deviations below normal Osteoporosis: T-score > 2.5 standard deviations < normal

1. Vitamin D and calcium are the best initial therapy 2. Bisphosphonates (alendronate, risendronate, ibandronate) 3 Estrogen replacement 3. 4. Raloxifene

MTBS2CK p.198

MTBS2CK p.198

Osteoporosis/Treatment

Osteoporosis/Treatment

1. Teriparatide is an analogue of PTH that stimulates new bone matrix formation 2. Used as a nasal spray, calcitonin decreases vertebral fractures risk
Bisphosphonates are very rarely associated with osteonecrosis of the jaw. Bisphosphonates that have prolonged contact with the esophagus can cause esophagitis (pill esophagitis).

When multiple treatment options are presented, choose vitamin D, calcium, and bisphosphonates.
Teriparatide has caused osteosarcoma in rats. It has also caused hypercalcemia.
MTBS2CK p.198

MTBS2CK p.198

RheumatoidArthritis/Definition/Etiology

RheumatoidArthritis,Systemic LupusErythematosus,& p p p Syndrome y Antiphospholipid

RA is an autoimmune disorder predominantly of joints More common in women

MTBS2CK p.178

14

RheumatoidArthritis/Definition/Etiology

RheumatoidArthritis/Presentation
PIP Hands MCP Hands

Chronic synovitis leads to overgrowth, or pannus formation, which damages all the structures surrounding the joint (bone, ligaments, tendons, and g ) cartilage)
Morning stiffness of multiple small, inflamed joints is key to diagnosis.
MTBS2CK p.178

Morning Stiffness > 30 min

Bilateral Symmetrical

Rheumatoid Nodules

Rheumatoid Arthritis

Vasculitis

Episcleritis

Lung nodules and effusions

MTBS2CK p.178

RheumatoidArthritis/Presentation

RheumatoidArthritis/Presentation Boutonniere and swan neck are classic deformities of the hands

C1 and C2 laxity - subluxation Baker cyst Pericarditis and pleural disease Carpal tunnel syndrome
DIP is spared in RA. DIP involvement happens in DJD.

MTBS2CK p.179

MTBS2CK p.179

Source: Nirav Thakur, MD.

RheumatoidArthritis

RheumatoidArthritis/DiagnosticTests

Rheumatoid factor (RF) in 70% to 80% RF is rather nonspecific Anti-cyclic citrulinated peptide (antiCCP) is > 80% sensitive and > 95% specific

Richard Usatine, M.D. Used with permission.

MTBS2CK p.179

15

RheumatoidArthritis/DiagnosticTests

RheumatoidArthritis/DiagnosticTests

Elevated ESR or C-reactive protein Anemia: normocytic Arthrocentesis on initial presentation excludes crystal disease or infection if diagnosis isnt clear Modest elevation in lymphocytes Sicca syndrome: dry eyes, mouth, and other mucous membranes
MTBS2CK p.179

Felty syndrome: RA Splenomegaly Neutropenia Caplan syndrome: RA Pneumoconiosis Lung nodules


MTBS2CK p.180

RheumatoidArthritis/DiagnosticTests

RheumatoidArthritis/Treatment

The most important issue in RA is stopping the progression of the disease. Any patient with erosive disease or X-ray abnormalities needs at least methotrexate to slow disease progression. The MCC of death in RA is coronary artery disease.

Disease-Modifying Antirheumatic Drugs (DMARD) Neither NSAIDs nor steroids stop RA from progressing Any patient with erosive RA needs DMARD as part of initial therapy

MTBS2CK p.180

MTBS2CK p.180

RheumatoidArthritis/Treatment Patient with long-standing RA is to have coronary bypass surgery. Which is most important prior to surgery?
a. Cervical spine X-ray Already diagnosed to have RA b Rheumatoid factor b. Doesnt change outcomes c. Extra dose of methotrexate Nonspecific test d. ESR e. Pneumococcal vaccination Has nothing to do with surgery

Erosive disease means: Joint space narrowing Physical deformity of joints X-ray y abnormalities

MTBS2CK p.180

MTBS2CK p.180

16

RheumatoidArthritis/Treatment

RheumatoidArthritis/Treatment

Methotrexate Best initial DMARD Adverse effects are:


Liver toxicity Bone marrow suppression Pulmonary toxicity

Tumor Necrosis Factor (TNF) Inhibitors (infliximab, adalimumab, etanercept) TNF inhibitors are first line as DMARDS Toxicity of anti anti-TNF TNF drugs:
Reactivation of TB Infection

MTBS2CK p.181

MTBS2CK p.181

RheumatoidArthritis/Treatment

RheumatoidArthritis/Treatment

Rituximab RA as a DMARD by removing CD20+ lymphocytes from circulation Excellent long-term +/- methotrexate

Hydroxychloroquine Rare as monotherapy as a DMARD More often used in combination with methotrexate as a DMARD Toxic to retina

Hydroxychloroquine leads to retinal toxicity. Do a dilated eye exam.


MTBS2CK p.181 MTBS2CK p.181

RheumatoidArthritis/Treatment

RheumatoidArthritis/Treatment

Symptomatic Control of RA NSAIDs are the best initial therapy for the pain of RA Steroids also work in a matter of hours to control the pain of RA secondary to inflammation Steroids for 2 purposes
NSAIDS arent working Bridge
MTBS2CK p.181

Use TNF inhibitors as a DMARD with methotrexate after methotrexate fails Adverse effects are mandatory for you to know Steroids dont prevent the progression of RA.
MTBS2CK p.181182

17

AdverseEffectsofRAMedications
Drug AntiTNF Hydroxychloroquine Sulfasalazine Rituximab Goldsalts Methotrexate
MTBS2CK p.182

Adverseeffect Reactivationoftuberculosis Ocular Rash,hemolysis y Infection Nephroticsyndrome Liver,lung,marrow

JuvenileRheumatoidArthritisor AdultStillDisease

Definition/Etiology Juvenile rheumatoid arthritis (JRA) is very difficult to define and theres no known etiology

MTBS2CK p.182

JuvenileRheumatoidArthritisor AdultStillDisease

JuvenileRheumatoidArthritisor AdultStillDisease/Presentation

Presentation The most important feature of JRA is the presence of high, spiking fever (often > 104 104F) F) in a young person that has no clearly identified etiology, but is associated with a rash

Features of JRA rash: Often only with fever spikes Salmon colored On chest and abdomen Other features of JRA: Splenomegaly Pericardial effusion Mild joint symptoms

MTBS2CK p.182

MTBS2CK p.182

JuvenileRheumatoidArthritisor AdultStillDisease

SystemicLupusErythematosus

Laboratory Abnormalities No clear diagnostic test; anemia and leukocytosis often present ANA is normal Ferritin level markedly elevated Treatment Half of cases improve with aspirin or NSAIDs If theres no response then use steroids
MTBS2CK p.183

Definition/Etiology Autoimmune disorder Inflammation diffusely through body

MTBS2CK p.183

18

SystemicLupusErythematosus

SystemicLupusErythematosus

Presentation Diagnosis of SLE is based on 4 of 11 known manifestations of disease Four skin-related manifestations:
1. Malar rash 2. Discoid rash 3. Photosensitivity 4. Oral ulcers

Alopecia is common in SLE, but isnt one of the official diagnostic criteria.
Richard Usatine, M.D. Used with permission.

MTBS2CK p.183

SystemicLupusErythematosus

SystemicLupusErythematosus

Presentation Joint: arthritis is present in 90% X-ray is normal Serositis: inflammation of pleura and pericardium i di chest h t pain i

Presentation Renal: any degree of abnormality can occur from mild proteinuria to end-stage renal disease requiring dialysis
Most common g glomerulonephritis p is membranous Red cell casts and hematuria occur

Neurologic: symptoms include psychosis, seizures, or stroke from vasculitis


MTBS2CK p.183 MTBS2CK p.183

SystemicLupusErythematosus

SystemicLupusErythematosus

Pneumonia, alveolar hemorrhage, and restrictive lung disease happen in SLE, but arent criteria for diagnosis g of the disease.

Ocular findings arent part of formal diagnostic criteria: Photophobia Retinal lesions ( (cotton wool spots) Blindness

MTBS2CK p.183

MTBS2CK p.184

19

SystemicLupusErythematosus

SystemicLupusErythematosus

Presentation Hematologic: hemolytic anemia is part of diagnostic criteria, but anemia of chronic disease is more commonly found Lymphopenia, leukopenia, and thrombocytopenia are also seen

Presentation Immunologic (laboratory) abnormalities - criteria include positive ANA, or any one of the following:
Anti-double-stranded DNA Anti-Sm False positive test for syphilis Positive LE cell preparation

MTBS2CK p.184

MTBS2CK p.184

SystemicLupusErythematosus

SystemicLupusErythematosus

Additional findings: Mesenteric vasculitis Raynaud phenomenon Antiphospholipid syndromes

Diagnostic tests ANA: found in 95% to 99% of cases Anti-double-stranded DNA (60%) and anti-Sm (30%):
Found Fo nd onl only in SLE Extremely specific for SLE

MTBS2CK p.184

MTBS2CK p.184

SystemicLupusErythematosus

Diagnostic tests Decreased complement levels:


Correlate with disease activity Drop further with acute disease exacerbations

Anti-SSA and anti-SSB: found in 10% to 20% of cases


Add little to diagnosis Tests most often found in Sjgren syndrome (65% of cases) Dont treat asymptomatic ANA
MTBS2CK p.184

34-year-old woman with history of SLE is admitted with pneumonia and confusion. As youre wrestling with the decision over a bolus of high-dose steroids in a person with an infection, you need to determine if this is a flare of lupus or simply an infection with sepsis causing confusion. Which of the following will help you the most?
a. Rise in anti-Sm Level doesnt change in acute disease b. Rise in ANA Level doesnt change in acute disease c. Decrease in complement and rise in anti-DS DNA d. MRI of the brain MRI doesnt diagnose regardless e. Response to steroids Not diagnostic

MTBS2CK p.184185

20

SystemicLupusErythematosus

SystemicLupusErythematosus

Treatment Acute lupus flare treated with high-dose boluses of steroids Hydroxychloroquine can control mildly chronic disease Lupus nephritis may need steroids either alone or in combination with cyclophosphamide or mycophenolate Only way to determine the severity of lupus nephritis is kidney biopsy Belimumab decreases symptoms
MTBS2CK p.185

Treatment Urinalysis is insufficient to determine severity of lupus nephritis Biopsy is the only way to diagnose simple glomerulosclerosis or scarring of the kidney, which will not respond to therapy
Young patients most commonly die of infection. In older patients, accelerated atherosclerosis makes MI the MCC of death.
MTBS2CK p.185

AntiphospholipidSyndrome/Definition

AntiphospholipidSyndrome

Idiopathic disorder with IgG or IgM antibodies made against negatively charged phospholipids The 2 main types are:
Lupus anticoagulant Anticardiolipin antibodies

Presentation/Diagnostic Tests Thromboses of both arteries and veins as well as recurrent spontaneous abortions Elevation of aPTT with a normal prothrombin time (PT) and normal INR APL = clotting + elevated aPTT and normal PT

MTBS2CK p.185

MTBS2CK p.185

AntiphospholipidSyndrome

AntiphospholipidSyndrome

Presentation/Diagnostic Tests False positive VDRL or RPR with a normal FTA Anticardiolipin antibodies - spontaneous abortion Lupus anticoagulant - elevated aPTT Best initial test is mixing study

Diagnostic Tests If the elevation in aPTT is from a clotting factor deficiency then aPTT will come down to normal If the APL syndrome antibody is present in plasma then aPTT remains elevated Most M t specific ifi test t t for f lupus l anticoagulant ti l ti is Russell viper venom test (RVVT) RVVT is prolonged with APL antibodies and doesnt correct on mixing with normal plasma

MTBS2CK p.185186

MTBS2CK p.186

21

AntiphospholipidSyndrome/Treatment

AntiphospholipidSyndrome/Treatment

Thromboses (DVT or PE) treated with heparin and warfarin as you would any other form of thrombosis with an INR of 2 to 3 Lifelong vs. 6 months primary occurrence Recurrent thrombotic episodes are treated lifelong

USMLE S2 CK questions have to be unequivocally clear. If an area is controversial, USMLE will avoid it and ask only what is clear clear. The exam will not trick you.

MTBS2CK p.186

MTBS2CK p.186

AntiphospholipidSyndrome/Treatment

Warfarin or steroids are wrong answers for preventing spontaneous abortion. Steroids arent effective.
Warfarin is contraindicated in pregnancy secondary to teratogenicity.

Scleroderma,Polymyositis,& Dermatomyositis

MTBS2CK p.186

Scleroderma(SystemicSclerosis)

Scleroderma(SystemicSclerosis)

Limited scleroderma is also known as CREST syndrome: Calcinosis Raynaud Esophageal h ld dysmotility tilit Sclerodactyly Telangiectasia

What is the most likely diagnosis? Look for a young (20s to 40s) woman (3 times more likely than men) with fibrosis of the skin and internal organs (e.g., lung kidney lung, kidney, and GI tract)

MTBS2CK p.186

MTBS2CK p.187

22

Scleroderma/Presentation

RaynaudSyndrome

Raynaud syndrome: increased vascular reactivity of fingers beginning with pain and pallor (white) or cyanosis (blue) followed by reactive hyperemia (red) Skin manifestations: fibrosis of hands, face, neck, k and d extremities; t iti t telangiectasia l i t i and d abnormalities of pigmentation occur

MTBS2CK p.187

Scleroderma

Scleroderma(SystemicSclerosis)

MTBS2CK p.187

Scleroderma/Presentation

Scleroderma/DiagnosticTests

GI: esophageal dysmotility with GERD, large-mouthed diverticuli of small and large bowel Renal: sudden hypertensive crisis Lung: fibrosis leading to restrictive lung disease and pulmonary hypertension Cardiac: myocardial fibrosis, pericarditis, and heart block; lung disease gives right ventricular hypertrophy

ANA: positive in 85% to 90%, but nonspecific ESR: usually normal SCL-70: most specific test (antitopoisomerase) Anticentromere: present in half of those with CREST syndrome Anticentromere antibodies are extremely specific for CREST syndrome.
MTBS2CK p.187

MTBS2CK p.187

23

Scleroderma/Treatment

PolymyositisandDermatomyositis

Penicillamine is ineffective Renal crisis: ACE inhibitors Esophageal dysmotility: PPIs for GERD Raynaud: calcium-channel blockers Pulmonary fibrosis: cyclophosphamide improves dyspnea and PFTs Pulmonary hypertension is treated like primary pulmonary hypertension with bosentan or ambrisentan (endothelin antagonist) or Sildenafil
MTBS2CK p.187

Presentation Proximal muscle weakness They dont affect facial or ocular muscles as occurs in myasthenia gravis Dysphagia

MTBS2CK p.188

PolymyositisandDermatomyositis

PolymyositisandDermatomyositis

Presentation Dermatomyositis presents with: Malar involvement Shawl sign: erythema of face, neck, shoulders, , upper pp chest, , and back Heliotrope rash: edema and purplish discoloration of eyelids Gottron papules: scaly patches over the back of hands, particularly PIP and MCP joints
MTBS2CK p.188
Commons.Wikimedia.org. used with permission

PolymyositisandDermatomyositis

PolymyositisandDermatomyositis

Commons.Wikimedia.org. used with permission Commons.Wikimedia.org. used with permission

24

PolymyositisandDermatomyositis/ Presentation

PolymyositisandDermatomyositis

Dermatomyositis is associated with cancer in 25% of cases. Common sites are: Ovary y Lung GI Lymphoma

Diagnostic Tests Best initial test is CPK and aldolase Most accurate test is muscle biopsy ANA is frequently positive MRI Electromyography

MTBS2CK p.188

MTBS2CK p.188

PolymyositisandDermatomyositis

Treatment Steroids are usually sufficient When patient is unresponsive or intolerant of steroids, use: Methotrexate Azathioprine IVIG Mycophenolate Hydroxychloroquine helps skin lesions
MTBS2CK p.188189

SjgrenSyndrome, Vasculitis,&Seronegative Spondyloarthropathies

SjgrenSyndrome/Definition/Etiology

SjgrenSyndrome/Presentation

Idiopathic autoimmune disorder secondary to antibodies predominantly against lacrimal and salivary glands 90% of those affected are women Sjgren syndrome is associated with:
RA SLE Primary biliary cirrhosis Polymyositis Hashimoto thyroiditis
MTBS2CK p.189

Sjgren presents with dryness of mouth and eyes Keratoconjunctivitis sicca Need to constantly drink water Dysphagia D h i Dental caries

MTBS2CK p.189

25

SjgrenSyndrome/Presentation

SjgrenSyndrome/DiagnosticTests

Less common manifestations are: Vasculitis Loss of vaginal secretions Lung disease leads to dyspareunia. Pancreatitis Renal tubular acidosis (20%)

Best initial test is called a Schirmer test Most accurate test is a lip or parotid gland biopsy Reveal lymphoid infiltration in salivary glands

Lymphoma is the most dangerous complication of Sjgren


MTBS2CK p.189 MTBS2CK p.189190

SjgrenSyndrome/DiagnosticTests

SjgrenSyndrome/Treatment

Best initial test on blood: SS-A and SS-B These are also called Ro and La and each are present in about 65% of patients
SLE is associated with SS-A and SS-B in 10% to 20% of cases Other abnormalities that are present, but are nonspecific: ANA, RF, anemia, leukopenia, and eosinophilia
MTBS2CK p.190

Best initial therapy is to water the mouth Use frequent sips of water, sugar-free gum, and fluoride treatments Artificial tears to avoid corneal ulcers Pilocarpine and cevimeline increase acetylcholine t l h li , the th main i stimulant ti l t t to produce d saliva Cevimeline increases rate of saliva production No cure Evaluate for lymphoma
MTBS2CK p.190

Vasculitis

PolyarteritisNodosa/Definition

Etiology unknown Symptoms develop over weeks to months All vasculitides give:
Fever Fe er Malaise/fatigue Weight loss Arthralgia/myalgia
MTBS2CK p.190

Polyarteritis nodosa (PAN) is a disease of small- and medium-sized arteries leading to a diffuse vasculitis that inexplicably spares the lungs Chronic hepatitis B and C are associated with PAN

MTBS2CK p.190

26

PolyarteritisNodosa/Presentation

PolyarteritisNodosa/Presentation

Common Features of PAN Renal: glomerulonephritis without a biopsy cant be diagnosed


UA isnt enough to confirm its PAN

Gastrointestinal: abdominal pain is worsened by eating from vasculitis of mesenteric vessels


Bleeding also occurs Nausea and vomiting are common

Neurological: any large peripheral nerve can be involved Peroneal neuropathy leading to foot drop Look for a stroke in a young person
MTBS2CK p.190191

Skin: lower extremity ulcers are most common; livedo reticularis, purpura, nodules, and rarely gangrene occur
Lung is spared in PAN
MTBS2CK p.191

PolyarteritisNodosa/Presentation

PolyarteritisNodosa/DiagnosticTests

Mononeuritis Multiplex Mononeuritis multiplex is multiple peripheral neuropathies of nerves large enough to have a name

Most accurate test is a biopsy of a symptomatic site Angiography of renal, mesenteric, or hepatic artery shows abnormal dilation or beading. P-ANCA is present in < 20% Test all PAN patients for hepatitis B and C.

MTBS2CK p.191

MTBS2CK p.191

PolyarteritisNodosa/Treatment

PolymyalgiaRheumatica

Prednisone and cyclophosphamide Treat hepatitis when found

Polymyalgia rheumatica (PMR) occurs in those over age 50 with:


Pain and stiffness in shoulder and pelvic girdle muscles Difficulty combing hair and rising from chair Elevated ESR Normochromic, normocytic anemia

No Lab Findings CPK and aldolase are normal Steroids even at low doses great response
MTBS2CK p.191 MTBS2CK p.191

27

GiantCell(Temporal)Arteritis

GiantCell(Temporal)Arteritis

The difference is the presence of: Visual symptoms Jaw claudication (pain in jaw when chewing) Scalp tenderness Headache Symptoms in other arteries such as decreased arm pulses, bruits near the clavicles, or aortic regurgitation
MTBS2CK p.191192

ESR and C-reactive protein are elevated Most accurate test is a biopsy of affected artery (e.g., temporal artery) Treat with prednisone Starting high-dose prednisone quickly is more important than waiting for biopsy
Blindness is irreversible.

MTBS2CK p.192

WegenerGranulomatosis

WegenerGranulomatosis/Diagnostic Tests

Presents with:
Sinusitis Otitis media Mastoiditis Oral and gingival involvement

Best initial test is antineutrophil cytoplasmic antibody (ANCA) Most accurate test is a biopsy Cytoplasmic antibodies are also called CANCA.
C-ANCA = anti-proteinase-3 antibodies P-ANCA = anti-myeloperoxidase antibodies Wegener: C-ANCA Churg-Strauss and microscopic polyangiitis: P-ANCA
MTBS2CK p.192

Wegener is also associated with skin, joint, and eye lesions Look for combination of upper and lower respiratory tract findings in association with renal insufficiency
MTBS2CK p.192

WegenerGranulomatosis/Diagnostic Tests

WegenerGranulomatosis/Treatment

When asked about the best test for Wegener, lung biopsy is better than renal biopsy with sinus biopsy being the least accurate When all 3 are in the choices choose lung biopsy

Treat with prednisone and cyclophosphamide The clue to answering the most likely diagnosis question is unresolving diagnosis pneumonia not better with antibiotics. You will not first think of Wegener when presented with the case.

MTBS2CK p.192

MTBS2CK p.192

28

ChurgStraussSyndrome

HenochSchnleinPurpura

Pulmonary-renal syndrome, ChurgStrauss also has: Asthma Eosinophilia Biopsy Bi i is the th most t accurate t test t t Treat with prednisone and cyclophosphamide

Vasculitis more frequently seen in children, Henoch-Schnlein purpura (HSP) is characterized by involvement of:
GI tract: p pain, , bleeding g Skin: purpura Joint: arthralgia Renal: hematuria

MTBS2CK p.193

MTBS2CK p.193

HenochSchnleinPurpura

HenochSchnleinPurpura When the case describes leukocytoclastic vasculitis on biopsy, the answer is Henoch-Schnlein purpura. Leukoplastic reactions are painless, palpable purpura of buttocks and legs
MTBS2CK p.193
Source: Shreya Patel and Nishith Patel

HSP is most often a clinical diagnosis; however, biopsy is the most accurate test
Serum IgA levels are the wrong answer. They Th are unreliable when testing for Henoch-Schnlein purpura.

MTBS2CK p.193

HenochSchnleinPurpura/Treatment

Cryoglobulinemia

Most cases resolve spontaneously Steroids if there are severe extrarenal mainfestations associated with renal failure

Most commonly associated with chronic hepatitis C +/- endocarditis and/or Sjgren syndrome Dont confuse cryoglobulins with cold agglutinins

Both are IgM antibodies


Neither respond to steroids

MTBS2CK p.193

MTBS2CK p.194

29

DifferencesbetweenCryoglobulinsandColdAgglutinins Cryoglobulins Associatedwith HepatitisC Coldagglutinin EBV,Mycoplasma, Lymphoma Hemolysis

Cryoglobulinemia Lab tests show a positive rheumatoid factor and cold precipitable immune complexes Steroids NOT effective Treat the underlying cause, especially hepatitis C, with interferon and ribavirin Despite the rarity of the condition, the USMLE loves cryoglobulinemia questions.

Manifestations

Jointpain Glomerulonephritis Purpuricskinlesions Neuropathy Interferon, Ribavirin,and boceprevir (or telaprevir)

Treatment

MTBS2CK p.194

Staywarm Rituximab, cyclophosphamide, cyclosporine

SLE decreased C3 or 3 letters (SLE) = C3 Hep C decreased C4 or 4 letters (Hep C) = C4 .


MTBS2CK p.194

BehetSyndrome

BehetSyndrome

The most common Behet questions are:


What is the most likely diagnosis? What is pathergy? pathergy ?
Pathergy: sterile skin pustules from minor trauma (e.g., needle stick)
MTBS2CK p.194

Asian or Middle Eastern person with painful oral and genital ulcers +/erythema Also with:
Ocular lesions leading to uveitis and blindness Arthritis CNS lesions mimicking multiple sclerosis
MTBS2CK p.194

BehetSyndrome/Treatment

SeronegativeSpondyloarthropathies

Corticosteroids To wean patients off of steroids, use:


Azathioprine Cyclophosphamide Colchicine C l hi i Thalidomide

The 3 types of seronegative spondyloarthropathies are:

Ankylosing spondylitis Psoriatic arthritis Reactive arthritis (Reiter syndrome)

MTBS2CK p.195

MTBS2CK p.195

30

SeronegativeSpondyloarthropathies

SeronegativeSpondyloarthropathies

Men < 40 years: Involvement of spine and large joints Negative rheumatoid factor (hence the name seronegative) Enthesopathy (inflammation where tendons and ligaments attach to bones) Uveitis HLA-B27

Corticosteroids arent a good treatment for seronegative spondyloarthropathy

D Despite it th the association i ti with ith HLA HLA-B27, B27 this is never the best initial or most accurate test for seronegative spondyloarthropathies.
MTBS2CK p.195

MTBS2CK p.195

AnkylosingSpondylitis/Diagnosis

AnkylosingSpondylitis/Diagnosis

Young man with low backache and stiffness of his back has pain that radiates to buttocks with flattening of the normal lumbar curvature and decreased chest expansion Eventually E t ll th the spine i will ill not t expand di in any direction Enthesopathy occurs at the Achilles tendon Look for back pain worsened by rest and relieved by activity
MTBS2CK p.195

Other Findings of Ankylosing Spondylitis Transient peripheral arthritis of knees, hips, and shoulders (50%) Cardiac: atrioventricular block in 3% to 5%; aortic insufficiency Uveitis
Bamboo spine is a late finding with fusion of vertebral joints.

MTBS2CK p.195

AnkylosingSpondylitis/DiagnosticTests

AnkylosingSpondylitis/DiagnosticTests

Best initial test is an X-ray of sacroiliac (SI) joint Most accurate test is an MRI MRI detects abnormalities years before the X X-ray ray becomes abnormal ESR is elevated in 85%

MTBS2CK p.196

MTBS2CK p.196

31

AnkylosingSpondylitis/DiagnosticTests

AnkylosingSpondylitis

HLA B27 is not a confirmatory diagnostic test since 8% of general population is positive Treatment Exercise p program g and NSAIDs are best initial treatment If NSAIDs are insufficient, use anti-TNF drugs (e.g., etanercept, adalimumab, or infliximab)
MTBS2CK p.196

Source: Conrad Fischer, MD.

MTBS2CK p.196197

PsoriaticArthritis

PsoriaticArthritis

Psoriatic arthritis 80% will have preceding psoriasis Besides SI joint involvement, characteristic findings are: Sausage digits from enthesopathy Nail pitting

MTBS2CK p.197

MTBS2CK p.197

PsoriaticArthritis/DiagnosticTests

PsoriaticArthritis/Treatment

ESR is elevated nonspecific Best initial test is an X-ray of the joint showing a pencil in a cup deformity

NSAIDs are best initial therapy Methotrexate used when question describes severe disease or no response to NSAIDs Anti-TNF Anti TNF agents are the answer when methotrexate doesnt control disease Steroids are a wrong choice

MTBS2CK p.197

MTBS2CK p.197

32

ReactiveArthritis(ReiterSyndrome)

ReactiveArthritis/Diagnosis

Reactive arthritis occurs secondary to: Inflammatory bowel disease (equal sex incidence) Sexually transmitted infection (far greater in men) GI infection (Yersinia, Salmonella, Campylobacter)

Look for triad of: Joint pain Ocular findings (uveitis, conjunctivitis) Genital G it l abnormalities b liti ( (urethritis, th iti balanitis) Keratoderma
blennorhagicum is a skin lesion unique to reactive arthritis that looks like pustular psoriasis.

MTBS2CK p.197

MTBS2CK p.198

ReactiveArthritis/Diagnosis

ReactiveArthritis/DiagnosticTests/Treatment

No specific test for reactive arthritis Rule out septic joint Treat underlying cause/use NSAIDs Sulfasalazine > NSAIDS

Antibiotics dont reverse reactive arthritis once joint pain has started.
Source: commons.wikimedia.org. Used with permission

MTBS2CK p.198

SepticArthritis

SepticArthritis,Gonococcal Arthritis,&Osteomyelitis

Definition Septic arthritis is an infection of joint space Etiology Septic arthritis is relatively rare in an undamaged joint Risk of infection is directly proportional to degree of joint damage

MTBS2CK p.199

33

SepticArthritis

EtiologyofSepticArthritis Etiology Frequency

Etiology (contd) Osteoarthritis (DJD) provides slight risk RA has greater risk Greatest risk is with prosthetic joint Bacteremia can spread into joint space, space which is why endocarditis and injection drug use causes septic arthritis

Staphylococcus Streptococcus Gramnegative rods

40% 30% 20%

MTBS2CK p.199

MTBS2CK p.199

SepticArthritis/Presentation

SepticArthritis/DiagnosticTests

Joint is warm, red, and immobile often with palpable effusion Chills and fever happen because of bacteremia

Best initial and most accurate test is aspiration of the joint with a needle (arthrocentesis); X-ray, CT, and MRI arent useful and are the wrong answers Joint fluid shows:
Leukocytosis: more than 50,000 to 100,000 cells, predominantly neutrophils Gram stain: positive (50%) Gram-negative bacilli; (75%) with Staphylococcus Synovial fluid culture: 70% to 90% sensitive Blood cultures: 50% sensitive

MTBS2CK p.199

MTBS2CK p.199

SepticArthritis

OtherOptionsforTreatmentofSepticArthritis Gramnegative bacilli Grampositivecocci (sensitive) Grampositive cocci(resistant)

Treatment Ceftriaxone and vancomycin are best initial empiric therapy

Quinolones Aztreonam Cefotaxime Piperacillin Aminoglycosides

Oxacillin Cefazolin Piperacillinwith Tazobactam

Linezolid Daptomycin Ceftaraline Tigecycline

MTBS2CK p.199

MTBS2CK p.200

34

SepticArthritis/Treatment

SepticArthritis

Adjust antibiotics according to culture results.


If Staphylococcus p y is sensitive, , vancomycin is associated with a worse outcome than betalactam antibiotic (e.g., oxacillin or cefazolin). Switch drugs if organism is sensitive.

Prosthetic Joint Infection Infected prosthetic joint gives a warm, red, immobile, and tender joint Must do imaging

MTBS2CK p.200

MTBS2CK p.200

SepticArthritis

SepticArthritis

Prosthetic Joint Infection (contd) MRI difficult to perform with prosthetic joints because they are made of metal If there is lucency around implantation of the joint on radiologic imaging or if joint is physically loose, infection is likely present at implantation site

Treatment of Infected Prosthetic Joint Remove joint, treat with antibiotics for 6 to 8 weeks, and then replace joint The most common organism for recently placed artificial joints is Staphylococcus epidermidis.

MTBS2CK p.200

MTBS2CK p.200

GonococcalArthritis(Gonorrhea)

GonococcalArthritis/DiagnosticTests

The difference in presentation from septic arthritis is:


Polyarticular involvement Tenosynovitis (inflammation of tendon sheaths, making finger movement painful) Petechial rash

Detecting gonorrhea is much more difficult than detecting Staphylococcus, Streptococcus, and Gram-negative bacilli of septic arthritis

Gonococcal arthritis is more frequent during menses.


MTBS2CK p.200 MTBS2CK p.200

35

SynovialFluidAnalysisforInfectiousArthritis Testsensitivity Leukocytosis Gramstain C l Culture Bloodcultures Septicarthritis >50,000100,000cells/L 5070%sensitive 90%sensitive ii 50%sensitive Gonococcalarthritis 30,00050,000cells/L 25%sensitive <50%sensitive ii <10%sensitive

GonococcalArthritis/DiagnosticTests

In order to reach maximum sensitivity, multiple diffuse sites must be cultured for gonorrhea such as: Pharynx What tells you to Rectum Rectum culture everywhere? Urethra Rash Cervix Tenosynovitis
Polyarticular involvement

MTBS2CK p.201

MTBS2CK p.201

GonococcalArthritis/Treatment

Osteomyelitis/Definition/Etiology

Ceftriaxone, cefotaxime, or ceftizoxime is the best empiric therapy for disseminated gonorrhea Fluoroquinolones are not the best initial therapy If recurrent gonorrhea infection is described, test for terminal complement deficiencya favorite subject of USMLE Step 2 CK.

Osteomyelitis is an infection of bone Staphylococcus aureus is MCC Children get osteomyelitis through hematogenous spread, but adults get it from a contiguous (nearby) infection Salmonella is the most commonly identified organism in patients with sickle cell disease.
MTBS2CK p.201

MTBS2CK p.201

Osteomyelitis/DiagnosticTests

Osteomyelitis/DiagnosticTests

Best initial test is an X-ray Most accurate test is a biopsy If X-ray is normal, the most appropriate next step in management is MRI CT scan isnt very y useful

When is ESR the answer? To follow the response to therapy


MTBS2CK p.202
Source: Eva M. Smietana

36

Osteomyelitis/DiagnosticTests

Osteomyelitis/Treatment

When is culturing the drainage the answer? Never. You cannot reliably distinguish superficial colonization from whatever organism is inside the bone causing the bone infection. Bone scan is the answer only if you want to get an MRI and its contraindicated (pacemaker).
MTBS2CK p.202

Osteomyelitis takes weeks to progress Must biopsy MSSA Oxacillin, cefazolin, nafcillin, or ceftriaxone MRSA Vancomycin or linezolid

MTBS2CK p.202

Osteomyelitis/Treatment

Osteomyelitis/Treatment

Gram-negative bacilli such as E. coli are treated with fluoroquinolones (e.g., ciprofloxacin)
Its essential to confirm the sensitivity of the organism prior to treating with ciprofloxacin
Ciprofloxacin is the only oral therapy for osteomyelitis, but should be used only if the organism is confirmed as a sensitive Gram-negative bacillus.
MTBS2CK p.202

Toxicity of Quinolones Fluoroquinolones can cause Achilles tendon rupture


They are also contraindicated in pregnancy and children because they interfere with bone growth

MTBS2CK p.202

37

Surgery
MatthewKinney,MD OrthopedicResident UniversityofCaliforniaSanDiego

PreoperativeEvaluation, PostoperativeEvaluation,& VascularSurgery

PreoperativeEvaluation Objective: Identify factors that increase risk of complications in perioperative period Age Cardiac history
#1 predictor of perioperative complications

PreoperativeEvaluation
Cardiovascular Pulmonary y Renal

Diabetes status
Risk equivalent to Coronary disease

History of
Pulmonary disease Renal disease Stroke
MTBS2CK p.379

PreoperativeEvaluation/Cardiac Must obtain detailed cardiac history Look for indicators of previous MI or CHF Recent MI
Defer surgery for 6 months Follow-up stress test to ensure adequate perfusion

PreoperativeEvaluation/Cardiac Management If age < 35 and no history of cardiac disease


EKG

If history of cardiac disease (MI, CHF)


EKG Stress testing
Ensure appropriate cardiac perfusion

Evidence of CHF (JVD, edema)


Patients with EF < 35% at increased risk ACE-Is, B-blockers, spironolactone must be optimized Proven to decrease overall mortality!!!

Echocardiography
Monitor EF Evaluate structural damage

MTBS2CK p.379

MTBS2CK p.380

PreoperativeEvaluation/Pulmonary

71-year old man undergoing femoro-popliteal bypass for severe claudication of left leg which causes unbearable pain with exercise. Past medical/surgical history is significant for remote appendectomy and insulin-dependent type 2 DM. What preoperative testing is recommended? a.Basic Metabolic Panel (BMP) only High cardiac risk b.BMP + EKG c. BMP + EKG + PFTs Must get stress test/ECHO d.BMP + EKG + Exercise Stress Test Cannot exercise e.BMP + EKG + Thallium Stress Test

Must evaluate for history of lung disease (including smoking) PFTs required for all patients with known lung disease
Vital capacity - most important predictor of perioperative complications

MTBS2CK p.380

PreoperativeEvaluation/Pulmonary

PreoperativeEvaluation/Pulmonary Must evaluate for history of lung disease (including smoking) PFTs required for all patients with known lung disease
Vital capacity - most important predictor of perioperative complications

Smokers
PFTs Smoking cessation for 6-8 weeks preoperatively
Nicotine patch acceptable

MTBS2CK p.380

PreoperativeEvaluation/Renal Renal disease increases surgical risk


Intravascular fluid losses occur during surgery Results in hypoperfusion of kidneys Physiologic response to decreased intravascular volume is activation of Renin-Angiotensin system Constricts renal vasculature

PostoperativeEvaluation
FeverAssessment Complications

Renal hypoperfusion correlates with increased mortality Management


Aggressive IV hydration beginning 24 hours pre-op Dialysis patients must be dialyzed 24 hours pre-op
MTBS2CK p.380

PostOperativeFever
WIND WATER WALKING POD WOUND WEIRD

PostOpComplications/Confusion
Confused Patient Obtain ABG, CXR, CBC
Evidence of Hypoxemia Abnormal ABG Evidence of Infection Abnormal CBC

1-2
Atelectasis #1 Pneumonia CXR -Sputum Culture (if pneumonia suspected) -Incentive spirometry -Antibiotics Vanc. + Pip/Tazo

3-5

5-6
DVT Thrombophlebitis (IV site infection) Doppler US Anti-coagulation Heparin p Warfarin Replace IVs

8-15
Drug Reaction Deep Abscess D/C likely medication CT Scan Drainage of Abscess

Changes on CXR?
Yes No

UTI Urinalysis Nitrite + Leukocyte Esterase + Antibiotics MTBS2CK p.400

Incision-site Infection Cellulitis Physical Exam Erythema Pus Swelling Abscess Incision/Drainage Antibiotics

Culture Likely Sources Blood (Bacteremia) Urine (UTI)

Atelectasis vs. Pneumonia Incentive spirometry Antibiotics


MTBS2CK p.401

Consider PE Spiral CT

Treat with empiric Antibiotics

AdultRespiratoryDistressSyndrome Etiology Endothelial damage allows fluid to fill alveoli


Prevents O2/CO2 exchange, acts as shunt

AdultRespiratoryDistressSyndrome

Signs/Symptoms HR, RR
Labored breathing (accessory muscle use) Fever may be present

Diagnosis ABG: pO2, pCO2 CXR: Bilateral pulmonary infiltrates


MS-4 USU Teaching File, Uniformed Services University

MTBS2CK p.401

AdultRespiratoryDistressSyndrome

PulmonaryEmbolism

Treatment Mechanical ventilation


Maximize Positive End-Expiratory Pressure (PEEP)

Etiology
Passage of a venous blood clot to lungs
Origin: Deep leg vein > 90%

Clot lodges in lung vasculature


Prevents O2/CO2 exchange
Source: nlm.nih.gov

MTBS2CK p.401

PulmonaryEmbolism

Risk Factors Stasis: Immobility (post-surgical, travel, etc.), obesity Endothelial damage: Surgery, trauma Hypercoagulability yp g y: Oral contraceptive p pills (OCP), malignancy, genetic disorder Signs/Symptoms HR, RR, Temp Pleuritic chest pain
MTBS2CK p.401

62-year-old woman with no significant PMH undergoes right total hip replacement 3 days ago. Recovery is uncomplicated until 30 minutes ago, she reported moderate SOB and chest pain with deep inspiration. Whats the next step in evaluating this patient?

a. b. c. d. e.

EKG only Insensitive for pulmonary embolism EKG + V/Q Scan No reported contrast allergy EKG + Spiral CT scan EKG + D-Dimer Sensitive, but not specific EKG + Heparin Injection Must diagnose PE first

PulmonaryEmbolism Diagnosis ABG


pO2, pCO2

PulmonaryEmbolism

EKG
Nonspecific ST-segment and T wave changes
Most common

S1

S1-Q3-T3

Q3

T3

MTBS2CK p.401

PulmonaryEmbolism Diagnosis ABG


pO2, pCO2

PulmonaryEmbolism

EKG
Nonspecific ST-segment and T wave changes
Most common

S1-Q3-T3
Infrequent during acute PE Can be found in massive acute PE and cor pulmonale

Spiral CT
Source: James Heilman, MD, commons.wikimedia.org

MTBS2CK p.401

PulmonaryEmbolism Diagnosis ABG


pO2, pCO2

PulmonaryEmbolism

EKG
Nonspecific ST-segment and T wave changes
Most common

Treatment Respiratory support Anticoagulation


Heparin acutely, bridge to warfarin longterm IVC filter (if anticoagulation contraindicated)

S1-Q3-T3
Infrequent during acute PE Can be found in massive acute PE and cor pulmonale

Spiral CT
Consider V/Q scan (if IV contrast allergy)
MTBS2CK p.401 MTBS2CK p.401

Vascular
AbdominalAorticAneurysm AorticDissection Claudication

69-year-old male with 50 pack-year smoking history is brought to the ER by his wife who reports he seems confused. He feels weak and has pain in middle of abdomen. He is a pale, elderly male in moderate distress. BP of 84/55, pulse 120. Palpable, pulsatile mass in patients abdomen. Whats the most likely diagnosis? a. b. c. d. Ruptured peptic ulcer Would expect peritoneal signs Hemorrhagic gastritis Would expect hematemesis Hemorrhagic pancreatitis Would expect flank bruising Ruptured abdominal aortic aneurysm

MTB S2CK - p. 399

AbdominalAorticAneurysm Etiology Weakening of aortic wall secondary to atherosclerosis


Aortic diameter expands > 1.5x normal Involves all layers of vessel wall (True Aneurysm) 90% arise from infrarenal aorta

AbdominalAorticAneurysm Risk Factors Male > Female Age Hypertension, Hyperlipidemia Smoking Si Signs/Symptoms /S t Frequently asymptomatic May report pulsatile abdominal mass

MTBS2CK p.399

Source: csm.ornl.gov

MTBS2CK p.399

AbdominalAorticAneurysm Diagnosis CT/MRI


Provides determination of level and surrounding structures

AorticDissection Treatment 3.0-4.0 cm


US every 2-3 years

4.0-5.4 cm
US or CT every 6-12 months

Etiology Tearing of aortic intima forms false lumen


Blood flows into false f lumen, extends tear
Sopurce: J. Heuser commons.wikimedia.org

Ultrasound for size measurements

5.5 cm, asymptomatic


Surgical repair

MTBS2CK p.399

AorticDissection/Etiology

AorticDissection

Can occur in ascending or descending aorta

Risk Factors Male > Female Age > 40 Hypertension (#1 risk factor) ) Marfans disease, Ehlers-Danlos syndrome

Signs/Symptoms Sudden onset, tearing chest pain


Radiates to the back

Elevated BP
May be asymmetric (R > L)

Sopurce: J. Heuser commons.wikimedia.org

AorticDissection/Diagnosis

AorticDissection

Demonstration of dissection on imaging CXR


Widening of mediastinum

Source: NNMC

AorticDissection/Diagnosis

AorticDissection

Demonstration of dissection on imaging CXR


Widening of mediastinum

Transesophageal ECHO (TEE)


Acute chest pain and/or clinically unstable

MRA
Chronic chest pain and hemodynamically stable

Treatment Ascending dissection = Emergent surgery Descending dissection = Medical therapy


Beta-Blockers (#1) Anti-hypertensive meds

CT angiogram
TEE and MRI contraindicated

Claudication Etiology Atherosclerotic plaques prevent sufficient perfusion to extremities (lower > upper)
Associated with smoking, DM, hyperlipidemia

Claudication/Management

Medical
Risk Modification Smoking cessation (#1) Graded exercise Pharmacologic therapy Cilostazol Cil t l Antiplatelet agents: Aspirin, Clopidogrel

Symptoms Calf/leg pain with exercise


Relieved by rest

Diagnosis Ankle-Brachial Index


< 0.9 = pathologic < 0.4 = symptomatic

Percutaneous
Stenting, angioplasty

Surgical

Trauma/ABCAssessment Initial assessment rely on ABC algorithm

Shock
TraumaAssessment(ABCs) yp ofShock Types Hypovolemic Cardiogenic Neurogenic Septic Anaphylactic

Airway Breathing Circulation Disability (CNS) Exposure ABCs are a roadmap, but you must know what to do at each step
MTBS2CK p.380381

Trauma/ABCAssessment A = Airway Must assess for airway compromise


If patient can talk airway is clear Look for traumatic obstruction, evidence of smoke inhalation

Trauma/ABCAssessment
Concern for airway compromise? AMS Facial trauma Apnea
Yes Intubate

No

Evaluate need for intubation


Indications
Facial Trauma Altered Mental Status Apnea

Assess Breathing/ Oxygenation

Yes

Facial Trauma?
No

Cricothyroidotomy
Yes

Cervical Spine Injury?


No

Careful Orotracheal Intubation Flexible bronchoscopy


MTBS2CK p.381

Orotracheal Intubation

MTBS2CK p.381

Trauma/ABCAssessment B = Breathing Assess breath sounds Monitor oxygenation status with pulse oximetry
Goal is O2 Saturation > 90% If O2 Sat < 90% consider
a. Supplemental O2 via nasal cannula b. O2 face mask c. Intubation

Trauma/ABCAssessment

C = Circulation Evaluate pulses (distal first, proximal if absent) Manage hemorrhage sites
Direct pressure slows blood loss

Monitor blood pressure


If the patient is hypotensive, place 2 largebore IVs and begin aggressive fluid resuscitation Start with normal saline (only use blood products as secondary measure)
MTBS2CK p.381

Source:UusiAjaja, commons.wikimedia.org

MTBS2CK p.381

Trauma/ABCAssessment D = Disability (Altered Mental Status) Assessed with Glasgow Coma Scale
Eye Response (1-4) Verbal Response (1-5) Motor Response (1-6)

Score < 8 requires intubation E = Exposure Remove all clothing on patient


Assess for hidden injuries

Thorough physical examination

52-year-old woman ejected from her car during a high-speed motor vehicle accident . Upon arrival to ED, she complains of severe, left-sided chest pain. Pale, cool patient in severe distress. Heart rate 130 bpm, BP 86/44 mmHg. JVD along angle of jaw. Chest X-ray shows anterior rib fractures on the left. Which is the most likely diagnosis? a. Hypovolemic shock Would not see JVD b. Neurogenic shock c. Anaphylactic shock d. Cardiogenic shock Warm, flush patients e. Septic shock
MTBS2CK p.381

Shock Definition Inadequate perfusion/oxygenation that impairs organ function Signs/Symptoms Vitals Signs
Decreased BP Increased HR

TypesofShock Hypovolemic Causes: Hemorrhage (#1), Dehydration, Burns

CNS: Confusion, altered mental status Kidney: Decreased urine output, increased BUN/Cr ratio Liver: Massively elevated AST/ALT (Shock Liver) Heart: Chest pain, shortness of breath
MTBS2CK p.381 MTBS2CK p.382
Source: U.S. Navy photo by Mass Communication Specialist 2nd Class Michael Russell, public.navy.mil

TypesofShock/Hypovolemic Signs/Symptoms
Pale, cold Trauma

TypesofShock

Cardiogenic
Causes: MI (#1), CHF, arrhythmia Signs/Symptoms
Pale, cold Symptoms associated with MI (Chest pain, SOB) JVD

Lab Findings
CO = SVR = CVP = PCWP =

Lab Findings
SVR = CVP = PCWP = CO =

Treatment
Treat cardiac problem Do NOT give fluid!!!

Treatment
Aggressive IV fluid replacement Pressors
MTBS2CK p.382

MTBS2CK p.382

TypesofShock

TypesofShock Lab Findings


SVR = CVP = PCWP = CO =

Neurogenic
Causes: CNS damage (cervical/thoracic spinal cord - #1) Signs/Symptoms:
Warm, , flush Evidence of CNS damage (trauma)

Septic Causes: Infection


E. coli and S. aureus (most common organisms)

Treatment
Aggressive IV fluid delivery Pressors

Signs/Symptoms
Warm, flush Possible nidus of infection (UTI, pneumonia, wound)
Source: cdc.gov

MTBS2CK p.382

MTBS2CK p.382

TypesofShock/Septic

TypesofShock

Lab Findings SVR = CVP = CO = PCWP =

Treatment Broad-spectrum antibiotics Fluid and pressors Dopamine Norepinephrine

Anaphylactic Causes: Allergy (insects, food, medication) Signs/Symptoms


Warm, flush Wheezing, hives, associated evidence of allergic reaction

Source: James Heilman, MD, commons.wikimedia.org

MTBS2CK p.382

MTBS2CK p.382

TypesofShock/Anaphylactic

ShockAlgorithm
Pale/Cold

Lab Findings
CVP = SVR = PCWP = CO =
Elevated

Is patient pale/cold or warm/flush?


Decreased

Warm/Flush

PCWP change?
Decreased

CO change?
Elevated

Treatment
Epinephrine

Cardiogenic Treat cardiac problem

Hypovolemic Fluid and pressors

Neurogenic Fluid and pressors

PCWP change?
Decreased No Change

Anaphylactic Epinephrine

Septic Antibiotics Fluids and pressors

MTBS2CK p.382

AbdominalTrauma

Trauma
AbdominalTrauma ThoracicTrauma PelvicTrauma

Penetrating Gunshot wounds Must do exploratory laparotomy in ALL patients Stab wounds If hemodynamically stable, do a FAST ultrasound scan If hemodynamically unstable, perform an exploratory laparotomy

10

AbdominalTrauma

AbdominalTrauma

Diaphragmatic Rupture

Cause
Penetrating or blunt trauma
Left > Right

Symptoms
Respiratory distress Kehrs sign = Left shoulder pain

Loops of Bowel

Diagnosis
CXR: Bowel loops in thorax
Absent Hemi-Diaphragm
MTBS2CK p.383
Source: Hariharan D, Singhal R, Kinra S, Chilton A, commons.wikimedia.org

AbdominalTrauma/Blunt

AbdominalTrauma/Blunt

Splenic/Liver injury Causes Most commonly injured in blunt abdominal trauma


Spleen #1 Liver #2

Pancreatic Injury Causes Blunt trauma to epigastrum


Bike handlebars Car dashboard

Associated with lower rib fractures Signs/Symptoms Hypotension (due to hemorrhage) Kehrs sign
MTBS2CK p.383

Signs/Symptoms Cullens sign = Bruising around umbilicus

MTBS2CK p.383

AbdominalTrauma/Blunt

AbdominalTrauma/Blunt Diagnosis FAST scan (ultrasound)


To evaluate for intraabdominal bleeding
Fluid = Blood

CT scan If negative FAST FAST, but suspect splenic rupture To evaluate retroperitoneal bleed
Source: Herbert L. Fred, MD and Hendrik A. van Dijk, commons.wikimedia.org

Source: commons.wikimedia.org

MTBS2CK p.383

MTBS2CK p.383

11

AbdominalTrauma/Blunt

Management If hemodynamically stable


Close monitoring Serial abdominal exams IV fluids

33-year-old female jogging at night was struck by a drunk driver. She complains of severe abdominal pain that radiates to the back. Vital signs are stable. After ER work-up, patient is admitted for evaluation. Three days later, the following picture is taken.

If hemodynamically unstable
Exploratory laparotomy

MTBS2CK p.383

33-year-old female jogging at night was struck by a drunk driver. She complains of severe abdominal pain that radiates to the back. Vital signs are stable. After ER work-up, patient is admitted for evaluation. Three days later, the following picture is taken. What is the most likely diagnosis?

Source: Herbert L. Fred, MD and Hendrik A. van Dijk, commons.wikimedia.org

a. b. c. d. e.

Pancreatic pseudocyst Delayed appearance Hemorrhagic pancreatitis Ruptured AAA No pulsatile mass Aortic dissection Stable BP, no chest pain Splenic rupture Wrong side, no L shoulder pain

ThoracicTrauma Pneumothorax Etiology Air in pleural space pulmonary collapse Signs/Symptoms Chest pain Decreased breath sounds Hyperresonance to percussion Tracheal deviation toward affected side

ThoracicTrauma/Pneumothorax

Diagnosis CXR

Collapsed Lung Border

Treatment Chest tube

Source: John Yasmer

MTBS2CK p.384

MTBS2CK p.384

12

ThoracicTrauma TensionPneumothorax AMEDICALEMERGENCY Etiology


Chest wall defect acts as one-way valve
Air can enter pleural space, but cannot exit Resulting increase in pressure can compress vital structures Trachea, vena cava
Source: army.mil

ThoracicTrauma/TensionPneumothorax
Resulting from laceration of cervical trachea and mediastinal pleura Resulting from sucking wound of valvular type

a. Laceration of cervical trachea b. Laceration of mediastinal pleura c. Tension pneumothorax d. Collapsed lung e. Shift of heart f. Subcutaneous emphysema g. Partial collapse of contralateral lung

a. Valvular sucking wound of chest wall b. Tension pneumothorax c. Collapsed lung d. Cardiac shift to intact side e. Partial collapse of contralateral lung

MTBS2CK p.384

ThoracicTrauma/TensionPneumothorax Signs/Symptoms
Chest pain Hyperresonance Decreased breath sounds Tracheal deviation (away from affected side)

ThoracicTrauma

Diagnosis
CXR

Source: James G. Smirniotopoulos, M.D.

Treatment
Immediate needle decompression Chest tube placement
MTBS2CK p.384

Hemothorax Etiology Blood in pleural space Signs/Symptoms g y p Chest pain Absent breath sounds Dullness to percussion
MTBS2CK p.384

Source: army.mil

ThoracicTrauma/Hemothorax

ThoracicTrauma/Hemothorax
White-Out

Diagnosis CXR
Blunted costophrenic angle

CT scan Treatment Chest tube drainage Thoracotomy


Posteroanterior CXR made shortly after wounding and showing hemothorax 10 days later, after 6 thoracenteses, three 150 cc of blood and other fluid was removed
Source: army.mil

MTBS2CK p.384

13

Hemo/PneumothoraxDiagnosis
Symptoms
Chest Pain Decreased/Absent Breath Sounds

ThoracicTrauma
Pericardial Tamponade

Etiology
Trauma to pericardium
Broken ribs Penetrating trauma

Response to Percussion?
Dullness Hyperresonance Tracheal Deviation? Toward lung Away from lung

Signs/Symptoms
JVD Hypotension Decreased heart sounds

Hemothorax
Chest tube drainage

Pneumothorax
Chest tube

Tension pneumothorax
Immediate needle thoracotomy Chest tube

MTBS2CK p.384

Source: James Heilman, MD, commons.wikimedia.org

ThoracicTrauma/PericardialTamponade Diagnosis
EKG
Electrical Alternans

ThoracicTrauma/PericardialTamponade Diagnosis
EKG
Electrical Alternans

ECHO
Diagnostic test of choice

Treatment
Pericardiocentesis

Source: army.mil

MTBS2CK p.384

MTBS2CK p.384

26-year-old man suffers blow to chest with a baseball bat. He is brought to the ED with severe right-sided chest pain and difficulty breathing. He is tachypneic at 26 breaths/minute, with a heart rate of 130 beats/minute, diminished breath sounds on the right, and left tracheal deviation. What is the next step in management?

14-year-old boy hits a pothole while riding his bike and falls directly onto the central bar. He presents to the ED with severe groin pain and swelling. Physical examination reveals blood at the urethral meatus and a high-riding prostate. What is the next appropriate step in management?

a. b. c. d. e.

Pericardiocentesis Not pericardial tamponade Chest X-ray Too slow. This is an emergency Needle thoracotomy Chest tube placement EKG Not indicated in this patient

a. a b. c. d. e.

Place a Foley Can C f further th damage d urethra th Get a retrograde urethrogram Empiric antibiotics Must assess urethral patency CBC and electrolytes Discharge the patient with reassurance

MTBS2CK p.385

14

AbdominalPainDifferentialDiagnosis
RUQ LUQ

TheAbdomen Part1
AbdominalPain MesentericIschemia EsophagealPathology GastricPerforation

Cholecystitis Splenic Rupture - Radiates to R shoulder - Radiates to L shoulder Cholangitis Ischemic Bowel Disease Perforated Ulcer Mid-Epigastrum Pancreatitis Peptic Ulcer Disease Aortic Dissection - Radiates to back

RLQ
Appendicitis Ovarian Torsion () Ectopic Pregnancy () Diverticulitis (Cecal)
MTBS2CK p.387

LLQ
Diverticulitis (Sigmoid) Sigmoid Volvulus Ovarian Torsion () Ectopic Pregnancy ()

ChronicMesentericIschemia
66-year-old man in the ED with acute onset, severe abdominal pain. Abdominal exam benign, without guarding or rebound tenderness. White count 18,000/mm3 (4,500-11,000/mm3), lactic acid 4.2 mg/dL (4.5 to 19.8 mg/dL). Only medication is coumadin. Whats the most appropriate next step in management? a. MRI scan of f th the abdomen bd T k too Takes t long l b. Exploratory Laparotomy c. Colonoscopy Risk of bowel perforation d. Oral antibiotics Doesnt treat primary concern e. EKG Symptoms not consistent with cardiac etiology

Etiology Mesenteric artery atherosclerosis insufficient blood flow to bowel


Patients often have other atherosclerotic diseases
Angina Claudication
MTBS2CK p.385386
A lack of blood flow causes ischemia to the bowel wall and sloughing of the mucosa. Source: Niket Sonopal, MD.

ChronicMesentericIschemia

ChronicMesentericIschemia Diagnosis CT abdomen (initial test)


Fast, non-invasive

Signs/Symptoms Diffuse, postprandial abdominal pain


Severe, crampy, associated nausea Due to O2 requirement by gut after meals May lead to weight loss due to fear of eating

Bloody diarrhea Progressive disease begins with mild ischemia, progresses to full occlusion of blood flow
MTBS2CK p.386

CT angiography is most accurate Treatment Surgical revascularization via arterial bypass


NPO Hydration with IV fluids Bowel prep 24 hours preceding surgery

Nitrates may provide short-term symptom improvement


MTBS2CK p.386

15

AcuteMesentericIschemia

AcuteMesentericIschemia

Etiology Acute occlusion of mesenteric arteries


Most commonly the superior mesenteric Causes:
Embolism secondary to A A-fib fib (#1)

MTBS2CK p.386

Source: Mikael Hggstrm from original image created and uploaded by Dr. I-Chen Tsai, commons.wikimedia.org

AcuteMesentericIschemia

AcuteMesentericIschemia Signs/Symptoms Sudden onset, severe abdominal pain


Pain out of proportion to exam

Etiology Acute occlusion of mesenteric arteries


Most commonly the superior mesenteric Causes:
Embolism secondary to A A-fib fib (#1) Acute thrombus formation on atherosclerotic plaque

Diagnosis Labs: WBC, pH, Lactate Abdominal X-ray/CT


Air in bowel wall (Pneumatosis ( Intestinalis) Edema Ileus

Nausea, , vomiting g Bloody diarrhea

Results in bowel infarction


Splenic flexure (#1) and hepatic flexure (#2) are most common sites
MTBS2CK p.386

CT angiography (Most accurate)

AcuteMesentericIschemia

AcuteMesentericIschemia

Treatment
Emergent Laparotomy Resection of necrotic bowel

Mesenteric artery thromboembolism. Source: Joel McFarland, Medpix 28818

MTBS2CK p.386

16

AcuteMesentericIschemia 52-year-old woman, alcoholic, with severe chest pain after an episode of persistent vomiting. No blood in vomitus, only stomach contents. Exam reveals crepitus over upper anterior chest wall. What is the next step?
a. b. c. d. e. Ethanol level Not useful in this case Chest X-Ray Will not show esophageal rupture Upper Endoscopy Risk further perforation Barium Esophagogram Not water-soluble Gastrografin Esophagogram

Source: haitham alfalah, commons.wikimedia.org

MTBS2CK p.387

EsophagealInjuries
MucosalTear MalloryWeissSyndrome EsophagealPerforation BoerhaaveSyndrome

Cause

Vomiting/Retching Alcoholics Hematemesis Odynophagia

Symptoms

Location Diagnosis

Gastroesophageal junction

Gastrografin esophagogram Noleakage Treatment Supportive Cauterizationifnecessary Complications Rare


MTBS2CK p.387388

Iatrogenicis#1(Endoscopy) Vomiting/Retching Alcoholics Retrosternal chestpain Severe,acuteonset RadiatestoLshoulder SubcutaneousEmphysema Distalesophagus LeftPosterolateral Aspect Gastrografin esophagogram Leakage EmergentSurgery Highmortality(25%) Acutemediastinitis Veryhighmortality

57-year-old male with BMI of 37 (18.5-24.9 normal) presents to ED with weakness. Vomiting large amounts of blood twice in last hour and passing one bright red stool. He has persistent pain in his mid-abdomen. Takes omeprazole for heartburn. BP 100/60, HR 120. What is the diagnosis? a Gastric Perforation No a. N bl bleed. d P Peritoneal it l signs. i b. Hemorrhagic ulcer c. Boerhaaves syndrome No hematemesis d. Diverticulosis No hematemesis e. Acute mesenteric ischemia No hematochezia

GastricPerforation Etiology Secondary to peptic ulcer disease


Increased production of gastric acid, combined with compromise of stomach lining results in ulcer formation
Source: commons.wikimedia.org

GastricPerforation Risk Factors H. Pylori infection NSAIDs Burns Trauma Head trauma Cancer Ethanol Tobacco
Source: User:KGH, commons.wikimedia.org

MTBS2CK p.388

MTBS2CK p.388

17

GastricPerforation

GastricPerforation/SignsandSymptoms

Pathophysiology
Perforation
Ulcer completely erodes through visceral wall
Gastric contents are released into abdominal cavity Result is damage to peritoneal structures Peritonitis (anterior + posterior ulcers) Pancreatitis (posterior ulcers)
MTBS2CK p.388

Acute onset mid-epigastric abdominal pain


Hemorrhage
Ulcer erodes into gastroduodenal artery
More common than gastric perforation Seen with posterior ulcers Requires upper endoscopy to evaluate/treat bleed

May radiate to right shoulder (phrenic nerve involvement) Worsening over time

Peritoneal signs (if any delay in seeking t t treatment) t)


Rigidity Guarding Rebound tenderness

MTBS2CK p.389

GastricPerforation

GastricPerforation
Management 1) Make Patient NPO
Prevents further extrusion of gastric contents into peritoneal cavity

Diagnosis Upright chest X-ray


Free air under diaphragm

2) Place NG Tube
Suction gastric contents Mitigates risk from newly-formed newly formed acid

Abdominal CT
Source: commons.wikimedia.org

3) Medical Management
Broad-spectrum antibiotics to combat infection IV fluids in preparation for surgery

4) Emergent Surgery
Exploratory laparotomy - Repair perforation
MTBS2CK p.389

MTBS2CK p.389

TheAbdomen Part2
AbdominalAbscess InflammatoryGIConditions AbdominalObstruction

16-year-old boy comes with new onset RLQ pain. He reports vague, mid-abdominal pain for last day, but when he awoke this morning it was much sharper and had changed location. He displays guarding and rebound tenderness, and palpation of LLQ hurts on right side. WBC count of 16,000/mm3 (4,5003) 11 000/ 11,000/mm What is the next step in management?
a. b. c. d. Abdominal X-ray Non-specific findings Colonoscopy Risk of perforation RLQ Ultrasound Non-specific findings Laparoscopic Appendectomy

MTBS2CK p.389

18

RightLowerQuadrantPain
Age > 60 yo?
Yes No

AbdominalAbscess Cause Surgical complication (#1) Inflammatory disease Trauma Signs/Symptoms Abdominal pain/distension Non-specific symptoms
Fever/chills GI symptoms nausea/vomiting, diarrhea

Cecal Diverticulitis
Make NPO, Place NG Tube Broad-Spectrum Abx
Male

Male or Female?
Female

Order -HCG, Pelvic US Appendicitis


Emergent Laparoscopic Appendectomy
Normal Elevated or Abnormal

Ovarian Torsion
Doppler US to diagnosis Laparoscopic surgery

Ectopic Pregnancy
Emergent Surgery

Rectal fullness
MTBS2CK p.390

AbdominalAbscess Diagnosis Abdominal CT


Visual evidence of abscess Information about surrounding structures

AbdominalAbscess

Source: Joel McFarland, Medpix 28716

MTBS2CK p.390

AbdominalAbscess Diagnosis Abdominal CT


Visual evidence of abscess Information about surrounding structures 78-year-old woman with severe LLQ pain, fever, and anorexia of one day. Shes had dark, tarry stools in the past. The patients husband brings in a medication list, which includes a stool softener and iron supplements. She displays guarding and rigidity. What is the diagnosis? a Ectopic pregnancy Patient a. P ti t is i postmenopausal t l b. Acute appendicitis Rare in elderly, RLQ in location c. Cholecystitis RUQ pain d. Acute diverticulitis e. Acute pancreatitis Central abd pain, radiates to back

CBC
May show elevated WBC count

Treament Broad-Spectrum Antibiotics Incision + Drainage


Percutaneous (CT-guided) versus Open
MTBS2CK p.390

19

InflammatoryGIConditions
Appendicitis
Cause Fecolith obstructing appendiceal orifice Anorexia Fever Periumbilical Pain RLQ Phys.Exam CTScan

Pancreatitis
Alcohol Gallstone obstructing pancreaticduct N/V Fever AbdominalPain Radiatesto back Amylase/Lipase CTScan(#1)

Diverticulitis
Fecal obstructionof bowelwallout pouchings N/V Fever LLQ Pain

Cholecystitis
Gallstone obstructingcystic duct N/V Fever RUQ pain Worsewith inspiration Ultrasound Fluid,Stones, ThickWall HIDAscan(#1) Laparoscopic Removal Perforation

Symptoms

Diagnosis

CTScan

Treatment

Laparoscopic Removal

IVFluids NPO Pseudocyst

Complication Abscess

Antibiotics(x1) Resection(if recurrent) Abscess

55 year-old man presents to the ED with severe, intermittent abdominal pain and nausea/vomiting. He cannot recall when he last passed flatus, but is certain he didnt have a bowel movement this morning. He has type 1 diabetes and had an open appendectomy in his 30s. Temperature of 101.8. Loud, high-pitched bowel sounds noted on auscultation. An abdominal plain film shows a partial small bowel obstruction. What is the best next step in management? a. Prescribe a laxative Will not relieve obstruction (may worsen) b. Place an NG-tube c. Emergent exploratory laparotomy Medical management first d. Prescribe antibiotics No evidence of bowel perforation e. Perform a colonoscopy Will not relieve obstruction

MTBS2CK p.391

BowelObstruction

BowelObstruction

Pathophysiology
A mechanical or functional obstruction of intestines
Leads to fluid/gas accumulation proximal to site of f obstruction Resulting pressure increase leads to
1. Pain 2. Decreased perfusion (and risk of necrosis)

Causes/RiskFactors
Obstruction can be partial or complete
Partial = GI contents are able to pass obstruction site Complete = No avenue for passage
Represents a much more severe condition

Adhesions from previous abdominal surgery


#1 in developed countries

Neoplasms Intussusception = Telescoping bowel


Most frequently seen in pediatric population

Hernia
#1 in undeveloped countries

Volvulus = Twisting of bowel on its mesentery Crohns disease


MTBS2CK Only p.393

Foreign bodies Intestinal atresia = Blind Pouch


Only seen in neonates

MTBS2CK p.392

BowelObstruction

BowelObstruction

Signs/Symptoms Severe, crampy abdominal pain


Colicky in nature

Diagnosis Labs
WBC, Lactate, pH

Nausea/vomiting Fever High-pitched, Tinkling bowel sounds


MTBS2CK p.392393

Abdominal X-ray
Air-fluid levels, dilated loops of bowel

Source: James Heilman, MD, commons.wikimedia.org

20

BowelObstruction Signs/Symptoms Severe, crampy abdominal pain


Colicky in nature

BowelObstruction Diagnosis Labs


WBC, Lactate, pH
Management 1) Make Patient NPO
Prevents further increase in bowel pressure

Abdominal X-ray
Air-fluid levels, dilated loops of bowel

Nausea/vomiting Fever High-pitched, Tinkling bowel sounds

2) Place an NG Tube (w/suction)


Lowers bowel pressure proximal to obstruction

Abdominal CT w/ oral contrast (#1)


Clear transition zone correlates with obstruction

3) Medical Management
IV Fluids Volume is lost due to third-spacing

4) Surgical Decompression (if indicated)


Complete obstruction (Emergent) Lack of improvement with medical management
MTBS2CK p.393

MTBS2CK p.392393

FracturesAlgorithm
Pain Swelling Bony deformity

Orthopedics
FractureTypes Upper pp Extremity yInjuries j BackPain FatEmbolism CompartmentSyndrome KneeInjuries

X-rays

Closed (skin intact)

Management
Open (skin puncture)
MTBS2CK p.393394

FractureTypes Stress Fractures Cause


Overuse injury secondary to repetitive insult to bone

FracturesTypes

Presentation Clue
High-performance athlete Common sites
1. Metatarsals 2. Tibia

Diagnosis
CT/MRI

Treatment
Rest and rehabilitation
MTBS2CK p.394

Compression Fractures Cause Vertebral fracture associated with poor bone quality Osteoporosis p ( (classical example) p ) Presentation Clue Elderly patient with back pain 33% thoracic spine, 33% thoracolumbar, 33% lumbar
MTBS2CK p.394

Diagnosis

Presentation

Symptoms:

Timing:
Acute onset with trauma

Neurovascular exam Closed reduction Surgery (ORIF) Emergency surgery (I&D)

21

FracturesTypes/CompressionFractures

FracturesTypes

Diagnosis X-ray CT, if inconclusive Treatment Controversial


MTBS2CK p.394
Source:commons.wikimedia.org

PathologicFractures
Cause
Fracture in bone weakened by disease
Metastatic cancer Multiple myeloma Pagets disease

Diagnosis
X-ray

Treatment
Treat fracture ID & treat primary disease

Presentation Clue
Patient with fracture after minimal trauma

MTBS2CK p.394

UpperExtremityInjuries

UpperExtremityInjuries/ClavicleFracture Workup
X-ray Careful distal neurovascular exam
Must rule out subclavian artery/brachial plexus injury

Clavicle Fracture Cause History of fall Blunt shoulder trauma Presentation Clue Pain over anterior shoulder Clavicle step-off

Treatment
Arm sling
Source: Mark D. Travis

MTBS2CK p.395

MTBS2CK p.395

UpperExtremityInjuries

Anterior Shoulder Dislocation Cause Fall on outstretched hand Sports, blunt trauma Signs/Symptoms Severe shoulder pain, swelling Arm is held in external rotation

UpperExtremityInjuries/ AnteriorShoulderDislocation Diagnosis X-ray, MRI (if necessary) Careful distal neurovascular exam Must rule out axillary artery or nerve injury Treatment Reduction w/sling immobilization
MTBS2CK p.395

Source: Daniel Patrick Moloney

MTBS2CK p.395

22

UpperExtremityInjuries

Posterior Shoulder Dislocation Cause


History of seizure or electric shock

UpperExtremityInjuries/ PosteriorShoulderDislocation

Diagnosis
XR, MRI (if necessary)

Signs/Symptoms g y p
Severe shoulder pain, swelling Arm is held in internal rotation

Treatment
Reduction w/sling immobilization

MTBS2CK p.395

MTBS2CK p.395

Source: Medpix 20030

19-year old with persistent wrist pain of 3 days duration. Pain began when he braced himself from a fall. Right wrist is notable for point tenderness and mild swelling at base of dorsal aspect of thumb, but no deformity. An X-ray is shown on the next slide.

Source: Medpix 6094

UpperExtremityInjuries 19-year old with persistent wrist pain of 3 days duration. Pain began when he braced himself from a fall. Right wrist is notable for point tenderness and mild swelling at base of dorsal aspect of thumb, but no deformity. An X-ray is shown below. What is the best next step in management? Trigger finger Cause
Inflammation of finger flexor pulley system
Leads to catching/locking of flexor tendon

Signs/Symptoms
Lone digit caught in flexion
Popping sensation if digit is manually extended

Scaphoid may be fractured a. Reassurance b. MRI Scan Unnecessary, worse than CT c. Thumb Spica Cast d. Amputation No indication e. Open Reduction + Internal Fixation No fracture (yet)

Moderatesevere pain

Diagnosis
Clinical Exam

Treatment
Corticosteroid injection
MTBS2CK p.395

23

UpperExtremityInjuries

DupuytrensContracture Cause
Thickening of palmar fascia, leading to flexion contracture
Digits cannot fully extend
Source: commons.wikimedia.org

UpperExtremityInjuries/ DupuytrensContracture

Risk Factors
Male > Female Age > 40 Northern European descent

Diagnosis Di i
Clinical Exam

Treatment
Surgery
MTBS2CK p.395

MTBS2CK p.395

BackPain A 66-year-old man comes to his PCP with bilateral leg pain of several months duration. The pain seems to be worst when he has to walk several blocks, and improves when he sits down. Leaning forward (on a bench, shopping cart, etc.) alleviates the pain. He is a non-smoker. What is the most appropriate next diagnostic step?
a. b. c. d. e. Lower Extremity XR Useful for bony trauma Doppler US of the calf Useful for DVT evaluation Ankle-Brachial Indices Diagnoses claudication Spine MRI Leg MRI Useful for soft-tissue injury/stress fractures

Spinal Stenosis Cause Arthritic changes result in narrowing of spinal canal Lumbar #1, Cervical #2 Symptoms Neck/Back Pain Bilateral leg/buttock pain + numbness Pseudo-claudication Worse with walking, improves with spine flexion

BackPain/SpinalStenosis

BackPain

Diagnosis MRI Treatment NSAIDs vs. surgery g y

Herniated Disk Disease Cause Intervertebral disk herniates, compressing spinal nerve root
Often associated with lifting injury Most frequently seen in elderly Symptoms

Electric Pain following a dermatome distribution

Source: nih.gov, commons.wikimedia.org

24

BackPain/HerniatedDiskDisease

BackPain/HerniatedDiskDisease Diagnosis Clinical Exam (Straight Leg Raise)


Consider MRI if red flags present

Treatment NSAIDS, Activity Modification


Source: user:debivort, commons.wikimedia.org

Source:Mjorter, commons.wikimedia.org

BackPain

BackPain

RedFlags ConcernforMetastasisorAbscess

Pain not relieved by rest


Not worse with activity

Fever Neurological deficits


Bowel/Bladder incontinence Abnormal reflexes

Management MRI to evaluate for mass lesion Emergent glucocorticoids if neurological findings

Night pain Constant, dull pain > 6 weeks

History of Cancer

FatEmbolism AMedicalEMERGENCY
22-year-old woman was struck by a car 3 days ago while walking her dog. The impact broke her femur, which required immediate surgical repair. This morning the patient is confused and has difficulty catching her breath. ABG shows PaO2 55 mmHg, and exam reveals a petechial rash. What is the most likely diagnosis? a. Myocardial infarction No rash b. Pancreatitis Expect abdominal pain c. Rhabdomyolysis No hypoxemia d. Fat embolism

Etiology Traumatic long bone fracture (#1 = Femur) Releases marrow fat into circulation Fat vesicles are too large to pass through capillaries Result is vascular occlusion

H&E stain, lungs blood vessel with fibrinoid material and an optical empty space indicative of the presence of lipid. Source: Boris L Kanen, Ruud JLF Loffeld, commons.wikimedia.org

MTBS2CK p.396

MTBS2CK p.396

25

FatEmbolism

FatEmbolism Diagnosis ABG: PO2 < 60 mmHg CBC: Decreased platelet count Chest X-ray: Infiltrates Urinalysis: Fat droplets in urine T t Treatment t Respiratory Support
Goal O2 Sat > 90% Consider intubation/mechanical ventilation if severely hypoxic

Signs/Symptoms Onset 0-5 days after fracture Respiratory Distress (V/Q mismatch) SOB Tachypnea yp Confusion (Involves brain vasculature) Petechial Rash (Involves skin capillaries) Chest wall, upper extremities

MTBS2CK p.396

MTBS2CK p.396

CompartmentSyndrome

CompartmentSyndrome/Signs&Symptoms
Early Findings Pain Severe, worse with muscle stretch Pallor Pale skin from blood flow
MTBS2CK p.396397

Pathophysiology
Injury occurs, resulting in Pressure builds, swelling leading to severe tissue compression Fracture #1 (tibial,
forearm) Nerves Burns Muscle Crush injuries Vessels (Reperfusion syndrome) Resulting damage can

The 6 Ps

Late Findings Pulselessness Absent distal pulses (ominous finding)

Parathesias Pins and needles nerve involvement

Poikilothermia Cold, due to decreased blood flow

In closed compartment (fascial sheath, cast), theres no escape for increasing pressure
MTBS2CK p.396

lead to Volkmanns ischemic contracture, limb loss, or death

Paralysis Inability to move distal musculature

CompartmentSyndrome

KneeInjuries Basic Principles


Ligamentous/meniscal knee injuries have similar symptoms
Knee Pain Swelling Instability The key factor is time course in which symptoms present

Treatment AMedicalEMERGENCY

Surgical fasciotomy Releases compartment p pressure

Pay attention to mechanism of injury! Physical exam provides clues to etiology


Special tests for each type of knee injury

Diagnostic test of choice is always MRI


MTBS2CK p.397
Source: Sarte, commons.wikimedia.org

26

KneeInjuries
Mechanismof Injury Symptom Onset Exam Maneuver Management

MCL/LCL Traumato Immediate Injury contralateral aspectofknee ACLTear Twistingor directimpact

Medial/Lateral Conservative Instability

Immediateand AnteriorDrawer Arthroscopic Repair Severe sign LachmanTest Repairvs. Conservative management

Immediatebut Posterior PCLTear Forced Hyperextension Mild Drawersign

22-year-old hockey star is checked by an opposing player, and his leg gets tangled as he falls. He experiences a popping sensation in his left knee, which immediately swells. The team physician performs a physical exam and notes a positive anterior drawer sign as well as medial knee instability. He recommends an MRI to evaluate the likely ACL and MCL tears. What other structure will likely be injured on MRI?

Meniscal TwistingInjury Delayed(1224 Jointline Conservative hours) tenderness Tear McMurrays Clicking/Locking
MTBS2CK p.398

a. b. c. d.

Lateral Collateral Ligament Different injury mechanism Lateral Meniscus Medial Meniscus Posterior Cruciate Ligament

MTBS2CK p.398

27

You might also like